You are on page 1of 160

Contents

Introduction 1 ColllillUilY ":l/II.llillll lilr 1{"di,,1 I.'IIJ\\' I(HI


Flow I.&lW~ IIII
1. Fluid Flow in Porous Media 2 Sillgl~-Ph&l~ I:hlw of Slightly
1.1 Inlr(>duction 2 C()\llpr~~~ibl~ Fluid~ I () I
1.2 Thc Idc&l1Rc~crvoir Modcl 2 Sillgl~-Ph&l~~ G&I~FI(IW I().:?
1.3 Solutions 10 Diffusivily EI.JU&llion 3 Simull&ln~ou~ Flow of Oil, W"ll'r, &llId (Ja~ I ().:?
1.4 R&ldius of Invc!iligalion 13
1.5 Principle of Superposition 15 Appendix B: Dimensionless Variables 103
1.6 Homer's Approxim&ltion 18 Inlr(>ducli(m I().~
RaJi&l1 Fill"' (If a Slightly Colllrr~.'i~ihl~ I'lllilll().~
2. Pressure Buildup Tests 21 Radi&ll Flow With ColI~lalll BliP I(~
2.1 Introduction 21
2.2 The Ide&ll Buildup Te!il 21 Appendix C: Van Everdingen and Hurst
2.3 Aclual Buildup T~~I~ 23 Solutions to
2.4 DcvialuJl1~ From A~~umplu)n!i Diffusivity Equations 106
in Ideal T~~I Th~OI)' 24 1lllrlklul'lillll II)(J
2.5 Qualilaliv~ 8~havior of Fi~ld Te~l~ 26 ('(III:.lanl Rale al 1111I~r1i('Ulldal)'.
2.6 Effecls and Durdlion of Aflertlow 27 No Flow Al'ro~~ ()UI~r lillulldal)' 1(ltJ
2. 7 ()el~mlillalu)n of Peml~,lbilily 2() ('oll~lalll Rail' al Imll'r 1illllJlllal)' ,
2.M W~III)&lIII&lg~ ,llIll Slillllllalioli .~() (illl:.I"111 Prl'~~llrl' al ()Ull'r li'llllillal)' 1117
2. () Pre~~urc l..cvel in Surrounding ('(III.'ilanl Pr~.'i.'illr~ al 1IIIIcr lillllllll"I)'.
FOml&lIU)n 35 No l"low Al'ro~~ ()(lIl'r IilIUlIll,,'Y 11.'\
2.10 Rcservoir Umil!i Tesl 41
2.11 Modilic&llions for Ga!ics 44 Appendix D: Rock and Fluid
2.12 Modifications for Mullipha~e Flow 45 Property Correlations 119
Inlr()(juclion II Y
3. Flow Tests 50 Psclld(Il'rilical T~mPl..'I.llure alill
3.1 Introduction 50 Prc~slIrc of l.il/uid IlyJrlll'iln)(llI~ IIY
3.2 Pressure Drdwdown Tests 50 Bubblc-lX)int Prcssur~ of ('l1Id~ Oil IIY
3.3 Multirate Tests 55 Solution GOR IIY
Oil FOmlalU)n Volume Filclor 12()
4. Analysis of Well Tests Compressibility of Und~~illural~d Oil 121
Using Type Curves 63 COInpressibilily of Sillllral~J ('ruu~ ()il 122
4.1 Introduction 63 Oil Viscosity 124
4.2 Fundamenlals of Type Curves 63 Solubility of Ga~ in Wal~r 124
4.3 Ramey's Type Curve!i 64 Wal~r Fomtalion Volllm~ Fal"lor 125
4.4 McKinley's Type Curve!i 68 Compr~ssibilily (Jr Wal~r ill
4.5 Gringarten L't ill. Type Curve!i Und~r~illurill~J Rl'~~lvlJir~ 12(J
for Frdclurcd Wcll!i 71 Cl)mpr~~sihilily or Wall'r in
a S,IIUrall'd Rl'~l'lv(lir 126
5. Gas Well Testing 76 Wall'r Vi~l'o~ily 12X
5. I Introduction 76 P~l'ullocrilicill Prol~11i~~ or GilS 12X
5.2 Ba!iic Theory or G&I!i Flow (J.I~-I..IW I)~vialioll F"l"l(lr (/-I"'aclllr) ..
in Rescrvoir!; 76 anll GilS Foml"lillli Vlllulll~ F.Il'llJr 12X
5.3 Flow-Artcr-Flow Tc!its 77 (jil~ Clllllrr~~~ihiliIY 1.11
5.4 Isochron&ll Tcsls 79 (ja~ Vi~c(l~ily 131
5.5 Modifi~d I~ol'hrollal Tc~t~ HJ 1:llnllalilln (")lIlrrl'~~ihilily 1.~2
5.0 U!ie of P!i~ulilipres~ure in
Gas Well Tesl Analysis M5 Appendix E: A General Theory
of Well Testing 134
6. Other Well Tests 89
6. I IlIlr()UllCli'llI HI) Appendix F: Use of SI Units in
0.2 1111~rl~r~lIcl.' Tl.'~lillg X') Well-Testing Equations 138
6.3 Pulse Tc!iling 91
6.4 ()rill~tcm Tcst~ 97 Appendix G: Answers to
6.5 Wirclinc Fontlalioll Te~l~ 9M Selected Exercises 148

Appendix A:Development of Differential Nomenclature 151


Equations for Flow Bibliography 154
r in Porous Media 100 Author Index 156
Introduction 100 Subject Index 157
Continuity Equation for
Three-Dimcn~u)nal Fk)w I(X)

www.petroman.ir ~Ul~;c",
-
Introduction
I'
This textbook explains how to use well pressures and dis~llssions of pressure buildup tests; pressure
now rates to evaluate the formation surrounding a drawdown tests; other now tests; type-curve analysis;
tested well. Basic to this discussion is an un- gas well tests; interferen~e and pulse te~ts; and
derstanding of the theory of fluid flow in porous drillstem and wireline formation tests. Fundamental
media and of pressure-volume-temperature (PVT) principles are emphasized in this discussion, and little
rt:lation~ for fluiJ ~ystL'm~of practil;:al iI1tere~t. Thi~ I:ffort i~ made 10 bring lhe intended audi~1ce-
book contains a review of these fundamental con- undergraduate pelroleum engineering students -to
cepls, largely in summary form. Llie frontier~ of tile subje~t. Tliis role is tilled mu~11
One major purpose of well testing is to determine better by other publications, such as the Society of
11I~abililY ofa format")n to pr()du~e reservoir Iluius. Pelroleum Engineers' monographs on welliestingl,2
Furtller, it is importallL to determine the underlying und Alberta Energy Re~ourcL's and Con~ervation
reason for a well's productivity. A properly de~igned, Board'~ gas well testing manual.3
executed, and analyzed well test usually can provide Basic equations and examples use engineering
informal ion about formulion pemleabilily, exlent of unil~. However, to ~mooth lile expecled transition to
wellbore damage or stimulalion, re~ervoir pres~ure, lhe Inti. System of Units (SI) in the petroleum in-
and (perhaps) reservoir boundaries and hetero- dustry, Appendix F Jis~usses lhis unit system and
geneities. restates major equations in SI units. In addition,
The basic test method i~ Lo create a pressure answers to examples worked out in the text are given
drawdewn in the well bore; this causes formation in SI units in Appendix F.
nuids to enter the wellbore. If we measure the flow
rate and the pressure in the well bore during Iteferences
production or the pressure during a shut-in period I. Mallhews,C.S,and Russell, D.G.: PressureB"i/dupandRow
following production, we usually will have sufficient ~ests;1' ~Vells.
Monograph Series.SP~.Da~las (1967)I. .
infor tion to characlerize the tested well. 2. Eilrlullgher.R:C. Jr.: AtI"",,('t'S '" II ell Test AnalysIs,
?"a Monograph Senc£,SPE,J}illla£(1977)s.
ThIs book beglJ}~ wllh Ii dl~cus~lon of basIc 3. 171('uryuntlJ'rut't;('euflhe
'It-!,.,;,,}:
ufGus ~~/('Ils,
IlairdI:dilion,
equations that describe the unsteady-state Ilow of I Pub.ECRIJ-75-34, EncrgyRI:£our~c£ andConservalion Uoard,
fluids in porous media. It then moves into Calgary,Atla.(1975).

r- www.petroman.ir
I; '

Cllapter 1
Fluid Flow in Porous Media

1.1 Introduction
In this initial chapter on nuid now in porous media, oil), we obtain a partial differential equation that
\\'c hcgin with a discussion of the differential simplifies to
Cqllation~ t hat are u~~d most often to model un- a2p J ap cf>JlC ap
~tcady-~tate now. SImple statements of these a:z+-a= ka' (1.1)
cqllations are provided in the text; the more tedious r r r 0.<XX>264 t
mathcmatical details are given in Appendix A for the if we assume that compressibility, c, is small and
in~tructor or student who wishes to develop greater independent of pressure; permeability, k, is constant
lInderstanding. The equations are followed by a and isotropic; viscosity, Jl, is independent of
di~cll,~sionof some of the most useful solutions to pressure; porosity, cf>,is constant; and that certain
these equations, with emphasis on the exponential- terms in the basic differential equation (involving
intcgral solution describing radial, unsteady-state pressure gradients squared) are negligible. This
now. An appended discussion (Appendix B) of equation is called the diffusivity equation; the term
dimcnsionless variables may be useful to some 0.OOO264klcf>Jlc is called the hydraulic diffusivity and
readcrsat this point. frequently is given the symbol '7.
of
The chapter concludes with a discussion of the
sllperposition.
radills-of-investigationSuperposition,
concept and ofillustrated in
the principle p,
Eq. 1.1 is written in terms of field units. Pressure,
feet;
is inporosity,
pounds percf>,square
is a fraction; viscosity,
inch (psi); distance,Jl,r, isis in
in ~
,
t

mlilt i\vell infinite reservoirs, is used to simulate centipoise; compressibility, c, is in volume per
simple reservoir boundaries and to simulate variable volume per psi [c=(I/p) (dpldp)]; permeability, k,
rate production histories. An approximate alter- is in millidarcies; time, t, is in hours; and hydraulic
native to superposition. Horner's "pseudopro- diffusivity, '7,has units of square feet per hour.
dlldiml time," completes this discussion. A similar eqllation can be developed for the.adial
now of a nonideal gas:
1.2 The Ideal Reservoir Model I a a cf> a
To .dcvelop a~alysis and design techniqu~s fo~ \~ell -a (~ r -£) = 0.000264 k at ( '!), (1.2)
tCStlllg, we first must make several simplifYing r r JlZ Z
a~sumptiOJ1S about the well and reservoir that we are where Z is the gas-law deviation factor.
nlOdcling. We Ilaturally make no more simplifying For simultaneous now of oil, gas, and water,
assllillptions thall are absolutely necessary to obtain I a ap cf>c ap
simple, useful solutions to equations describing our -a(r a)= O-()(X)2~ at' (1.3)
sitllal ion -but we obviously can make no fewer r r r. ,
assllmptions. These a~sumptions are introduced as where c, is the total system compressibility,
Ilccdcd, to comhine (I) the law of ~onservation of c =S c +S c ,+S c +c. (1.4)
mass, (2) Darcy's law, and (3) equations of state to (0 0 WM g P, f
achieve our objectives. This work is only outlined in and the total mobility ~, is the sum of the mobilities
'his cllapter; detail is provided in Appendix A and the of the individual phases:
Refercnces. k k k
Consider radial now toward a well in a circular .~,'= (-.£ + :.:.I.+ ~). (1.5)
re~crvoir. If we comhine the law of conservation of P-o Jlp, P-w
ma~~ and Darcy's law for thc isothermal now of In Eq. 1.4, So refers to oil-phase saturation, Co to
n\lid~ of small alld constant compressibility (a highly oil-phase compressibility, ,,>,
M'and c M'to water phasc,
satisfactory model for single-phase now of reservoir S" and c" to p,asphase; and c f is the formation

Il'" h~ dcc, lill..


www.petroman.ir
..lI.~ "
~;.;. ~
FLUID FLOW IN POROUS MEDIA 3

compressibility. In Eq. 1.5, ku i~ the effe\:live per- al1u where Jl and YI are BI.'S~I.'I fun\:tion~. (Total
meability to oil in lhe presence of the other phases, ~ompre~sibililY, CI' is used in all equalion5 in lhi5
and 1J.0 is the oil viscosilY; k and p. refer to the gas chapter becau~e even formalions thaI produce a
phase; and k wand p.w refer to tte water phase. single-phase oil contain an immobile waler pha5e and
Because the formation is considered compre5sible have formatioll compre5~ibility.)
(i.e., pore volume decreases as pressure decrea~es), The reader unfamiliar with Bessel function5 ~hould
porosity is not a constanl in Eq. 1.3 as it was assumed not be alarmed at this equation. It will nor be
to be in Eqs. 1.1 and 1.2. necessary to use Eq. 1.6 in its complele form to
, ',' " calculate numerical values of Pw/; instead, we will
1,3 Solutions to Dlffu~lvlty Equation use limiting forms of the 50lutlon in mo~t com-
This section deals with useful solutions to the dif- putations. The most imporlant facl about Eq. 1.6 i5
fll~ivity equation (Section 1.2) uc~~ribing Ihe Ilow of that, unu\.'r the a5~umptioll~ nwdl.' in il~ dl.'v\.'lopnl\.'lll,
a slightly compressible liquid in a porous medium. it i~ an exaci sohllion to Eq. 1.1, It ~ometime~ i~
We also have some comments on solutions to Eqs. called the van Everdingen-Hurst constant-terminal-
1.2and 1.3. rate solution.2 Appendix C discusses this solution
There are four solutions to Eq. 1.1 that are par- more colllpletely. Because it is exacl, it serves a5 a
licularly useful in well resting: the solution for a standard wilh which we may compare more useful
bounded cylindrical reservoir; the solution for an (but more approximate) solutions. One such ap-
infinite reservoir with a well considered to be a line proximate solution follows.
source
state solution;
with zero
and well
the bore
solution
radius;that the
includes
pseudosteady-
well bore Infinite Cylindrical Reservoir With Line-Source W~"

storage for a well in an infinite reservoir. Before we Assume that (I) a well produces at a constant rate,
discuss these solutions, however, we should sum- qB; (2) the well has zero radius; (3) the reservoir is at
marize the assumptions that were neces~ary to uniform pressure. Pi. before prodllction begins; and
develop Eq. 1.1: homogeneous and isotropic porous (4) the well drains an infinite area (i.e.. that P-Pi as
medium of uniform thickness; pressure-independent ,- CX». Under those conditions. Ihe solution to Eq.
rock and fluid properties; small pressure gradients; 1.1 is
radial flow; applicability of Darcy's law (sometimes qBp. ( -'- 948 ~p.CI,2
called laminar flow); and negligible gravity forces. P=Pi+70.6~ Ei k ) (1.7)
We will introduce further as~umptions to obtain -I
solutions to Eq. 1.1. where the new symbols are p, the pressure (psi) at
distance, (feet) from the well at time I (hours). and
Bounded Cylindrical Reservoir ~ -u
Solution of Eq. 1.1 requires that we specify two Ei( -x) = -~ ~dl',
boundary conditions and an initial condition. A x U

realistic and practical solution is obtained if we the Ei function or exponential integral.


assume that (1) a well produces at constant rate. qB. Before we examine the properties and implications
into the well bore (q refers to flow rate in STB/D at of Eq. 1.7, we must answer a logical question: Since
surface conditions, and B is the formation volume Eq. 1.6 is an exact solution and Since Eq. 1.7 clearly
factor in RB/STB); (2) the well. with wellbore radius is based on idealized boundary conditions, when (if
r w' is centered in a cylindrical reservoir of radius, e' ever) are pressures calculated at radius, w from Eq.
and that there is no flow across this outer boundary; 1.7 satisfactory approximations to pressures
and (3) before production begins, the reservoir is at calculated from Eq. 1.67 Analysis of these solutions
uniform pressure. Pi. The most useful form of the shows3 that the Ei-function solution is an accurate.
desired solution relates flowing pressure, Pwf' at the approximation to the more exact solution for time
sand face to time and to reservoir rock and fluid 3.79x 105 <PIJ.CI'I~.lk<I<948 <plJ.c,,;/k. For times
properties.Thesolutioni~1 lcss than 3.79xlo5 ~/'C",~.lk, Ihe assllmption of
qBIJ. [ 21 3 ll.'ro well ~ize (i.e.. a~~umillg the well 10 be a line
Pwf=Pi -141.2- -2!!-.- + In' eO --SOllrcl.' or ~ink) lilllil~ IIII.' uc\:uracy of IIII.' c'llluliull; ,II
kh 'eO 4 times grealer than 948 <P1J.(.'/,;lk. Ihe reservoir's
~ 2 ht)lllldari\.'s hl.'gill Iu afft.'\:1 III\.' prl.'SSllrC distrihlili0l1
+2E e-",I/JJ1(u,,'eO) 1. ..(1.6) ~11..III.t.' rl.':il.'.rvoir,!;u tllulillt.' rt.'!;t.'rvuir i~ 110 lulIgt.'r
- l .
n- "
a2rr2/_.
[J l (a lIe
'
-\~12/~
O )-J2 1 (a
n
\
»)j1 1111.ll1lteactll1g.
f I l fi .
A urt ler sImp I Icatlon
. . 0
f h I
t e so utlon to t h e fl ow
where. for efficiency and convenience. we have equation is possible: for x<0.02. Ei( -x) can be
introduced the dimensionless variables approximated with an error less than 0.6070 by

'eO='e/rw Ei(-x)=ln(I.78Ix). , (1.8)

and To evaluate the Ei function, we can use Table 1.1 for


2 0.02 <XS 10.9. For xsO.02, we use Eq. 1.8; and for
to=0.OOO464kl/~p.CI'W' x>10.9. Ei(-x) can be considered zero for ap-
where the an are the roots of plications in well testing.
In practice. we find thal most wells have reduced
Jl(an'eO)Yl(an)-Jl(an)Yl(an'eO) =0; permeability (damage) near the well bore resulting

www.petroman.ir
4 -, ~'""""" WELL TESTING
~- --

from drilli'tg or colttplction opcralion~. Many otllcr q/lll.


wcll~ arc ~ti,nt,lalcu by acidimtion or Itydraillic ~/J.f= 141.2'-k-' -111(rflrlt.)
fracturing. Eq. I. 7 fail~ to modcl such wcll~ properly; .f '
its derivation holds the explicit assumption of
uniform permeability throughout the drainage area qBp.
ofthewelluptothewellbore. Hawkins4 pointed out -141.2~ In(rslr".)
that if the damaged or stimulated zone is con~idered
eqtlivalent to an altered zone of uniform permeability
(kf) and outer raditls (r s)' the additional pressure qBp. k
drop across this zone (L\IJ.f)can be modeled by the =141.2~(k -1)ln(rslrw). (1.9)
steady-state radial now equation (see Fig. 1.1). Thus, ' s

TABLE 1.1. -VALUES OF THE EXPONENTIAL INTEGRAL, -E/(- x)

-EI ( -x), 0.000 < 0.209, interval -0.001


x 0 1 2 3 4 5 6 7 8 9
0-:00 -.;:;;; 6:332 5:639 5:235 ~ ~ 4:545 4:m- 4:259 ~
0.01 4.038 3.944 3.858 3.779 3.705 3.637 3.574 3.514 3.458 3.405
0.02 3.355 3.307 3.261 3.218 3.176 3.137 3.098 3.062 3.026 2.992
0.03 2.959 2.927 2.897 2.867 2.838 2.810 2.783 2.756 2.731 2.706
0.04 2.681 2.658 2.634 2.612 2.590 2.568 2.547 2.527 2.507 2.487
0.05 2.468 2.449 2.431 2.413 2.395 2.377 2.360 2.344 2.327 2.311
0.06 2.295 2.279 2.264 2.249 2.235 2.220 2.206 2.192 2.178 2.164
0.072.151 2.138 2.125 2.112 2.099 2.087 2.074 2.062 2.050 2.039
0.08 2.027 2.015 2.004 1.993 1.982 1.971 1.960 1.950 1.939 1.929~
0.09 1.919 1.909 1.899 1.889 1.879 1.869 1.860 1.850 1.841 1.832
, 0.10 1.823 1.814 1.805 1.796 1.788 1.779 1.770 1.762 1.754 1.745
0.11 1.737 1.729 1.721 1.713 1.705 1.697 1.689 1.682 1.674 1.667
'" 0.12 1.660 1.652 1.645 1.638 1.631 1.623 1.616 1.609 1.603 1.596
0.131.589 1.582 1.576 1.569 1.562 1.556 1.549 1.543 1.537 1.530
0.14 1.524 1.518 1.512 1.506 1.500 1.494 1.488 1.482 1.476 1.470
0.15 1.464 1.459 1.453 1.447 1.442 1.436 1.431 1.425 1.420 1.415
0.16 1.409 1.404 1.399 1.393 1.388 1.383 1.378 1.373 1.368 1.363
-0.17 1.358 1.353 1.348 1.343 1.338 1.333 1.329 1.324 1.319 1.314
0.18 1.310 1.305 1.301 1.296 1.291 1.287 1.282 1.278 1.274 1.269
0.19 1.265 1.261 1.256 1.252 1.248 1.243 1.239 1.235 1.231 1.227
0.20 1.223 1.219 1.215 1.210 1.206 1.202 1.198 1.195 1.191 1.187

-Ei( -x), O.OO<x< 2.09, Interval = 0.01


0.0 + ~ 4.038 3.335 2.959 2.681 2.468 2.295 2.151 2.027 1.919
0.1 1.823 1.737 1.660 1.589 1.524 1.464 1.409 1.358 1.309 1.265
0.21.2231.1831.1451.1101.0761.0441.0140.9850.9570.931
0.3 0.906 0.882 0.858 0.836 0.815 0.794 0.774 0.755 0.737 0.719
0.4 0.702 0.686 0.670 0.655 0.640 0.625 0.611 0.598 0.585 0.572!
0.5 0.560 0.548 0.536 0.525 0.514 0.503 0.493 0.483 0.473 0.464
0.6 0.454 0.445 0.437 0.428 0.420 0.412 0.404 0.396 0.388 0.381
"
'.' 0.7 0.374 0.367 0.360 0.353 0.347 0.340 0.334 0.328 0.322 0.316
...lc 0.8 0.311 0.305 0.300 0.295 0.289 0.284 0.279 0.274 0.269 0.265 ..
0.9 0.260 0.256 0.251 0.247 0.243 0.239 0.235 0.231 0.227 0.223
1.0 0.219 0.216 0.212 0.209 0.205 0.202 0.198 0.195 0.192 0.189
1.1 0.186 0.183 0.180 0.177 0.174 0.172 0.169 0.166 0.164 0.161
1.2 0.158 0.156 0.153 0.151 0.149 0.146 0.144 0.142 0.140 0.138
1.30.1350.1330.1310.1290.1270.1250.1240.1220.1200.118
1.4 0.116 0.114 0.113 0.111 0.109 0.108 0.106 0.105 0.103 0.102
1.5 0.1000 0.0985 0.0971 0.0957 0.0943 0.0929 0.0915 0.0902 0.0889 0.0876
1.6 0.0863 0.0851 0.0838 0.0826 0.0814 0.0802 0.0791 0.0780 0.0768 0.0757
1.7 0.0747 0.0736 0.0725 0.0715 0.0705 0.0695 0.0685 0.0675 0.0666 0.0656
1.8 0.0647 0.0638 0.0629 0.0620 0.0612 0.0603 0.0595 0.0586 0.0578 0.0570
1.9 0.0562 0.0554 0.0546 0.0539 0.0531 0.0524 0.0517 0.0510 0.0503 0.0496
2.0 0.0489 0.0482 0.0476 0.0469 0.0463 0.0456 0.0450 0.0444 0.0438 0.0432

2.0<x< 10.9, Interval = 0.1


x 0 1 2 3 4 5 6 7 8 9
2 4.sg-x1Q-=~ 4.26 x 10~~ mx1r 3:2W0-=2 2.84x-1~2 2.49 x 10 -~ 2.19 x 10 -2 '1:92X~ 1.69 x 10 -2 1.48 x 10 -2
3 1.30x10-2 1.15x10-2 1.01x10-2 8.94x10-3 7.89x10-3 6.87x10-3 6.16x10-3 5.45xI0-3 4.82x10-3 4.27x10-2
4 3.78x10-3 3.35xI0-3 2.97x10-3 2.64x10-3 2.34x10-3 2.07x10-3 1.84x10-3 1.~x10-3 1.45x10-3 1.29x10-3
5 1.15x10-3 1.02x10-3 9.08x10-4 8.09x10-4 7.19x10-4 6.41x10-4 5.71x10-4 5.09x10-4 4.53x10-4 4.04x10-4
6 3.60xI0-4 3_21x10-4 2.86xI0-4 2.55x10-4 228x10-4 2.03x10-4 1.82x10-4 1.62x10-4 1.45x10-4 1.29x10-4
7 1.15xI0-4 1.03x10-4 9.22x10-5 8.24x10-5 7.36x10-5 6.58x10-5 5.89x10-5 5.26x10-5 4.71x10-5 4.21x10-5
8 3.77x10-5 3.37x10-5 3.02x10-5 2.70x10-5 2.42x10-5 2.16x10-5 1.94x10-5 1.73x10-5 1.55x10-5 1.39x10-5
9 1.24x10-5 1.11x10-5 9.99x10-6 895)(10-6 B02x10-6 7.1Bx10-8 6.44)(10-6 5.77x10-8 5.17x10-8 4.64x10-8
10 4.15x10-8 3.73x10-6 3.34)(10-6 3.00x10-6 2.68x10-6 2.41)(10-8 2.16x10-8 1.94,<10-6 1.74x10-8 1.56x10-6
.Adapl@d'rom Nlsle, RG.: "How To Use The Expon@nlialinleoral," Pel Eng.(~uO. 1956)8171.173.

www.petroman.ir
FLUID FLOW IN POROUS MEDIA -5-,.:

fonnation the damage extelld~, tll~ larger the


t I numerical value of s. There is no uPI1Crlimit for ~'.
Some newly drilled wells will not flow at all before
stimulation; for these wells, ks =0 and s-~. If a
well is stimulated (ks >k), s will be negative, and the
P deeper the stimulation, the greater the numeril.:al
~e value of s. Rarely does a stimulated well have a skin
S factor less than -7 or -8, and such skin factors
f? arise only for wells with deeply penetrating, highly
W I conductive hydraulic fractures. We should notc
finally that, if a well is neither damaged nor
r ~ stimulated (k=ks)' s;O. We caution the reader that
W S Eq. 1.10 is best applied qualitatively; actual wells
rarely can be characterized exactly by such a sim-
plified model.
r ..Before leaving the discussion of skin factor, we
should point out that an altered zone near a par-
F' 1 1 S h t' f ' , , ticular well affects only the pressure near that well-
Ig, .-w~II~~r~,'c 0 pressure distribution near i.c., the pressure in the unaltered formation away
from tile well is 110laffected by the I.'xi~ll.'l1l.:cof till.'
altered zone. Said another way, we use Eq. I. II to
Eq. 1.9 simply states that the pressure drop in the calculate pressures at the sandface of a well with an
altered zone is inversely proportional to k rather altered zone, but we use Eq. 1.7 to calculate pressures
than to k and that a correction to the pressur: drop in beyond the altered zone in the formation surroun-
this region (which assumed the same permeability, k, ding the well. We have presented no simple equations
as in the rest of the reservoir) must be made. that can be used to calculate pressures for radiu!i, r,
Combining Eqs. 1.7 and 1.9, we find thalthe tolal ~llchthatrw<r<.rs,butthiswilloffernodifficultic!i
pressuredrop at the well bore is In well test analysIs.

pj-Pwf= -70.6~
qBJJ. .
E, -kt
( 948 tPlJoC
tr~
) +Aps Example 1.J-Calculation of Pressures

Beyond the Wellbore Using


q BJJ.,
[ ( 948.1. c r2
'I'll t w
) the Ei-Function Solution
; -70.6 ~ E, -kt Problem. A well and reservoir have the following
characteristics: The well is producing only oil; it is
producing at a constant rate of 20 STB/D. Data
( --Ik ) ( r s )] describing the well and formation are
-2In -.
ks rw Il ; 0.72 cp,
For r=rw, the argument of the Ei function is suf- k; 0.1 md'_5 .-1
ficiently small after a short time that we can use the c, ; 1.5 x 10 pSI
logarithmic approximation; thus, the drawdowl1 i!i Pj ; 3,000 psi,
1,688 tPJJ.ctr~,, [( ) r I! ;
,
3,()()()It,
--

pj-Pwf- -qBJJ.
-70.6-:- In k rw ; 0.5 ft,
kh t, Bo ; 1.475 RB/STB,
h ; 150 ft,
( --Ik ) ( r s )J tP ; 0.23, and
-2 In --.
r k s ; o.
s w
It is convenient to define a skin factor, s, in terms of Calculate the reservoir pressure at a radius of I ft
the properties of the equivalent altered zone: after 3 hours of production; then, calculate the
) (~)
( ~k - Iln. pressur~ at radii of 10 and 100 ft after 3 hours of
$-- (1.10) produl.:tlon.
s rw
Thus, the drawdown is Solution. The Ei function is not an accurate solution
[ ( 1,688tPJJ.Ct"~ ) ] toflowequationsuntilt>3.79XI05tPIlCtr~,,/k.Here,

pj-Pwf-
-qBJJ.
-70.6- In -2s. 3 79x 105tP ~

kh kl .IlC t w = [(3.79 x 105)(0.23)(0.72)


(1.11) k
Eq. 1.10 provides some insight into the physical .(1.5x 10-5)(0.5)2]/(0.1)
significance of the sign of the skin factor. If a well is ; 235 <I; 3 hours
damaged (ks <k), s will be positive, and the greater ..
the contrast between ks and k and the deeper into the Thus, we can use Eq. 1.7 with satisfactory accuracy if

www.petroman.ir
rr
6 ,,":;, -WELL TESTING-
",j""

the re~ervoir is still infinite acting at this time. The of Eq. 1.6, which de~cribes pressure behavior with
rl'~l'rvoir will act a~ an infinite reservoir until 1 > 948 time for a well centered in a cylindrical reservoir of
1/>1(("";
1 k. radius r (" The limiting form of interest is that which
is valid for large times, so that the summation in-
Here, volving exponentials and Bessel functions is
948 cf>1Lc,r~ negligible; after this time (I >948 cf>1(C,r~/k),
k = r (948)(0.23)(0.72) -qB1L
( 21 D 3
P,vf-Pi-141.2- y+lnrl'n--, )
.(1.5 x 10 -5)(3,000)2 J/0.3 = 211 ,900 hours. kl, r l'D 4

or
Thll~, for times ~css than 211,900
Eq. 1.7. At a radius of 1ft,
hours, we can use
P1vf
=
P,
.-141.2~
kh
l 0.000527kl
cf>1Lc,r~

p=p. + 70.6--qB1L £1. ( -948cf>1LC,r2 )


I kl, kl ( r (' ) 3
+In 1 (1.12)
r II' 4
(70.6)(20)(1.475)(0.72) Note !h~t during thi~ time period we find, by dif-
=3,000+ ferentlatlng Eq. 1.12,
(0.1)(150) a
~ = --~-=-Li-.
0 0 744 B
al ct>c,hr('
.Eil-(948)(0.23)(0.72)(1.5X 10-5)(1)2 ] Since the liquid-filled pore volume of the reservoir,
(0.1)(3) V p (cubic feet), is
= 3,()()() + (1()(»Ei(-0.007849) V =..?I,-I.
p "(' ,#"

=3,000+ 100 In [(1.781)(0.007849») then

=3,000+(100)(-4.27)
~ot -c,Vp.
--0.234qB (1.13)

Thus, during this time period, the rate of pressure


= 2,573 psi. ., -decline is inversely proportional to the liquid-filled
pore volume V p. This result leads to a form of well
At a radius of 10 ft, te~ting sometimes called reservoir limits testing,
which seeks to determine reservoir size from the rate
p = 3,000 + 100 of pressure decline in a wellbore with time.

1 -(948)(0.23)(0.72)(1.5X 10-5)(10)2 ] Another form of Eq. 1.12 is useful for some ap-
.E, (0.1 )(3) plications. It i~volves replacing origi!,al !es.ervoir
.pressure, Pi' with average pressure, P, within the
= 3,000 + 100 E,( -0.7849) drainage volume of the well.

= 3,000 + (100)( -0.318) The volumetric average pressure within the


.drainage volume of the well can be found from
= 2,968 pSI. material balance. The pressure decrease (Pi -p)

In t!lis calculation, we find the value of the Ei resulting from removal of qB RB/D of fluid for t
fll11rt ion from Tablc 1.1, Note, a~ indicated in tIle !lours [a total volume removed of 5.615 qB (1124) cu
tablc, that it is a negative quantity. ft] is
At a radius of 100 ft, -~V 5.615 qB(1124)
Pi-P= -= 2
- 3 000 I 00 c, V c, ( 7rr (' I,cf>)
p-, +
.£)-(948)(0.23)(0.72)(1.5XIO-5)(IOO)2
t. (0.1)(3)
] =~~~~j~.
cf>c,hr('
(1.14)

= 3,000 + 100 Ei( -78.49) Substituting in Eq. 1.12,

0.0744 qBt 0.0744 qBt


= 3,()()() psi. Pwf=P+ ..1.- 1.-2
h
4>c, r~
-..L- 1.-2
ct>c,h r~
Ilcrc wc notc tllat for an argul1lcnt of 7R.49, tile Ei
function is essentially zero. qBp. l ( r (' ) 3
-141.2-ln ---, J
P~l"ldosteady-State Solution. We now discuss the kh r w 4
next ~olution to the radial diffusivity equation that or
we will use extensively
analysi~. Actually,
in this introduction
this solution
to well test
(the pseudosteady-
~tate~olution)isnotnew.ltissimplyalimitingform
P-P
w
B
1=141.2~ln(~)--.
kh
~
rH,
3
4
] (1.15)
I

www.petroman.ir
FLUID FLOW IN POROUS MEDIA 7

Eqs. 1.12 and 1.15 become more useful in practice if formation volume factor is 1.5RI3/STB.
they include a skin factor to account for the fact that 1. Estimate the productivity index for the tl:~tl:d
most wells are either damaged or stimulated. For well.
example, in Eq. 1.15, 2. Estimate formation permeability from thl:~1:
B ~ r 3 ] data. ""

P-Pwj=141.2~111(-!.)-

kh rw 4
-+(Ap)

S ,

perml:ability to oil of 50 md. Does this imply thatlh~


3. Corl: data from thl: wc:lImdll:ate an efll:l:tlVC:

well is either damaged or stimulated? What i~ the


-q
B
IJ. re
I 31 apparent skin fal:tor?
P-Pwj=141.2-ln(-)--+s, (1.16) Solution.
kh rw 4 . d '"
.I. To estimate pro Ul:tlVIlY Index, we use E
'q.
and 1.19:

-qBIJ. [0.000527 kt q 100


P;-Pwj-141.2- kh 2
cPlJ.c,re J= P-Pwj = (2,000-1,500)

=0.2 STB/psi-D.
+In(~)- ~ +s ] (1.17) 2. We do not have sufficient information to
rw 4 estimate formation permc:ability; we can I:all:tllatc:
Further, we can define an average permeability, kJ' average permeability, kJ' only, which is not
such that necessarily a good approximation of formation
-qBIJ. ~ re 3J permeability,k.FromEq.I.19,
P-Pwj= 141.2kh
J
In( -,:-) -4
w
141.2 JBIJ.[ ln (
r
-~4 ~) I
w
kJ=
-qBIJ.I;
-h
( re ) 3
] I 000 )-0.75]
-141.2~~n ;: -4 +s , (141.2)(0.2)(1.5)(0.5)[ln( iis

from which, = 10

J/[ ln(~)-~+s ] (1.18) =16md. -


kJ=k [ln(~)-~
rw 4 rw 4
This average permeability, kJ, proves to have 3.. Core.data frequen.t~yprovide a better esti.n!~te
considerable value in well test analysis, as we shall see of formation permeability than do permeabilities
later. Note that for a damaged well, the average derived from the productivity index, particularly for
permeability kJ is lower than the true, bulk for- a well that is badly damaged. Since cores indicate a
malion permeability k; in fact, these quantities are permeability of 50 md, we conclude that this well is
equal only when the skin factor s is zero. Since we damaged. Eq. 1.18 provides a method for estimating
sometimes estimate the permeability of a well from the skin factor s:
productivity-index (PI) measurements, and since the k r 3
i productivity index J (STB/D/psi), of an oil well is s = (k -1)[ In( -!. ) -4]
defined as J rw
-.
q kJII
1=_=,...(1.19)
P-Pwj 141.2BIJ.[ln(~)-~] (50
= 16-1 )[(Iniis1000 ) -0.75 J
rw 4
= 16.
.Ihis method does not necessarily provide a good
estimate of formation permeability, k. Thus, there is flow Equations for Generalized Reservoir Geometry
! a need for a more complele means of characterizing a Eq. 1.16 is limited to a well centered in a cirl:ular
I producing well than exclusive use of PI information. drainage area. A similar equationS models pseudo-
steady-state flow in more general reservoir shapes:
.qBIJ. [ 1 ( IO.06A ) 3
Examplel.2-AnalystsofWell P-P,vj=141.2kh 2/n C 2 --+s, ]
From PI Test Arw 4
Prublem. A well produces JOO STB/D oil at a (1.20)
measured flowing bottomhole pressure (BHP) of
.were h
1,500 pSI. A recent pressure survey showed that
2 000 ..
L d .= . . A d .
average reservoir
..A pressure IS, pSI. ogs In Icate C = Shralnage area sq fI " and
ape fac tor 'f or SpeCI
fiICd ralnage-area .
a .net sand thickness of 10 ft. The well drains an area
th d . d .IUS, re'
f I 000 ft th b h I sh ape and weIII ocat Ion, d Imenslon Iess. . ..
WI ralnage ra 0, ; e ore 0 e
radius is 0.25 ft. Fluid samples indicate that, at Values of C A are given in Table 1.2; further ex-
current reservoir pressure, oil viscosity is 0.5 cp and planation of the source of these CA values is given in

L _!
www.petroman.ir ~
8 WELL
TESTING

TRANSl:NT
REGION

PWI Pwl
PSElroST[,J)Y-STAT[
REGION
~TEAOY-STATE
REGION

"l. log t t

Fig. 1.2-Flow regions on semilogarithmic paper. Fig. 1.3-Flow regions on Cartesian.coordinate graph.

Chap. 2. ...
Productivity index, J, can be expressed for general pseudosteady-state region, the reservoir IS modeled
drainage-area geometry as by Eq. 1.20 in the general case or Eqs: 1.15 a~d I: 12
0.00708 kh for the special case of a well cente~ed In a cyll.ndrlc~1
J= ~ =. reservoir. Eq. 1.12 shows the linear relationship
P-Pllf
Bp.
I! (
10.06 A -~ +s l
21n C r 2
) 4
between Pwf and I durin~ p~eudostea~y-state. Th~s
linear relationship also exists In generalized reservoir
..1 II' .
(1.21) geometries.
At times between the end of the transient region
Other numerical constants tab~lated in Table .1.2 and the beginning of the pseudosteady-state region,
allow us to calculate (I) the maximum elapsed time this is a transition region, sometimes called the late-
during which a re~ervoir is infinite acting (~o that.the transient region, as in Figs. 1.2 and 1.3. No simple
Ei-function solution can be used); (2) th~ time equation is available to predict the relationship
required for the p~eudosteady-sta~ solution to between BHP and time in this region. This region is
predict pressure drawdown within IOJoaccuracy; ~nd small (or, for practical purposes nonexistent) for a
(3) time required for the pseudosteady-state solution well centered in a circular, square, or hexagonal
to be exact. ..drainage area, as Table 1.2 indicates. However, f?r a
For a given reservoir geometry, the maximum time well off-center in its drainage area, the late-transient
a reservoir is infinite acting can be deter!11~nedusing region can span a significant time region, as Table
the entry in the column "Use Infinite-System 1.2 also indicates.
Solution With Less Than IOJoError for IDA < ." Note that the determination of when the transient
Since IDA =0.000264 kllf/1p.c/A, this means that the region ends or when the pseudosteady-state region
time in hours is calculated from begins is somewhat subjective. For~ example, tl~e
f/1p.c
/A IDA limits on applicability of Eqs. 1.7 and 1.12 (st~ted ~n
1< .the text earlier) are not exactly the same as given In
0.(xx)264 k Table 1.2 -but the difference is slight. Other
Time required for the pseudosteady-state equation authors I consider the deviation from Eq. I~ to be
to he accurate wit hin 1"/0can be found from the entry sufficient for I> 379 f/1p.c/r~ I k that a late-transient
in the column headed "Less Than IOJoError for region exists even for a well centered in a cylindrical
I f)..t >" and the relationship reservoir between this lower limit and an upper limit
q.IC AI of 1,1.l6 f/1/tc/r;lk .These apparently contradictory
I > --~ _/__-1J~!_-.opinions are nothing more than different judgments
0.()()()264 k about when the slightly approximate solutions, Eqs.
Finally, time required for the pseudosteady-state 1.7 and 1.12, can be considered to be identical to the
equation to be exact is found from the entry in the exact solution, Eq. 1.6.
coltlmn "Exact for If).t > ." These concepts are illustrated in Example 1.3.
AI this point, il is Ilelpful to depict graphically Ihe
Ilow regimes that occur in different lime range~.
rigs. 1.2 and 1.3 show BIfP, !'1I:f: in a w~llllowing al Exa/71ple 1.3 -Flow Analysis in
con~l~nl r.ale, pl<;,tled as a function of time on both Generalized Reservoir Geometry
logarIthmIc and linear scales. ..
In the transient region, the reservoir is infinite Problem. I. For each <;,fth.e following rese.rvolr geo-
acting andis'a
that 1~II:f is modeled by Eq. 1.11
linear fllnction which
of , log I. implies
In the melries, calculate
reservoir is infinite the tIme (b)
acting; In hours for whIch (a)state
the pseudosteady the

www.petroman.ir .
I
FLUID FLOW IN POROUS MEDIA 9

TABLE1.2-SHAPEFACTORS
FORVARIOUS
SINGLE.WELL
DRAINAGE
AREASfo
t Use Infinite System

( 2.2458)
0.51n -Exact
Less Than
1%'DA
Error
Solution With Less
Than
In Bounded Reservoirs CA In CA CA for IDA> for > for 1'DA
% Error
<
-~ (:) 31.62 3.4538 1.3224 0.1 O.~ 0.10

() 31.6 3.4532 -1.3220 0.1 0.06 0.10

6 27.6 3.3178 -1.2544 0.2 0.07 0.09

/-:7 27.1 3.2995 -1.2452 0.2 0.07 0.09


L!~
'" 21.9 3.0865 -1.1387 0.4 0.12 0.08
I/){~
.
~{§ ]. 0.098 -2.3227 1.5659 0.9 0.60 0.015

c:J 30.8828 3.4302 -1.3106 0.1 0.05 0.09

ffi 12.9851 2.5638 -0.8774 0.7 0.25 0.03

rn 4.5132 1.5070 -0.3490 0.6 0.30 0.025

m 3.3351 1.2045 -0.1977 0.7 0.25 0.01.

,.
~ I 21.8369 3.0836 -1.1373 0.3 0.15 0.025.
l

E=I=~' 10.8374 2.3830 -0.7870 0.4 0.15 0.025


Z

E:I~j, 4.5141 1.5072 -0.3491 1.5 0.50 0.06


2

E=:I~:~31 2.0769 0.7309 0:0391 1.7 0.50 0.02


2

m. 3.1573 1.1497 -0.1703 0.4 0.15 0.005


2

-
www.petroman.ir
10 ~ ~ WELL TESTING

TABLE 1.2 -SHAPE FACTORS FOR VARIOUS SINGLE.WELL DRAINAGE AREAS1o

Use Infinite System


( 2.2458 ) Less Than Solution With Less
0.51n -Exact 1 % Errbr Than 1 % Error
In Bounded Reservoirs CA In CA CA for 'DA > for tOA> for tOA <

EHB 1 0.5813 -0.5425 0.6758 2.0 0.60 0.02

EEB31 0.1109 -2.1991 1.5041 3.0 0.60 0.005

L. .~I 5.3790 1.6825 -0.4367 0.8 0.30 0.01

E- t- 31 2.6896 0.9894 -0.0902 0.8 0.30 0.01

E=I =~I 0.2318 -1.4619 1.1355 4.0 2.00 0.03

Eo 19, 0.1155 -2.1585 1.4638 4.0 2.00 0.01

C- .~ I 2.3606 0.6589 -0.0249 1.0 0.40 0.025

~ -
In vertic~lIy _fractured reservoirs: use (r~/L/)2 in place of A/r~ lor ~r_~I?:~r~~_sy~~!~~

[-oJ"l x//x"
I L:J= 2.6541 0.9761 -0.0635 0.175 0.06 cannot use

r-'Ojl
I L.:J 2.0346 0.7104 0.0493 0.175 0.09 cannot use

r-~l
1L:J 1.9686 0.6924 0.0583 0.175 0.09 cannot use

a .
10r~l 1.6620 0.5080 0.1505 0.175 0.09 cannot use

r~l
1 L:::J 1.3127 0.2721 0.2685 0.175 0.09 cannot use

[=:!iJ
I[ ~ 0.7667 -0.2374 0.5232 0.175 0.09 cannot use

I
In water-drive reservoirs
(:) -19.1 2.95 -1.07 ---

In reservoirs of unknown production character

0 25.0 3.22 -1.20 ---

www.petroman.ir
www.petroman.ir
c
:,
~' "r""'r~~C
0
r ('1
',ML;
"""", ,.'j.~
.,-, --

q
I

q
r.-
1- '- -AREA = Awb (ft2)

Z
;.

q ~Pw Q.sf ~R W

Fig. 1,4-Schematic of wellbore with moving liquid/gas Fig, 1,5-Sch~matic of wellbore containing slngle"phase
interface. liquid or gas.

d(p", -p,) p g dz Sub~titutiI1g,


= (1.24)
dl 144 gc dl ~d = -q; B IL X 0OOO264kdn
' _2 ~
TI
.IU~, dl 0.00708 kh cPlLC,rw dl D

(24)(1"~41 ~: A , ~'-~~ 0.0373q;B ~


5.615 of "" dl =- L_2 (1.31)
p <l>lLc,hr M' dID

=(q.~f-q)B. (1.25) .Thus,


" qif=q- ~:~~~?~, (1.32)
J}efillc a wellbore storage constant, Cs: s ct>c,',r w dID

144 A"",, gc If we define a dimensionless wellbore storage con-


('
~=--- 5.615p g (1.26)
stant, C so, as
Then, CsD~0.894CslcPclhr~, (1.33)
q = q +~d(P,,'-P,). (1.27) then
~f B dl
..~, qf =q.
lq --C
.~
v
dt-'
n
D
-."""'"
I .
(
I 34
. )

I:or zero or \Inchanging ~\Irface pressure, p, (a major .q; dr D


and not nece~~arilyvalid a~~umption), For c0l1~tant-rate production (q(/) = q;], Eq. 1.34
r 24 C d becomes
q.~r=q+--!.-~. (1.28) qs' dPD
.q .B dl ~=I-CD-. s dl V )
( 135

To ullderstand the soluti.ol~ to now prob~ems that Eq. 1.35 is the inner boundary condit,ion for the
include wellbore storage, It ~~neces~aryto. Introdu~e problem of constant-rate flow of a slIghtly com-
dinlen~ionle~s variables, ~imllar to those dl~cus~cd In pressible liquid with wellbore storage. Note t~at, for
Appendix B. Let .qi ~e the su~face !ate at 1.=0 and small C.~Dor for small dpvldlD' qsflq= I (I.e:, the
introduce the deflnltlon~ of dImensionless tlmc and effect of well bore storage or sand face rate wIll be
dil1lel1~ionlessprcssure: negligible).
As a second example, consider a weJlbore (Fig. 1.5)
, O.{)()708kh (Pj -p".) that c0l1tains a single-phase fluid (liquid or gas) and
1',,= '- '--, (1.29) that is produced at some surface rate, q. If we.let
qjB,c V "" be the volume of wellbore open to formatIon
(b~rrel~) and c It'h be the compressibility of th~, fluid
O.(xx)264 kl (I 3() in the well bore (evaluated at wellbore cof,dlllons).
I" = -:;:-
' 'I" /~-
( :-;2-.
, '" the mass-balance components are (1) rate of nuid

r .l"j~lio",,-.- www.petroman.ir
.
www.petroman.ir
www.petroman.ir
-~"

FLUIDFLOWIN POROUSMEDIA Ib

ll1u~, ,I pl'\.'~~ur\.'trull~i\.'llt to r\.'ul:lI tll\.' b'luII,I,lri\.'~ 'II' a


t =r~/4 =948q,' ~/k te~tedre~ervoir).I"orexample,il'uwclli~,-"\.'lIt\.'r\.'Jill
11/ , 11 1"1'" acyJilldricaldraillageareaofraJius't.,tlll.'ll,~l.'ttillg

Stated another way, in time t, a pressure distur- r;='e' the time required for stabilizati,)II, t,\, i~
ballce reaches a distance r;, which we shall call radius found to be
of investigation, as given by the equation t s = 948 q,1'£'lr; / k. (1.4M)

r;=
( --.!!-_ ) Y2. (1.47) It is no coincidence that thi~ist~letimcat.wlli,-"II
948 q,1lC, pseudosteady-state now begins (I.e" the tlml.' at
The radius of investigation given by Eq. 1.47 also w~lich Eq. 1.12 b~comes an a~~~lr~l~approxi..n,ltiOiI
proves to be the distallcl.' a sigllificallt prl.'s~url.' 01 thc l'~,Ict ~~)IUIIOl~tu tIll.' dlllu~lvlty 1.'411,ItIOll).A
di~turbance is propagatl'd by produl..'tion or illjl'l..'tiUII \~ord ul C,I~I~IOII:I'l)r l)tl!l.'r l,lr:,.IIII,lgl.'-arl'.,1
~11,lpl.'~,
at a constant rate. For example, for the formal ion ~lllle to slabllize call be qUltl' c.lIllcrl'IlI, u~ IlIustr,It\.',1
wilh pressure distribulions shown in Fig. 1.7. ap- In Example 1.3.. ...,
plication of Eq. 1.47 yields Ihe following resulls. Useful ~s the radlus-of-lnvc~tl~alloll conccpl I~.~WI.'
must caullon the reader Ihat It IS no panacea. 1'lr~I,
, rj we nole that it is exaclly correct only for u
(hours) -.!!!L homogeneous, isolropic, cylindrical reservoir -
0,1 32 reservoir helerogeneilies will decrease the aCCllral..'Y l)1'
t~:g ~~ Eq. 1.47. Fllrlher, Eq. 1.47 is exal..'l onl~ r'll'
100.0 t,<XX> describing Ihe lime the maximum prCSSllre Jislur-
bance reaches radills r; following an inslanlallel)lls
Comparison of these resulls wilh the preSSllre bllrst of injection inlo or prodllclion from a well.
distributions plotted shows Ihat r; as calculated from Exacl lacalion of the radius or investigation becl)mes
Eq. 1.47 is near the point at which the drawdown in less well defined for continuous injeclion or
reservoir pressure caused by producing the well production at constant rare following a change in
becomesnegligible. rate. limitations kepI in mind, though, the radills-
We also use Eq. 1.47 to calculate the radius of of-investigation concept can serve us well.
investigation achieved at any time after any rate
change in a well. This is significant because the ..
distance a transient has moved into a formation is Example J ,4 -CalculatIon of RadIus
approximately the distance from the well at which of Investigation
formation properties are being investigated at a Problem. We wish to run a now test on an ex-
particular t.ime in a. well t,est.. ,ploratory well for sufficiently long to ensure thaI the
The radius of Investlgallon has several uses In well will drain a cylinder of more lhan I,OOO-ft
pres~ur~ transi~nt test analysi~ and design. A radius. Preliminary well and nllid data anallsis suf-
qualitative use IS to help explain the shape of a gests thaI k = 100 md, q, = 0.2, ", = 2 x 10 -psi -,
pressure build~p or pressure drawdown ~u~ve. For and I' = 0.5 cpo What length now test appears ad-
example, a buildup curve may have a dlfflcult-to- visable? Whal now rare do YOll suggest?
inlerpret shape or slope al earliest times when lhe ,. ..
radius of invesligalion is in the zone of altered Scllll8lCtn. The minimum. length fl,?w lest WOlII~
permeabilily, ks' nearest the wellbore. Or, more propagale a pressur~ tranSlell1 ,al?proxlmal~ly 2,()()(~It
commonly, a pressure buildup curve may change fro~l t~e well (twice I~e mmll~lum. radius 0" In-
shape al long times when the radius of invesligation vestlgatlon for safelY). Time required IS
reachesthe gencral vicinilY of a reservoir bollndary t = 948 q,J'c r? / k
(~uch as a sealing falllt) or some massive rescrvl)ir 1, /
helerogeneilY. (In practice, we find Ihal a (948)(0.2)(0.5)(2 x 10 -S )(2,000)2' .
heterogeneity or boundary inlluences pressure = 100
response in a well when the calculated radius of
invesligation is of the order of twice the dislance to = 75.8 hours.
the heterogeneity.)
The radius-of-investigation concept provides a In principle, any now rate would suffice -lime
guide for well tesl design. For example, we may wanl required to achieve a particular radius of in-
10 sample reservoir properties at least 500 ft from a vestigation is independent of now rate. In praclice,
rested well. How long a tesl shall be run? Six hours? we require a now rate sufficiently large that pressure
Twenty-four hours? We are not forced to guess -or change with time can be recorded with sufficient
to run a lest for an arbitrary length of time that could precision to be useful for analysis. What constitutes
be either too short or too long. Instead, we can use sufficient precision depends on the particular
the radius-of-investigation concepl to estimate Ihe pressure gauge used in the lest.
..
time required to test to the desired depth in the
formation.
The radius-of-investigation equation also provides
a means of estimating the length of time required to
I. 1 5 Pnnclp I eo f S uperposl ' ( Ion
'

At th!s point, th~ mos~ useful s.olution to the now


equation, the El-function solullon, appears to be
i
i

achieve "stabilized" flow (i.e., the time required for applicable only for describing the pressure

--- www.petroman.ir
16 WEll TESTING

Image Actual
Well Well
Well A
L L
q q
\

rAC rAB ,

No Flow
Well C Well B Boundary

Fig. 1.8-Multiple.well system in infinite reservoir. Fig. 1.9-Well near no.flow boundary Illustrating use of
imaging.

distribution in an infinite reservoir, caused by the produces; qB' Well B; and qc, Well C. Note that this
production of a single well in the reservoir, and. most equation includes a skin factor for Well A. but does
restrictive of all, production of the well at constant not include skin factors for Wells Band C. Because
rate beginning at time zero, In this section, we most wells have a nonzero skin factor and because we
demonstrate how application of the principle of are modeling pressure inside the zone of altered
superposition can remove some of these restrictions, permeability near Well A, we must include its skin
and we conclude with examination of an ap- factor. However, the presence of nonzero skin
proximation that greatly simplifies modeling a factors for Wells Band C affects pressure only inside
variable-rate well. their zones of altered permeability and has no in-
For our purposes, we state the principle of fluence on pressure at Well A if Well A is not within
superposition in the following way: The total the altered zone of either W~1I B or Well C.
pressure drop at any point in a reservoir is the sum of Using this method, we can treat any number of
the pressure drops at that point caused by flow in wells flowing at constant rate in an infinite-acting
each of the wells in the reservoir, The simplest reservoir. Thus, we can model so-called interference
illustration of this principle is the case of more than tests, which basically are designed to determine
one well in an infinite reservoir. As an example, reservoir properties from the observed response in
consider three wells, Wells A, B, and C, that start to one well (such as Well A) to production from one or
produce at the same time from an infinite reservoir more other wells (such as Well B or Well C) in a
(Fig. 1.8), Application of the principle of super- reservoir. A relatively modern method of conducting
position shows that interference tests, called pulse testing, is based on
these ideas. 10
(Pi -P"1) lotal al WcllJ\ Our next application of the principle -4)f super-
I W II A position' is to simulate pressure behavior in bounded
= (p. I -p) d IICOC/\
c 'd h II ' F 19d .
reservoIrs. onsl er t e we In Ig, , a Istance, L ,
.
+ (Pi-P)dllcloWcIiB from a single no-flow boundary (such as a sealing
+ (Pi-P)dllctoWcIIC' fault). Mathematically, this problem is identical to
.' " the problem of a well a dist~nce 2L from an "image"
In t~rm~ of £, functIons and logarIthmic ap. well (i,e., a well that has the same production history
prl'Xlmatlons, a~ the actual well), The reason this two-well system
.f ) A simulates the behavior of a well near a boundary is
(p.I -P "tola I I W II
.11at a c 1\ I .. ' ant between t he two weII scan be
a IIne equi d 1st

qABlL In
l (
1,688r/>ILC,'M'A2 ) -A2S" I shown to be a no-flow boundary -i,e., along this line
= -70.6 ~ kt the pressure gradient is zero, which means that there
can be no flow. Thus, this is a simple two-well-in-an-
infinite-reservoir problem:
( -948r/>ILC"AU2 ) qBIL
(1n-k)
1,688r/>lLc,r~ ""-
-70.6~E;k'
Ir
kt PI.- p WJ
.r=-706- .kh kt )
-70.6
qcBIL .( -948<1>ILC"AC2)...(1.49)
---£, -706~£i
.kh ( -948 <l>p.ct(2L)2
kt ).
kl, kt.
.wllcrc qA rcfcr~ to the rate at which Well A ,., (1.50)

-~
www.petroman.ir
I:
FLUID FLOW IN POROUS MEDIA r' ~ 1/

(1 (Ap)1 = (Pj-PII } ") I = -70.6 Jtql8


---
~=~==::~~J==~= kh
~ 2
~ ~ .[In(~~~t~6')-2S"].
°L ql I-,,-tl
.t:ell l2 Startillg uttimc II' th\.' Il\.'W total rutc is q2. W\.' ill-
q t -..trodll\.'\.' a Wcll 2, produ~illg at rate (q2 -lIl) starlillg
at time II' so that the total rate after II is the
Well I required q2. Note that total elapsed time sin~e this
well started producing is (I -I I); note further that
L [~:=~~1~:::: = this well is still inside a zone of altered pcrmeahilily.

( - ) l'tllls, the contribution of Wcll 2 to drawdowll of


q 2 q I reservoir pressure is

Well 2 , = -70.6 jJ.(q2-QI)B


(AJ1)2 =(jl '-P II'1 } 2 kh

II
l
2
[ I I ,688 cPJtc/r;v -~. }
.In 1
k(/-/I)
Well 3 Similarly, the contribution of a third well is

(
~ 3 - q 2) (Ap)3=(Pj-Pwj)3=-70.6
jJ.(Q3-Q2)B
kh
Fig. t.tO-Produclion schedule for variable-ratewell. .f In t i ,688 cPjJ.C
/r~
1-~1.
l k(/-/2) J

..-Tllus, Ihc total druwdown for thc well with two


Here agaIn, note that whether the Image well lIas a challges in rate is
nonzero skin factor is immaterial. Its influence
outside its zone of altered permeability is in- Pj -Pwj= (Ap) 1+ (Ap) 2 + (Ap) 3
dep'ende~t of whether.this ~one exist~. -Jl.Qt 8 1,688 cPjJ.c
tr~.
Extensions of the Imaging tecllnlq'le also ~an hI.' --70.6-: lln( : ) -2'i1
used, for example, to model (I) pressure dislrib'ltion kh kl
for a well between two boundaries intersecting at
90°; (2) the pressure behavior of a well between two jJ.(Q2-ql)8
parallel boundaries; and (3) pressure behavior for -70.6--~-
wells in various locations completely surrounded by
no-flow boundaries in rectangular-shaped reservoirs.
This
studylast
by case has been
Matthews el studie?
al. II ISquite
one ofcompletely; the
the methods r
'tln /r~I' 1 -~
[ 1,688 cPjJ.c 1 ..
most frequently used to estimate average drainage- k(/-/t)
area pressure from pressure buildup tests. jJ.(Q3 -Q2)B
Our final and most important application of the -70.6 kl
superposition principle will be to model variable-rate '
producing wells. To illustrate this application,
consider the case (Fig. 1.10) in which a well produces
atrateQrfromtimeOtotime/l;at/l,therateis
[
.In
r 1,688 cPjJ.cr2 1
-k(---=-~ -~J' ) (1.51)
changed to Q2; and at time 12' the rate is changed to I 12

Q3' The problem that we wish to solve is this: At


some time I> 12' what is the pressure at the sandface Proceeding in a similar way, we can model an
of the well? To solve this problem, we will 'Ise a\:tual well with dozens of rate changes in its history;
superposition as before, but, in this case, each well we also can model the rate history for a wcll with a
that contributes to the total pressure drawdown will continuously changing rate (with a sequence of
be at the same position in the reservoir -the wells constant-rate periods at the average rate during the
simply will be "turned on" at different times. period) -but, in many such cases, this use of
The first contribution to a drawdown in reservoir s'lperposition yields a lengthy eq'lation, tedious to
pressure is by a well producing at rate ql starting at use in hand calculations. Note, however, that such an
I=; O. This well, in gcnl.'ral, will be inside a zonl.' of eq'lation is valid only if Eq. 1.11 is valid for the total
altered permeability; thus, its contribution to time elapsed since the well begal\ to flow at its initial
drawdown of reservoir pressure is rate -i.e., for time I, 'j must be ~ re.

www.petroman.ir -
-18 WELL TESTING

Example 1.5 -Use of Superposition p~eudoproducing lime:


Problem. A nowing well i~ compleled in a reservoir I (hours) =
lhal hasI the following properties. /'
.
p.B = 21:32
500RB/STB,
psla 24 cumulative
mostproduction
recentrate,from well, Np(STB)
Qla.,(STB/D) (I 52)

JL = 0.44 cp, Then, to model pressure behavior at any point in a


k -= 25 md, reservoir, we can use the simple equation
/1 = 43 ft. 2
c, = 18x 10-6psi-l,and .-= -70.6JLQlas1B Ei
4> = 0.16. P, P kh -9484>JLC,r
kIp ( .(1.53) )
What will the pressure drop be in a shut-in well 500 ft Two questions arise logically at this point: (I) What
from the flowing well when the nowing well has been is the basis for this approximation? (2) Under what
shut in for I day following a flow period of 5 days al conditions is it applicable?
300 STB/D? The ba~is for the approximation is not rigorou~,
Solution. We must superimpose the contributions of but intuitive, and is founded on two criteria: (I) If we
two wells because of the rate change: use a single rate in the approximation, the clear

p. -p= -~~
[q Ei ( -948 4>JLc,r2 choice is the most recent rate; such a rate, maintained
I kh I kl ) for any significant
distribution nearest period, determines
the wellbore the pressure
and approximately

out lo the radius of investigation achieved with that


2 rate. (2) Given the single rate to use, intuition
+ (q -q ) Ei l ::::-948 4>JLc,r
I] .Sllggc~ls
2 I k(I-II) such that that
the we choose of an
product the effective
rate and production
the productionti.me

Now ' , lime way,


lhi~ re~ults in lhe correcl
material cumulative
balances production. ac-
will be maintained In
948 4>JLCr
'= [(948)(0.16)(0.44)(1.8XI0-S) I
curatey.. "
k Bul when IS the approximation adequate? I f we
maintain a most-recent rate for too brief a time
.(500)2J/25 = 12.01. interval, previous rates will playa more important
role in determining the pressure distribution in a
Then ' tested
First, reservoir. We can
if the most offerrate
recent twoishelpful guidelines.
maintained suf-
p. -p = -(70.6)(0.44)( 1.32) ficiently long for the radius of investigation achieved
I (25)(43) at this rate to reach the drainage radius of the tested

well, then Horner's approximation is always suf-


ficiently accurate. This
findrule
that,is for
quitea new
conservative,
.[ (300) Ei [ -12.01 J howevcr. Second, we well that,
(6)(24) undergoes a series of rather rapid rate changes, it is
't usually sufficient to establish the last constant rate
" for at lea~t twice as long as the previous rate. When
+ (0 -300) Ei [ ~~ Il IlIcrc is any doubl about whet her these guidelines are
(I )(24) J satisfied, the safe approach is to use superpo.\1tion tol
model the production history of the well.
I. = 11.44[-Ei(-0.0834)+Ei(-0.5)j

= 11.44 (1.989 -0.560) Exal11ple 1.6 -Application of


= 16.35 p~i. Horller's Approximation

1.6 Horner , s Approximation


..ProlJlem. for a ~hortFollowing completion,
time and then shut in aforwell is produced
a buildup test.
In 1951, Hornerl2 reporled an approximalion that Theproduclionhistorywasasfollows.
can be used in many cases to avoid the u~e or
superposition in modeling the produclion history or a Production
Time TotalProduction
variable-rate well. With this approximation, we can ~~~~ -(STB)
replace the sequence of Ei functions, renecting rate :~ S~
changes, with a single Ei function that contains a 26 46
single producing time and a single producing rate. 72 68
The single rare is the most recent nonzero rate at
which the well was produced; we call this rate qla~t I. Calculate the pseudoproducing time, tp'
for now. The single producing time i~ found by 2. Is Horner's approximation adequate for this
dividing cumulalive production from the well by the case? If not, how should the production history for
most recent rate; we call this producing lime Ip. or thi~ well be simulated?

www.petroman.ir ~
r
!
FLUID FLOW IN POROUS MEDIA 19

,I
" Suilltiun. Ilow IOllg wolilu il lakt: for tilt: wt:llto ~Iaoili,t: al lIlt:
'
1. new rate?
I 68 STB 24 h 1.7 Suppose the well descr~bed in EX4:rCfSe 1.2
l/ = x -~~ = 22.7 STB/D. flowed at a rate of 700 STOll> lor 10 days. Prep..rt:..
I"~I 72 hours day plot of pressure vs. logarithm of radius for tllis

TIII.'II, situation
a r..te of on
350the same graph
STOll>. Is theasradiu~
the plot
of devl.'lopl.'d for
invt:~tigatk)11

I = ~4(cumulilli~~~CI~"~ ~:~~) calcillat\.'u from Eq. 1.47 Otfft:d\.'d by I.'llallgl: ill Ilo~'
II ql..,..STUll) ralt:'! I)ot:~ tilt: exlrapolOtliol1 of tilt: ~Iraiglll lilll:
referred to in Exer~ise 1.5~llangl."! Wllal i~ tilt: t:fft:l:t
(24)( 166) of illcrt:a~t:d ral~'!
= 227 = 176 hours. I.M Writ4: all equation simil..r to ~q. 1.49 fur tIll.'
( .) cast: in which Wells, A, 0, and C bcglll to prod lice ,It
2. In this case, uiffl.'rent times from onc ..nolhl:r. What do you
~/I asl = 72
-= 2.77> 2. ..ssumc
when youabout
write the
tllis location
equation?of rest:rvoir boulldarit:~
A//lI:XI-lo-lasl 26 ...1.9 (a) Suppose a well is250 ft uuewcst of an or 111-
Thus, Horner's approximation IS probably south trending fault. I'rom pressurc transit:nt tcst~,
ilc.I~ljllalefor this case. It should not be necessaryto the skin factor, s, of this wcll has bcen founu 10 Ot:
II~I.'~llperposition, whi~h is required when Hofller'~ 5.0. Suppose further that the wt:11ha~ been tlo,,'illg
ilpproximation is not adequate. for 8 days at 350 BID; reservoir and well properlit:s
I L'" .'. " are those given in Exercise 1.2. Calculate pres~urt:..t '
I'.xercises the nowing well.
1.1 Compare values of Ei (-x) and In (I. 781x) (b) Suppose there is a shut-in well 500 ft due north
for the following values of x: 0.01, 0.02, 0.1, and I. of the producing well. Calculate the pressure at the
Wllal do you conclude about the accuracy of the shut-in well at the end of8 days.
Ill~arilhmic approximation? About its range of 1.10 A reservoir has the following propcrtics.
ilpplicability? .
1.2 A well has nowed for 10 days at a rate of 350 Pi = 2,500 psla,
Sill/D. Rock and nuid properties include B= 1.13 B = 1.32 RB/STB,
RIl/STO; Pi = 3,000 psia; II.= 0.5 cp; k = 25 md II. = 0.44 cp,
(ulliform to wellbore-i.e., s=O); h=50 ft; k = 25md,
'> xl() -S P " '
I-I. A.=() 16' and r =().333 ft. h = 43ft,
'1--
"
'" 'i' ., II' -6 '-1 d
l'al..:uIOttepressures at radii of 0.333, I, 10, 100, c{ = 18 x 10 pSI, an
l.tKX),and 3, 160 ft, and plot the results as pressure cP = O.16.
\). Ih~ logarithm of radius. What minimum drainage In this reservoir, a well is opened 10 now at 250
i JilJiu~have you assumed in this calculation? STUll) for I day. The s~conu Jay its now is ill-
1.3 I:or the well described in Exercise 1.2, plot creased to 450 OlD and thetlliru to 5CX) OlD. What is
pJ~)~llr~in the well bore vs. logarithm of time atlimcs the pressure in a shut-in well 66() ft away after the
III' 0.1, I, and 10 days. What minimum drainage third day?
I ril,lill~ have you assumed in this calculation? I. I I In Example 1.6, Application of Hoflll:r's
! 1.4 Calculate (a) elapsed time required for t~~ Ei- Approximation, what innuence did the 12-hour ~IUt-
I I'lIllction solution to be valid for the conditions in time have on the calculation? How would the
I c.I~~..:rib~d in Exercise 1.2; (b) time required for the innuence of this shut-in period have changed had tile
I hl~arilhmic approximation of the Ei function to sllut-in period been 120 hours? How do you suggest
ilpply for calculations at the wellbore; and (c) time tllat the calculation procedllre be modified to take
Jl.'ljuircdfor the logarithmic approximation to apply inlo account long shut-in periods prior to producing
Illr I.'alculalions at a radius of I,O{}O ft. Is the at the final rate?
Illgarilhmic approximation valid by the time the Ei 1.12 Consider a well and formation with the
fullction ilself is a valid solution to the now equation following properties.
illlhl: wellboreJ At a ra~!us of .I.O{}O.ft? ..B = 1.0 RB/STB
1.5 Estimate the radius of Investlg~tlon ac.hlev~d = I 0c '
illll.'r 10 days now time for .the re.servolr ~escflbed In ~ = 10 ftP,
I:x~r..:ise1.2. Compare. this estlm~te w~th the ~x- k = 2S md,
I IJilpolalion to 3,000 pSI of the straight hne passIng = 0 2
r tilruligh radii of 0.333 and 100 ft on the plot of cP -3' ~ .
prc)~urevs. logarithm of radius. Pi -, P~t"-1
On this plot. how far into the formation has a C{ :: ~O x ~O PSI.
)jgllifil.'ant pressure disturbance been propagated? s:= ';~
What is the size of the pressure disturbance at the r w -I. t.
radiu~ of investigation calculated from Eq. 1.47? The well produced 100 STB/D for 3.0 days, was
1.6 If the drainage radius of the well described in shut-in for the next 1.0 day, produced 150 STB/D for
I:xcrl.'ise 1.2 were 3,160 ft, and if the now rate at the the next 2.0 days, produced 50 STB/D for the next
"~II ~llddenly was changed from 350 to 500 STB/D, 1.0 day, and produced 200 STB/D for next 2.0 days.
www.petroman.ir
I t w
f ~-
r (a) Calculate the pseudoproducing time, t p. Referencesi
Compare
b C I
this
I
with the actual total producing time. I I'L 1 a Ith ews, C ..an
S d R us~e,II D ..:
G P re.\'sllrl' BId
III lip all d
() a cu ate and plot the pressure dIstributIon In Flo"' Tests in Wells, Monograph Series, SPE, Dallas (1967) I.
the reservoir at the end of 9 days using Horner's 2. van Everdingen,A.F. and Hurst, W.: "The Application of the
al"proximation. Laplace Transformation 10 Flow Problems in Reservoirs,"
(c) On the same graph plot the pressure T~an,\'.,AIME(1949) .186,305-324. ...
...' ...3. Slider, H.C.: PractIcal Pt!troleliln ReservoIr Engmt!t!rmg
dIstributIon at the end of 9 days usIng superposItIon. Methods, Petroleum Publi~hing Co., Tulsa (1976) 70.
(d) What do you conclude about the adequacy of 4. Hawkins, M.F. Jr.: "A Nole on the Skin Efrect," Trans.,
Horner's approximation in this particular case? AIME (1956)207,356-357.
1.13 A well and reservoir have the following 5. Odeh, A.S.: "Pseudosteady-Slale Flow Equation and
I"roperties Productivity Index for a Well With Noncircular Drainage
.Arca," J. Pet. Tech.(Nov. 1978)1630-1632.
A = 17.42 X 10(' sq ft (40 acres), (,. A~arw;1I, R.G., AI-IIII~~;1iIlY,R., ;1l1dR;1mcy,II.J. .Ir.: "All
Ii> = 0 2 Invc~tigationof Wellhorc Storageand Skin Efrcct in Un~teady
I' , Liquid Flow -I. Analytical Treatment," Soc. Pt!t. Eng. J.
p. = Cp, (Scpt. 1970)279-290; Tran,\'., AIME, 249.
Ct = IOxI0-6psi-l, 7. Wattenbarger,R.A.andRamey,H.J.Jr.:"Anlnvestigation
k'= 100 md, of Wellhore Storage and Skin Erfect in' Un~leady l.iqllid
" = 10 ft Flow-II. Finite-Dirference Trcatment," Soc. Pt!t. Eng. J.
30 ' (Scpt.1970)291-297;Tran.\'.,AIME,249.
S = ., R. Kat7, D. L. f't 01.: lIandhook of Natural Gas Engineering,
r,., = 0.3ft,and Mc(iraw-llilllJfIOkCo.lnc.,NcwYork(1959)411.
B = 12 Rn/STIJ. 9. (ar~l;1w, II.S. and Jaegcr, J.C.: Conduction of "('Ot in
,lilllill\', ~ccondcd., Oxford althe ClarendonPress(1959)25R.
For each of the drainage areas in Table 1.2, deter- 10. Earlougher, R.C. Jr.: Advances in Wt!1I Test Analysis,
mine (a) the time (hours) up to which the reservoir is Monograph Serie~,SPE, Dallas (1977)5.
infinite-acting; (b) the time (hours) beyond which the II. Matthews, ~.S". Brons, F., and Hazebroek,~.: "A Method
, .for Determination of Average Pressure In a Bounded
pseu?ost~ady-state solutIon IS an adequate. .ap- Re~ervoir," Trans.,AIME(1954)20I,182-191.
proxlmatlon; (c) PI of the well; and (d) stabIlIzed 12. Horner, D.R.: "Pressure Build-Up in Wells," Proc., Third
production rate with 500-psi drawdown, World Pet. Cong., The Hague(1951)Sec.11,503-523.

www.petroman.ir
.Chapter 2
Pressure Buildup Tests ,
2.1 Introduction
This chapter discusses the most frequently used test in an infinite, homogeneous, isotropic reservoir
pressure transient test, the pressure buildup test. containing a slightly compressible, single-phase Iluid
Basically, the test is conducted by producing a well at with constant fluid properties. Any well bore damage
constant rate for some time, shutting the well in or stimulation is considered to be concentrated in a
i (usually at the surface), allowing the pressure to build skin of zero thickness at the wellbore; at the instant
up in the wellbore, and recording the pres~ure of ~hut-in, flow into the well bore I.'easestotally. No
': (usually downhole) in the well bore as a function of actual buildup test is modeled exactly by this
! time. From these data, it is frequently possible to idealized description, but the analysis methods
estimate formation permeability and current developed for this case prove useful for more realistic
drainage-area pressure, and to characterize damage situations if we recognize the effect of deviation from
or stimulation and reservoir heterogeneitie~ or ~ome of these a~~umptions on actual test behavior.
boundaries. Assume that (I) a well is prod~lcing from an in-
The analysis method discussed in this chapter is finite-acting re~ervoir (one in which no boundary
based largely on a plotting procedure suggc~ted by effects are felt during the entire flow and later shut-in
Horner.1 While this procedure is strictly correl.'t only period), (2) the formation and fluids have uniform
for infinite-acting reservoirs, these plots also can be properties, so that the Ei function (and, thus, its
interpreted correctly for finite reservoirs,2 so only logarithmic approximation) applies, and (3) that
this plotting method is emphasized. Another im- Horner's pseudoproducing time approximation is
portant analysis technique for buildup t~~t~, u~ing applil.'able. I f the well ha~ produced for a tim~ I p at
type curves, is discussed in Chap. 4. rate q before shut-in, and if we call time elapsed since
The chapter begins with a derivation of the Horner shut-in ~I, then, using superposition (Fig. 2.1), we
plotting technique and the equation for calculating find that following shut-in
skin factor. Differences in actual and idealized test 8 I 688 2
behavior then are discussed, followed by comments Pi-Pws= -70.6~[lnl-!--~!-!:!]-'-2sJ
on dealing with deviations from assumptions made in kh k (Ill + ~/)
developing the Horner plotting technique. We then
examine qualitatively the behavior of actual tests in ( -q) 8 I 688 cI> 2
,i common
develops inreservoir situations.analysis
detail a systematic The procedure
chapter next
for -70.6 kh 1J./ln( ~---~)k~1 -2 S] '

buildup tests: (I) effects and duration of afterflow which becomes


(continued production into the well bore following 8
sllrface shut-in),and
(3) well damage (2)stimulation,
determination of permeability,
(4) detc.:rnlination of PlY.\"
=Pi -70.6~ kit lnf (I II + ~/) I ~/l,

pressure level in the surrounding formatioll, and (5) or


reservoir limits tests. 8
Up to this point, the analysis procedure discussed Pw~'=Pi -162.6!!~ log! (Ip + ~I) I ~/]. ...(2.1)
is applicable only to single-phase flow of a slightly kh
compressible liquid. The chapter concludes with a The form of Eq. 2.1 suggests that shut-in BHP, p, t"
~
discussion of how the procedure can be modified to recorded during a pressure buildup test should plotW~s
analyze tests in gas wells and in wells with two or a straight-line function of log [(I +~/)/~/]. i
three phases flowing simultaneously. Further, the slope m of this straight line fhould be !'

.q81J.
1.1 The Ideal Buildup Test m= -162.6- .
.. ..
I n t h IS section we denve an equation d escn" b'Ing an kh
ideal pressure buildup test. By ideal test we mean a It is convenient to use the absolute value of In i~ test

www.petroman.ir
ri{- ' WELL TESTING

t ""'"'" Pi
Q. p "'"
ws
m
w
~ ~tp ..~l\l
a:: 1000 100 10 I

0 ~t =0 tp + ~t
TIME ~t

Fig. 2.1 -Rate history for ideal pressurebuildup test. Fig. 2.2-Plotting technique for pressurebuildup test.

analysis; accordingly, in this text we will use the -I 688 2:onvention


that ," is considered a positive number s= 1.151 ( ~~ ) + 1.15110g( ' </JJl.C('W
that In kill )and
-~
111-162.6
kh
(2.2) +1.15110g ( !2~ )
Iprhus,
(2.3)

formation permeability, k, can be determined It is conventional practice in the petroleum industryfrom


a buildup test by measuring the slope In. In to choose a fixed shut-in time, ~/, of I hour and the!dditio
if we extrapolat:e this straight line to infinite corresponding shut-in pressure, PI hr' to use in this
illlit-in time [i.e., (II!+~/)/~/=I) the pressure at equal ion (although al'Y shut-in time and the
:hislime will be the original formation pressurePi' corresponding pressure would work just as well). The
Conventional practice in the industry is to plot PII'.~ pressure, PI hr' must be on the straight line or its(S.
(I" +il/)/~1 (Fig. 2.2) on semilogarithmic paper extrapolation. We usually can assume further thatNith
values of (/p+~/)/~1 decreasing from left to log (/(,+~/)/lp is negligible. With these sim-'ight.
The slope "1 on such a plot is found by simply plificatlons,iubtracting
the pressures at any two points on thc -k
i!raight line tha~ are one cycle (i.e., a factor of 10) S=I.151 [ (Plhr-Pwj) -Iog( _2)+3.23 ].
Ipart on the semllog paper. In </JJl.C('
w
We also can dctermine skin factor s from the data (2.4)Ivailab
in the idealized pressure buildup test. At the. "
Ilslant a well is shut in, the nowing BliP, Pwj' i~ Not.e, again that. the .slope '~1 IS consIdered t~ be a
.2 pO.~ltlvenumber In thIs equation.
P 11./'
'=P.+70.6~
l (
ln 1,688 </>Jl.(('"') -2S / In summary, from the idcal buildup test, we can
kll kl" determine formation permeability (from the slope m
of the plotted te~t re~ults), original reservoir
B pre~sure,Pi' and skin factor, s, which is a measure of
=Pi + 162.6;{- damage or stimulation.

2 Example 2.1-Analysis of Ideal


.[log(!~~c:h)-0.869S] Pressure Buildup Test
kip Problem. A new oil well produced 500 STB/D for 3
day~; it then was shut in for a pressure buildup test,
1 688 </JC ,2 during which the data in Table 2.1 were recorded.
=Pi +I"llog(~-~) -0.869sJ. For this well, net sand thickness, h, is 22 ft; for-
kIp mation volume factor, Bo, is 1.3 RB/STB; porosity,
It ~hut-in time ~I in the buildup test, </J,is 0.2; total compressibility, c" is 20x 10-6; oil
-viscosity, Jl.o' is 1.0 cp; and well bore radius, , w' is 0.3
PII..~-Pi-I,'log[(/p+~/)/t1/]. ft. From these data, estimate formation per-~omhin
these equations and solving for the skin meability, k, initial reservoir pressure, Pi' and skinactor
S, we have factor, s.

www.petroman.ir
--!~,
';-~t'. -
PRESSURE BUILDUP TESTS 23

TABLE 2.1 -IDEAL PRESSURE TABLE 2.2 -BUILDUP TEST DATA


BUILDUP DATA FOR HORNER PLOT

Time After At ~ PWI


Shut.ln At PWI (hours) ~t (pslg)
(hours) (pslg) 2 31.0 1,194
0 1,150 4 19.0 1,823
2 1,194 8 10.0 1,850
4 1,823 16 5.5 1,876
8 1,850 24 4.0 1,890
16 1,876 48 2.5 1,910
24 1,890
48 1,910
EARLY MIDDLE LATE
ZOO() t TIMES TIMES TIMES

I P" P ws
'

.'"
,---
01
Vi
a.
VJ
~
"
""
<-P,.-
100
~ «> I I
og tp + At
l1t -
~t .-
Fig. 2.3 -Ideal pressure buildup test graph. Fig. 2.4-Actual buildup test graph.

Solution. To estimate permeability, original reservoir


pressure,and skin factor, we must plot shut-in BHP,
I
-og
( (0.2)(1.0)(2 48 ) 3 23
x 10-5)(0.3)2 + . 1
P ,vs. log (I + AI) / AI; measure the slope /11and
u;: Eq. 2.2 toPcalculate formation permeability, k; = 1.43.
extrapolate the curve to [( I P + AI) / AI] = I and read This means the well has a now restriction.
original reservoir pressure, Pi; and use Eq. 2.4 to .
calculatethe skin factor s. (See Fig. 2.3.) 2.3 Actual Buildup Tests
Producing time, I , is given to b~ 3 ~ays., or 72 Encouraged by the simplicity and ease of application
hours (in this case, Horner's approXimation ISe~-acI of the ideal buildup test theory, we may test an actual
bccau~ethe well was produced at con~tant rate since well and obtain a mo~t di~collraging re~ult: Instead
timezero). Thus, we develop Table 2.2. .of a single straight line for all times, we obtain a
We plot these data, and they fall along a straight curve with a complicated shape. To explain what
line suggested by ideal theory. The slope /11 of the went wrong, the radius-of-investigation concept is
straight line is 1,950 -1,850 = 100 psi (units are useful. Based on this concept, we logically can divide
actuallypsi/cycle). The formation permeability k is a bllildllp curve into three regions (Fig. 2.4): (I) an
q81J. (162.6)(500)(1.3)( 1.0) early-time region duri.ng wh~l:ha pressure transient is
k = 162.6 = =48md. Illoving tllrough the lormatlon n\:ar\:~t tile wcllbur\:;
fllh (100)(22) (2) a middle-time region during which the pressure
From extrapolation of the buildup curve to transient has moved away from the wellbore and into
1(/p+~/)/A/)=I,Pi=I,950psig.Theskinfactors the bulk formation; and (3) a late-time region, in
isfound from Eq. 2.4: which the radius of investigation has reached the
[ (Plhr -Pwj) ( k ) ] wel.I's ~rainage b~undaries. Let us examine each
s=I.151 -log 2 +3.23. regIon m more detatl.
m <P1lC/' w
Thevalue for P ws is PI hr on the ideal straight line at Early-Time Region
(/p+A/~/A/=(72+ 1)/1 =73; this value is PI hr = As we have noted, most wells have altered per-
1,764pslg. Thus, meability near the well bore. Until the pressure
[ (1,764-1,150) transient caused by shutting in the well for the
s= 1.151 (100) buildup test moves through this region of altered

~
www.petroman.ir
.,' """"'I'IU
-~-

permeability, there is no reason to expect a straight-


.j ,line slope that is related to formation permeability.
'( (We should note that the ideal buildup curve -i.e.,
'one with a single
is possible straight line
for a damaged wellover
onlyvirtually
when the alldamage
time -t Iq
i~ concentrated in a very thin skin at the sand face.») W
There i~ another complication at earliest times in a ~
pressure buildup test. Continued movement of fluid ~
into a well bore (afterflow, a form of wellbore l P ~t
storage) following the usual surface shut-in com-
pre~sesthe fluids (gas, oil, and water) in the wellbore.
Why should this affect the character of a buildup
curve at earliest times? Perhaps the clearest answer
lies in the observation that the idealized theory
J\
L1L -- 0
leading to the equation P"'.\,=Pj-'" log TIME ~
.(I" + ~I) / ~IJ explicHly assumed that, at ~I =0, flow. .
rate abruptly changed from q to zero. In practice, q Fig. 2.5-Rate history for actual pressurebuildup test.
declines toward zero but, at the instant of surface
shut-in, the downhole rate is, in fact, still q. (See Fig.
2.5.) Thu~, one of the assumptions we made in 2.4 Deviations From Assumptions
deriving the buildup equation is violated in the actllal in Ideal Test Theory
te~t, and another question arises. Does afterflow ever ..,..
diminish to such an extent that data obtained in a In suggest~ngthat tests 10glc~lly can .be divided Into
pre~sure buildup test can be analyzed as in the ideal early-,. mIddle-, and late-tIme re~lons, we ha~e
test? The answer is yes, fortunately, but the im- recognl~ed that several. assu~ptlons made .In
portant problem of finding the point at which af- developIng ~he theory of Ideal bulldu~ test ~ehavlor
terflow ceasesdistorting buildup data remains. This are n,ot valid for actu.al t~sts.. In this section, we
is the point at which the early-time region usually ~xam.me further. t.he Implications. of ~hree ove~-
end~, becau~e afterflow frequently lasts longer than Ideallze~ assumptlo~s: (I) the. In,finlte-reser~olr
the time required for a transient to move through the assumptIon; (2) the single-phase. lIquid as~umptlon;
altered zone near a well. We will deal with this and (3) the homogeneous reservoir assumption.
problem. more ~omplctelY when we. ~iscu~s a Infinite Reservoir Assumption
systematIc
te~t~. analysIs procedure for pressure buIldup '~I(t In developing the equation suggesting the Horner

.
III
M IC .
C e-' I '.Ime ,(,~ICt"
,
-.acting
...thc
plot, we assumed that the reservoir was infinite
during
.' both the production period preceding
buildup te~t alld the buildup test Itself.
.
Wh711the radll~~ of Illvc~tlgatlon ha~ moved hCYOlld l:rcqllcl1tly, the re~ervoir i~ at p~elldosteady-statc
thc Illflucl1ce 01 thc altcrcd 7,one near thc tc~ted well, hcforc ~hut-il1; if ~o, neither the Ei-function solution
and when arterflow has ceased di~torting the pre~~ure nor its logarithmic approximation should be used to
buildup test data, we usually observc the idcal describe the pressure drawdown caused by the
straight lil1e ~hose slope is related to formation producing well:
permeability.' (This straight line ordinarily will B
conlilluc until tIle radius of il1vestigati0l1
.. rcachc~ OI1C (p.I -p \I f ) pro d "c II ~ -70.6~~
or more reservoir . b d I
OUI1 aries, massIve letero- kl1"
gencities, or a fluid/fluid contact.) 2
Sy~tematic analysis or a pressure buildup test using .f In [ 1,688 c/>ILc,r"'
1 -2s1.
the Horner method of plotting IIl\l'S vs. log {k(l" + ~I) j
<,f
I' + ,~I) / ~f] requ!res th~t we recognize thi~ middl~- II1~tead, if the well is centered in a cylindrical
tllnc llnc
with falseal1d tlInt, Inlil1cs
straigllt partin
Icular, wc do al1d
the carly- not con fu~c It
late-til1lc rc~crvoir '

regions. As we have seen, detcrmination of reservoir (p. -p) = 141 2~


pcrmeability and skin factor depends on recognition ,..f prodwell. kh
or the middle-time line; estimation of average
drail1age-area pressure ror a well in a developed field
al~o requires that thi~ line be derined.
. 1 0.000527 k (I p + ~I) + In
c/>/lc,r; ---rlt.
( ~ ) -~4.] .I

Thus, we must conclude that in principle, the Horner


1.l1tl'-llml' Itl'~lctn plot is incorrect whcll the reservoir is not infinite
Given enough time, the radiu~ or invc~tigation acling during the flow pcriod preceding the buildup
eVcl\tl;ially will reach the drainage boundaries of a , test. Boundaries become important as rj -r f'.
wrIt. (In this late-time region pre~~ure bchavior is The problem is compounded when rj -r (' arter
innllcnccd hy boul1dary col1figuration, il1tcrfcrcl1cc shut-in. Then, too, the Horner plot is incorrect in
from nearby wel15, signiricant reservoir hetero- pril1ciple.
gcncities, al1d nuid/fluid contacts.) This difficulty is resolved in different ways by

~..~
I ";;,,,;;
www.petroman.ir
- -
PRESSURE BUILDUP TESTS 25

MTR
I t (I a 2) t ~ I -

Pws Pws ETR-1M'TiR


I-!:~f (I)
Pwf(2) Pwf(182)

tp + At tp + At
-+-- log -15 -+-- log ~f

Fig. 2.6 -Buildup test with no alterllow: (1) without Fig. 2.1- Buildup test with formation damage: (1) williout
wellbore damage arid (2) witll wellbore dafllage. alterllow and (2) willi altcr Iluw.

different analysts, In this text, we will use a method Single-Phase l.iquid Assumption

supported by the research of Cobb and Smilh,2 We The assumption that a petroleum reservoir contains
will use the Horner plot for all tests (even when the only a single-phase liquid must be modified. Even
reservoir has reached pseudosteady-state during the reservoirs in which only oil flows contain an im-
production period preceding the test) for the mobile water saturation; many also contain an

following reasons. immobile gas saturation. Also, in many cases,


I, This method of plotting is correct theoretically compressibility of the formation cannot be ignored.
for an infinite-acting reservoir (i,e., one for which, al These factors are taken into account if we use total

time tp.. + At. r; <r e). compressibility, Ct' in solutions lo flow equations:
2, The Horner plot offers a convenient means of
extrapolating to At-~ not found in some other CI =coSo +cwSw +CgSg +cf' .,..,..,... (1.4)

plots; the pressure al this shut-in lime is a useful Even in single-phase flow, when S ~O, evaluation of
checkpoint for the test analyst. oil compressibility, "0' and wat~r compressibility,

3. For finite-acting reservoirs, formation per- "w' are somewhat complicated:

meability can be determined accurately from the -I d8 8 dR


slope of the Horner plot at even greater shut-in times Co = --!!- + ~ -2, , (2.5)

than from a plotting method developed specifically 80 dp 80 dp

for reservoirs at pseudosteady state at shut-in.2 The

, curve will begin to deviate from the ideal slope bef?re -~ ~ ~ ~ (2 6)


'j during shut-in reaches reservoir boundaries. C II' -8. dp + 8 dp.' , ':,.' .
Th " 'b '
However, the middle-time region still can be iden- l I h II .W . . .1..1;
t f' d
' t f I I t '. ese compressl Iity re at Ions IpS a ow us to mou'-'. ...
I Ie ,excep or ong ear y- Ime regIons, , I I fl f 'I L .. h.
Other analysis methods for finite-acting reservoirs sl.ng e-p 1ase, ow.o 01, ater sections I~ l IS C I 1arter
d ' d b M II . D and H tchl . nson 3 discuss modIfications to the flow equatIons required
are Iscusse y I er, yes, u , .
(MDH) and Slider.4 Many analysts use the data- for (1) single-phase gas flow and ~2) slmultane.ous
plotting method suggested by MDH because it is flow of ~wo or three phases. Ap~endlx D sum~~r!~es

simpler than the Horner method. Consider a buildup correlatIons ~seful for.calculatm~ compre.sslbliltles
t t ' th ' ddl t ' . d ' b d b Eq 2 1 .and other fluId propertIes needed In analysIs of well
es WI a ml e- Ime region escfl e y ...
tests.
p ws = p; -m log (t + AI) / AI
p Ilomogcneou~ Re~ervoir A~~"mption

=p;-mlog (tp+At)+mlogAt. No reservoir is homogeneous, yet solutions to the.


now eqilatim1S are valid only for homog~llcous :
: If t p )10AI during the range of shut-in time values re~ervoir~, The ~olutions prove to be adequate for ;
f examined, then log (/p +A/) =log tp = constant, and most real reservoirs, particularly eariy in time while:

\ p = constant + m log At coQditions nearest the tested well dominate test i


ws .behavior, Rate of pressure change is dominated by !

This leads to the plotting technique suggested by average rock and fluid properties. When massive I
MDH: p ys. log At. It has the same slope m as the heterogeneities are encountered (particularly in a
Horner ;iot (in the time range of applicability)., localized portion of the reservoir), the simple
Further insight into this plotting technique is solutions to flow equations lose accuracy. Examples
provided by Exercise 2.2. include changes of depositional environment, with

www.petroman.ir
-MTRILTR-

t E R p!,

Pws

w
-4- L TR -+

tp + L\t tp + At
-+- log ._-~t 0-40- log/-At

Fig. 2.8 -Buildup test in hydraulically fractured well, Fig. 2.9 -Boundary effects in pressure buildup test: (1) well
centered in drainage area and (2) well off.center
in drainage area,

resultant changes in permeability or thickness, and Fig. 2.6 illustrates the ideal buildup test, in which
some nuid/nuid contacts. The longer a test is run, the MTR spans almost the entire range of the plotted
the higher the probability that a significant data. Such a curve is possible for an undamaged well
het.eroge~eitr will be enc?mpassed within the radius (Curve I, with the level of Pwi' the nowing pressure
of InvestIgatIon and thus Influence the test. at shut-in, is shown for reference) and for a damaged
Modifications to the simple reservoir models have well with an altered zone concentrated at the
hccn developed for some important reservoir wellbore. This latter situation, shown in Curve 2, is
IIctcrogcncitic~. Still, in actllal lIeterogcncou~ tnuicated by a rapid ri~e in pressure from nowing
rc~crvoir~, tile te~t analy~t mu~1 be aware con~talltly pre~~ure at sllut-in to tile pressures along the MTR.
of tllc po~~ibility of an unknown or impropcrly Ncitllcr ca~c i~ ob~crved often in practice with a
modelcd IIctcrogcncity. Thc~e IIcterogcncitics makc ~urface ~hut-in bccau~c afternow usually distorts the
analysis of late-time data in tran~icnt test~ more early data that would fall on the straight line.
difficult -reservoirs are rarely uniform cylinders or Fig. 2.7 illustrates the pressure buildup test ob-
I'arallclcpipcds, and thc analy~is technique that is rained for a damaged well. Curve I would be ob-
I'a~cd on tllc~c a~~uml'tion~ for Ircatlncnt of lale- tained with a shut-in near the perforations
timc data can bc difficult to apply witllOllt am- (minimizing tile duration of afternow); Curve 2
biguity. would be obtained with the more conventional
What is the test analy~t to do with late-time data7 surface shut-in~' Note in this figure that the nowing
Opini{~n~ vary: One frequent approa~h i~ t? use BHP at shut-in, PIII(' is the same for either case, but
anaIY~I~ tccllnlque~ ~uggested by published simple that the afternow that appears with the ~urface shut-
m{,dcl~ -but to try to find other models tJlat also fit in (I) completely obscures information rcnecting
the observed data. One then chooses the most near-well conditions in the ETR and (2) delays the
probable reservoir description, and recognizes that beginning of the MTR) A further complication in-
thc analysis may be ah.~o/"tel_I'"'correct. troduced by afterno~ is that several apparent
Q I.. I) I . r L" Id " straight lines appear on the buildup curve. The
2...lIa
S .faflve Je lavlor 0 ..Ie I e~f~ qllestion ari~e~, how do we find Ihe straight line (the
We IIOWhave devcloped the background requircd to MTR line) whose ~Iope is related to formation
under~tand the qualitative bellavior of commonly permeability7 We will deal with this question shortly.
occurring pressure buildup curves. There is an im- Fig. 2.8 shows characteristic behavior of a buildup
portant reason for this examination of behavior. It test for a fractured well without afternow. For such a
provides a convenient means of introducing some well, the pressure builds up slowly at first; the MTR
factor~ that innuence the~e curve~ and that can develops only when the pressure transient has f'\1oved
(,h~cure il,lcrl,rclation unlc~~ Ihey are rccognized. In beyond the region innuenced by the fractureliin a
thc figllre~ that follow, the carly-, middlc-, and late- buildup test for a fractured well, there is a possioility
time regions are dcsignated by ETR, MTR, and LTR,. that boundary effects will appear before the .ETR has
rc~pcctivcly. In tl,ese curves, the most important ended (i.e., that there will be no MTR at all).)
region is the MTR. Interpretation of the te~t using Fig. 2.9 illustrates two different types of behavior
the ll{,rncr plot I" \I',tvs. log (f l' + d/) I d/] i~ usually in the LTR of a buildup test plot. Curve I illllstrates
iI'lpos~ible unlc~s the MTR can be recognized. middle- and late-time behavior for a well reasonably, j
~

~ -j
www.petroman.ir
buildup te~t alone i~ not ~uffil:ient to ilJdil:atl.' IIII.'
presence or absence of afterllow -it is merely a clue
that sometimes indicates presence of afterflow.
t A log-log graph of pressure change, p",s -Pili' in a
buildup test vs. shut-in time, L\t, is an even more
diagnostic indicator of the end of afterllow
Pws distortion. Fig. 1.6, based on solution~ to the Ilow
equal ions for I:onstant-rate produl:tion with wellborl.'
sl(lragl.' cJislorliolJ. d(."scrib(."sprl.'SSllr~ builcJllp Il.'sls,
as Wl: Ji~~u~!i in !iom(."Jl:tail in <"Ilap. 4. I.'or u~(."()f
thi~ figure for buildup test~, dimen~ionle~~ pre~~url.',
PD' is defined as
0.00708 kh(pws -P"'f)
PD = '. (2.7)
q8J1.
log tp! At Dimen~ionless time, I v' and dimen~ionless wellbore
At storage constant, CsD' are defined e~sentially as for
con~tant-rate production:
Fig, 2.10-Characteristic influence of afterflow on Horner 0.000264 kAle
graph. IV= 2' (2.8)
<pJl.c,r w

centered in its drainage area; Curve 2 illustrates 0 894 C,


behavior for a well highly off center in its drainage CsD = _':' ;f, " (2.9)
area. For simplicity, the ETR is not shown in either <PJl.c,w
case. where
Many curve shapes other than those discussed A
above appear in practice, of course. Still, these few Cs =25.65 ~
examples illustrate the need for a systematic analysis P
procedure that allows us to determine the end of the for a well with a rising liquid/gas interface in the
ETR (usually, the time at which after flow ceases wellbore, and
distorting the test data) and the beginning of the C -,
LTR. !)-(tt-lJ V wb

Without this procedure, there is a high probability for a w~llbore containing only single-phase fluid
of choosing the incorrect straight-line segment and (liquid or gav.
using it to estimate permeability and skin factor. We define
2.6 Effects and Duration of Afterflow Ale = AI/( I + AI/I p). (2.10)
In our discussion thus far, we have noted several
problems that after flow causes the buildup test As noted in Chap. I, wellbore storage distortion
analyst. Summarizing, these problems include (I) (afterflow in the case of a buildup test) has ceased
delay in the beginning of the. MTR, making its when the graphed solutions for finite CsD become
recognition more difficult; (2) total lack of identical to those for CsD = O:~/Also,a line wiOJ unit
development of the MTR in some cases, with slope (450 line) appears at earl~ times for most values
relatively long periods of afterflow and relatively of CsD and s. The meaning of this line in a buildup
early onset of boundary effects; and (3) development test is that the rate of afterflow is identical to the flow
of se~eral false straight lines, anyone of whil:h could .r~te just before shut-in. ).
be mistaken for the MTR line. We note further that A,lf the un1t-slope line is present, the end of af-
recognition of the middle-time line is essential for terflow distortion occurs at approximately one and a
successful buildup curve analysis based on the half log cycles after the disappearance of the unit-
Horner plotting method Ipws vs. log 1(/p+A/) slope line. Regardless of whether the unit-slope line is
/ AI JI. becau~ethe line mU!it b~ identified to e!itimate prc~clll, thc end of aftcrllow di!itortion can be
reservoir permeability, to calculate skin factor, and determined by overlaying the log-log plot of the test
to estimate static drainage-area pressure. The need. data onto the Ramey solution (Fig. 1.6) -plotted on
for methods to determine when (if ever) afterflow .graph or tracing paper with a scale identical in size to
ceaseddistorting a buildup test is clear; this section the Ramey graph -finding any preplotted curve that
fills that need. matches the test data, and noting when the preplotted
The characteristic influence of afterflow on a curve for finite value of Cosvbecomes identical to the
pressure buildup test plot is a lazy S-shape at early curve for CsD =0. This point, on the actual data
times, as shown in Fig. 2.10. In some tests, parts of plot, is the end of afterflow or wellbore storage
theS-shape may be missing in the time range during distortion)
which data have been recorded -e.g., data before If the unit-slope line is present, we can use a
Time A may be missing, or data for times greater relationship developed in Chap. I to establish the
than Time B may be absent. Thus, the shape of the value of CsD that characterizes the actual test. There,
www.petroman.ir
IY~L.L. I t:~ I IN\,)

,
Ii. ,-
.,
TABLE2.3-OILWELL PRESSURE
BUILDUPTESTDATA

I +AI ~t.=~1/(1+~)
AI L- Ip Pws Pws -Pwf
-(~~~~~~-- ~- ~~ours) (psla) (psla) ,
0 --3,534-- 0 -
0.15 90,900 0.15 3,680 146
0.2 68,200 0.2 3,723 189
0.3 45,400 0.3 3,800 266
0.4 34,100 0.4 3,866 332
0.5 27,300 0.5 3,920 386 .
1 13,600 1 4,103 569
2 6,860 2 4,250 716
4 3,410 4 4,320 786
6 2,270 6 4,340 806
7 1,950 7 4,344 810
8 1,710 8 4,350 816
12 1,140 12 4,364 830
16 853 16 4,373 839
20 683 20 4,379 845
24 569 24 4,384 850
30 455 29.9 4,393 859 /
40 342 39.9 4,398 864
50 274 49.8 4,402 868
60 228 59.7 4,405 871
72 190 71.6 4,407 873

we noted that any point on the unit-slope line must


satisfy the relationship ".443141-
CsoP 0 .4431+tf7a
= I, (1.42)
to
which, in terms of variables with dimensions, leads to .~
qB At ~
Cs= 24~' (2.11) 0.: -

where ~t and At] are vailles read from a point on the


lIllit-slopc line. If we can estahlish C.t in this way (a
less acceptable alternative is to use the actllal
mechanical properties of the well- e.g., Ct = 25.65 Id'.,. r/
A\I'/I/PI,.h for a well with a rising liquid/gas in- lpt~t
terface), we then can establish CsO from EQ. 2.9 and
thus determine the proper curve on Fig. 1.6 on which ~t
to attempt a curve match. (It is difficult to in- Fig. 2.11 -Semi log graph of examplebuildup test data.
terpolate between values of CsO on this curve; ac-
cordingly, many test analysts prefer to find a match
with the preplotted value of Cso closest in value to a s.olution to the flow equ~tions for .an iijfinite-
the calculated value.) With Cso established, and actIng, .homogeneous res~rvolr; when, In an actual
permeability, k, and skin factor, s, determined from reservoIr, a pressure transIent reaches a boundary or
complete analysis of the test, we can use the em- important heterogeneity, the actual test data plot will
pirical relationships below to verify the time, tl,'h.t' deviate from Fig. 1.6. This characteristic of curve
marking the end of well bore storage distortion. matching is illustrated in Example 2.2.

to=50CsoeO.14S, (2.12)
Exal11pte2.2 Finding the End
or 170 (XX)C O.14,t of Wet/bore Storage Distortion
, ",/I.t= ' se. (2.13) Pr()blem.ThedatainTable2.3wereoblainedina
(kh/lJ.) pressure buildup test on an oil well producing above
We will illustrate (I) the application of the basic the bubble point.
curve-matching procedure in Example 2.2; (2) the The well was produced for an effective time of 13,630
check provided by Eq. 2.13 in Example 2.4; and (3) hours at the final rate (i.e.. , p = 13,630 hours). Other
COillplctc, quantitative curve-matching procedurcs in data include the following.
Chap. 4.
In thi~ di~clls~ion of the qualitative application of qo = 250 STB/D,
curve matching, we al~o should note that appearance 1J.0= 0.8 cp,
of boundary effects or the effects of heterogeneiti'es <I>= 0.039,
frequently can be verified from the curves. Fig. 1.6 is B = 1.136 RB/STB,

www.petroman.ir
~
-~'"
PRESSURE BUilDUP TESTS 29

f ", = 17 x 10 -6 psi -I, Then from


t. 'w = 0.198 ft, , Eq ..,2 9

'e = 1,489 ft (well is centered in square C -0.894 Cs


drainage area, 2,640 x 2,640 ft; r is sD -/f>c hr2
d. f I. .e , w
ra IUS 0 clrc e with same area),
Po = 53 Ibm/cu ft, (0.894)(0.0118)
l Awb=0.0218sqft,
h = 69 ft, and = (0.039)(1.7x 10-S)(69)(0.198)2 -=5880I
' .

rising liquid level in well during shut-in. Thu~, matchinl ~hould be attempted in Ihe rangc
A semilog graph [Pws vs. log (/p+A/)/A/] of 103<CsD<10.
(Pws
these -Pwf
data isvs.shown
Ale) in Fig. 2.12.
2.11, and
Froma log-log
these graph~,
graph 2. 71)etermlnatlon of Pernleability

answer the following questions. In chis scction, we examine techniques for Ihe nexi
.I. ~t what shut-in time (AI) does afterflow cease step in the sy~tematic analysis of a pre~sure buildup
dIstorting the pre~~ure buildup tc~t data? or falloff test: determining bulk-formation pcr-
2. At what shut-in time (AI) do boundary effects meability. llecau~e bulk-formation pcrmeability i~
appear? obtained from the slope of the MTR line, correct
Solution. From the semilog graph (Fig. 2.11), it select!~n of this region is critic.al. Average per-
seems plausible that afterflow distortion disappears meablhty, kJ' also can be estimated from in-
at (I p + AI) I A/.= 2,270 or AI = 6 hours because of t he formati~n available in.bu.ildu~ !.ests:
end of the characteristic lazy-S-shaped curve. ~he II~st problcm IS I~entlllc~tlon of Ihe MTR.
However, other reservoir features can lead to this This region cannol begm until aflerflow cease~
same shape, so we confirm the result with the log-log distorting the data; indeed, cessation of afterflow
graph. After plotting Ap = P -P vs effects usually determines the beginning of the MTR.
Ale = AII(I + Alii) on log-log paperwswe fi~d tha~ If the altered zone is unusually deep (as with a
the actual data m well. curve~ for s::: 5 for scveral hydraulic fracture), passage of the transient through
valuesofCsD(e.g.,CsD=103,104,andIOs).lneach Ihe region. of Ihe ~rainage a~ca.influenced by Ihe
case, the curve fitting the earliest data coincides with fracture will determine the beginning of the MTR.
the CsD =0 curve for s= 5 at Ale =A/=4 to 6 hours. .P~edicting the ti!11~ at which. the ~TR en~s is more
This, then, is 1he end of wellbore effects: I .= 6 difficult than predIcting when It begIns. Basically, the
hours. The data begin to deviate from the ~g:nilog mid~le-!ime li~e ends when .the radius o! in-
straight line at (t + AI) I A/.= 274 or At = 50 hours. vestlgatlon begins to detect drainage boundarIes of
On. the log-log gr~ph, data begin falling below the Ihe leste~ well; at this time, the .pressure b~ildup
filling curve at At = At .= 40 hours consistent with curve begins to bend. The problem IS that the time at
the semilog graph. e' which the middle region ends depends on (I) the
In summary, basing our quantitative judgment on di~tance. from the te~ted well to the re~ervoir
the more sensitive semilog graph, we say that the boundarIes, (2) the geometry of the area drained by
MTR spans the time range of AI = 6 hours to AI:; 50 the well, and (3) the duration of the flow ~eriod as
hours. This judgment is verified qualitatively by the well as the shut-in period. Cobb and Smith present
log-log graph curve matching. Even though the c.harts that allow the analyst to predict the shut-in
semilog graph is more sensitive (i.e., can be read with time At at which the MTR should end if drainage-
greater accuracy), it alone is not sufficient to area geometry and producing time are known. If this
delermine the beginning and end of the MTR: i~l:ormation is available and if .the reservoir i~suf-
matching Ramey's solution is a critically important ficiently ~omoge~eous that, untIl. the L TR begIns, it
part of the analysis. behaves In the Ideal way requIred by Cobb and
The log-log curve-matching analysis was per- Smith's theory, their charts can be used to check
formed without knowledge of CsD' Note that CsD results. ...
can be established in this case, at least ap- One useful generalizatIon can be made from their
proximately: from the curve match, we note that the result~. If a well was at pseudosteady-state before
data are near the unit-slope line on the graph of shut-in~ the time AI at which the L TR begins. is
Ramey's solution; the point Ap = 100, AI = 0.1 is approximately A/j" := 38 /f>J!cIA I k for a well cenlered
essentially on this line. Thus, from Eq. 2.11, in a square or circular drainage area. In Ihe equation,
qB AI (250)( 1.136) (0.1) A (sq ft) is the drainage area of Ihe teste~ well. I f the
Cs'= --= -wcll was not at pseudosleady-stale, A/f/lS larger than
24 Ap 24 (100) calculated by Ihe rule above. In many cases, we
= 0.0118 bbl/psi. ~imply a~sume thai Ihe straight line spanning the
..times between Ihe end of after flow distorlion and a
Alternatively (and, m general,less accurately), later bend of the Horner plot constitutes the MTR.
C -25.65 A wb -(25.65)(0.0218) Use of the log-log graph and curve matching, as in
s -p -53 Example 2.2, can help confirm this assumption.
=00106 bbl/ .The calculated radius of investigation (r;) at the
.pSI. assumed end of the MTR provides a qualilative
'Data are plotted on 3 x 5 cycle log. log graph paper (11 x 16'1. in.) and matched
w,~~~~~ Ramey solu,tion (such as provided In the SPE type-curve package) 'Cholce of time at which l TR begins Is somewhat arbitrary. The rule slated Is
p n the same size scale. based on a 10% deviation in slope ollhe Horner plot from the true MTR.

-
www.petroman.ir
estimate only of the radius of the infinite-acting reservoir sampled during the MTR; that region is
drainage area in the reservoir. given roughly by the radius of investigation achieved
I In summary, the procedure for determining bulk- by the shut-in transient at the start and end of the
formation permeability is as follows. MTR. From Eq. 1.47,
1. Determine the probable beginning of the MTR kt Y2
I by estimating when afterflow effects disappear. r; = () .
2, Assume that the probable end of the MTR 948 <l>p.c,
occurs when the Horner plot becomes nonlinear, Thus, at dt = 6 hours,
mIddle-tIme
ve!i fied ?y a data
deviation
on a log-log
from a curve
graph fitt~ng
using early-
the curve-
and r; = I (7.65)(6) -s ] Y2
matching technique. (948)(0.039)(0.8)(1.7 x 10 )

3. J r thcrc ;.\' an apparcnt lincar MTR, calculatc -302 rt


the slope of the middle-time line, and estimate -,

permeability from and at dt = 50 hours

, ,

I
qBII. 50 Y2
I k=162.6-;;;h. r;=302(6) =872 ft.

4. Radius-of-investigation estimates at the Thus, a significant fraction of the well's drainage


beginning and end of the assumed MTR may hclp area ha~ been sampled; its permeability is 7.65 md.
establish its plausibility but should be viewed a~ ..
qualitative only. 2.8 Well Damage and Stimulation
5. If there is no clear-cut MTR or if it is so short This section shows how to use data available in a
that its slope cannot be determined with confidence, buildup or falloff test to estimate damage or
bulk-formation permeability estimates from stimulation quantitatively. The basic technique when
quantitative type-curve analysis (i.e., curve- analyzing data from a plot of Pws vs. (tp +~t)/dt is
matching) can be used and probably are essential. called the skin factor method. It involves calculating
It can be helpful in MTR analysis to calculate the skin factor and translating it into a more easily
average permeability, kJ' from data obtained in a visualized characterization of a well. We will deal
buildup test. From Eq. 1.19, which is valid only if with another method of analyzing damage or
pseudosteady-state is reached during the production stimulation in a later chapter -a method using type-
period, curve analysis.
141 2 B [I / -~ ] Before we examine the skin factor method, it is
~ .q II. n (r e r w) 4 useful to consider briefly the physical reasons for

k J = -.damage or stimulation of a well. Wellbore damage is


h (p -P wf ) a descriptive term applied when permeability is
: For a well that is neither damaged nor stimulated, reduced near a wellbo~e. This.P,ermeability reduct~on

kJ should equal bulk-rormation permcability, k, u~uall~ occurs du!lng drIllIng. or completl?n


determined from the slope of the MTR; for a operations: C:auses In.cI~de pl~gglng of p.ores wIth
damaged well, k J < k; and for a stimulated well, fine m~terlal.ln the drillIng fluId an~ ~eactlo~ of the
kJ > k. Average permeability is valuable in checking form~tlon wIth filt!ate from the .drllllng fl~ld (e.g.,
consistency in buildup test analysis. If k and k J do swellIng <?f clays I~ ~he formatIon result!ng fr~m
not bear the proper relationship to each other, contact Wlt.h I.ow-sallmty ~~trate). C<?mpletlon fluIds
something is wrong -pos~ibly an incorrect analysis can cause s!mllar permeabIlIty reductIon as they enter
due to incorrect choice of MTR. the f?rmatl.on. ..
StImulatIon usually results from delIberate at-
tempts to improve a well's productivity. Common
Example 2.3 -Estimating Fonl1ation techniques include acidization and hydraulic frac-
l'emleability ttlring. Acidi7.alion i~ di~~olving plugging malerial~
..and the formation near the wellbore with acid in-
Pr()hle~. For tl~e test dlscu.s~ed In Example 2.2, jection through the perforations. Hydraulic frac-
determIne formatIon permeabIlIty. turing is creating seams in the formation with high-

Solution. In Example 2.2, we established that the pressure injection of special fluids, usually ac-
MTR spans the time range of ~t ~ 6 hours to ~t ==50 companied by sand or some other agent that will
ho\lr~ [2,270 ~ (t p + ~t) / ~t ~ 274] .From Fig. 2.11, prop the fracture open when the pressure creating the
note that the slope 111 of this straight line is fracture is removed.
.Eq. 2.4 shows how we can calculate skin factor, S,
III = 4,437 -4,367 = 70 psI/cycle. once the MTR is identified and bulk-formation

Then, permeability is estimated:

k=162.6-=
qBII. (162.6)(250)(1.136)(0.8)
S-
_ I 151
.og.
[ (PI hr -Pwf) _I ( k
, )+ 3 ..23]
lnh (70)(69) m I/>p.ct~

=7.65md. (2.4)

It i~ of interest to ,determine the portion of the We recall that PI hr is the value of Pws at shut-in

~~ -
www.petroman.ir
PRESSURE
BUILDUPTESTS -31
"...;-~.;.;-
tI

~~. --

I Pws
I 0
: in I(XX)
, C-
..:G- I»"
p-. EffiC1$
J t = I hr
I
I/)
~ I

01 I G m log t p +.1t
At
t1te, hr
Fig. 2.12-Log-log graph of examplebuildup test dala Fig. 2.13-Determination of PI h,.

time AI of I hour on the middle-time line, or its pleting our consideration of skin factor solely caused
extrapolation as shown in Fig. 2.13. It is not possible by formation damage or stimulation.
to calculate the skin factor until the middle-time line We now turn our attention to methods for trans-
has been established because values of k, m, and lating values of s into less abstract characterizations
PI hr are found from this line. If an accurate skin of the well bore. We consider three methods:
factor is to be calculated from a buildup test, the estimation of effective wellbore radius, 'h'U;
flowing pressure PWf mu~t be me&1surcdbeforc ~llut- call.:ulatioll of additional prcssurc drop near the
in. wcllbore; and calculation of now efficiency.
Interpretation of a given numerical value of the
skin factor can be summarized as follows. ":SCimaCion or ":ffccCive Wellbore Radiu~
I. A positive skin factor indicates a now The effective wellbore radius 'I~'Uis defined as
restriction (e.g., wellbore damage); the larger the --s
skin fal.:tor, the more severe the restriction. 'wu -, we (2.14)
2. A negative skin factor indil.:ates stimulation; the To ulldcrstand thc siglliricallcc of this quantity,
larger the absolute value of the skin factor, the more note that from Eq. 1.11,
effective the stimulation. q81J. I 688 4>c ,2
3. Conditions other than well bore damage can pj-Pwf= -70.6-fln(' IJ. ( w)-2S]
cause an apparent skin factor. The reason is that any kh kl
deviation from purely radial now near a well, which
results in to!al wel~ production squeezing through a
smaller vertical thickness near the well than away
--70
-.kh
6~
11n
( 1,6884>IJ.C('~
kl )
from the well, increases the pressure drop near the ..
wl.'ll. This is precisely tIle ~alIICI.'ffcct that wcllborc +In (e-2s) I
damage has; damage also results in an increased
pressure drop near the well. The basic equation used -2s
in constructing our theory of pressure buildup and = -70 6~ fln( 1,688 4>IJ.C(~e
falloff test behavior, Eq. 1.7, is based on the .kh kt )1
assumption that flow is radial throughout the
drainage area of the well up to the salldfac~; a
deviation from this assumption invalidates the
q81J.
= -70.6-ln'
(
I 6884>IJ.C,2,
(~ ) .
equation, but Eqs. 1.11 alld 1.16 arc u~U,llly CKl.:l.:llcll1 kh kl
approximations when th(: nollra<.Jiall"low OI.:I.:Uf!i Ilcar
th~ well bore only. l11is shows. that the efrel.:t of ~. on total p~cssurc
Conditions leading to nonradial flow near the drawdown IS the .same as that o~ a well wIth no
wellbore include (I) when the well docs not com- alt~!cdlon.ebut wlth.aw~llboreradlusof"ru'. .
pletely penetrate the productive interval, and (2) C~ll.:ulatlon of effel.:tlv~ wellbore r~dlus IS. of
when the well is perforated only in a portion of the special value for a.nalyzlng wells wIth ve~tlcal
interval (e.g., the top 10 ft of a 50-ft sand). In these fracture~. Mod~1 studIes have s~own that for hIghly
cases, the analyst will calculate a positive skin factor c~nductlve vertIcal fractures with two equal-length
even for an undamaged well. (In addition, the wlllgS of length Lf'
perforations themselves-their size, spacing, and Lf=2'wo. (2.15)
depth -also can affect the skin factor.) We will

I
examine results of this non radial flow after com- Thus, calculation

www.petroman.ir -of skin factor from a pressure


buildup or falloff test can lead to an estimate of producing about twice as much fluid with a given
fracture length -useful in a post fracture analysis. pressure drawdown as it would had the well not been
However, this analysis technique for a fractured well slimulated.
is frequently oversimplified; more complete methods Use of the skin factor method is illustrated in
are discussed later. Example 2.4.
.
Calculation of Additional Pressure Drop
Near Well bore Example 2.4 -Damage Analysis
We defined additional pressure drop (t:\p).~ across the Problem. Consider the buildup test described in
altered zone in Eq. 1.9 in terms of the skin faclor s: Example~ 2.2 and 2.3. Make the following
n calc\llation~ with lho~e data.
I (AI})s=141.2~-~s. I. c.'alculatcthc~kinfactorforthcte~tedwell.
:: kh 2. Calculate the effect ive wellbore radius r M'Q'
r In terms of the slope II' of lhe middle-time line, 3. Calculate the addilional pressure drop near the
&
r An ) s = 0869
( "'YJ .," ( s,) ..."""".,...' (216)
, wellborecausedbythedamagethatispresent.
4, Calculale the now efficiency,
Calculation of lhis additional pressure drop across 5, Verify the end of wellbore storage distortion
lhe altered zone can be a meaningful way of trans- using Eq. 2,13.
lating the abstract skin factor into a concrete
h " f h d
c aractenzatlon 0 t e teste we.
II F I
or examp e, a
S I
0 II Ion.
t
.. I th k' f '
we
II b d .
may
100 STB/D 1 'tl
e pro uclng
.
01 WI 1 a
1,InSk rOC or. n e s In actor equatIon,
.I:'

" .
d
raw
d
...Ine own 0
f 1 000
, pSI,
. A I
na YSIS0 a
' f b . Id
UI up es
t t
.
Iwe need
t 0 a s h u t -In'
PI
tIme
'
hr
0 f I
from
h our. At
extrapolatIon of the mIddle-time
UI
A. = I
mIght show that (t:\p)fls900psland,thus,that900 ho (t + A t)/ A t 13631 F t I t .
pSI
"
of the
Th '.
total
I.
drawdown
h 'f h
occurs
d
across the altered
d
ur,
(F' 2 I PI ) f th
Ig..
ddl t
0
u
I'
e
u
t th ' t '
ml
.
= ,

e-
.
Ime
,rom

Ine
an

0
ex

IS
rapo

Ime
a

PI
Ion

hr
zone. IS Imp les t at I t e amage were remove, 4 295 ' (N t h d ' ff t th ' '
the well could produce much more nuld ..= wIth the ' pSI, 0 e ow I eren IS IS , f rom th e

same drawdown or, alternatively, could produce the actual pressure at 6t = 1 ~our: 4,103 pSI.) Then,
same 100 STB/D with a much smaller drawdown. becausekl<t>IJ.C,
= 1.442 x 10 ,

Calculation of Flow Efficiency s = 1.151


(p t hr -P wf)
-log"
l k
) + 3.23 J (
'" m <t>p.c
I'W
f The final method that we will examine for translating
swell
intois abyphysically
calculationmeaningful characterization
of the now efficiency, E. ofWea f (4 295 -3 , 534)
= 1.151{'
define now efficiency as the ratio of actual or ob- 70
served PI of a tested well to its ideal PI (i.e., the PI it
would have if the permeability were unaltered all the 1.442 x 107
way to the sand face of the well). In mathematical -IOg/ 98 2 1+ 3.23J = 6.37.
terms, (0. I )
J In Example 2.2, we found from curve matching that
E= ~ s:5, which is good agreement.
Jidcnl 2. I:.jfedive U'elllJore Radius. From Eq. 2,14,

j =P-P~if-(/1f)s. " , ,.,(2.17) rM.u=rwe-'~


P-Pwf ..
.For rapid analysis of a pressure buildup or falloff = (0.198)e -6.37 ..
test, Eq. 2.17 can be written in approximate form as
.= 0.00034 ft.
P -P"f- (l1p)s
E=. ' ..".,...,.."., (2,18) The physical interpretat ion of this result is that the
P -PM:! tested well is producing 250 STB/D oil with the same
..,., pressure drawdown as would a well with a wellbore
wllere p ,the extrapolallon of the mIddle-tIme lIne to d' f 0 00034 ft d b 'l'
, ,-, ra IUSO, an permea II t y una It ered Up to
I I . FI
(t,,+6t)/6t=I,lsfoundmorereadllythanp,whlch
, I ff '" th esan df ace,
can requIre cngt lY ana YSI~, oW e Iclency IS ac-
tually time dependent unless a well reaches 3, Additiol,al Pre.5sure Drop Near tl,e U-"elllJore,
p~cudo~teady ~tate during the producing period (only I:rom Eq, 2,16,
tllclli~p-P\l'rinEq,2,17con~tant),
flow efficiency is unity for a well that is neilher (~) -0869
S-, III ()S
, dal!l~ged I~or stimulated, For a d,amaged well, now =(0,869)(70)(6,37)
~ erflclency ISIcss than one; for a stlmulatcd well, now
efriciency is grcatcr, t!lan one, A d~magcd ,,:ell wilh a = 387 si.
calculated now efflclcncy or 0.1 IS producIng about p
10% a~ much nuid with a given pressure drawdown Thu~, of the total drawdown of approximately
a~ it would if the damage were removed; a stimulated 4,420 -3,534 = 886 psi, about 387 psi is caused by
wcll with a calc\llated nO\V efficiency of two is damage, Much of this additional drawdown could be

---~ -
www.petroman.ir
~ "
PRESSUREBUilDUP TESTS ~:~ :- 33 I

avoided if the skin resulted from formation damage -


(rather than from parlial penetration, for example) Example 2.5 -Incompletely
and if the well were srimulated. Perforated Interval'
4. F/ow Efficiency. To calculate flow efficiency, Problem. A well with disappointing prodllctivily i~
we need p., the value ofpws on Ihe middle-time lille pt:rforated in 10 ft of a total formatioll thil.:kllC:~Sof
at (tp+At)/At= I. We cannot extrapolate dircl.:tly 50 fl. Vertil.:al and horizolltal permeabilitic~ "art:
on our plot because there are no values of believed to be equal. A pressure buildup lesr wa~ run
(t p + AI) I At less than 100, bur we nOle that the Oil the well; results and basic propcrties are ~um-
pressure increases by 70 psi over each cycle; thus, we marized as follows.
can add 2 (70) psi to the value of P at I 190 .
(lp+At)/At=IOO: PIYj=' ps~,
PI hr = 1,940 pSI,
p' = 4,437 + 2 (70) = 4,577 psi. <p = 0.20,
Then, from Eq. 2.18, m = 50 psi/cycle,
II. = 0.5 cp,
£=P.-Pwj-(.:1p)s rw = 0.25 ft,
p' -Pwj CI = 15 x 10-6/psi -I, and
k = 3.35 md.
4,577 -3,534 -387 Calculate s, sd' and s p; on the basis of these resulls,
= 4 577 -3 534 determine w~ether the productiviry problem results
, , from formarlon damage or from orher causes.
= 0.629.
Solution. From Eq. 2.4,
This means that the well is producing about 621170
as -k
much fluid with the given drawdown as an un- S=I.151[Plhr-PWj -Iog( 2)+3.23
damaged well in a completely perforated interval m <PII.Clfw ]
would produce.
5. £ndof Wet/bore Storage Distortion. From Eq. = 1.1511 (1.940- 1,190)
2.13and Examples 2.2 and 2.3, 50
I = 170000
' Cs eO.14s -
wbs kh
III.
-10 ( 3.35
g (0.2)(0.5)(1.5xI0-S)(0.25)2
) + 3. 23 ]
(170,000)(0.01 1S)e(O.14)(6.37) = 12.3. ~
= (7.65)(69)/0.8 .The I.:ontribution <:>fan in~omplelely perforated
Interval to the total skin factor IS, from Eq. 2.20,
=7.42 hours. h h m
This agrees closely with Ihe results of Example 2.2. ~/J= (f -1)[ln(/ V~ )-2]
p w V
Effect of Incompletely Perforated Interval
When rhe complered interval is less rhan total for-
~ation rhickness, the pressure d~op near Ihe well is
(50
= 10 -I
)[In (~v50 ~~ ) -2 ] .l
Increased and the apparent ski!) factor becomes :
increasingly positive. In a review of technology in = 13.2. l
this area, Saidikowski6 found thai total skin factor, !
5, determi.ned from ~ pressure transient re.stis related Rearranging Eq. 2.19, skin factor, sd' resulting from
10Irue skin factor, sd' caused by formation damage formation damage is
and apparent skin factor, sp' caused by an in- h f
completely perforated interval. The relationship Sd= :.:2. (s-sp)
betweenthese skin factors is hi
hi 10
S=j;-Sd+Sp' (2.19) =-(12.3-13.2)
p 50
where hi is total interval height (ft) and hlJ is the = -0 IS
pcrforaled interval (1'1). ..
r Saidikowski .also verified that Sp can be eslimated A~ a practical matter, the well is neither damaged nor ~I
rrom the eqllatlon stll1\lllaled. The obscrvcd produclivilY problem is ;,i
h
I ( h r-k- I ,-"all~cdcnl!rcly by Ihe effc,-"l~ of an in,-"omplclcly ~
~"J= (-L -I) In -Lv ---}l- ) -2, (2.20) pcrloruled Inlcrval. f
hp rw kv
where k if is horizontal permeability (md) and k v is Anulysis of Ilydruulicully "'ruclured Wells
vertical permeability (md). Use of these equations is Type curves provide a general method of analyzing
b~stillustrated with an example. hydralllically fractured wells -particularly because

www.petroman.ir ~
Sla'E
E.~

PWS FR/(;~E T TABLE 2.4 -BUILDUP TEST SLOPES FOR


DC».4INATB HYDRAULICALLY FRACTURED WELLS
ClRVESHAPE
L,lr. mm..'m'ru.
0.1 0.87
0.2 0.70
0.4 0.46
0.6 0.32
t +6t 1.0 0.28

logT
Fig. 2.14 -Buildup curve for hydraulically fractured well,
bounded reservoir.

finite condl,lctivity can be considered. Some con- Thus,


vention~1 .methods are a~so o~ value for infinite- L L
conductIvIty fractures. ThIs sectIon summarizes some -210g( ~ ) = -2 log ~ -210g (~
of the useful conventional methods. 2r w 2 r w)
When fractures are highly conductive (i.e., when
there is little pressure drop in the fracture itself) and = (PI hr -Pwj) -IOg (~
~hen there is ulliform nux of nuid into the fracture, m )
<l>IJ.C,
lInear now theory de~cribes well behavior at earliest
t~mcsnow
tical
ill a rates
buildtlp
of
te~t. (Uniform
fonnation nuid
nux
illtO
mcall~
thc
idcll-
fracturc
-210g ( ~ ) + 3 23
r. .

..w
pcr Ullit cro~s-sectlonal area at all points alollg the This simplifies to
fracture.)
product ion,
From Eq.

qB
1.46, for collstallt-rate
I
IJ.f V2
log ( L j) = ~
2
l(~ mI
~)
Pi-PI~1=4.064-(- ) I .
11Lj k<l>c, k
f Forabuilduptest,forlp)..ill, +log;;;;;--2.63. 1 (2.22)
qB ill V2 .'
Pws -Pwj=4.064 -(£.- ) .~slng Eq. 2.22, fracture I~ngth, LI' can be estimated
hLj k<l>Ct if the MTR can be recognIzed, whIch allows m,PI hr'
Thus, the slope ',1 of a P vs. v'A7 plot is and k to,be determined.
L wYi In buIldup tests from some hydraulically fractured
"'L =4.064 ~ (~ ) 2. (2.21) :-veils, the ~rue.middle-time line does not appear, as
f IlL f k4>c, Illustrated In FIg: 2.~4. (~fter~ow can cause the same
From measurements of this slope, fracture length, c.urve shape.) ThIS sltua.tlon a.nse~bec~u~e, at earl~est
/4f' can be estimated. This procedure requires that an tlme~, the depth of InvestIgation .'s In a region
independent estimate of permeability be available -d°m.lnate~ by. the fracture; at lat.er tImes,. the.depth
from a prefracture pres~ure buildup test on the well, of Investigation reaches a point dominated by
for cxamr;le. boulldar~ effect~. (Se~ Fig. 2.14.) When the length L,r
When linear now cannot be recognized (i.e., when of ~ vertical rracture IS greater than one-tenth of the
there is f10 _early straight-line relationship between draln~ge radius r e of a7well centered in its drainage
p",Sand v'A1), we can use the observation that L area, It ha~ been found that boundary effects begin
= 2 r, for infinitely conductive fractures to estimat{ before the Innuence of the fracture disappears. For a
fractl~'~elellgth. Rather thall calculate s directly, we given drainage radius, the greater the fracture length,
can note that the greater the discrepancy between the maximum
slope achieved on a buildup test and the slope of the
s= 1.151 1 (PI hr -Pwj) -IOg ( -~ ) + 3 23] .true middle-time line. Table 2.4 summarizes the ratio
nl <l>JA.c,r~.' of the maximum slope attained in a buildup test to
and, bccau~c the ~Iope of the true middle-time line (from the work
L of RII,~~cll and Truitt') for infinitely conductive
, = ~ =r e-s fracturcs.
"'0 2 w , The implication is that if the test analyst simply
then does the best he can, and finds the maximum slope
L L on a buildup test from a hydraulically fractured well
s= -In( ~ ) = -2.303 log( ~ ). and ass.ume~that this maximum slope is an,adequate
2, w 2r w approxImation to the slope of the true middle-time

www.petroman.ir
~
-~ ':. ~-
-
PRESSURE BUILDUP TESTS 35

ETR MTR L TR1/


RI

ETR MTR t-p. P I


"" I ws

Pws ~...

tp + .1t tp + .1t
log --.11--- log --~t -/

Fig. 2.15-Buildup test graph for infinite-acting reservoir. Fig. 2.16-Buildup test graph for well near reservoir
limit(s).

line, then the permeability, skin factor, and fracture For a reservoir with one or more boundaries
estimates will be in error, with the error growing as relatively near a tested well (and encountered by the
fracture length increases. radius of investigation during the production
Correlation of reservoir model results by Russell period), the late-time line must be extrapolated (Fig.
and Truitt 7 showed that an equation similar to Eq. 2.16). (This can be quite complex for multiple
2.22 can be used to estimate true fracture length even boundaries near a well.) Note that our discussion is
when L f > 0.1 r e. still restricted to reservoirs in which there has been
We again emphasize that all methods in this negligible pressure depletion. Thus, even in the case
section assume highly conductive, vertical fractures under .consideration, the well must be relatively far
with two equal-length wings. When fracture con- from boundaries in at least one direction.
ductivity is not high, fracture length estimated by
thesemethods will be too small. Static Drainage-Area Pressure
.For a well in a reservoir in which there has been some
2.9 Press~re Level In. pressure depletion, we do not obtain an estimate of
Surrounding Formation original reservoir pressure from extrapolation of a
A pressure buildup test can be used to determine buildup curve. Our usual objective is to estimate the
average drainage-area pressure in the formation average pressure in the drainage area of the well; we
surrounding a tested well. We have seen that ideal will call this pressure static drainage-area pressure.
pressure buildup theory suggests a method for We will examine two useful methods for making
estimating original reservoir pressure in an infinite- tllese estimates: (I) the Matthews-Brons-Hazebroek
acting reservoir-that is, extrapolating the buildup (Mllll)8 p. method and (2) the modified Muskat
t~st to infinite shut-in time (tp+dt)/dt=IJ and l1Il:thod.9
reading the pressure there. For wells with partial The p. method was developed by Matthews et al.
pressure depletion, extrapolation of a buildup test to by computing buildup curves for wells at various
infinite shut-in time provides an estimate of p., positions in drainage areas of various shapes and
which is related to, but is not equal to, current then, from the plotted buildup curves, comparing the
average drainage-area pressure. In this section, we pressure (/).) on an extrapolated middle-time line
will examine methods for estimating original and with the static drainage-area pressure (p), "r'hich is
current average drainage-area pressures. the value at which the pressure will stabilize given
sufficient shut-in time. The buildup curves were l:om-
Original Reservoir Pressure puted using imaging techniques and the principle of
For a well with an uncomplicated drainage area, superposition. The results of the investigation are
original reservoir pressure, Pi' is found as suggested summarized in a series of plots of kh (p' -p) /70.6
by ideal theory. We simply identify the middle-time q/lB vs. 0.000264 ktplt/>/lc,A. [Note that kh(p. -p)
line, extrapolate it to infinite shut-in time, and read /70.6 q/lB can be written more compactly as 2.303
the pressure, which is original reservoir pressure (Fig. (p* -p) 1m. Also, the group 0.000264 ktp/t/>IJ.C,Ais
2.15). This technique is possible only for a well in a a dimensionless time and is symbolized by t DA .The
new reservoir (i.e., one in which there has been group kh(p*-p)/70.6 q/lB is a dimensionless
negligible pressure depletion). Strictly speaking, this pressure and is given the symbol PDMBHJ. The only
is true only for tests in which the radius of in- new symbol in these expressions is the drainage area,
vestigation does not encounter any reservoir A, of the tested well expressed in square feet. Figs.
boundary during production. 2.17A through 2.17G (reproduced from the Mat-

www.petroman.ir
Chapter 6
Other Well Tests

6.1lntroduction
This concluding chapter surveys four well-testing In an infinite-acting, homogeneous, isotropic
techniques not yet discussed in the text: interference reservoir. the simple £i-function solution to the
tests; pulse rests; drillstem lesls; and ~'ireline for- diffusivity equalion describes Ihe pressure change al
mation rests. These tesls and olhers covered in Ihe observation well as a function of lime:
previous chaplers by no means exhausl the subjecl; qBIL IP/LCr
however. the comprehensive Irealment needed by the Pi -Pr = -70.6kh£i( -948 -t- ). ...(6.1)
practitioner is provided by SPE monographs 1.2 and 1
Ihe Canadian gas well tesling manual.3 This is simply a restatement of a familiar result.
.The pressure drawdown at radius r (i.e., the ob-
6.2 Interference Testing servation ~.ell) resulting from production from the
Interference tests have two major objectives. They active well al rare q, slart1ng from a reservoir initially
are used (I) to determine whether two or more wells at uniform pressure Pi, is given by the £i-function
are in pressure communication (i.e., in the same solution. Eq. 6.1 assumes thaI the skin factor of the
reservoir) and (2) when communication exists, to aclive well does nor affecl the drawdown al the ob-
provide estimates of permeability k and poros- servation ~'ell. Wellbore storage effects also are
ity/compressibility product, d>£i, in the vicinity of assumednegligible al bOlh Ihe aclive and observation
the tested wells. wells "hen Eq. 6.1 is used 10 model an interference
An inlerference test is conducted by producing test. JargonS shows thaI bOlh Ihese assumptions can
from or injecling inlo al least one well (the active lead 10error in testanalysis in some cases.
well) and by observing Ihe pressure response in at A convenienl analysis lechnique for interference
least one olher ~'ell (Ihe observalion well). Fig. 6.1 lesls is Ihe use of Iype curves. Fig. 6.3 is a type curve
indicates the typical lest program with one active ~'ell presentedby Earlougher; I it is simply the £i function
and one observalion well. expressedas a function of its usual argument in now
As the figure indicales. an active ~'ell starts problems. 948 oJJ.'ir2/kl. Note that Eq. 6.1 can be
produl.:ing from a reservoir at uniform pressure at expressed complelely in terms of dimensionless ..
Time O. Pressure in an observation well, a dislance r variables:
a\\ay, begins to respond after some lime lag (related I I
to the lime for the radius of investigation Pi -Pr = --£i
1(- -)
corresponding 10 the rate change at Ihe active well to qBIL 2 4
reach the observation well). The pressure in the active (141..!~)
well begins to decline immediately, of course. The
magnitude and timing of the deviation in pressure 2 ".
response at I~e observ~tio~ well d~~?ds on reserv?ir .! QILC,r'" )( ~ ).1,
rol.:k and fluId propertIes In the VICInity of the actIve 0.000264 kl 'r.., ,
and observation wells. or
Vela and MI.:Kinley~ showed thaI Ihese properties I 2
are values from the area investigated in the test -a PD = --£i ( ~), , .-_(6.2)
reclangle \\'ith sides of length 2ri and 2ri + r (seeFig. 2 41D
6.2). In Fig. 6.2. ri is the radius of investigation \vhere
achieved by the active well during the t.esland r is the -kh
distance bel ween active and observallon wells. The PD = (p, Pr),
essenlial point is thaI the region investigated is much 141.2qBIL
grealer than some small area bel ween wells, as in-
tuition might suggest. rD =rlr ""

www.petroman.ir
PRESSURE BUILDUP TESTS ---=~lr";'
_~i;"" ~~fltff~- ;: 1i, 37

, r -~ --1- -~ -l! 1--- I I


, !
6

. i
: I
:
.
.
-1

I ~~ I I ! "" "" :
c.'m
II
-.
~
I
!
I
I;
I
':
tIJ,
/"
, i
i
i
.,a
c.jCD: --' 1 " ",- : -' / !
0 .' m /1 I,
' I i : u..o-;;;; "
I : ..'1"

I I i /1.. //
i : /1' --1 /1
I .' /1 1
I :: 1 i
I
: ~ I "'" .
0,01--r- 01 I 01: 04 06 ~ 01 I 1-"'I ..1 I I' 6 ..
0,000264 kt

~,.cA
Fig, 2.178 -MBH pressure function for differenl well locations in a square boundary,

0.<XX>264kl pss general drainage-area shal'N.'Srollow eq,.ations of Ihe


IDA = =0.1, form
t;I£C,A -
or. for this case (with A = 160 acres =6.97 x 106 sq P-Pwj= 141.2~ 1~In(~~
~)- ~ +s. J
ft).. kh 2 CAr. 4
Ipss=183 hours. (1.20)

" The reader can verify that use or I ss in the Horner


plot and in the p' method leads to t~e same results as after pseudosteady state now is achieved. Values for
;; in Examples 2.3 and 2.6. the shape factor CA can be derived from thePDM8H
.vs. I D.~ charts for the various reservoir geometries in
Shape Faclors ror Re~ervo.rs Figs. 2.17 A through 2.17G. Note (hat (he definition i
Brons and Millert2 observed that reservoirs with of p' implies that f
I

I
!,
f II'.-.I I ii
p -p ,!
:-' ~ 706QjJ.B/kh .I:
I ---!,
" :
:~ 5 --~ .-1"" ;\

4. -':- ,.,1/ "" / ;

~~..-1
3 ~ ,,~ :'
"~~--:-- 2. -"""",: /, ~ ~ ~ / ---, [

" ""
"" ~
/' '
I ~,..,. i
I

0
--"" :'
01 02 03 04 061 QI 2 3 4 , I 2 3 4 , 10 .I

0.000264 kt
+jJ.cA
Fig, 2.17C -MBH pressure function for different well locations in a 2:1 reclangular boundary

-~__~~JI_II.. www.petroman.ir
38 -WELL TESTING

~ ;~~-r-] ::rrrj--"- 1---; ;-!"1;--'---' I


706Q1/.B/kh;:i!':i ! ! ,~I
4 '
!

.2

1I

0 I

-I

-2
06 01 2 3 4 6 1 2 3 .6 10

0000264 kt
~1/.CA
Fig, 2.17D -MBH pressure function for different well locations in a 4:1 rectangular boundary,r

f
q 8 IL
p' -P":f =70.6T In(tp +~t)/~t, ~
h
and lhal, allhe instant of shut-in (~t = 0), -In
( 10.06 A
,... _2 ) + 1.5]
P.q81L
-p"j=70.6- kl f In ( -~-:-::-:!
kt ) +2s. I CA'",
I ,~,688tPlI.£."", -q8Jl 0.<XXJ264ktpCA

These relationships result from replacing Pi with p' -70.6kj;- In( tPlJ.CA )
in Eqs. 1.11 and 2.1, which are valid for infinite- I I
i acting reservoirs only. :
Eliminating P"f between equations, - 70 6!!!!!:-
I
I (C ) l

p' -jJ = 70.6- q8p. In ( kt ) -.n

kh
At DA ' .

k/1 i~6si:;~~. 0r

, I,
i
i
..!
.--I
-!r !
~I
--'
:
loCo .,

,oX
a,ljQ'
II ~ ' I
.' C' .
0.1 co ,
I o. .,

,... ,
, "

:.

..., M I' ..., .G


0.000264kt
~llcA
Fig. 2.17E -MBH pressure function for rectangles of various shapes.

-
www.petroman.ir
PRESSURE
BUILDUPTESTS -39
.c
~
Q.m
I ~
.a
0..0
d
roo
,'
!fit
.'
GO
4IllcA
Fig. 2.17F-MBH pressure function in a squareand in 2:1 rectangles.
p. -p determine CA. For example, consider a circular
PD MBH = 70.6 q81J./kh drainage ~rea with a centered well.
From Fig. 2.17A, for
=In(CAIDA)' O.OCXJ264
kip
IDA = = I,
This equation implies a linear relationship between cPlJ."
,A
P D MBH and IDA after pseudosteady state now has
been achieved. Indeed, inspection of the curves in p. -p
Figs. 2.17A through 2.17G shows that, for suf- PDMBIJ = 706 8 /kl =3.454.
ficiently large IDA' a linear relation does develop. .q IJ. I
Further, any point on this straight line can be used to Thus, In (C A (I») = 3.454 or C A = 31.6 -essentially
4
,
3 .o'
,.
f.
2
;1'; .c
~
i:;;! ICI.~ I :
:,; ; .1 ;; ;
Q.
.j...
!;., " I \D
0 0
1Oi po.
!' :\~; 0
III ;' t
.'!!o.-"" -I
,.
tu',r; :
-':
.-2 '
" '
0
.0'
"):1' :
,
iI
r -3 I
~ ~' 001 01 10 10 100 !',
;~' 0OOO264kl
.,.cA
[
.. I
Fig, 2.17G-MBH pressurefunction on a 2:1rectangleand equilateraltriangle. ii.
r
,.;:;~:;:,~:;,. www.petroman.ir ~
"C;L.L II;;;) IINI,j

TABLE 2.5 -APPLICA TION OF MODIFIED MUSKA T METHOD

30
(ho.;}~rS)~;i)
4.393 --15
-i;4.408 o;;~~1f;~~=4.422
19 29 -;, ~ ASSUMED
TOO HIGH p

40
50
60

72
4.398
4.402
4.405

4,407
10

3
6

1
14
10
7

5
24
20
17

15
~

I
'e::.
~
~ ASSUMED P

g CORRECT
thc val'Ic given in Table J .2. Note al~o that the linear -ASSUMED 15
rclatioll...hip between P/)~tmf and IDA begills at TOO LON
11>..t ~O.I; ill Table 1.2, this is the value at which the
p~cudosteady-state now equation becomes exact (in
the column "Exact for '0/1 > "). At

Modiried Muskat Method


The d ' fi d M k h d b d I .., . Fig. 2.18 -Schematic graph lor modilied Muskat method.
mo I Ie us at met 0 IS ase on a Imltlng
form of Eq. 1.6, which is a solution to the now
equations for a well producing from a closed, The modified Muskat method has two important
cylindrical re~ervoir at constant rate. Using super- advantages over the p. method: (I) it requires no
position to simulate a buildup rollowing stabilized estimates of reservoir properties when it is used to
now (depth of investigation has reached reservoir establish jJ (except to choose the correct portion of
boundaries) and noting that, once boundary effects data for analysis); and (2) it has been found to
are felt in the buildup, the equation can be ap- provide satisfactory jJ estimates for hydraulically
proximated as fractured wells and for wells with layers of different
B permeability that communicate only at the wellbore.
P-PII'.f = 118.6~exp( -0.00388 k4//</>IJ.c,~). In these cases, the p. method fails. The Muskat
kh ~ method has serious disadvantages, too: (I) it fails

(2.23) when the tested well is not reasonably centered in its


F ..drainage area (although the drainage area need not
.or a~lalysls of buildup tests, we usually express t~is be cylindrical, as implied in derivation of this
equation as -method); and (2) the required shut-in times of (250

I - ( QBIJ. ) 0.OO168k111 </>IJ.c,r;)/k to (750 </>Jl.C,r;)/k are frequently im-


og(j>-PII:f)=log 118.6kh -</>p.cr2. practically long, particularly in low-permeability
, ~ reservoirs.
(2.24) We now illustrate this method with an example.

Approximations used in developing this equation are


valid ill tllc ..II'lt-ill timc ral1gc Exalt'p/£, 2. 7- Modified
2~f) ""(',r; 7~f) 4>/(.,r; Mliska/ Me/hod To Ca/cli/a/e
k- ~ 111~ -k-' A verage Pressure in Drainage A rea

Note tllat Eq. 2.24 has the form Problem. Consider the buildup test described in
Examples 2.2 and 2.3. Estimate the average presStire
log<l; -PI\:f ) = /1 + 8111, in the well's drainage area by using the modified
whcrc A and 8 are constants. This form of the Muskat method.

cq'Iatiol1 suggests how it is applied, We assume a Solution. The data that can be examined by this
value for p and plot 10g(p-PM's) vs. 111 until a method are those in the range 6./= (250 </>IJ.c,r;)/k to
straight line results; when it does, the correct value of 111 = (750 </>IJ.C
,r;)/k. In this case we are fortunate to
p (thc !itatic drainage-area pressure of the tested well) have estimates of k and r~, so we can eliminate data
has been found. Experience with the method in- outside the time range of interest. Often, of course,
dicatc!i that it is quite sel1sitive. An assumed value of we do not have these estimates -a situation that does
p that i~ too low producc!i a plot of log(jj -PII'f) vs. not limit the applicability of the method, but one lhat
6.( that ha~ a noticeable downward curvature for data slightly complicales the trial-and-error nature of the
in the time range calculations.-Here. ---

I 2S0 4>#(clr~ 750 4tIJ.C,r~ 250 4tIJ.C,r~ (250)(1,320)2


--~ ~11/~ k' k = (1.442x 107) =30.2 hours,

An a~sumed value of p that is too high produces a and


noticeable upward curvature in this same time range. 750 </>C ~
l11e correct assumed value of p produces a straight IJ. , ~ = 90.6 hours.
litlC in (and only in) the proper time range. (See Fig. k

2.lft.) Thu~, we can examine data from 4/=30 hours until

~ ~-
B
www.petroman.ir
BUILDUP TESTS 41

I
!

TABLE 2.6 -BUILDUP DATA FOR WELL NEAR BOUNDARY tOO

~( Pws ~( Pws ~( Pws


(hours) (psia) (hours) (psia) (hours) (psi a) (/)
0 3,103 8 4,085 30 4,614 0.- ~
1
2
3 , 488
3,673
10
12
4, 1 7 2
4,240
36
42
4 , 700
4,770
I
10.- ; ~:=:~-Q~~~~
o ;~.:~~--o
P' 4422
3
4
5
6
3,780
3,861
3,936
3,996
14
16
20
24
4,298
4,353
4,435
4,520
48
54
60
66
4,827
4,882
4,931
4,975 K>
~:::::~~~~~ p. 4412

we stop recording pressures at ~ = 72 hours.


We make Table 2.5 for three trial values of p.
From plots of (p-Pws)' we see that p=4,412 psi is I
clearly the best choice; we also note the sensitivity of M) ~ ~ 70

this method. There is a noticeable curvature for ~ t hr


p = 4,408 psi and p = 4,422 psi (i'"ig. 2.19). '

This example application of the modified Muskat Fig. 2.19 -Modified Muskal melhod applied to example'
method is intended to illustrate only the mechanics of buildup tesl.

using the m~thod. The pr~ssurc had built up to withil1


5 psi of its static value at a shut-in time of 72 hours;
in such a case, there is little value in applying either
this method or the MBH p' method. Both methods
are of value only when the final pressure is a

significant distance from stabilization.

2.10 ReservoirLimits Test -70.6~ Eil~92 tjI~ 1


In this section, we deal briefly with techniques for kh k (Ip + AI)

estimating reservoir size and distance to boundaries.


These comments are introductory only, and deal only Bp. -3 792 tjlp.C L 2
with the simplest cases. The intent is to illustrate an -70.6 ( -q) -Ei( ~- I --).
approach to these problems rather than the state of kh k~

the art. The techniques presented are based on (2 25)


on
buildup-test
drawdown data
test analysis.
analysis has
Much
been technology
developed and
basedis For a shut-in time sufficiently large that the

summarized by Earlougher. t3 logarithmic approximation is accurate for the Ei


We demonstrate that a single boundary near a well functions, the equation becomes
causes the slope of a buildup curve to double, and qBp. 1 +.-11 I +.-11
then develop a method for estimating distance from a Pi -P ws = 70.6 -=-Iln( .p .--) + In( .p '--.)]
tested well to a single boundary. kh ~ .-11

In Chap. I, when we illustrated application of the


superposition
pressure in
principle,
a well a
we showed
distance L
that
from
the flowing
a no-flow = 141.2-ln
qBp. ( I'P' +.-11
-.. )

l
boundary (such as a sealing faull) is given by Eq. kh.-11

1.50. Rearranged, it becomes This can be written as

' P;-Pw/'= -70.6- qBp. [In ( 1,688cf>p.Clr~) -2s ] qBp.


Pws=Pi-325.2-log(lp+.-1/)/.-1/]. ., (2.26)
;I kh kip kh
" 2 Two observations can be made: (I) for a well near a
kh [ -948 kip
cf>p.CI(2L)_ ] single
shows boundary,
that the slope
such of as a abulldu,P
s~ling curve
fault, will
~q. even-
.2.26
...': -," -70.6~Ei
We can develop an equation describing a buildup test tuall~ double (~ompare Eq. 2.26 with Eq. 2.1) and (2)
for such a well. Note that the time r~qulred for the slope to double can be
long -specifically, 3,792 .JI.C,L 2 /kA/<O.02, or
[ ~~88cf>Jl.CA ] -2s- J .-1/>1.9xIO' tjlp.c,L2/k. For large values of L or I.
P, .-= Pws -706~rln
.kh l k(1 +~) small values of permeability, shut-in time required
p for the logarithmic approximation to be valid can be
, longer than the time ordinarily available for a
[ In ( ~~ cf>p.CE ) -2 sl buildup test. For .this reason, ~waiting a doubl~ng in
ti.J -70.6 (- q ) ~kh k~ j slope on a buildup test IS not necessanly a

www.petroman.ir
-PRESSURE
--
,; ~;..r:1CIO"~'~e~~oc Or ~cen!!fy;~~
:1 no-now.. ,
!,. , "",
""', '- \ " --,,- .:\\\:.a~\:.es'.~'~~"':.::-:s~a!1\.".,j"..
," , ." o '"
~. ."
, .' '.\'; :- "-~. ~'l)U!~Car\ "cr-t.'("'O,,"v SO"""
..."'.."p".".,,.
...\"..-'.
..."--"--"-' "",f:t.' 2 '~-""',:."."""
'."""'-"""'\'~.".". '."-~': :'..
:. I'j,;"':"'r~:,~;:-".:;.~":'..':"~,.',., *
"' ,:?, I-\r ,'l\~ V A
,. .:,-;- ( ", '] -' ~;)
-'.; .'.,,' .,!., ..-
..; :,' .":'j P ws ,V-rR ~
, ""~

-O,434£i( -3,792op.c(I_:,' 1""""""'- LTR. I


\ kIp /: L- "'--~

-70 ,6~£'
kJ1' ( ~~~~~\ktlt ,~ ~
J' ...~k,..7)
Reasons for arranging the equation in this form are I tp + ~t
as follows, 09 -~t--
1. The ~erm 162.6 (qB~/kh){log[(tp+~t)/4IJ
-0:~34 £, ( -3, 79~ lPp.c,.L2 / ~t p) ] I determines the,
position
flilidion ofis the middle-time
a constant; lane.
thus, it Note
affectsthat the the
only Ei FIg. 2.20- boundary,
Buildup lest graph lor well near reservo'Ir

position of the MTR and has no effect on slope.


2. At earliest shut-in times in a buildup test, Ei
(-3,7921Pp.c,L2/k~t) is negligible. Physically, this
mcan~ that the radius of investigation has not yet, .
cllcountered the no-now boundary and, math- d~llled well. ~o confirm this fault and to estimate
ematically, that the late-time region in the buildup distance from It, we run a pressure buildup test. Data
tc~tha~ not yet begun. from the test are given in Table 2.6. Well and
Thcse observations suggest a method for analyzing reservoir data include the following.
thc buildup test (Fig. 2.20): IP = 0.15,
I. Plotp"~vs.log(tJ!+~t)/~t. Jl = 0.6cp
2. Establi~hthemiddle-timeregion.
J. Extrapolatc thc MTR into the L TR, rCO =
( = t7x ft10-6 psi-I '
05
~. Tabulate the differences, Ap~'S' between the Aw: = 0:00545 sq ft,
bu~ldup c~rve and extrapolated MTR for several Po = 54.8 Ibm/cu ft,
p<.)lnt~(At)I':(=P":f-PMT). q = 1221 STB/D
5. Estimate L from the relationship implied by Eq. B: = 1:310 RB/STB, and
2,2ft: J1 = 8 ft.
.11':,:\ ~ 70.6 fill'l 1 I/I/lc',1,2)).llic
-1:.'i( --::_~~792 ~'cll prodllccd only oil I1lld di~~olvcd ga~. fkforc
kh k~t shut-In, a total of 14,206 SloB oil had bccn produccd.
Analysis of these data show that afternow distorts
, (2.28) none of the data recorded at shut-in times of I hour
,. i~ Ihc only unknown in this equation, so it can be or f!1°re. B?sed ?n the slope .(~50 psi/cycle) ef the
~ol\'cd dircctly. Remember, though, that accuracy of earliest straight II~e, p~rm:ablhty, k, a~pe~rs to be
Illi~ cqualion requires that ~t.cft .when this con- 30 md. Depth of Investigation at a shut-In tIme of I
dil iOiI is not satisficd, a compuf~r history match h~ur is 1.44ft: lending confidenc~ to ~hechoic~ of the
u~ing Eq. 2.25 in its complete form is required to middle-time line. Pseudoproduclng time, tp' IS279.2
detcrminc I~. hours.
111i~ calculation implicd in Eq. 2.28 should be From these data, determine whether the buildup
madc for several values of ~t. I f the apparent value testdata.indicate that the w~ll is beh,avingas if it were
of I. tcnds to increa~c or to decrease systematically near a single fault ~nd estImate distance to an ap-
with timc, thcrc is a strong indication that the model parent fault from buildup data at several times in the
doc!; nol dc~cribe thc reservoir adequately (i .e., the LTR.~-1\'c~I..i~
not .~having a~ if it ~ere in a rcscrvoirof S()luli()n.-Our attack is to plot/?~ VSi(/p+At)/~t;
unllmm thlckncss and porosity, and much nearer extrapolate the middle-time line on into the L TR;
oll,c.houndary t-,lan any others): , rcad pre~sures, PMT' from this extrapolated line;
IIIC fulluwlng cxamplc Illustratcs this com- sublraclthosc Prcssurcs from obscrved values of /J
.
pu tatlona I tecI1l1lque.
.M~ in the LTR (Ap~ =Pws -PMT); estimate values of
L from Eq. 2.28; and assume that, if calculated
E.\'Ol11ple 2.8 -Estinloting Distance va~uesof L ~re fairly constant, the well is indeed near
a single sealing fault.
toaNo-Flo},'BoUlldary From Fig. 2.21, we obtain the data in Table 2.7.
(Jr()I)frm. Geologists suspect a fault near a newly We now estimate L from Eq. 2,28. Note that

www.petroman.ir
PRESSURE BUILDUP TESTS 43

TABLE 2.7 -ANAL VSIS OF DATA FROM


3,792 4>,i.tC, = (3,792)(0.15)(0.6)( 1.7 x 10 -S) WELL NEAR BOUNDARY

k 30 ~t P..s PMT' (P..~ -PMT) = ~p:",


=1.934xI0-4. (hours) (t"t~tl/~t (psi) (psi) (PSI)

6 475 3.996 3.980 16


We first estimate L at At = 10 hours, which assumes 8 359 4.085 4.051 34
that the approximation I =1 + Al is adequate in 10 289 4.172 4,120 52
.P p. 12 243 4240 4,170 70
thts case. 14 209 4:298 4,210 88

B-3 792 <f>ilC L 2 16 18.5 4.353 4.250 103


Ap;"s = 52 = -70.6~ Ei ( ' ' ) 20 15.0 4,435 4,300 135
kh kAI 24 126 4.520 4.355 165
30 10.3 4,614 4,410 204
36 8 76 4,700 4,455 245
-(70.6)(1,221)(1.310)(0.6) 42 7.65 4,770 4,495 275
= 48 682 4,827 4,525 302
(30)(8) 54 6.17 4,882 4,552 330
60 565 4,931 4.578 353
2 66 523 4,975 4,600 375

( -3,792 q"i.tC(L ).
'Ei
kAt .
, I

-. ( -1.934x 10-4 L2
) -0.184.
-
EI " .
10 "
Sl~ .1300 poi"C~"'\v/,,1
""
2 (1.107)(10) 4 /
L = 4 =5.72xIO, .<1;

1.934x 10- C-

or I.n
~
L=239 ft.
-.11

For larger values of shut-in time, the approximation


I p = I p + At becomes decreasingly accurate, and no cycle

terms in Eq. 2.25 can be neglected, but L =240 ft

satisfies the equation for all values of shut-in time. KX) I


for the case In which the slope of the buildup test
has time to double, estimation of distance from well tpt ~ t

to boundary is easier. From the buildup tests plot, we -"Kt-

find the time, Al x' at which the two straight~line


sections intersect (Fig. 2.22). Gray 14 suggests that the Fig. 2.21 -Estimating distance to a no-flow boundary.

distance L from the well to the fault can be calculated


from

L=J~~~~~~~~!.!. (2.29)
ct>IJ.C,
In Fig. 2.21, the slope did double, and the figure
shows that (/p+Alx)/A/_,,=17, from which
AI x = 17.45 hours. Eq. 2.29 then shows that L = 225
ft, in reasonable agreement with our previous P ws

calculation.
The results of pressure buildup tests sometimes can lp+~lx I
be used to estimate reservoir size. The basic idea is to log A
compare average static reservoir pressure before and u t X
after production of a known quantity of fluid from a
closed, volumetric reservoir, with constant com-
pressibility, c,. If VR is the reservoir volume
(barrels), AN p is the stock-tank barrels of oil t + i1t
produced between Times I and 2, and PI and P2 are log p
the average reservoir pressures before and after oil i1t ,

production, then a material balance on the reservoir

shows that
(ANp) (Bo)
P2 -p I V Rc «f> ' Fig. 2.22 -Dislance to boundary f,om slope doubling.

www.petroman.ir ...
or pressed in thousands of standard cubic feet per day
(~N ) (8 ) (Mscf/D), and gas fC?rmation volume factor, Bg, is
J-'R= '~'-J1'-'~o'. ,..,..,.,.(2.30) then expressed in reservoir barrels per thousand
(p, ~P2)Ct(/) standard cubic feet (RD/M~cf), so that the product
q~ B~ is in reservoir barrels per day (RB/D) as in the
analogous equation for slightly compressible liquids.
Exailiple 2.9- Estimating Reservoir Size 2, ,A}I gas prope!ties (Bg, JJ.,and ~g) are evaluated
I'rcthltm. Two pressure buildup tests are run on the at original re~ervolr pressure, Pi. (More gener.ally,
only well in a closed reservoir. The first test indicates these pro:pertles shoul~ be eval~~t~d .at the uniform
an average pressure 0,f 3 000 pSI, . the Second I'nd,' cates pressure
E 2 31 In the reservoir before Initiation of flow.) In
2,1()() psi. The well produced an average or 150 q,.,
STB/lJ of oil in the year between tests. Average oil 8 -178.1 Zj Tpsc(RU/M f)
furlllationvolumcfaclor,Bo,isl,3RD/STD;total ~j- poT sc,
comprcssibility, ct' is 10 x 10 -6 psi -I; porosity, (/), I SC
i~ 22%; and avcragc ~and thicknc~s, h, i~ 10 fl. C.,j=C~i'\'R+CII.SII,+cf~C~j'\"~.
Eslilllalcarca,AR,ofthcrcscrvoirinacrcs. 3 Th f D .
.e actor IS a measure 0 f non- Oarcyor
Solution. From Eq. 2.30, turbulent pressure loss (i.e., a pressure drop in ad.
AN B dilion to that predicted by Darcy's law). It cannot be
VR = --.p-~o calculated separately from the skin factor from a
(p I -P2)C, (/) single buildup or drawdown test; thus, the concept of
apparent skin factor, S'=s+Dq", is sometimes
= (q/)B 0 convenient since it can be determined from a single
test.
(p I -P2 Xc t (/) For many cases at pressures below 2,CXX>
psi, flow
in an infinite acting reservoir can be modeled by
= (ISOSTB/O)(365daYS)(I.3RB/STB). 2 = .2+ 1,637q"Jl.iZ;!
(3,<XXJ-2,IOO)psi(IOx 10-6 psi-J)(0.22) Pwf P, kh
Thus,
J'H = 3S.9x 106 bbl ./
( 1,688 (/)JlCti) -<::!+Dqg )
].
llog kIp 1.151
= 43,560ARh , (2.32)
5.615 :
anI.! 6 superposition
Using these basic
to develop
drawdown
equations
equations,
describing
we can use
a ~
,.1 = (35.9 x 10 bbl)(5.615 cu ft/bbl) buildup test for gas wells.
H (10 ft)(43.56 x 10.1sq ft/acre) Forp> 3,000 psi,

=463acrcs. P,'s=p;-162.6'fg-glr-1
q 8 -JJ.. ,log( I'/1'+ ~I
-., )1 ..(2.33)
I kh ~I
2.11 Modificariol1s for Gases anI.! .
l11is scclioll prcscllts modifications of the basic (p -P )
ura\\uo\vn anI.! buildup equations so tha~ they can be s' =s+D(qg) = 1,ISll \PI hr -Pwfl
applicl.! to analysis of gas reservoirs. These 111
Illuuificalions are based on results obtained wilh the
ga~ pscudopressure.IS although a more complete k
uiscll~sionoflllatsubjcctislerltoChap.S. -Iog( ~)+3.231. (2.34)
Wattcnbarger and Ramey 16 have sho\vn that for (/)Jl.;Ct;II'
some gascs at pressures above 3,000 psi, flow in an For P < 2,000 psi,
in finitc-act illg reservoir can be modeled accurately by q Jl.-z. T
tile cqualion -P~I'S =pf -1,637 'fgr"'1 'log ( 'P
I +, -.,
~/ ) ...(2.35)
.kh ~I
Pll'r = P,+
-162.6QgBg;Jl.; [ 1.og ( 1,~88q,Jl.;~!;) and ---
.kh kIp 2 2
s' =s+D(q
g
l
) = 1.151 (PI hr -Pwf)
mN

__(S+DQg~', (2.31)
1.151 .k
This equation has the same form as the equation for -log( .£... -_2) + 3.23], (2.36)
a slightly compressible liquid, but there are some q,p.iCt;' w
important diffcrcnccs: where /11N is the slope of the plot p~ Ys.
I. Ga~ production rate, q~, is conveniently ex- log(tp+~t)/~/J,whichisl,637qgJl.;z;T/kh.

www.petroman.ir
An obvious question is, what technique shollld be c=(' S =(3.44x 10-4)(0.7)
used to analyze gas reservoirs with pressures in the II g/ g
range 2,(XX)<p<3,(XX) psi? One approach is to use =2.41 x 10-4 psi-I,
equalions writlen in terms of the ga~ pseudopre~~ure
inslead of either pressure or pressure squared. T!li~ is alld
at least somewhat inconvenient, so an alternalive,
approach is to use eqllalions wrillen in terms or s =s+D(qg)=1.151
I ~PI hrIII-PHi)
either Pws or P~s and accepl the resultant inac-
curacies, which, in real, heterogeneous reservoirs,
may be far from the most significant over-
simplification on which the te~t analy~is procedure is -log
( ~;7?k ) + 3.23 I
based. The smaller the pressure drawdown during Ihe / II H'
le~l, Ihe less the inaccuracy in this approach.
~(2,525 -1,801)
= 1.151t
Example 2. 10- Gas Well Buildup 81
Test Analysis ~

lest. Test data include the following.


l)rctltlem.Agaswclliss!luliI1roraprcssurchuildup l ?96.x 10 -4)(0.3)2
-lo' g (0. .18)(0.028)(2.41 I
qg
T =
= 5,256 Mscf/D,
181°F=64I°R, + 3...23J = 4 84

h = 28 ft, From results of the P~vsplot,


Ili = 0.028 cp, T
Sw = 0.3, k= I 637 q_~Jl.iZj.
~ = 0.18, , mwh

Zj = 0.85,
r w = 0.3 ft, (1,637)(5,256)(0.028)(0.85)(641) I
Cgj = 0.344 x ~O-J psi-I, = (4.8x 105)(28)
Pi = 2,906 psla, and
Pwf = 1,801 psia: -=9.77md,
Most of the test data fall in the intermediate pressure d
range, 2,(XX)<p<3,<XX> psia. On a plot of Pws vs.log an 2 2
(tp+At)/~t, t~e MTR had a slope, m, of 81 '=1151 [ (Plhr-Pwf) -IOg-~+3.23
psI/cycle; on this plot, PI hr was found to be 2,525 s. mw ~JI.-c
,I.r2.H
1
.. Alternatively, on a plot 0f Pws
psla.
2 I
vs. °g
/

(ti +~/)/~/, the MTR had a sloRe of 0.48x 10 6"


psl1/cycleandprhr of.7.29 x 106 psi2. = I 151f E.29x 10 -(1_,801)-]
From these data, estimate apparent values of k and .t 4.8 x 105
s' (I) based on characteristics of the P ws plot and (2)
based on characteristics of the P~s plot.
-10 I 9.77 --+ I 3..23!
.g (0.18)(0.028)(2.41 x 10-4)(0.3)2 ..
Solullon. From results of the Pws plot, for standard
conditionsofI4.7psiaand60°F, =4.27.

8 .=178.1~~ Neilll\.'r ~ct of r\.'slllts (k and ~") is ncc\.'~~arilymm\.'


gl Pi Tsc accurate than the other in the general case; as in t!lis
particular case: use of an analysis procedure based on
(178.1)(0.85)(64I)(J4.7) .~as.pseudopress~lre can be used 10 improv~ accura~y
= If dl~agreement In results from P..'s and PHosplots IS .,
(2,906)(520) unacceptably large. Iii

= 0.944 RB/Mscf, ...' !


~.~~-~-~ 2.12 Modifications for Multlphase Flow -1
i 1
B ..Uasi..: buildup and dra,,:down cqualions can be !
k= 162.6 qg-gIJl./ modified to model multlphase flow.17.18 For an
mh infinite-acting reservoir, the drawdown equationI
becomes

=p. + 162.6 ~ [IOg ( ! ,68~q,c,r~I:


= ~162.6)(5,256)(0.944)(0.028)
(8 J)(28)
p
wf, )., h
s 1
}., II )
.
,. --, (2.37)
=9.96 md, 1.151

www.petroman.ir
alld I Ilc buildup cljual iOll bccomc~
q Rt
p",~ =Pi -;- 162.6 -10g(.p I +, -.
AI E.\"a/l1p/e 2. II -;\-Ili/tip/lase Blii/dlfp
) .,.',., (2,38) Test Ana/ y sls
Ath AI
In the~e equations, tlte total nO\\' rale qRt is in P,roblem. A buildup t,estis run in a well that prod\ll:t'\
rc~cr\'oir barrels per day (neglecting solulion gas Oll~water, an~ gas simultaneously, Well, rock, an,d
liberated from produced water) nuld propertIes evaluated at average reservoIr
R ' pressure during the test include the following.

l/Rt =l/II/l'1 +(l/.1' -.~:~- )8.1' +(111,1111" SO = 0.58,


5,f = 0,08,
,..,." (2.39) 5",=0.34,
36 10 -6 '-I
I b . I '~. c",=.x pSI,
.1
allutola
,. mo I Ily, "t,IS 35
cf = ,x 10 -6 pSI, '-I
-A:!I ~-". A:8 C = 0.39 x 10 -3 psi -1 ,
A, -+ + ,. ., ..., .., (2.40) ,f
IL" ILII' It.1' It,} = 1,5cp,
.T(ltalcomprcssibility,ct,wasdcfincdinEq.I.4. 11."' = 0.7cp,
"llc~c C4lmtion~ imply that it i~ po~~iolc to It" = 0.03 cp,
uctcrmillc A, from the ~Iopc III of a buildup tc~t run B() = 1.3 R8/STB,
011 a \\'cll that prodllcc~ two or tllrcc pha~c~ 1111'= 1.02 RB/STB,
~imultallcoll~ly: B.1' = 1.4ROR8/M~cf,
q H, R~ = 685 scf/STB,
A,=162.6-. Illir ,...,...(2.41) ~-
'#' -., 017
Perrlne
.17 1las s110wn t I lat It IS aIso pOSSI .bl e to.. '" ' = 0.3ft,and
c~(imate thc permeability to eoch pl,ose nowing from h = 38 ft.
the samc slope, ",: From plots of Bo vs. p and Rs vs. p at avcra~t'
q B II. pressure in the buildup test,
kn=162.6 (} () (I, (2.42) dR
",1, -:! =0.0776 scf/STB/psi,
R lip
(q~- -~
q(} s ) B~II.~ an
d -
k.e = 162.6
",1,
, (2.43) dBo
-=
I 6 R
2.48 x 0 -B/STO/psl.
.
dp
alld The production rates prior to the buildup tc~t \Vcrc
B qo = 245 STB/D, q", = 38 STO/D, and q" = 4R9
"- q", 1I,/tll' Mscf/D.
A:1I,-162.6. ...,.., (2,44)
",1, A plot of P"'S vs. log (I l' + At) / ~t shows tltat tht'
.nlc term «(I -(loR~/ I,OOO)B~, wltich appcar~ in slope of tlte ~TR, "'~ IS 78 psI/cycle and tl~:lt
1:4~, 2..19 anJ 2.4.1, i~ (lte jrc'£' ga~ now ratc in tlte 1'1 hr = 2,466,psla. FlowIng .pressure,P'vf' at thc In.
rt"'cr\'oir. It i~ f()llnd oy suotract ill!! thc di."s()I\.cd!!a~ sta~lt of shut-In wa~2,~28 psla.
ratc «(I"/?\/I,()()(») fronl tltc total ~urfacc ga~ ratc l'romtltc~cdata,cstlmatcA"ko.k""k-i,and\",
«(IE,)and con\'crting t? a reservoir-condition ba~i~. Sfllution. Permeabilities to each phase can hl'
Slmullancou~ ~olutlon of Eq~, 2.37 and 2.38 re- determined from tlte slope ", of the MTR:
"..Its in the follo\\lng expre~sioll for tlte skin factor s.
- ,~-log(-~)+3.23,
q0 8 011.0
...=1.151 /" Ihr , A I k(} =162.6 ",II
III 1/>£." II'
(162.6)(245)(1.5)( 1.3)
(2.45) ==26.2md,
(78)(38)

Stat ic drainage-area pressure, p, is calculated just q",8".11.'"


a~ for a single-pl\a~e reservoir. In use of the MBH kw=162.6 h
charts to determine p (and in the Horner plot itsel0, ",
thc effective production time tp is best e~timated by
dividing cumulative oil prOduction by the oil (162.6)(38)(0.7)(1.02)
prodllction rate just before shut-in. = 78 38 = J.49 md,
An important. assumption required for accurate ( )( )
use of these equations for multiphase now analysis is q R
that saturations of each phase remain essentially (q" --fiiii )BgII.g
uniform Ihroughout the drainage area of the tested k = 162.6 '
\vcll. " ",h

www.petroman.ir
PRESSURE BUILDUP TESTS -47

I489 (245)(6X5) (1.480)(0.03)I 1;~"


.,'~ r " .I""":
, ,
= 162.6 (1,000) 2. I. In Examplc 2. I, ~'hat error arises h\.'I:"II~C:\\'\.'
(78)(38) II~cd Eq, 2.4 to call:ulate skin'fador il1stc"d or thc
= 0 782 md more e,\"ct Eq. 2.3? What dirfercnce would it Ilavc:
..made in the value of s had we used a shut-in timc: or
To calculate total mobilit A we first need lotal 10 hours in Eq. 2.3. and lIte corre~ponJing v~lue or
now rale. y, I' P\l'~? What assllmptlon have we made about dlslanl:c
, q RI' rrom lested well 10 reservoir boundaric:s in Exampll.'
= B + (
-(} q RS )B + _..
') I ?

qRr qo 0 qg I (XX) g q..,BI" 2.2. I)rove thallhe slope of a plol of shut-in HI IP


, vs. log (/,,+.1/)/011 is, as asserted inlhe lext, Illc
difference in pressure al two points one cycle apart,
= (245)( 1.3) + [489 -(245)(685) ] Also prove that, for .11 cC1P' w~ obtain Ihe same slope
1,<XX> on a plot ofPM'S vs. log dr. Finally, prove that on a
plot of Ph'S vs. log ~/, we obtain the same slope
.(1.480)+(38)(1.02) regardless. of Ihe units used for shut-in t!me, .~, on
the plot (I.e., that ~I can be expresscd In mInutes,
= 833 RB/D. hours, or days \vithout affecting the slope of Ihl.'
plOI),
Then, 2.3. A well produl:ing only oil and dissolved gas
has produced 12,173 STD. The well has not been
A = 162 6 ~ = ~~~~ stimul"tcd, nor is there any reason to believe that
I 'mh (78)(38) thl'rc: is a signifil:ant amount of formation damagc. A
pressure buildup test is run with lIte primary ob-
=45.7 md/cp. jeclive of estimating static drainage-area pressure. I
During buildup, there is a rising liquid level in the
To calculate skin factor, S, we first need Co and C,: wellbore. Well and reservoir data are:
-~ dRs .I dBo cf>= 0.14,
c-0 Bo dp B dp Jl. = 0.55 CPt
0
-CI = 16 X 10 -6 pSI,
'-1
r w = 0.5 ft,
= (1.480 RB/Mscf)(0.0776 scf) A wb = 0.0218 sq ft, i
(1.3 RB/STB)(STB-psi) r e = 1.320 ft (well centered in cylindrical I
drainage area), !
p = 54.8 Ibm/cu ft, '
.1 Mscf -(2.48 x 10-6) q = 988 STB/D. 1
1,<XX>scf 1.3 (psi) B=I.126RB/STB,and I
h = 7 ft. '
= 86.4 x 10 -6 psi -I.
Data recorded during the buildup test are given in
Then. Table 2.8. Plot PI"S vs. (lp+.1/)/~1 on semilog !
c =5
r oCo
+5
gCg
+5 +
I"CW Cf
= (0.58)(86.4 x 10 -6) + (0.08)(0.39 x 10 -3)
paper and (PI"S -PI'f) vs. .1/t' on log-log paper-yand
estima!e the time at \vhich aflerno\v ceased distorting
the buildup t~st data.. ..
I
2.4. Consider the buildup test described In
+(0.34)(3.6x 10-6)+3.5x 10-6 Pro?lem 2.3. ~~I:ate the MTR and estimate for-
mation permeability.
= 860 x 10 -6 .-t 2.5. Consider the buildup test described in
.pSI, Problems 2.3 and 2.4. Calculate skin factor, s;
and pressure drop across the altered zone. (i1p)s; now

S=I.151 [ Pthr-Pwf_1
m
(
og ~.
AI )
I III
323 1
+ .single
efficiency. E; and effective wellbore radius. r wu.
.2.6. Provethatinab.uilduptestfora.wellneara
fault, the technique suggested In the text
(extrapolating (he rate-time line to infinite shut-in
I
-

time) is the proper method for estimating original


= 1.151\ 2,466 -2,028 re~e~voir pressu~e. Comment o~ the p~ssible errors in
l 78 original reservoir pressureestimatesIn thesecases:
(I) some LTR data were obtained, but final straight
line was not established; and (2) no L IR data were
-log [ (0.17)(86.0 45.7
x 10 -6)(0.3)2 ] + 3.23 J . dConsider the buildup lest described In
0 b2.7.
talne.

Problems 2.3 and 2.4. Estimate static drainage-area .;


= 1.50. pressure for this well (I) Iising thep. method. and (2)

www.petroman.ir ..
.TABLE 2.8 -PRESSURE BUILDUP TEST DATA

~t P., ~t Pw. TABLE2.9-BUILDUPTESTDATA


(hours) (pSla) (hours) (pSia) FORWELL NEAR FAULT
0 709 19.7 4,198
197 3,169 246 4.245 ~I Pw~ ~t Pw~
2.95 3,508 296 4,279 (hours) (pSI) (hours) (pSI)
394 3,672 345 4,306 20 1.373 500 2,225
492 3,772 39.4 4.327 30 1,467 ROO 2,360
591 3.813 -144 -1,343 -10 1,533 1,000 2,434
788 3,963 49.3 4,356 50 1,585 1,500 2,545
9.86 4,026 59,1 4,375 100 1,752 2,000 2,616
14.8 4,133 200 1,940

USilig tIle mouificu Muskal melhod, R~ = 748 scf/STB,


2.8, In Exalilple 2,7, explain how we could ha\'e It> = 0,18,
applied the modified ~1uskat method to estimate r M' = 0,3 ft, and
SIalic drainage-area pressure if we had not had II = 33 fl,
I eslimalesofk/lt>Jl.c, orrt" "
2 9 E . f . b 'l ' d k .2,11, A pressure buildup test was run on an 011
i "stlmate ormation permea I Ity an s In. ",
,I r t
ac or
f
rom t
h
e
f II
0 owIng
' d
ala aval
' I bl
a e
f
rom a gas,
well believed
"
to be near
"
a sealing fault In
,
an otherwIse
'
we
II
pressure
b
UI
' Id
up les
I
,
Infinite-acting
f I .
all I, given t e we ,roc, an
h
reservoIr,
II k
EstImate
d n UI'd
the dIstance
'
propertIes
to

below
the

T = 199°F=659°R, anc.JthebuildupdatainTable2,9.
II = 34 ft,
Jl.i = 0.023 cp, q = 940 STB/D,
Sit' = 0.33 (water is immobile), Jl. = 50 cp,
('!!; = 0.000315 psi -t , It> = 0,2,
It> = 0.22, CI = 78xl0-6 psi-I,
z; = 0.87, and h = 195 ft,
rlt, = 0.3 ft. P; = 2,945 p~i, .
i .. A Np = 84,500 51'0,
1hc wcll produccu 6,068 Mcl/O bcfore the te~l. /1 = I,ll RB/STII, anu
plot ()f IIIIIJ, PIt~' v~, log (I'I+~/)/~I gavc a I (I < 20 hours
miuulc-time line with a slope of 66 psi/cycle. ,,'bs .

An~lysis of the buildup. curve showed. that static 2.12. ' A well nowed for 10 days at 350 STB/D; it
uralnage-~rea p~essur,e, p, was 3,171 psla. Pressure was then shut in for a pressure buildup test. Rock,
ou thc mluulc-tlme line at ~I = I hour, PI hr' wa~ " nuid and well properties include the following
2,745 psia; nowing pressure al shut-in, Plti' wa~ ' ,
2,4R6p~ia. Bo = 1.13 RB/STB,
2,10, Estimate total mobility, X" oil, water anu P; = 3,000 psi,
ga~ pcrmeabilities, alld skin factor for a well thaI Jl. = 0,5 cp,
proullceu oil, waler, alld ga.~simultaneously before a k = 25 md,
press\lrc buildup lest, Production rates before the S = 0,
test wcre qo=276 STB/D, qlt,=68 STB/O, and II = 50ft, -6 '-I
({,f =689 Mcf/O. A plot ofplt'.~ VS, log (I" +~/)/~I c, = 20x 10 pSI,
sho,,'cu tllal the slope m of tile middle-time line was It> = 0,16, and ..
~9 psi/cycle, Flowing prcssllre at shill-in, Plti' wa~ r It' = 0.333 ft.
1,5RI p~ia; Illc prC~Sllrc 011 thc middlc-liltlc slrniglll ) De ' d I h d ' t ' b ' ,

lillcal~/=lhour,Ptllr.'\a~I,744psia,PlotsofB(} (a te~mlnean P,otl, epressure ISri utlonln

V~. P and R~ VS, P ~ho"cu thaI dR~/dp=0.263 Ihe reservOIr for ~h~I-!n tln~es of 0,0:1, I, and 10
~cf/S1'B/psi ' and that dB(}/dp=O,248 x 10-" uaY~,(~~Sllmeanlllflnll.eactlll,grese~VOI!,)
RO/51'0/psi, Rock, nuid, anu well properties in- (b) Calculate the radius ?f Investigation al ~,I, I,
cludc the following, and 1.0days, Compare ri with the depth to which the
tranSient appears to have moved on the plots
'~f) = 0.56, prepared in Part a,
'~.l' = 0.09, 2,13, In Example 2,6, jJ was determined to be
")'It' = 0,35, 4,411 psi, Bolh the Horner plot and the abscissa of
('It. = 3.5 x 10 -6 p~i -I, -~ tlJe MOH chart used tp = 13,630 hours. It can be
,. c{ = 3.5 x 10 -6 psi-I, shown that for a well centered in a square drainage
c'!! = 0.48 x 10 -3 psi -I , area, the time required to reach semisleady state is
1«(1 = 1.lcp, tp.t~=(It>p.cIA/O.000264 k)(IDA)p.u and that
It", = 0.6 cp, (' /)/1) s = 0.1. Show that if tp,t\' is used instead of t p
It.~ = 0.026 cp, in boththe Horner plot and in the abscissa of tne
Bf) = 1.28 R8/STB, MBH chart, the resulting estimate of fi is essentially
Bit, = 1.022 RB/STB, unchanged, Buildup data (from the MTR only) are
IJ" = 1,122 R B/M~cf, given in Table 2.10. Other data include:

r www.petroman.ir
.-

TABLE 210 -MTR DATA TABLE 2.11 -PRESSURE BUILDUP TEST DATA
FROM BUILDUPTEST
~, P.. ~, P..
~, P..s (hours) (psla) (hours) (psia)
(hours) (psi) 0 2.752 10 4.272
8 4.354 0.3 3.464 12 4.280
12 4,366 0.5 3,640 14 4.287
16 4,376 1 3,852 16 4,297
20 4,382 2 4,055 20 4,303
24 4,388 3 4.153 24 4,308
4 4,207 30 4.313
5 4.244 36 4,317
6 4.251 42 4.320
8 4.263 50 4,322

q = 250 STB/D presented al the SPE-AIME 491h Annual Fall ~1"'l.'ling,


8 = 1.136 RB/STO, I !~U!oI.~I.I,
OCI.,6-9, 1974.An ~~~idgl.'dvcr!oi(~n appt."ilr!oin J.
-08 '(/. ILL". (Aug. 1975)991-()(XI,Irultl., AI~1I::,25'.
II. -.cp, 3. ~1illl.'r, C.C., Dyc:s,A.H., anJ Hutchinson, C.A. Jr.:
h = 69 ft, "Eslimation of PermeabilityanJ Reservoir Pressurc From
ct>= 0.039, Boltom-flole Prc:ssure8uilJ-Up Chara(.1eristics," TrulIs.,
CI
r = 17xI0-6psi-I,
I 320 fl d 4. ~~ME(195~)I~~,91.-.I~
SIIJl.'r, 11.( ,: A SlIllplllll.'ll ~11..lllllll lIt I'rl.'!o~url.' 1111/111111

k -7'65 d' an Anilly)i~ fur iI Slubilii.I.'J WI.'II," J. P...I. Tet.h. (Sc:pt. 1971)
-.m, 1155-11fX>; Trulls.,AI~1E,271.
5. Agar,,'al, R.G.: "A N.:w ~1I.'thL~ To Al.'l.'ountfor PrtXlul.'ing-
2.14. A well producing only oil and dissolVl.'d gas Ti/llC l:ffcl.'l) Whl.'n l)rilw"Jo\\n IYJ'l(.'(,Ilrv(.~ Arc lI!o(.."J1(1
has produce.d 13,220 STH. To chara~lenze
.Analyze
t~e severe Prc))ure
9289presenled Buildup
atlhe anJ Olh.:r
SPE551h AnnualTc)1 Data," Conf.:rcnl.'c
Te\:hnical paJ'l(.'r
SPE
damage believed present, the well IS shut In for a and Exhibition, Dallas,Sept.21-24,1980.
buildup test. Well and reservoir data are given below. 6. Saidikowski,R.M.: "Numerical Simulationsof the Combined
ct>= 0.17, Effects
SPE 8204of presenled
Wellbore Damage and Partial
atlhe SPE-AIME 54th Penetration," pa()t:r
Annual Te\:hnicul
II. = 0.6 Cp, Conferenceand Exhibition,Las Vegas,Sepc.23-26,1979.
CI = 18 X 10 -6 psi -I 7. .Russell.,
D.G. and Truitl, N.E.:."~~ansicntPressureBc:hilvior
r~ = 1 "320 ft well cenle red in Squa r e draina ge In Vertically
-1159-1170; FracluredRe~rvolrs, J. Pet. Tn.h. (Dl:t. 1%4)
Trans.,AIME,23I.
area (160 acres), 8. Malthews, C.S., Bron), F., and Hazebroek,P.: "A MelhoJ
rw = 0,5 ft, for Determination of Average Pressure in a BounJ(.oJ
A b = 0.036 sq ft, Reservoir," Trans.,AIME (1954)201, 182-191.
;0 = 54.8 Ibm/cu ft, 9. Larson~ V.C.: "Un~erslanding t~e ~Iuskal Method .of
q = I , 135 STB/D ( sta b.Ilze
l. d for severa I d ays,) Analyzmg PressureBullJup Curves, J. Cdn. Pet. Tech.(I'all
1963)2,136-141.
B = 1.214 bbl/STB, and 10. Matlhews, C.S. and Russell,D.G.: Pressur~ Buildllp Ulld
h = 28 ft. Flclw Testsill "Ielb, MollogrilphSeries,SPE. Dalla) (1967)I.
II. Pinson,A.E. Jr.: "Concerningth.: Value of Producing Tim.:
When the well was shut in for the buildup test, the Used in Average PressureDelerminations From Pressure
liquid level rose in the wellbore as pressure increased. BuildupAnalysis,"J. Pel. Tech.(Nov. 1972)1369-1370.
Data recorded during the buildup test are given in 12. Brons, F. and Miller, W.C.: "A Simple Melhod for
Table 2 II Correcting SPOI PressureReadings," J. Pet. Te(.h. (Allg.
..' ..'. 1961)803-805; TruIIs.,AIME,222. ..
DetermIne (a) time at which aftertlow distortIon 13. Earlougher, R.C. Jr.: Adl'ullce.l' ill Il'ell Test AII(/ly~is.,
ceased; (b) time at which boundary effects begin; (c) Monograph St:rie),SPE,Dilllas(1977)s.
formation permeability; (d) radius of invesligalion al 14. (ir:lY, K.E.: "Appro\i/llilling WclI-to-I';11I11 I)isl:lnl.'c I'rll/ll
bl.'ginning and end of MTR; (I.') ~kin factor, ~P.l" and Prl.')sll~':1.~lIillllipI~~I),','J. Prl. I.'ch. (J!lly ..%5) 761-7.6.~..
..
fl 0\\ . e ffi Iclency, (
f) -.
p using t
h
e
MBH
p
.
met
h
0
d '
,an
d 15. AI-Hussamy,
Flo\\. of Real
R., Raml.'),
Gases Through
H.J. Jr., and
Porolls
Cra\\forJ,
Media " J.
P.U..
P...I.
Tilt:
Tech.
(g) p using the modified Muskat method. (~1ay1966)624-636;Trulls.,AI ME, 237. '
16. Wattenbarger, R.A., Ramey,H.J. Jr.: "Gas Well Testing
References With Turbulence, Damage,and Wellbore Storage," J. Pel.
Tech.(Aug. 1968)877-887;Trulls.,AI~1E,243.
,. Horner, D.R.: "Pressure Buildup in Wl.'lIs," PrO(.., ThirJ 17. Perrine, R.L.: ,. Analysisof PressureHuilJup Curves," Drill.
World Pet. Cong., The Hague(195I) Sec. II, 503-523;also ulld Prod. Pruc., API, Dallas(1956)482-509.
Pressure Analysis Methods, Reprint Series, SPE, Dallas 18. Murtin, J.C.: "Simplilil.'<! Equalionsof Flow in Gas Drive
(1967)9, 25-43. Reservcirs and the Theoretical Foundalion of Muliipha)e
.2. Cobb, W.M. and Smith, J.T.: "An Invesligationof Pressure- PressureBuildup Analyses,"J. Pel. Tech.(Oct. 1959) 309-
Buildup Tests in Bounded Reservoirs," paper SPE 5133 311; Trans.,AI ME, 216.

www.petroman.ir ,
'\
, \

Chapter
3 ~ "\"\:
FlowTests ~~ ",

~ o 'J

-~
, i

~.\ '.'
-' '

\.-
~. \
-q
3.1 Introduction
This chapter discusses now tests in wells, including estimated by qualitative comparison of a log-log plot -
constanl-rale drawdown lesls, continuously of (P;-Pwf) vs. t wilh lhe solulion of Fig. 1.6 or
dcclining-rale drawdown Icsls, and mulliratc tcsls in with the empirical equation based on that figure,
infinite-acting reservoirs. T~e more g~neral (and t 2::(60+3.5s) C (1.43) I
more complex) case of multlrate tests In bounded D sD' ,
rescrvoirs is discussed in Appendix E. or the equivalent form,
3.2 Pressure Drawdown Tests -(200, <XX>
+ 12.<XX>s)Cs I
t"'bs- (3.2)
A pressure drawdown test is conducted by producing kh/p.
a well, starting ideally with uniform pressure in the If the effective radius of the zone of altered per-
reservoir. Rate and pressure are recorded as func- meability is unusually large (e.g., in a hydraulically
tions of time. fractured well), the duration of the ETR may depend
The objeclives of a drawdown test usually include on the time required for the radius of investigalion to
estimates of permeability, skin factor, and, on oc- exceed the fracture half-length. (More exactly, for an
casion, reservoir volume, These tests are particularly infinile-conductivity vertical fracture with half- #
applicable to (I) new wells, (2) wells that have been length Lf' shut-in time must exceed 1,260 ct>p.c(L}/k
shul in sufficiently long to allow the pressure to or r; must exceed 1.15Lf'
slabilize, and (3) wells in which loss of revenue in- The MTR begins when the ETR ends (unless
curred in a buildup tesl would be difficult to accept. boundaries or important heterogeneities are i
Exploratory wells are frequent candidates for lenglhy unusually near the well). In the MTR, a plot ofPwf .
drawdown tesls, with a common objective of vs. log t is a straight line wilh slope, m, give.p by
dclcnllinillg minimum or lolal volume being draincd qBp. ;
byAn
the idealized
well. constanl-rale drawdown tesl in an 111=162.6-.kh (3.3) 1
!

infinite-acling reservoir is modeled by the logarilh- Thus, effective formalion permeability, k, can be
mic approximalion to the Ei-function solulion: estimaled from this slope: ,
l
i

Pwj=p;+162.6-log
qBIL ( 1,688ct>lLc,r~) k=162.6-qBp.
h (3.4)
j

kh kt m
J After lhe MTR is identified, skin factor, s, can be
-0.869s. (3.1) determined. The usual equation results from solving I

Like buildup tesls, drawdown tests are more Eq. 3.1 for s. Setling t = I hour, and letling P~ =
complex thanusual
Eq. 3.1. The suggested byan
test has simple
ETR,equations
an MTR, such as
and an PI hr be
time, thethe pressure
result is on the MTR line at I-hour flow J
'
LTR. The ETR usually is dominated by wellbore i
[ (P;-Plhr)-IOg ( k
unloading: the rate at which nui~ is re~oved from
the wellbore exceedsthe rate at which fluid enters the
S=I.151
m ~~
)+3.23].:
wellbore until, finally, equilibrium is established.
Until that time. the constant now rate at the sand face (3.5)
required by Eq. 3.1 is not achieved. and the straight-
line plot of Pwj vs. log t suggestedby Eq. 3.1 is not The LTR begins when the radius of investigation:
achicved. Duration of wellbore unloading can be reaches a portion of the reservoir innuenced by

www.petroman.ir
FLOWTESTS -51

TABLE3.1 -CONSTANT.RATEDRAWDOWNTESTDATA
P,-Pwl P, -Pwl P,-Pwl
~(~?~r~) Pwl (psia) (psia) t (hours) Pwl(psia) (PSIa) t (hours) Pwl(psial (psla)
0 4,412 0 144 3,573 839 891 3.515 897
0.12 3,812 600 17.3 3,567 845 107 3,509 903
1.94 3,699 713 20.7 3,561 851 128 3,503 909
2.79 3,653 759 24.9 3,555 857 154 3.497 915
4.01 3,636 776 298 3,549 863 185 3,490 922
4.82 3,616 796 35.8 3,544 868 222 3.481 931
5.78 3,607 805 430 3,537 875 266 3.472 940
6.94 3.600 812 51.5 3.532 880 319 3,460 952
8.32 3.593 819 61.8 3.526 886 383 3,446 966
9.99 3,586 826 74.2 3,521 891 460 3,429 983

reservoir boundaries or massive heterogeneities. For


a well centered in a square or circular drainage area,
this occurs at a time given approximately by ...

.1(1=
380 <PJlC
k'
,A (3.6) F? ETR
where A is the drainage area of the tested well. for wf R
more general drainage-area shapes, 1(( can be
calculated from the number in the column "Use
Infinite System Solution With Less Than I 0/0Error
for 'DA <" in Table 1.2. I The dimensionless time
I DA is defined as I0 9 t I
0.CXX>264 kl
r 'DA=. <PJlC
,A Fig. 3.1-Typical constant.ratedrawdowntest graph.

For this more general case. then.

I 1(1= 3.800 <PJlCIAIDA


k-. (3.7)
Thus.
has the the typical
shape constant-rate
shown draw
in Fig. 3.1. Todown test plot
analyze the 0' ,' k

typical test. the. following steps are sug~este~. .~ ;


1. Plot flowIng BHP. PwJ. vs. flowIng tIme. " on .\
semilog paper as shown in Fig. 3.1. ~
2. Estimate I wbs from qualitative curve matching 0.:
marks athe
(with beginning
full-size of the
version of MTR (except
Fig. 1.6); thisforusually
frac- .

tured wells).
3. Estimate the beginning of the L TR, 1((. ll~ing )4 '
deviation from a match with Fig. 1.6 to confirm I C ~
deviation from aninapparent
mu~t be calltiou~ semilog
dr",wdowl1 straight line.
tc~t aI1alysi~, We
though. R-ONING TIME. hr

Even small rate changes can causea drawdown curve Fig. 3.2-Semilog graph of exampleconstant-rate
to bend just as boundaries do (a method of analyzing drawdowntest.
this possibility is presented later).
4. Determine the slope In of the most probable
MTR. and estimate formation permeability from Eq.

i 3.4.
-_s. Estimate the skin factorsfromEq. 3.S..~
~
a: ~~===?-
.:: ",
-.-
.
I .u.x.--
Example 3.1- Constant-Rate Drawdown Q.-
TestAnalysis .
Problem. The data in Table 3.1 were recorded during 1.-
a constant-~ate p:res~uredra.wdown test. The wellbore l .t1'
.had a failIng liquid/gas Interface throughout the
drawdown test. Other pertinent data include the Fig. 3.3-Log.log graph of exampleconslant.rate
I
'

following. drawdowntest.
II

www.petroman.ir
I
q = 250 STB/D, At the end of the MTR (I = I SOhours),
B = 1.136 bbI/STB, -~ 4 --
p. = 0.8 cp, ri -(1.521 X 10 )(150)l
r", = 0.198ft,I
II = 69ft, =I,510ft.
!/J = 0.039, and
c, = 17 x 10 -6 psi -I .A substantial amount of formation has been sam-
pled; thus, we can be more confident that the pcr-
The tubing areas is 0.0218 sq ft; the density of the meability of 7.65 md is representative.
liquid in the well bore is 53 Ibm/cu ft. Determine the We next calculate the skin factor s.
fonnation permeability and skin factor.
Sctl"liu~. We first plot flowillg UI~P'Pwf' vs. tilllc, t, .\'= 1.151
I Pi -P,m hr
-log ( ;j;~k ) +3.23 ]
on semllog paper and (Pi -Pwf) vs. I on log-log
paper. Then we determine when wellbore effects
ceas.eddistorting the curve. From the shape of the -I [ 4,412 -3,652
semllog grapll (Fig. 3.2), Ihis appears 10 be 31 abolll -.151. 70
12 hours; however, we can check this assumption
with Ihe log-log graph, Fig. 3.3. For several values of
CD (e.g., 103 to 104), the graph shows a good fit (1.442x 107)
with Fig. 1.6 for s = 5; wellbore storage distortion -log (0.198)2 + 3.23 ]
end~ at ~l = 5 hours, in approximate agreement with
the more sellsitive semilog graph. = 6.37.
We have no information about the location of
bolllldarics; therefore, we assume that boundary We now can verify more closely the expected end of
effects begin when the drawdown curve begins to wellbore storage distortion from Eq. 3.2, using
deviate from the established straight line on the 25 65 A
semilog graph at a flowing time of 150 hours. This is Cs.:' K'b
confirmed qualitatively on the less sensitive log-log P
graph by noticeable deviation beginning at t.: 260
hours. The slope of the middle-time line is =0.0106 bbl/psi.
", = 3,652 -J,5R2
--7
0
-pSI
/ . I
cyc e.
-(200, (XX)+ 12,(XX)s)Cs
t K'bs-
kh/1I.
Thus, the permeability of the formation is

k= 162.6~ [200, <XX>


+ (12,<XX»(6.37»)(0.0106)
mh = (7.65)(69)10.8

= (162.6)(250)( 1.136)(0.8) = 4.44 hours.

(70)(69) This closely agrees with the result from the log-log
= 7.65 md. curve fit.
..
We now check the radius of invest~gation at the Another use of drawdown tests is to estimate
beginning and end of the apparent middle-time line reservoir pore volume, VP' This is possible when the
10 ensure that we are sampling a representative radius of investigation reaches all boundaries during
portion of the formation. a test so that pseudosteady-stale flow is achieved.
At the begillning (t = 12hours), Eqs. 1.12 and 1.13 showed that, in pseudosteady-
slate flow, flowing BHP, Pwf' is related linearly lo
--~ -(7.65) time and that the rate of change in Pwf with time is
948 !/Jp.c,-(948)(0.039)(0.8)( I. 7 x 10-s) related to the reservoir pore volume. From Eq. 1.13,
this relationship is
= 1.521 x 104, -O.234qB
Vp= ,
,( ~ot )
and, from Eq. 1.23, c

I kl where oPwj/ol is simply the slope of the straight-line


r; =...Jii4i ~ Pwl vs. 1 plot on ordinary Cartesian graph paper.
t .Even though Eqs. 1.12 and 1.13 were derived for a
=..J(1-.S-21X1.04)(12) cylindrical reservoir centered at the wellbore of the
tested well, the principles derived from them apply to
= 427 ft. all closed reservoir shapes. The graph of P wfvs. t is a

www.petroman.ir ~.-
FLOW TESTS 53

Ihe te)1-re)ulls oblained using Ihose Icchnique~ l.:al1


lead to inlerprelations thai are seriously in error. An
, analysis mer hod I hal leads 10 proper inlerprelat ion is
available, bur il can be used only if Ihe produl.:ing
rare is changing slol~'I_vand sl1/ooth~v. Abrupi rare
.V) c/1angl"swill make Ihe drawdowllle~1 uilla impos~iblc:
~ to inlerpret using either the method discussed earlier
-or this new method.-
~ Winestock and Colpitls2 show thai when rare is
changing slowly and smoolhly, the equal ion
modeling the MTR of the drawdown test becomes

0 --.iCiI- ~ = 162.6~ [IOg( ~~~t~~)

FLONlt'IK:; TIME, hr .

Fig. 3.4-Cartesian-coordinate graph of example + 0.869 s] + negligible terms2. ..(3.8)


conslant-rate drawdown test.
The analysis technique is to plot (p; -P wI) / q vs. 1
slraighl line once pseudosleady-stale now is on semilog paper;-- identify the middle-time straight
achieved; the volume of the reservoir can be found line; measure the slope Ill' in psi/STB/D/cycle;
from Eq. 1.13. It is important to remember, calculate kh from
however, that these equations apply only to closed, p.B
or volumetric, reservoirs (i.e., they are not valid if kh= 162.6 -; ,
Ihere is water innux or gas-cap expansion). Further, ",
they are limited to reservoirs in which total com- and, finally, calculates from
pressibility,of Ct'
dependent is constant (and, specifically, in-
pressure). s=).151 I W )
[( P;-PWj -;-I
We will illustrate pore-volume estimates with an q t hr m
example.
-log ( ..L..-k _2 ) +3.23. J .,- (3.9)
Example3.2-Estimation of Pore Volume cpp.Ctrw
P~oblem. Estimate the pore v.olume of the reservoir In Eq. 3.8, [(Pj-Pwj)/q]1 hr is the value of this
wIth drawdown data reported In Example 3.1. quantity on the middle-time line or its extrapolation
Solution. The first step is to plot Pw vs. t (Fig. 3.4). at a now~ng ~ime of I hour.. .
The slope of this curve is constant ~r t> 130 hours; We wIll Illustrate use of thIs method wIth an
Ihis slope, OPwj/at, is example.

~ -3,531-3,420
at -0-500 Example 3.3-Analysis of Draw down Test
- 0 222 .JVith Varying Rate ..
--.psI/hr. . T bl 2 b
Problem. The data In a e 3. were 0 lame In a
d. .
Thus urawdown lesl in which the rare q w.tS measllrcd as ..
,
0 23 function of lime. Other data include the following
-.4qB
Vp= 0 B = ).) 36 bbl/STB,
('t(~) p.=-0.8cp,
01 h = 69 ft,
p = 53 Ib/cu ft,
(-0.234)(250)(1.136) Awb = 0.02)8 sq ft.
= (1 7 )0 -5 )( 0 222) cP= 0.039.
.x-. ('t = ) 7 x 10 -6 psi -I .and
rw = 0.198 ft.
= 17.61 X 106 cu ft Determine formation permeability and skin factor.
Solution. We note immediately that conventional
= 3.14 X 106 res bbl. drawdown test analysis. using an average rate, would
.
.Verificalion 01 this method,s incomplete In cases with severe wetlbore
The method of P ermeability determination slorage ellects. A nonexhaustive numerical simulation study by this author ~s
..slluwn thai Ille method Yluills essenllatly c(J{recl permcabillty ancl skin I~clor
out lined above' applies only to drawdown tests e."n "!IOOse c~ses
d ted t .. , . t t If t varies '.'1... same all41ysil lechrnque. bul lor a IIllIsrenl application lan~tYllng
con UC a a strici y (onstan ra e. ra e w"llIlUlesturaye-dum.nilleIJ lIala) Wil. Sll!/!I"stcd eall",. by Glad'c"", ..,.., J

during a test -e.g.. if rate declines slowly Ihroughout anllRamey.

www.petroman.ir
TABLE 3.3 -DA T A FOR PLOTTING FROM
VARIABLE-RATE DRAWDOWN TEST
TABLE 3.2 -VARIABLE.RA TE DRAWDOWN TEST DATA t (hours) (P, -Pw/ ) Iq t_(h~ur_~ .!P-!-=1?:!'~!..~9-
'(hours) Pw/(psi) 'q(STB/D) '(hours) Pw/(psi) q(STB/D) 0.105--- --0.444 -8.32 3.299
0 4412 250 832 3927 147 0.151 0.621 9.99 3338
0105 4:332 180 999 3:928 145 0.217 0.851 14.4 3.364
0151 4.302 177 144 3.931 143 0313 1.140 207 3.414
0217 4.264 174 207 3.934 140 0.450 1.491 29.8 3.467
0313 4.216 172 298 3.937 137 0648 1.886 43.0 3.515
0450 4.160 169 430 3.941 134 0934 2.288 61.8 3.545
0648 4.099 166 618 3.944 132 1.34 2.640 74.2 3.585
0934 A 039 163 742 3.946 130 1 94 2911 89.1 3597
, 34 3.987 161 891 3.948 129 2.79 3.090 107 3.638
1 94 3.952 158 107 3.950 127 ..
279 3.933 155 128 3.952 126 4.01 3.197 128 3.651
401 3.926 152 154 3.954 125 5.78 3.240 154 3.664
5.78 3.926 150 185 3.956 123 185 3.707

be futile. Press~res fo~ now times greater than ab?ut s= 1.151 [( Pi -PWj ) (~
6 hours are Increasing even though production
continues for another 179 hours and even though the
q I h m'
r
)
rate decline from this time to the end of the test is
only 27 STB/D (from 150 to 123 STB/D). -log
( -~ k
) + 3.23 ]
Thus, we must use the variable-rate analysis 4>JJ.C1
W
technique; the first step is to tabulate (Pi -Pwf) Iq, 3.04
as in Table 3.3. These data are plotted in Fig. 3.5. On = 1.151[ 0"288
the basis of curve shape, wellbore storage appears to .
end at approximately
ass.umption 6 hours;
with Eq. 3.2 when kweand
willS check this
have been -log [ (0.039)(0.8)(17 7.44
x 10-6)(0.198)2 ]
estimated.
There is no deviation from the straight line for 1>6 + 3.23
hours; accordingly, we assume the MTR spans the J
time range 6 hours < I < 185 hoHrs. = 6.02.
From the plot, nl' = 3.616 -3.328 = 0.288 ..
psi/ST8/D/cycie. Then, Since Cs =0.0106 bbl/pSI, as In Example 3.1,
(200 , <XX>+ 12,<XX>s)Cs
p.B I =
k= 162.6- M'bs-khlp.
Ill' h
(200,<XX>+ 12,<XX»(6.02)(0.0106)
-
(162.6)(0.8)(1.136) -(7.44)(69)/0.8
-
(0.288)(69) I = 4.5 hours.
= 7.44 md, This qualitatively confirms the choice of well bore
lid storage distortion end. --

0 ,. t
(I) q2 q
~
.~
.I
0'
,
q Q.
I
I
Q.
f
qn-1
I
A I I
~-I I I
I '

-
0:..- 0 II -I

r 0 .m t
FLONIt...K; TIME, hr FIg. 3.6 -Rate history for multlrate test.

Fig. 3.5 -Example variable-rate drawdown test.

www.petroman.ir ~--
FLOW TESTS 55

3.3 Multirate Tests


We will develop a general theory for behavior of
multirate tests in infinite-acting reservoirs for ~lightly
compressible liquids. In Appendix E, we extend this t
general theory to reservoirs in which boundary ef-
fects may become important .before the test ends.
Consider a well with n rate changes during its q
production history, as indicaled in Fig. 3.6. Our
objective is lo,delermine Ihe wellbore pressure of a
well producini"- wilh this schedule. We will use
sllperposilion of the logarithmic approximation to
Ille Ei-function solution; 10 simplify the algebra, we
will write the solution as 2 t ~

pI - p WJ
.r= 162.6- kh
q81J. r
lo g
( 1,688 <l>1J.C
(r IV
)
kl
- ] Fig. 3.7-: Rate history for single.rate drawdown test.
0.869 s .

q8IJ.ti k
= l62.6-,log I+l.og 2
kh <l>IJ.C(r W

-3.23 + 0.869 S) , qI
=m' q(log 1 +sj,

where t
m'=162.6~ q f

kh'
and q2 =0

S=log~ -3.23+0.869s.
With this <l>lJ.c(rW
nomen,clature for n rates and for I> 1n -t ' t .
application of superposition (as in the discussion
leading to Eq. 1.27) leads to Fig. 3.8-Rate history for buildup test following single
Pi-pw/=m'QI(log/+sj+m'(q2-QI) flow rate,

.[log (/-II)+S] +m' (q) -q2)


.[log (1-/2)+S] + ...q2 ..

+m'(Qn-qn-I)[log (/-I,,-I)+S].
This can be written more compactly as t
q q,
Pi-PW/ -, ~ (qj-qj-l) I
-m LJ I
qn j= 1 qn tp,--r- tP2
.log(I-lj_I)+m's,qn~O ..(3.10) I f1t-+

In Eq. 3.10, we define qo =0 and 10 =0. In terms of tl t2


more
Pi fundamental
-Pwf =m' E
~uantilies,
!qj -qj-l)
Eq. 3.10 becomes t

qn j= I qn Fig. 3.9-Rate history for buildup test following two


[ ( k ) different flow rates.
.Iog (1-lj_I)+m' log ~2
<I>IJ.C ( w
,

-3.23+0.869S]. (3.11)

www.petroman.ir "
.JV

For lhespccial caseqn =O(a pressure builduplCSl). Lel 1-12=AI. II =Ipl' 12=lpl +lp2' and
, -, I I = I p2 + AI. Then.
Pi-PM's=m ql(logl+S) +111 (q2-ql) q28p. ql I 1+1 2+AI
Pi-PM's=162.6- [ -log('PI '.Pol'-')
.[log(I-II)+sl+ ...+m'(qn-1 kh q2 Ip2 +AI

-qn-2)[log (/-ln-2)+sl-I11'qn-1 I +A/


+log(..:2.L:-=. )J (3.J
.[log (I -In -I) + sl AI

p.B II Eq. 3.15 ha~ Ihi~ applicalion: when the producil


= 162.6 kh E (qj -qj-l) rate is changed a short time before a buildup tc
j= I begins, so that there is not sufficient time f.
.log(I-I.- ). (3,12) Horner's,approximation.to be valid,. we frequent
J I can consider all production before time I I to ha'
Eqs. 3.11 and 3.12 can be used to model several been at rate q I for time I pi and production ju
special cases of practical importance, but we must before lhe lest 10 have been al rate q2 for time Ip2'
remember that they have an important limitation: the To analyze such a test, we plot
reservoir must be infinite acting for the total time
elapsed I since the well began producing at rate ql. Pws vs.
[ -log
qI ( I'PII'1+ 'PlI 2~+ -+Iog
AI ) ( I 2 + 41
.:~= )]
Appendix E suggests a more general method of q2 p2 + I
modeling tests in which boundaries have been The slope nl of this plot is related to formatic
reached. pcrmeabililY by lhe equation

Prc!i!illrl' I)rll"'down Test m = 162.6~.


(:rom Eq. 3.11, forn= I (Fig. 3.7), kl,
Extrapolation of the plot to AI = Q) gives Pws =)
~I[ = 162.6 JJ.!! log I + log ~- ( because, at AI =~, the plotting function is zer."
q1 kl, <!>p.c,r;. Nole 11101semilog polJer is "0110 be used; instead, tl,'
) sum or two logarithms is plotted on an ordinal' :
-3.23+0.869s. .., :..(3.13) Carlesianaxis.
su
Pr "~ r e II Id T t P
UI up es rece e y
.
d d b -To
h n
calculate skin faclor, s, note that at the end 4';1
d . .
bef ore s h ut-m,.'~,
C n t t R t P d t .t e ow peno jUSl ,.
0 s an -a e ro uc Ion .'

From Eq.3.12for/l=2(Fig.3.8), Pi-Pws=162.6~1~log(lpl+lp2) ~


p.B kh Lq2 I p2 ..
Pi-P\I'." = 162.6 -[q,logl-q.log (I-II)]
kJl
I
+I og(lp2)+S, _
,
qlp.B
( I ) or I.
\
=162.6-log (3.14) i
kJl I-I I Pi-PMt='" I -log
ql 'pi' 'pol ) +log(/,'2)+.5.
( 11+12 J !I
Ifwclclql=q,I-II=~/,andll=I"lhefamiliar q2 1112
equal ion lhal serves as the basis for the Horner plot .. j

resulls: The equal ion of lhe MTR line on the buildup te

plot is :
qBp.
IJi-P\I,.~=162.6-log ( I.1"I + -..
~I )
kll ~I Pi -PM,~ =m / ql
-log ( I"l...,.-+ln2+A/ ) j
.q2 1II2+AI
I-rc!isllrc 811ildllp Tcst I-rcl'cdcd by
T,,'o
~. I)irrcrcnt I-low Rates + 10 ( I.112.+ -.41 )J
('rom Eq. 3.12(Flg. 3.9), g ~I .

Pi -PI!'", =",' Iql log 1+ (q2 -ql) log (I-I.)


-r
Subtracting,
ql [ (Ipi + tp2)(lp2 + ~t) ] I ,

-q210g (1-/2)]' PM'S-Pw/=I1'tq-;log (Ipl +lp2+~/)(lp2)


This can be written as

Pi -P\I:t = 162.6-q2BP
kl l -log
ql ( -+I ) log l (I 2)(AI)
"p""-' ]+$.J
r q2 I-II Ip2 +41

+ log. I-II ( )1 Assume Ipl + Ip2 + 41=lpl + IPl:: and Ip2 + ~I = II


1-12 for small 13.1
(e.g" ~I = I hour). Then,

www.petroman.ir
FLOW
TESTS 57

uc:livc:rability colltract~), all ~igllif"j~.allt ratc:~ .Ifl.'


Q. considered. Improvement in a,-,curacy when u~ing
I this approach is questionable; fllrther, the flIn-
t damental assumption on which Eq. 3.16 is ba~eu
(that for 1=lpt +lp2 + ...+lpn-1 +i11 Ihe
reservoir is infinite acting) rarely wIll be valid for
q q2 large values of I. Nevertheless, when Eq. 3.16 is u~ed
I to model a buildup test, the following analysis
f procedure can be used.
: I. Write a computer program to calculate the
plotting function,
tI I ill
---log -=-
I( +...
) I-I" -2
+ log -= --= )1 X.
(
q,,-1 "I I 'n-I

t ~ 2. Plot Pws vs. X on ordinary (Cartesian coor-


dinate) graph paper.
F. 3 -3. Determine the slope In of the plot and relate toI
Ig. .10-Rate history for two-rate flow lest. the formation permeability by the equationI

q,,-IBp.I
Pws-pwf=m(logl:ll+s). /1/=162.6 kh.

If we choose i1/=1 hour,pws=Plhr (ontheMTR 4. Cal,-,ulatetheskinfactorsfromtheequation


line) and, for I p2 » I, ( -) k
-Plhr-Pw! S=I.ISI [ 'Plhr-Pwfl -Iog( _2)+3.23 J .
s= m tPp.C
,T w
m
(The derivation and assumptions implicit in this
k equation closely parallel those used for a buildup test
= log -~ -3.23, preceded by two different now rates.)
tPp.C,w S. The original formation pressure Pi is the value
or of P W'\'on the MTR line extrapolated to X = O.
_ I ISI( PI hr -Pwf k
s- .-log
m
, + 3.23 ,
tPp.c,rw
) Two-Hale Flow Test
From Eq. 3.11, this test (Fig. 3.10) can be modeled5
asbefore. by
We also note that duration of well bore storage qB p. q (q -q )
distortion is calculated as in the previous analysis for Pi -Pwf= 162.6-1- [ -'. log 1+ 2 I
buildup tests. kh q2 q2

Pressure Buildup Test Preceded .Iog (1- 1 ) + 10g( -~


by (n -I) Different Flow Hales I tPp.c
,r;. ) ..

FromEq.3.12, -3.23+0.869S] (3.17)-

P,.-=1626~!!~
Pws .kh [~
qn- (~ )
t log I-II ...
If we rearrange and Introduce specIalIzed nomen-
clalure, II =Ipl and 1-lpl =i1/', then Eq. 3.17 !
becomes !

+ ~ (
log -~ )+ ...+ q n -2 -q2 Bp.r k (
q ,,- I 1-/ 2 q "-:.1I pwr-Pi-162.6- kh log tPp.(ir~.
~ -3.23 )
.Iog ( I-I n -3 + log) ( 1- 1n.-2 )1 q I Bp. 1pi + i11'
I- I ~ 21
n-
- 1 I
n-
+0.869s j -162.6-
kh
/ l0g () i1/'
-~-

(3.16)
Although we introduce no specialized nomenclature + ~ 10g(di')]. , (3.18)
for this situation, note that 1-ln-1 =61 (time ql
elapsed since shut-in) and that qn-1 is the
production rate just before shut-in. This type of test can be used when estimates of
Applications of Eq. 3.16 in which more than three permeability, skin factor, or reservoir pressure are
terms are needed are probably rare; sometimes, needed but when the well cannot be shut in because
though, to satisfy precise legal contracts (e.g., gas loss of income cannot be tolerated. This test shares a

www.petroman.ir
,
,. ".~o~

VI
Pws a.
.
~

t + At 14 .'I 20 22 24 Z8 2.1
log 'P .-, l tL)l'
At k>g(-~) t .3:z.log (L)t')
L)l q,
Fig. 3.11 -Buildup test with pressure humping. Fig. 3.12 -Example two-rate flow test.

fundamental analysis problem with the conventional


drawdown test. The second rate must be kept strictly I. Plot Pwf vs.
[
log
( tpl + ~t'
!JJ'
)
+
q2
-Iog(~t
,
) ] .

constant or the test interpretation may be sub- ql


stantially in error. 2. Determine the slope m from the plot and use it
Eq. 3.18 is rigorously correct only when the to calculate permeability, k, from the relationship
reservoir is infinite acting for time (/pl + ~/') [just as B
the Horner plot is rigorous only when a reservoir is k = 162.6~.
infinite acting for time (/p+~/)]. Nevertheless, mh
application of the plotting and analysis technique 3. Calculate the skin factor, s, from the equation
suggested by Eq. 3.18 allows identification of the
MTR and determination of formation permeability s= I.l~[ ql (~J!~)
in a finite-acting reservoir. (ql -q2) m
The two-rate flow test does not reduce the duration
o~wel~bor~storag~distortion-the.duration.ofthis -Iog ( k
) +3.23] (3.19)
dIstortIon IS essentIally the same as m any buIldup or ~~~
drawdown test. However, the test procedure may
minimize the effects of phase segregation in the In Eq. 3.19, PI hr is the flowing pressure at ~/' = I
wellbore, an extreme form of which is "pressure hour on the MTR line and Pwj1 is the flowing
II\lmping",6 whcre high-pressure gas trapped in a pressure at the time the rate is changed (~t' =0).
\\.cllbore in poor communication with a formation (Eq. 3.19 was derived by simultaneous solution of
may lead to pressures in the wellbore higher than Eqs. 3.18 and the drawdown equation for a single
formation pressures (Fig. 3.11). A buildup test with rate applied at t = tpi ' at which time Pwf =Pw/l .)
this humping is, at best, difficult to interpret; thus, a 4. Pi (or, more generally, p') is obTaIned by
te~t procedure that can minimize these phase- solving for Pi (p') from the drawdown equation
segregation effects can be of value. written to model conditions at the time of the rate
The following method of analysis can be used for change. (It is implied that sand m are known at this
two-rate now tests. point.)

TABLE 3.4 -TWO.RATE FLOW TEST DATA TABLE 3.5 -DATA FOR PLOTTING
FROM TWO-RATE FLOW TEST
.11' (hours) Pwl (psi) ~I_~ (hours) _PWI (pSi)
0 3.490 8.32 3,897 ..\,' Pwl ..\,. Pwl
0.105 3,543 12.0 3.903 'hours) PF (psi) ,hours) PF (psi)
0.151 3.564 17.3 3.908 0 -3.490 8.32 1.826 3.897
! 0.217 3.592 24.9 3.912 0.105 2.756 3.543 12.0 1.754 3.903
0.313 3.627 35.8 3.915 0151 2.677 3.564 17.3 1.686 3.908
0.450 3,669 51.5 3.918 0217 2.599 3.592 24.9 1.623 3.912
0.648 3.717 74.2 3,919 0313 2.519 3.627 35.8 1.566 3.915
~. :. 0934 3766 89 1 3918 0.450 2.441 3.669 51.5 1.517 3.918
..., 0.648 2.~ 3.717 74.2 1.478 3.919
1.344 3.810 107 3,917 0.934 2.283 3.766 89.1 1.462 3.918
1.936 3.846 128 3,916 1.34 2.206 3.810 107 1.450 3.917
2.788 3,868 154 3,913 1.94 2.127 3.846 128 1.442 3.916
4.01 3.882 184.7 3,910 2.79 2.050 3.868 154 1.436 3.913
5.78 3,891 4.01 1.974 3.882 184.7 1.434 3.910
5.78 1.899 3,891

www.petroman.ir
FLOW TESTS 59

P, =Pw./l +m log -~
I ( kl,l ) -3.23 +0.869s .
I 4. 1)<:ll.'r/11i/1I.'/'o.
CPJl.C
Ir IV p' =PI':/l +m log -~kl I ( I ) -3.23 +0.8695 I
(3.20) </>Jl.({rll'

= 3,490 + 70[ log[ (7.65)( 184.7)/(0.039)(0.8)


Example 3.4- Two-RateFlow Test
.JrcJblem. A lwo-rale now ll.'sl was run on a wl.'ll willi .(17 x 10 b)(0.1 '18)2/
properties given below. From these properties and
the data in Table 3.4, determine k, 5, andp..
ql = 250STB/D, -3.23+(0.869)(6.32)~
q2 = 125STB/D,
Jl. = 0.8cp, =4,407 psi.
B = 1.136 RB/STB,
Pi = 4,412 psi, 5. ~e then check on wellbore-storage duration.
CI = 17 x 10-6 psi -I, For lhlS wel" Cs =:=25.65A lI'b/p=(25.65) (0.0218) /
A wb = 0.0218 sq ft, 53 =0.0106 bbl/psi. Then,
r W = 0.198 ft, = (200,CXX>
+ 12,CXX>5)Cs
h = 69ft, III'h~- kh/Jl.
P = 53lb/cu ft,
q, = 0.039, and (200,CXX>
+ (12,CXX»(6.32»)0.0106
Ipl = 184.7hours. =

Solution.
(7.65)(69)/0.8
J. We first tabulate the plotting function (PF), = 4.4 hours.

PF=[log(!£!-~~)+ ~log(aJ')]' At this time, the plotting function is approximately


AI ql 1.9;thisconfirmsourchoiceofthestartoftheMTR.

and plot Pwj vs. PF. Note that 1 1 = 184.7 hours and n-Rate Flow Test
q2/ql ~12~/250=0.5 (Table 3.5). The data are FromEq.3.11 an n-rate flow test is modeled by
plotted In Fig. 3.12. '
2. Next, we determine permeability. Assume that Pi -Pwj = 162 61l}! [ t (qj -qj-l)
the MTR spans the time range 1.5<PF<I.9 (50 qn .kh j=1 qn
hours>~/' >6 hours). Then the slope m=(3,927-
3,857)/( 1.4 -2.4) = 70 psi/cycle and
k=162.6~ .log(/n-lj-l) ]+162.6k1; Jl.8

mh

(162.6)(250)(1.136)(0.8) .rIOg(~) -3.23 +0.86951. ..


= cPJl.(
Ir'i"
(70)(69)
This equalion suggestsa plot of
=7.65 md.
Pi-Pili
q ~I
vs. ""' (qj-qj-I)lo
q g (I n -I. J-I' )
3. We determine skin factor,
n J= n
S= 1.151 r qI (p I hr -P w}1)
(ql -q2) m Permeabilily is related to the slope m' of such a plot:
( k Jl.8
-log.
CPJl.C
I~
) ') +3.23 / , k=162.6-;-.
'" h
at aJ , = I hour, PF=log(184.7+ 1)/1 +0.5 log(l) = Irwe let funclion
plolling h' be the value thcn
iszcro of (p;-Pwj)/qn when the
2.269, andPI hr = 3,869 psi (on the MTR line): k '

-
s-I.151
[ 250 (3,869-3,490) h'=m' ( d>uc.~ ) -3.23+0.869S ].
( 'Og .')

(250 -125) 70 'f'r-t w


or
[ -:
(~
7.65 6 _
-Iog
(0.039)(0.8)(17 x 10-°)(0.198)"
]+3.23 } S=I.151 [~-IOg
m'
)+3.23]
4>#lCtfw
(3.21)

=6.32. Note that use of Eq. 3.21 and of the proposed

www.petroman.ir -
TABLE 3.6 -MUL TIRATE FLOW TABLE 3.7 -DATA FOR PLOTTING MUL TIRATE FLOW TEST
TEST DATA t ~,--Q,- 1
P, -Pwf ( PSia ) 1= 1 qn I
t Pwf I
(hours) (psi a) t(l)ours) Q,,(STB/D) P,-pwf(psia) q" RB/D_:'og(t,-t,-,)

00.333 3.000
999 0
0333 478.5 2,001 4.18 -0.478

0.667 857 0.667 478.5 2.143 4.48 -0.176


1.0 778.5 1.0 478.5 2,221.5 4.64 0
2.0 1.378.5 2.0 319.0 1.621.5 5.08 0.452
2333 2,043 2.333 159.5 957 6.00 1.459
2667 2.0675 2667 1595 923.5 5 79 1 232
3.0 2.094 3.0 159.5 906 5.68 1.130

plotting method implies that Pi is known from in- qn =478.5 STH/D,


UCPClldclltmca~lIrcmcllt~.
Odeh and Jones 7 discussed this analysis techniquc. (i = 1.0 hour,
l1tey pointed out that it can be applied to the analysis
of multirate now tests commonly run on gas wells (pi-Pwj)/qn=(3,CXX>-778.5)/478.5=4.64 and
and oil wells. In these applications of the technique, '

it is essential to remember the assumption that the ~ (qj -qj- I)


reservoir is infinite acting to the total elapsed time ( .4J q log (I n -Ii -I )
for all now rates combined. Further, note that the J= I n
technique ignores any wellbore storage distortion
c.:reatedby any discrcte rate c.:hanges. (478.5 -0)
= --og I ( I .-0)=0.
0
-478.5

E.\-oI11ple
3. 6 -Multirote Flow Test A 110lysis
.-r.,hlem. Odch and Joncs , prcscnt data from a 3- 1 = 3.0 hours. Here ,
Iiour drawdown tcst on an oil well; in this test, the
rate during the first hour averaged 478.5 STH/D; (Pi -Pw/)/qn =(3,CXX>-2,O94)/159.5=5.68, and.
during the second hour, 319 STB/D; and during the n
third hour, 159.5STB/D. Reservoirnuidviscosityis E (qj-qj-I)IOg(1 -:"1. )
0.6 cp; initial pressure is 3,<xx> psia; formation i= I qn n )-1
volume factor, 8, is considered to be 1.0; and the
rcscrvoir is assumed to be infinite acting for the
entire test. Assume that wellbore storage distortion is =~ (478.510g(3.0-0) + (319-478.5)
minimal at all times during the test. Pressures (PHf) 159.5
at various now times are given in Ta?~e 3.6: From .log(3.0-1.0)+(159.5-319)
Ihc~c data, determine the permeabliity/thlc.:kness
I'roduc.:t of the tested well. .log(3.0 -2.0)J
SClllIli.'II. We first prepare the data for plotting -i.e.,
at each tilllC, we must determine (pj-P"f)/q" and =1.130.

~ ~ -qi -I) -.calculations at theseand other times are summarized


t i=i.JI q" 10g(/" (i-I)' In We
Table next
3.7.I estimate the permeability/thicknessI

product. From Fig. 3.13, I


(= 0.333..'!our. Hcre, (6.0 -4.2)
m' = =0.942.I
Q,,=ql =478.5S'I'B/D, (1.459-(-0.452)J[
Then,I
(" = 0.333 hour, ILB (162.6)(0.6)( 1.0)
kh= 162.6 -=
(l'j-1111:r)/q,r=(3,()()()-999)/478.5=4.18,alld m' 0.942

" (il'-i/'I) =104nld-ft.


E ~ .'j-J'log«(,,-lj-l)
j= I q" Odeh and Jones do not state values for h, ~, Ct'
alld , w used 10 construct these example test data.
(478.5 -0) To illustrate skin-factor calculation, assume that
= log(0.333 -0) = -0.478 /, = 10 ft and2lhus, that k = 10.4 md. Also assume
478.5 that k/~ILCtrK.=4.81 X 10'. Then, since the graph
indicated that b' =4.63 (b' is the value of (Pi-
, = 1:0 hour. Here, Pll f ) Iqn when the plolting function is zero), I
-I.
I
I
www.petroman.ir
FLOW TESTS -~!~- 61

.FO~
S=I.t511l!:.--IOg(~-2)+3.231 J~-~'~.;- ~i ---I
",' ~IJ.('" ". ~

=1.151 I --log
4.63 1 .0

0.942 (4.8xI07 ) +3.23 ~.


c
:!it:
=0.53. ~ 4
I

-0:.-
Exercises
3.1. A constant-rate drawdown test was run in a 0 02 04 ~ ~ 10 II 14 .

well with the following characteristics: r


[
.qJ ( .-.)/1"' ll..v.(t
q J-I '-tnJ ~ ""n.t. J-I)
J=I
q = 500 STB/D (coll~tal1t),
ct>= 0.2,
08 .
FI g .3.13 -xample
E IIIUItlrate I low test.
IJ. = .cp,
c, = IOXI0-6psi-t,
'II' = 0.3 fl,
h = 56 fl,
Bo = 1.2 RB/ST8,
Awb = 0.022sq fl, TABLE 3.8-DATA FOR EXAMPLE
P = 50 Ib/cu fl, and CONSTANT.RATE DRAWDOWN TEST
liquid/gas interface is in well. 1(hOllrs) P.t(psil 1(hours) p./(psil '(hours) p.tlpsi)
From the test data in Table 3.8, estimate formation 0 3,000 0491 2,302 328 1.543
permeability skin factor and area (in acres) 00109 2.976 0.546 2,256 382 1,533
d .' -, 00164 2,964 1.09 1,952 43.7 1.525
rained by the well, -00218 2.953 1.64 1.828 491 1,517
3.2. A drawdown test in which the rate decreased 00273 2.942 2.18 1.768 546 1,511
continuous
.
1 h h t .
y t roug out t 1e test was run In a we
II 00328
00382
2,930
2,919
2.73
328
1,734
1,712
655
874
1.500
1.482
with Ihe following characteristics. 00437 2.908 3.82 1,696 1092 1,468
0.0491 2.897 437 1,684 163.8 1,440
00546 2,886 491 1.674 218.4 1.416
ct>= 0 2 0109 2,785 5.46 1,665 2730 1,393
., 0164 2.693 655 1.651 3276 1.370
II. = 1.0cp, 0218 2.611 874 1.630
Ct = IOxI0-6psi-t, 0273 2.536 109 1.587
0 25 f 0.328 2.469 16.4 1.561i
'w =.
t, 0437 2.352 27.3 1,554
h =
100 fl,
Bu =
1.3 RB/STB,
A I.-b =
0.0218 sq ft, ..
P =
55lb/cuft,and TABLE3.9-DATAFOREXAMPLE ..
I.. d/
Iqui
. t f
gas In er ace I~ In wc
II .. VARIABLE. RATE DRAWDOWN TEST

'(hours) Pw/{PSI) q(STB/OI tiholllsl P../IPsi) qISTBID)


0 5.000 200 303 4.797 122
From the lest data in Table 3.9, estimate formation 0114 4,927 145 364 4797 121
pcrmeabililyandskinfactor. 0136 4,917 143 437 4,7911 119
0164 4.905 142 524 4798 118
3.3. A constant-rate drawdown test was run on Ihe 0197 4,893 141 629 4,798 117
well described in Problem 3.2. Rate was held con- 0236 4,881 140 754 4.799 116
slanl at 600 STB/D. Afler 96.9 hours, Ihe sllrface 0283 4,868 138 905 4799 114
.0340 4,856 137 109 4800 113
rate was changed abruplly 10 300 STI3/0. Data lor 0401! 4.844 136 130 41101 112
Ihe lesls before and after Ihe rate change are givcn in 0490 4,833 135 156 4.1101 110
T bl 3 0 F I d b . d . I 600 0.587 4,823 133 188 4,1102 109
a e .1. rom t 1e ala 0 Caine Wit 1 q = 0705 4815 132 225 4.1103 lOll
STI'/O, cst im.ltc k, .\', alld .lrc.1 of I hc rcsl'rvoir. 0846 4:/j09 131 270 4.!!O3 107
I;'rom the data obtained in the two-rate now test arter 1.02 4,004 12'J 324 41104 1~
t d 300 STO/O f. h . fk 1.22 4.801 128 389 4.805 104
q C1ange to , con Irm t e estimates 0 1.46 4.799 127 467 4.~ 103
and s, and calculate the current value or p.. 1,75 4,798 126 56.1 4.~7 102
3 4 For the multirate flow test described in 2.11 4.797 124 67.3 4.~7 100
..2.53 4,797 123 807 4.808 99
Example 3.6, (a) calculate the value of the plotting ~.9 4.809 98
function at /=0.5, 1.5, and 2.5 hours, and (b)
calculate the flowing bOltomhole pressure at t = 3.5
hours, assuming that there is no change in rate for 3
hours< / < 4 hours.

~ www.petroman.ir
TABLE 3.10 -DATA FOR EXAMPLE TWO.RATE FLOW TEST

Pwl at Pwl at PwI at Pwl at


600 STBID 300 STB/D 600 STB/D 300 STB/D
~~h_O!!!:.~) (~~I) (psi) ---'-J!!_O~~~) ~,,-~IL- (psi)
0 5,000 3.833 3.03 4,012 ~,~
0.114 4,710 3,978 3.64 4,002 4,330
0.136 4,665 4,000 4.37 3,992 4,334
0.164 4,616 4,025 5.24 3,982 4,339
0.197 4,563 4,051 6.29 3,972 4,343
0.236 4,507 4,079 7.54 3,963 4,346
0.283 4,449 4,108 9.05 3,953 4,350
0.34() 4,390 4,138 10.9 3,944 4,353
0.408 4,332 4,167 13.0 3,935 4,356
0.490 4,277 4,194 15.6 3,926 4,359
0.587 4,227 4,219 18.8 3,918 4,361
0.705 4,182 4,242 22.5 3,909 4,363
0.846 4,144 4,261 27.0 3,900 4,364
1.02 4,112 4,276 32.4 3,891 4,365
1.22 4,087 4,289 38.9 3,883 4,365
1.46 4,067 4,299 46.7 3,874 4,364
1.75 4,050 4,307 56.1 3,865 4,362
2.11 4,036 4,314 67.3 3,855 4,359
2.53 4,024 4,319 80.7 3,845 4,354
96.9 3,833 4,349

~
He r ere.lces
r I. [:arlougher, R.C. Jr.: Advallces ill "'ell Tesl /1l1alysis, Effcct~ on Pressure Buitdup and Drawdown of Gas Wells," J.
Monograph Scrie~, SPE, Dalla~ (1977) ~. Pt'l. Tech. (Feb. 1965)223-233; Trans., AIME,134. J
, 2. Wine~I(}\:k, A.U. and <..olpitl~, (i.P.: "Advitncc~ in [:~til11itling 5. R"~~II, D.(i.: "Determinalion of Formation Otaradcri~liC!i
(,a~ Well [)cliverabilily," J, Cdll. Pel. T('('h. (July-Scpl. 1965) Froln Two-Rate Flow Tests," J. Pt'l. T«h. (Occ. 1963) 1347- ~
111-119, Also, Gas T('('hnoloK.II, Reprint Series, SPE, Dallas 1355; Trans., AIME, 228. I
(1977) 13, 122-130, 6. Stegemeier, G.L. and Matthews, C.S,: "A Study of
3. Uladfellcr,
Wcll~ Which R.E., Will
Tracy,Rcspond
G.W., and 10 Wilsey, L.E.: "Selecting
Production-Slimulitlion Anomalous
113,44-SO. Pressure Buildup Behavior," Trans.,AIME(19-'8)
.-
Trcalmcnl," Drill. alld Prod. Prac., API, Dalla~ (1955) 117- 7. Odeh, A.S. and Jones, L,G.: "Pressure Drawdown Analysis
129. Variable-Rate Ca~e," J. Pel. Tech, (Aug. 1965) 960-964; :
~'4. RitlllCY, H,J. Jr.: "Non-Darcy 1:10"' and Well bore Storage Trails., AI~1E, 234.

www.petroman.ir
Chapter 4 II

t Analysis of Well Tests


Using Type Curvesi

4.1 Introduction
This chapter discusses the quantitative use of Iype established in a pressure Iransient lest on a fraclured
r curves in well test analysis. The objeclive of Ihis well.
chapler is limited basically to illustraling how a Fundamentally, a type curve is a preplotted family
representative sample of type curves can be used as of pressure drawdown curves. The most fundamental
analysis aids. Other major type curves in use todar of these curves (Ramey's2) is a plot of dimensionless
are discussed in the SPE well testing monograph. pressure change, PD' vs. dimensionless time change,
However, type curves for specialized situations are t D. This curve, reproduced in Fig. 4.1 (identical to
appearing frequently in the literature, and even that Fig. 1.6), has two parameters that distinguish the
monograph is not completely current. We hope Ihat curves from one another: the skin factor s and a
the fundamentals of type-curve use presented in this dimensionless wellbore storage constant, CsD. For
chapter will allow the reader to understand and to an infinite-acting reservoir, specification of CsD and
apply newer type curves as they appear in the s uniquely determines the value of PD at a given value
lilerature. of t D. Proof of this follows from application of the
Specific type curves discussed include (I) Ramey el techniques discussed in Appendix B. If we put the
01.'s type curves2-4 for buildup and constanl-rate differential equation describing a flow test in
drawdown tests; (2) McKinley's type curvesS.6 for dimensionless form (along wilh its inilial and
Ihe same applications; and (3) Gringarlen el 01.'s7 boundary conditions), Ihen the Solulion, PD' is
Iype curves for vertically fractured wells with determined uniquely by specificalion of the in-
uniform flux. dependent variables (in this case, t D and rD)' of all
dimensionless parameters that appear in Ihe
4.2 Fundamentals of Type Curves equalion, and of inilial and boundary condition~(in
Many type curves commonly are used to determine this case, sand CsD)' Further, in most such
formalion permeabililY and 10 characterize damagc ~Olillion~. we are inleresled in wellbore pressures of a
and stimulation of the tesled well. Furlher, some are te~ted well; here, dimensionless radius, r D =r/r I'"
used to determine the beginning of Ihe MTR for a has a fixed value of unity and rhus does not appear as
Horner analysis. Most of these curves were generated a parameter in the solution.
by simulating constant-rate pressure drawdown (or Thus, type curves are generaled by oblaining
injection) tests; however, most also can be applied to solutions to the now equations (e.g., the diffusivity
buildup (or I'allorl) tesls if an equivalent ~hul-in equation) willI specified initial alld boundary COll-i!
time8 is used as the time variable on the graph. ditions. Some of Ihese solutions are analytical; others
Conventional test analysis techniques (such as the are based on finite-differcnc.e ~pproximations
Horner method for buildup tesls) share these ob- generated by computer reservoIr sImulators. For~_.
jectives. However, type curves are advanlageous example, Ramey's type curves were generated from
because they may allow te~t interpretalion even when analytical ~olulions 10 Ihe diffllsivity equ3lion, wilh
wcllbore storage dislort~ mosl or all of the lesl dala; Ihe initial condilion thai the reservoir be al uniform
in Ihal case, conventional mclhods fail. pressure before the drawdown tcst, and with
The use of type curves for fractured wells has a boundary conditions of (I) infinitely large outer
further advantage. In a single analytical technique, drainage radius and (2) constant surface withdrawal
type curves combine the linear flow that occurs at rare combined wilh wellbore storage. which resulls in
early times in many fractured reservoirs, the radial variable sandface wilhdrawal rare. A skin factor, ~'.is
flow that may occur later after the radius of in- used to characterize wellbore damage or stimulation;
vestigation has moved beyond the region influenced as we have seen, this causes an additional pressure
by Ihe fraclure and the effects of reservoir boun- drop, Aps' which is proportional to the in-
daries that may appear before a true MTR line is slantaneous sandface flow rare (which changes with
www.petroman.ir
~-
10' f

: : :::: .1
.\ -10 I

, 10

In

Q0 ,

10',
10' 10' 10' '0' 10' 10' 10'

'0
Fig. 4.1 -Type curves lor constant production rate. inifinite-acting reservoir (Ramey).

time while wellbore storage is a dominant innuence). note later in this chapter. Of major importance is
Dimensionless pressure drawdown at the well bore, that the curves can be used for buildup tests and for -
PD' predicted by these solutions thus can be plotted gas well tcsts.) The result of Ramey's work is shown
as a function of elapsed time, I D' for fixed values of in Fig. 4.1. ,:'
CsD and s. When curves are drawn for the range ofs Some important properties of these curves follow.
and CsD of greatest practical importance, the type I. Examination of the analytical solution on I
curve results (Fig. 4.1). which the type curves are based shows that, at earliest.,
To use a type curve to analyze an actual drawdown times when well bore unloading is responsible for I
test, the allalyst plots pressure change, Pi -P"1 vs. 100% of the now in a drawdown test (or afternow
now time. /, on the same size graph paper as the type rate equals rate before shut-in in a buildup test), Ap is
curve. Then one finds the preplotted curve that most a linear function of 111 (~ is pressure change since
nearly has the same shape as the actual test data plot. the test began and 6/ is time elapsed since the test.
When the match is found, s, CsD' and corresponding began). Thus, the log Ap-log61 curve is also linear \
values of lI'v. (Pi -p"'r)] and (tv. /) will have been with a slope of unity (a 450 line) and the wellbore
establishcd. and k thcn can be detcrmined. These slorage conslant Cs can be determined from any
sentences summarize the pril,ciple -but the practice point (6/, 6p) on this line (Fig. 4.2) from the relation ,
differs in del ail from Ihe princii"le and is not qR 6f
~Icce~sarily as straiglltforward as this brief discussion ('s = -( -) ...
ImplIes. 24 Ap unit.slope line

3 R '" r C Note that, in a well with a liquid/gas interface in the


4 .amey s ype urves wellbore,
~ame'y's2 type curves were generated for the 25.65A ,
sItuation of a constant-rate pressure drawdown test Cs = "b bbl/psi, (4.1)

in a reservoir with slightly compressible. single-phase P


liquid nowing; sufficient homogeneity such that the and for a well bore filled with a single-phase liquid or
radial diffusivity equation adequately models now in with gas,
the reservoir; uniform pressure in the drainage area C -V bbl/ .42 i
of thc \Vcll hcrorc pr(\dllctiOlI; inrillilc-actitlg ,f -CII'" "", pSI,.. ( .)

rcscrvoir
of interest (110 for
boulldary
test effccts
analysis durillg the
puri"oses); flow constant
pcriod andC -0894 C Icf>c }rr2 -, (4 3) i

and
\\'ithdrawal
concentratcd
rate at \Vellborc
thc surface; damagc
alld wellborc
or stimulatioll
storage Successful
.fD -.s application t M"
of Ramey's type curves for

characterized by a skin factor, s. This list of quantitative analysis depends significantly on our
assumptions is tedious, but it is also important. ability to establish the correct value of CsO to be used
Whcn one or morc of these assuluptions is not valid for curve matching -type curves for a given value of
in a sr>ccific case, tl1Cre is no assurance that use of the s and for different val tiCS of C.fv have very similar
type ctlrvcs can Icad to a valid test intcrprelalion. shapes, so it is difficult to find the best fit without
(Some of these limitatiol1s can be removed. as \ve will prior knowledge of Cso, Direct calculation of Cst I

www.petroman.ir Ii
I
i
:

09 u
I\ p ~ ~
V
' LINE WITH SLOPE.
= I CYCLE/CYCLE

USE ANYffilNT (i'll 1\1')


I
.'..'
.'~
~..~
,.=8
:

I I
I
/
/ :
I'

~
ON LINE TO CAl.CULATE Cs
MATCH POINT :.
..
,-

storage constant,
log t::.t
Fig. 4.2-Use of unit slope line to calculate wellbore . .,;
-'Iii:
I -.D rrD ,~~

log Po (.\0
~
.o
~d9/
Z.,..:..t'~
-::;;---
/-
, -f~,)fSTORA(;l
W(IL~(
CAST(1RII(~1
(C~VlS'1.~TI'::Al
CUHV[~~ ~.u)
10
I .
,~"
.

~ " 0)" s.o J


~ '-:g -;0 ---t D
I Fig-4.3-Use of type curves to determineend of wellbore Fig. 4.4-Horizontal and verticalshilting to Iind po:)IIIOll01
I storage distortion- lit and matchpOllltS. ~,
r and thus CsD' from known values of A wb ~";d p or (p i -p)lIj) drawdown-(p,,~ -P 1.1-)buildllp' :

Cwband V wb does not characte~ize test conditions as and


well as the value of Cs determined from actual test
performance as reflected in the unit-slope lines. 9 Idrawdown-Albuildup'
2. Well bore storage has ceased distorting the If these analogies can be used for the larger vallll:~
.pressure transient test data when the type curve for of At in the MTR then "'e would expect intllitively
the value of CsD characterizing the test ~comes thai the approxim~tion ust.-dto develop them woliid
identical to the type curve for CsD = 0 (Fig. 4.3). be even better for the smalll:r values of AI in the
(This usually occurs abollt one and a half to two ETR
cycles from the end of the unit-slope line.) Thus, TI..e practical impli,-,ation of thi~ analysis is thi~.
thesetype curves can be used to determine how ~lllCh I"or u~e with type Cllrvt.'S,we plot actual dru\vdown
data (if any) can be analyze~ by. conventional tc~t data as V'i -P"1) vs. I and buildllP test data a~
methods such as the Horner .plotlor build lip te~ts... (/'",\ -1'"".) vs. AI, hllt "'1: mllst rcmemhcr tll.lt AI"
3. The type curve~, whl,-,h wl:rc devel.opl:d 101 mllst be llsed instead of At \VII~llever~I >0.11/"
drawd~wn ~ests, also can. be used for b~lldlIP. test 4. A log-log plot of Pv vs. IV differs from a log-
analysIs If an .equlvalent .shut-l~ time, log plot of (Pi-P"f) vs. I (for a druwdown .test)
~/e~.~I/(I+AI/I ),.lsused.asthetlmevarlable.An only by a shift in the origin of the coordln.lte
Intuitive proof of this assertion for small AI (where system-i.e., log I D differs from log I by a constant
.1/~~/e) fol.lows.. and log PD differs from log (Pi -P":f) byaJJother
The equation of the MTR In a drawdown test can conSlant. To show this, we note that
be expressed as O.CXX>264 kl
Pi-pwj=nllog/+CI' IV= 4>lJ.c,r;, ,
The equation of the MTR in a buildup test is .Ind
Pi -Pws = 11110g[(lp + AI) / AI] kh(Pi -Plif)
Pv= 14l.2qBIJ. .
= nllog(/" + AI) -Ill log AI.
I flog (I p + AI) =:log Ip(.111ad~LJllat\:.1~~lllllptit,ll for 'llll1~,
A/maxSO.l/p)' O.OOO264k
log IV = log I + log ---:- '.-,
Pi-Pws=:l1Ilogll,-"/log~/ tI>,((,rlv -,-
and
= (Pi -PIli/) -Ill log AI-Ctt kh

or. 10gPn=log (P;-PMf)+logI4Uqs;.


(PIllS -PMf) =:111log AI + C I. The significance of this re~ult is that the plot of .in
Thus; the equations for MTR's in drawdown and actuul.draw.down test (log I vs. log Ap) should have a
buildup test plots have similar form if we use the shape Identical to that of a plot of log I D.vs. log P n,
ana I ogles
.bllt we have to displace both the horizontal and

www.petroman.ir
~ !
66
~~
~
-WELLTES

\'ertical axes (i.e.. shift the origin of the plot) to find z, T "
tlleposition?f.bestfit(Fig.4.4).. .Bg,=5.04~ RB/Mscf. (4.11)
Once a fIt IS found by vertical and horizontal Pi
shifting, we choose a match point 10 determine the Thus. when p.Zlp = constant, we can plot (Pi-
relalionship bel~een actual tinle and dimensiolllcss Pwf) for Iype-curve use juSI as for a slightly com-
time and between actual pressure drawdown and pressible liquid. Match-point interpretation is
dimensionless pressure for the test being analyzed. ,B '
AI~Yr<~intol1thcgraJ?h.papcrwill~lIfficca~a~lalch. k=141.2qgIJ.,-g,( PD) , (4.12)
polnl (I.e., the rcsult IS l/Jdepcndent of lhe cl'OICC01 I, Pi -PK1 ~11'
match point). For the match point chosen, we
determine the corresponding valu~s. ~f (I, ID) and 0.000264 k I
(Pi-PK1),PDJ.Then, from definItion ofPD and tPCti= u;~.(-) (4.13)
10' IJ.i K' ID MP i
qBIJ. P Nole that all gas properties are to be evaluated at '
k=141.2-( 0) , (4.4) originalreservoirpressureforatestinaninfinile- I
h Pi-PM:! MP acting reservoir (or, more generally, at lhe uniform
a/Jd reservoir pressure preceding the drawdown lesl or at
the current average reservoir pressure for a buildup

f/J<"t
-p.r~
-0.<XXJ264 k ( r;;I )Mp. (4.5) test).
In some other situations,.p.Z is constant (e.g., in
5
."
Altl
.10Ug
h tl
Ie
t
ype
.can
curves were
d
eve
I
ope
d f
rom
many cases for p <
be repIaced by th e defiInl t Ion
2 ,(xx)
..
psla); as a result, E4. 4.7

solutIons to now equallons for slightly compressIble 2 2


liquids, lhey also can be used to analyze gas well -kh(Pi -Pwf) (4 14)
tests. Transformation of the flow equations to model PO-1422
.10 ' qg IJ. I.z.T.
I
gas no~ In terms of the p~eudopressure , 1ft(p) and Thus, when p.Z= constanl, we can plot (pt -P~f)
c?lnpa~lson of these solutions expresse~ In lerm.s of for lype-curve use. Match-point interprelalion
dimensionless pseudopressure, 1fto , wIth solutions b '1
Po for slighlly compressible liquids, II show that, as ecomes
a high-order-accuracy approximation for lransient
n ow, - 1 422
k -, ~~~
h
( Pi_2_2
Po
-Pwf )MP, (4 I 5)

lfto(ID,rD'S',CsO) =PO (/o,ro,s,CsO) , -


where, for gases, -0.000264 k ( I
f/J<"ti- , ) (4.16)
0.<XXJ264 kl IJ.i~ 10 MP
10= ..I... -_2' (4.6) Determination of whether either approximation
tPIJ.iC gir w
(IJ.zlp = constant or p.Z= conslanl) is valid musl be
based on plots or labulations using lhe properties of
1ft0 = kh TSC"
[1ft(Pi) -1ft (Pwf) J, (4.7) lhe specific gas in lhe reservoir being tesled.
50300
, p scqg T Use of Ramey's Type Curves
,
s =s+D I qg'I (4.8) The theoryprocedure
following of Ramey's type curves
for using leadsfor
the curves lo test
the

all d analysis. Theliquid;


compressible procedure is through
Eqs. 4.6 given for' slightly
4.16 show the
\.,' I' changcs ncccssarywhcn a gas wcll test is analyzcd.
lft(p)=2dp. (4.9) 1 PIOl (p .- p ) vs I (drawdownlesl ) or (p -
'I' IIIJ.(p) Z (p) .I vs. ale =all(1wf + alllp)
PK1) .WS' (buildup tesl) on log-
To use this result as stated would require that we log paper the same size as Ramey's type curve.
(I) prepare a table or graph of values of 1ft(p) vs. P Caution: Unless a type curve undistorled in the
from Eq. 4.9 based on the properties of the specific reproduction process is used, it will not have the
gas in the well being tested; (2) plot [1ft(Pi) -same dimensions as commercially available graph
Ift(P"1) ) vs. Ion log-log paper; and (3) find the best paper and finding a fit may be misleading or im-
fit just as for a slighlly compressible liquid (with the possible. The best solution is to use an undistorted
values of s providing the best fit being interpreted for type curve; an acceptable alternative is to plot test
the gas well as s + Dlq RI). data on tracing paper, using the grid on the distorted
Sleps I and 2 can be simplified in some cases. Iype curve as a plotling aid.
When ItZ is directly proportional to pressure (for 2. If the test has a uniform-slope region (458 line
p>3,<XX> psia for some gases). Eq. 4.7 can be alearliestlimes).chooseanypoinl(l, (Pi-pwf)Jor
replaced by the definition [61, (PM'S'-Pwf) ) on the unit-slope line and calculate
-~h (Pi -Pwf) the wellbore storage constant Cs: '
pv- , (4.10) QB( I
141.2QRIJ.iBRi Cs= - ) I
where

www.petroman.ir
24 Pi -Pwf unil-slopeline

'
II
I
ANALYSISOFWELLTESTSUSINGTYPECURVES T
67
""'"
TABLE4.1-CONSTANT.RATEDRAWDOWN TABLE4.2 -DRAWDOWNDATATABULATED
TESTDATA FORPLOTTING
'(hours) Pwl(PSII '(hourSI Pwl (PSI) '(hours) Pwl (PSI) P, Pwl P, Pwl P. P."
1(lloI.rs) (psil '(IIour~1 (11:'11 IIIIuur!;1 111:'11

00109
0.0164 2.976
2.964 00218
164 2.611
2.693 218
273 1.734
1.768 00109 24 0164 301 218 1.232
00218 2.953 0273 2.536 328 1712 00164 36 0218 389 273 1.21i6
00273 2.942 0328 2.469 382 1.696 00218 41 0213 464 32U 1.2t1ti
00328 2.930 0382 2.408 4 37 1.684 00273 58 0328 531 382 1.:104
00382 2.919 0437 2.352 491 1.674 00328 10 0382 592 431 1.316
00437 2.908 0491 2.302 546 1.665 00382 81 0437 648 4.91 1.326
00491 2.697 0546 2.256 655 1651 00437 92 0491 698 546 1.3:15
00546 2.686 109 1,952 874 1630 00491 103 0546 744 655 1.349
0109 2.765 164 1.626 109 1614 00546 114 109 1048 874 13/0
~64 1.587 0109 215 164 1.112 109 I.~
164 1.413

Then calculate the dimensionless wellbore storage q = 500 STB/D,


constant: 4> = 0.2,
0.894 C p. = 0.8 cp,
C sD = -~h-T .c, = lOx 10 -6 psi -I ,
, rw r M' = 0.3 ft,
Note that estimates of 4>and c, are required a( this h = 56 ft,
point -with implications that are discussed later. Bo = 1.2 RB/STB, and
If a unit-slope line is not present, Cs and CsD must Pi = 3,(xx) psia.
be calculated
curacies from if well
may result thesebore properties,
properties do notand inac-
describe ..
Solution. We must first prepare !he ~ata for plotting
actual test behavior. 9 (Table 4.2). The data are plotted In Fig. 4.5.
3. Using type curves with CsD as calculated in From the unit-slope line (on which the data lie for
Step 2, find the curve that most nearly fits all the I :S0.0218 hour),
plotted data. This curve will be characterized by C -qB [ I
some skin factor, s; record its value. Interpolation s -24 (p. -P ). ] ..
between curves should improve the precision of the ' wf pomton unit-slope
line
analysis, bu! may:prove difficult. Even for fixed.CsD (500)(1.2) (0.0218)
from the unIt-slope curve, the analyst may expenence = x
difficulty in determining that one value of s provides (24) (47)
a better fit than another, particularly if all data are .
distorted by wellbore storage or if the "scatter" or = 0.0116 R8/psl.

"noise" that characterizes much actual field data is Then,


present. If CsD is not known with certainty, the 0894 C
possible ambiguity in finding the best fit is even more C sD =' 2s
pronounced. <t>c,hrw
4. With the actual test data plot placed in the
position
(Pi -Pwf' of Pbest fit, record
D) and (I, I D)corresponding values of
from any convenient -(0.894)(0.0116)
-(0.2)(1 x 10 -.5)(56)(0.3)2 ,..

match point. ..
5. Calculate k and <t>c,
(Eqs. 4.4 and 4.5): = 1.03 x 103

k=141.2!!!!!--( PD) , =lxI03.


h Pi-Pwf MP For C sD = 103, the best-fitting type Cllrve is for 5 = 5. ,I'

0.000264 k I A time match point is 1= 1 hour when


ci>c,= 2(-)' ID=I.93XI04. A pressure match point is (p.-
p.rw 10 MP pw/)=IOOpsi,whenpo=0.85.'
Eq. 4.5 does not eslablish 4>c,based on test per- .Fro~ the match, we also note that wcllbore storage
formance unless it is possible to establish C V dt~lortlon end~ at 1=5.0 ho\lr~ (i.e., the typt: ctlrvt:
without assuming values for 4>C,-it simply for CsO = 103 becomes identical to the type curve for
reproduces those values assumed in Step 2. CsD = 0).
In summary, the procedure outlined in Steps 1 From the pressure match pl)int,
through 5 provides estimates of k, 5, and Cs. k= 141.2-qBIJ. PD ( )
h Pi -Pwf MP
Example 4.1- Drawdown Test Analysis
Using Ramey's Type Curves
I'rc)blem. Dctermine k, 5, and C s from the data = (141.2)(500)(1.2)(0.8)
(56)
0.85
..00-
( )
below and in Table 4.1, which were obtained in a I
I1rcsslIrc drawdown tcst t}J\ ,lIt oil wcll. = JO.3md. '1
I~

www.petroman.ir --~ ~
i(;" ;-
, ;
" Ii
.', ..., ..,,",'y."'" \." "'llllIlitl\:U OllllUUP a/lU
dra\\do\\'11 c~Jrvc,~~llo\\:d Illar. dllrillg tIle wcllborc-
r
~ ['C)'7W(ll~ storage-dol11llKlted portloll or a test. tfle paralllctcr
1 c.-,['r- /'~"[08_~ khAIIJlCs was much more important in determining
... I df11qB than was the parameter kAllcPJlC,~. Ac-
ii -""':.':' ,'.. cordillgly. he leI kl<t>p.ctr~,,=lOx 106 md-psi/cp-sq rt
-IN' "'"" ,.. c"..,'n' ! (all average value) ror all his type curves. It is im-
~ portant to emphasize that even when klcPJlc,r;.varies
'- from this average value by one or two orders of J
Q. magnitude. the shape of the type curves is not af-
fected significantly. The reason for this ap-
proximation was McKinley's judgment that the loss
t \ '- ~::~c
-1tJO~ of accuracy
sitivityin the istype
more than offsetthat
curves-i.e., by the
theshape
gain of
in each
sen-,

I C m curve is distinctly different at earliest times (Fig. 4.6).


FLON TIME hr 4. To take into account the remaining parameters
.tflat do have a significant influence on test results,
Fig. 4.5 -Drawdown test analysis with Ramey's type McKillley plotted his type curves as AI (ordinate) vs.
curve. 5.615 C.fAplqB (abscissa), with the single parameter
kh15.615 CsJl. A small-scale version of McKinley's
curves i~ .~hown in Fig. 4.6.
I:rom thc lilnc match point, 5. Note that the skin factor s docs not appear as a
O.<XX>2(>4
k I parameter in the McKinley curves. Instead, ..
1/>«= -~ (-) McKi/llcy's curvc.'; a~~e~~damage or stimulation by
Jtr". I I) MP nolillg that thc earlic.~t wcllbore-storage-distorted
data are dominated by the effective near-well
= (O.<XX>264)(10.3) I transmissibility (khIJl)
in a wb;
testthus,
shoulda type-curve match
(08)(0 3)2 (193 JO4 ) of the.earliest ~ata allow calculation
" .x of thl~ quantity. later, after well bore storage
= I 96 x 10 -6 psi -I distortion has diminished, the pressure/time
'. behavior is governed by the transmissibility in ttle

Compare those with values used to determine C formation, khlJl; this quantity also can be estimated
from C : sD rrom a type-curve match -but for the later data only.
s 6. McKinley approximated boundary effects by
t/J(',=(0.2)(1 x 10 -S) plotting the simulator-generated type curves for c'!
6 about one-fifth log cycle beyond the end of wellbore
= 2 x 10 values in storage distortion (where the curve has the same
shape as for Cs = 0) and then making the curves ",'
= values out. vertical. This step roughly simulates drainage
.,.
4.4 Mc Kiniry s rype Curves conditions
curves of 40-acre
early-, spacing.
middle-, Note that regions
and late-time this gives the
-bul
McKinley~ propo~ed type curves with the primary remember that the curves were designed to be used
ohjective of cllara;:lcrizing damage or stimulation in primarily to analyze earlY-lime data. When the
a dra\\'down or buildup test in which wellbore storage curves are applied to drawdown tests, they"niusl be
dislorls most or all of the data, thus making thi~ applied to early-time data only; they do not properly
dlaractcrization po~siblc \\'ith relatively short-term simulate boundary effects in drawdown tests.
te~ts.
In con~tructing hi~ typc Cllrves, McKinley observed Use ()f McKinley's Type Curves :
that .toc ratio of pressure ~hange, df!, to flow rate Bcfore providing a step-by-step procedure for using
callslllg the cflange, qB, IS a function of scvcral McKinley's type curves, we note that he actually I
dimcnsionless quantities: prepared three different curves: one for the time '

I1IJ ( k/lAI kA.I r AI ) range 0.01 to JO minutes; one for 1 to J ,(xx) minutes;
-=f -, -:-2:' -.!.-, -.and Olle for 103 to J06 minutes. The curve for the
qlJ ItC s cPJlc,rIt' r". IP time range I to I,<XX>minutes is by far the most
Type curves with thi~ many parameters would be useful; accordingly, i! is the only one provided with
difficult, if not inlpossible, to u~e. Accordingly, this text. The complete set is provided with Ref. 1.
McKinley sinlplified the problem in tIle following The steps for using McKinley's curves follow.
way. I. Plot A.I (minutes) as ordinate vs. Ap=P;-Pw/
I. Hc a~~ul11cdthat thc \vell has produccd suf- (or PM~-PM'/) as abscissa on 3xS cycle log-log
ficicntly long (essentially to stabilization) that tIle last paper the sanle size as McKinley's type curve if
group, AI I II" is not imporlant. uJldistorted type curves are used. Otherwise. use
2. He ignored bOUJldary effects except ap- tracing paper for actual test data plotting. The time
proxil1lately and, tflus, ignored r ('Ir It. in the basic range on the axis should correspond exactly to one of
logic u~ed to constrnct the type curves. the type curves (e.g., it should span the time range of I

www.petroman.ir
'M~ \'ILLt Il~l~ USING IYPE CURVES -69 I

0 0
0 .., I I
~ f

°
1'
~ ~ii1

III

.11 Ii
.,
.
.i 11 .

f
-t -1

.
I I: : ,I:II!

--.

I.'" 10-1
IS... 10-1 I
IS'" 10'
PRESSuRE 8UILOUP GROUP, ~ 6146 6p C. ~
q8 .R8

Fig. 4.6 -McKinley's Iype curve.

l_minUIl'~t"O.OIIO 10minUlesor 103 to 106 transmissibility), shiff the data plot horizontally 10
J. find another type curve that better fils the later data.
~,h fill' lillII.' axi~ of the lest data plot with A shiff 10 a higher value of (kh/JJ.)/5.615 Cs in-
~ '\1..~llIlc~'~. ;\ltlVt: Ille dala along tile plOI dicales damage; a shift to a lower valLII.' indicale£.
I) tnu 1,"li(',,/ }h{jiill.!,' u/lol.'ed) untillilc slimllialion.
4&1, f.all illlllIg 0111.' of I hI.' typt: curves. 8. Calculatt: formalionlransmis~ibilily:
11«~\l111~ p..r.lml.'ll.'r \'alill.' (kh/JJ.)/5.615 C.\ kh/J(
"car~II)I~I.'urvl.'. -.(kh/I()j=(--) x(5.615Cs)s,,:ps,
.~, ..I.IIJ 111-.1\.'11pOInt (any ~p Irom tIle 5.615 C5 SI.:p7
.'fIb I\IIlX"r illlli till.' l'()rrl:~pollding valliI.' of Nole Illal \\'1.'do I/O! find a nl.'\\' pre~sllrl.' m.ltcll point
JitC.'IlHir,lllltIlI.'IYPl.'l:urve). 10 redell.'rmine C5; Cs is found onct: and for all in
IMnminl' till: \\l.'lll1orl.' ~Ioragl.' I:Onsl"11t C.\ SICp 5. In fad, if only d.lla rt:llel:ling formal ion
,.Iuc, of ~p:;: ~/IMP al1d 5. 615 tran~llli~,ibiliIY (afll.'r wl.'llborl.' ~Iorag\.' di~lorlion lIa~
,'tl- C~ hi S ~/'C °) IfIll) MI' al tile ma.ch poilll: di~appl.'arl.'d) arl.' analYlcd. errur will r\.'~ull l,~il1g IIII.'
"t.
~ .'61'
-.x \"C' /1/11).
"'" ~It
(,1/1 McKil1ley method. (However.
wor. kl Icre, so no pr.o bl eJu arIses.
'
collvemiol1al
) '. ' 1II.' mall: I I poInt
'
melllod~

.1I)~tt. 5.615 must be found with early, wellbore-storage-distorled


.Cawt" n~..r.\\1:11tran~mi~sibility. (kif/II.) K'b' data (Figs. 4.7 and 4.8).
...f\AI~mctcl \O1lul.'rel:ordl-d in Step 3 and the Flow efficiency also can be estimated fairly directly
IICW',C ,,)n~IO1ntdl.'ll.'rmined in Step 5: from the data plotted for use with MI:Kinl(y's tylX'
4hl,. curves.6 The definition of now efficiency, E, is
,
...S.6IS
( -) x5.615Cs'
CJ Mob
..
P -Pwr-Aps Ap -4ps
E-" "~J
.~ d.llol Irl.'l1d a~'ay from the type curve -p' -Pwf "~- -~. .

Ik cilrli(,t fit (indicating thaI formation


I)" i\ Jiff~rt:nt from effective near-well The quantifies .1p. and ~s call be estimated from

www.petroman.ir
","'~U
~",
the McKinley type curves in the following manner
(Fig. 4.9):
I. ~e is the vertical asymptote approached by ~
in lhe McKinley plot.
2. Aps can be calculated from Apd' the time at
~t which the actual test data depart from the earliest-
( ) fitting type curve. Picking a time of departure is
k h/)J. subjcctivc, so no grcat accllracy is as~lIrcd for llli~
reason alone.
5.6/5 Cs W McKinley6 states that Aps and Apd are related by
(
k wb
Ap S= 1-- kf Apd' )
~p
3. Thus, E can be calculated:
Fig. 4.7-Early data fit on McKinley's type curve. E = Ape -Ap S .
Ape

Example 4.2 -Drawdown Test Analysis


Using McKinley's Type Curves
Problem. &timate near-well and formation per-
meability and now efficiency using the data
presented in Example 4.1 from a drawdown lest on
~t an oil well.
( kh/ ) S(tluCion. We firsl prepare lhose data, most of which
Kn/Jj, lie in lhe time range I minute<l< I ,<XX> minutes for
56/5 C plouing as ~ (minutes) Ys. ~ =Pi -Pwf (Table 4.3).
0 .5 f From the dala plot (Fig. 4.10) and the match with the
best-fitling McKinley curve for the early dala,

~
P ( kh
-x
) 1
= 5,<XX>.
p. wb 5.615 Cs
We also note that the data depart from the best-
Fig. 4.8 -later data fIt on McKinleys type curve. fitting curve (early time) at ~/d = 100 minutes; here,

Apd=I,180psi.
A match point for the early fit is ApMP = 107 psi
when 5.615 11pCs/qB=O.OIO. The best fit of the
later dala is for (kh/lJ.)f( 1/5.615 Cs) = 10,<XX>.
Thus, from the match-point data,
Cs--(5.615i1pCs) ( ~ ) x --!!!!.- ..
qB MP ~ MP 5.615
I
~t I
---i) td 'p*
/Y =(0.010)
( -! I )( 500X 1.2 )
I ~ 107 5.615 I
J\h 1 .I
urd 1 =0.01 RB/psl

~p The near-well transmissibility and apparent per-


meability, kwL. are lhen

(~ ) = ( -~!!!- (5.615Cs) )
Fig. 4.9 -Data for flow efficiency calculation from p. b 5.615 Cs wb
McKinley's type curve. W
= (5,<XX»(5.615)(0.010)

.:.'" =281 md-ft/cp.

:f;I)1~ Then.
","~"o/r .,.-
'J,;;.."t.~ k b--- (281)(0.8)
-..4 01 md
ri:I't3i1er: " ", w (56) ~~1
,'i'uf&ui\r::a\ly
Cr. Ii\1
-"c-,,;..,~, . www.petroman.ir
ANALYSIS OF WELL TESTS USING TYPE CURVES 71

---: ---fd; Ij~


-~"I" "0000-
1~c, ' )
:

C C-=~T~E flIOM a!IGINAL


.E
-TYPE-CURVE MATCH
6l4 ' 100MIN
6P4'IIIOPSI 1500~
W .' '

~ TABLE 4.3- DRAWDOWN DATA FOR


i= McKINLEY CURVE ANAL YSIS

:> I I~J'~,OOO .11 P, -Pw/.11 P, -Pwl ..11 P, P..,


:> ~~c. (minutest (pSI) (minuiesl (pSI! (mlnules) (P51!
S 131 47 164 464 197 1.288
LL 164 58 197 531 229 1.304
197 70 229 592 262 1,316
229 81 262 648 295 1,326
'001 262 92 295 698 328 1,335
~ 295 103 328 744 393 1.349
I ~ 328 114 654 1,048 524 1,370
0 00 OX) 654 215 984 1.172 654 1.3t16

Pi -Pwf ' psi 984 307 131 1,232 984 1,413


131 389 ,~ 1.266

Fig. 4,10 -Orawdown test analysis with McKinley's type


curve.

The formation transmissibility and permeability are fractures with two equal-length wings were created.
(
kh ) (khlll.x5.615 Cs)f ( kh ) The ~urves discussed in this section assume '!"ijOffll
-= x -flux Into the fracture (same flow rate per Unit cross-
II. f (khl II.x 5.615 Cs) wb II. wb sectional area of fracture from wellbore to fracture
.:. tip). High fracture conductivity is required to achieve
-~~ x 281 uniform flux, but this is not identical to an infinitely
-5 ,(XX) conductive fracture
tip to wellbore), (no pressure
as Gringarten drop
el al. from fracture
demonstrated. 7
= 562 md-ft/cp, The study was made for finite reservoirs (i.e.,
boundary effects become important at later times in
and the test). The reservoir is assumed to be at unirorm
kh pressure, Pi' initiaUy. The type curve (Fig. 4.11),
kf= (-) x ~ developed for a constant-rate drawdown test ror a
II. f h slightly compressible liquid, also can be used ror
buildup tests (ror .11maxsO. I lp) and for gas wells.
08 using the modifications discussed earlier. Wellbore
= (562)( .-:.--) storage errects are ignored. ..
56 All the dimensionless variables and parame(ers
-8 03 d considered important are taken into account ill Fig.
-.m. 4.11, which is a log-log plot of Po vs. lor;; L} with
.parameter xelLj- In these parameters, Lf is the
Flow efficIency becomes fracture half-length and xe is the distance rrom the
.1p. ~ 1,500 psi (Fig. 4.10), well to the side of the square drainage area in which it
k is assumed to be centered. Dimensionless pressure
.1ps = (I --f) .1pt/ has the usual definition,
f P [) = kh(Pi' -Pwf)
w (drawdown test),
141.2qBII.
.1ps = (1- 401
~)(1,180) = 590 psi. and
8.03 I o~ 0,000264 kl
-u- = ~..~ L2 =IOL/. (4.17)
f I 500-590 1 IPlJCtL-f
l E~.= 0.~7. Several features of Fig. 4.11 are of interest:
1,500 1. The slope of the log-log plot is 1/2 up to
4.5 Gringarten et al.7 Type Curves 1OL ~0.16 for x!/~/> 1. This is linear flow. We
for Fractured Wells ha~ shown that, In linear flow,
I
Grin garten el al.7 developed type curves for ~
'ThIS slalemenl may be an orerslmplll'callOfl Some ~as wells exllltJ'l IlIlIe I'
hydraulically fractured wells In whIch vertical d"l"'ndellln"n.Darcyll"wlntllelracturt!.ulII'~t!llqUld~ 2 I

I
www.petroman.ir
..", ,L."""~U I

0 '
.
.

. .
. ,
..
I I
I .
.
,

,
,
.
'~
.I0 .
, I I

f I!,
I

. ,

,
...

10...
10.1 10.' I 10 G 10"
, ~,. 000026370'
.,.C,L,.
Fig. 4.11 -GrillQallen et a/. Iy~ curve 'or vertically. fractured well centered in closed square. no wellbore storage.
ullifo"" flux. I

Pi -PI,:r=C/I/I, PwJ) (buildup test) on the ordinate vs. 1 (drawdown j


or test) or ~/t' (buildup test) on the abscissa on a 3 x 5

I cycle log-log paper if an undistorled version of the


p/)=C'I/)/., "z. type curve is availablc. Otherwise, use tracing paper.
111cn 2. Select the besl malch by sliding the aclual test "

t data plot both horizontally and vertically.

logp/J=logc' + 1/210g1DL. 3. Nole the value of the match points [(PD)MP' ~


, (Pi -PMj) MP) and [(I DLj) MP' 'MP). ,
2 Altl I t t I I I l 4. Eslimate formation permeability from the 'i
..10Ug 1 no appar~n ?n a og- og pOt a ressure match oint: '
semllog plot of lhe data In Fig. 4.11 (p D vs. log P P ..

1m.,> is a straight line, signifyin.g rad}al flow.when, k=141.2~ (PD)MP. -.

for.\'('IL.r>S, 1lJ/~,=2. Thestralght-llnet.ermlnates, h (Pi-PMj>MP


of course, when boundary effecls b~com~ Important,S. Eslimate fracture length from the time match
but a match of actual test data wllh Fig. 4.11 can. t
sho\v the amount of data in lhe radial flow region poln : ~
[ 0.000264 kIMP ] 2 (4 J 8)
(and, thus, can be analyzed for permeabililY by the L -
thus
conventional
combines, PII.r in vs.
a single
log I or
graph,
Horner the plots).
linear Fig.
flow 4.1
and J f- <l>IJ.CI (I DL f ) MP

radial now regions (and a region wilh neither), Three useful checks are somelimes possible:
boundary effects, and the effect of various fracture I. If a half-slope (linear now) region appears on
Icngt hs. If fract ure conductivity is high and constant the test data plot, replot ~ata from the region as P wJ

1hro\tghout the test and if wellbore storage has (or PMOS) vs~ ..;t (or ~I ); from the slope mL and

ncgligible effect on earliest data, then this figure linear flow theory,

allows a rather complete analysis of a hydraulically 4 064 qB r;;


fractured well- specifically, estimation of fracture LJIt= -.:--~~,
Icn~tht LJt and formation permeability, k. The h"'L ~I

method is frequently superior to the nontype-curve which should agree with the result from the type-

methods discussed in Chap. 2. curve analysis.


Stcps in use of Fig. 4. I I as a type curve for test 2. If a radial now region appears (before boun-
analysis include the following. dary effects become important -that is, before the

J. Plot (Pi-PMj) (drawdown test) or (PM~- datadeviatefromthex~/Lf=~curve),aplotofPwf

www.petroman.ir
ANALYSIS OF WELL TESTS USING TYPE CURVES 73 ,

of;-
';;~
1i;
""oil

U)
a.
j
a.

C-
I
.
~

TABLE 4.4 -FRACTURED WELL BUILDUP I


TEST DATA

~
l
~~Lh,!u!S)
0
0.0833
Pws -Pwl
--(psi)
0
31
~t (hours)
0.75
0833
Pws -Pwl
(psi)
89
100
2::.:: Fff::~
TIM~O~;~~lroINT
6l"O062 Iv
MAltHfQHT
0.167 43 0.917 100 A -01
0.250
0.330 54
66 1.00
1.25 100
114 10 01 P"-P:f'150~ : --
0.417 66 2.00 136 II)
0.500 72 2.50 159 ~ t , hr
0.583 78 4.00 181
0.667 83 4.75 206 Fig. 4.12 -Buildup test analysis for vertically.fractured
6.00 218 well with Gringarten type curve.

vs. log 1 (pw!" vs. log ~I or log (lp+~I)/~I) should q = 2.750 STO/D.
show that k = 162.6 q8p.lmh. in agreement with type- p. = 0.23 Cpo
curve analysis. 8 ~ 1.76 RB/STB.
3. If a well proves to be in a finite-acting reservoir. h = 230 ft,
it may be possible to estimate xe from a matching t/> = 0.3, and
parameter, xelLf' to compare with the known (or c( = 30x10-6 psi-I.
assumed) value
match
of xe to check the quality of the S I r
0"100.
.
F 4 12 .
tg. .lsapoto
I .
f AI -..~
...,}-PII'~-PII'1'V!i.ul.
.An adequate fit is characterized by the maid. point!i .
(1=0.062 hour, IDL=O.OI) and (AjJ= 15.2 p!ii,
Example 4.3 -Buildup Test Analysis Po =0.1). From the p/essurematch point, :

fora Vertically Fractured ~Vell k=141.2(i!~~~.~~!!!~t'-'- ..


Problem. Gringarten el ul.13 presented build IIp tC!it II (~p) MP
data for a well believed to be fractured vertil:ally.
From these data, presented below and in Table 4.4, (141.2)(2.750)( 1.76)(0.23)(0.1)
estimate fracture length and formation permeability. = (230)(15.2)

Producing time, 1p' was significantly greater than


maximum shut-in tIme, so that ~1=~le. =4.5 md.

TABLE 4.5 -CONSTANT. RATE DRAWDOWN TEST DATA

t (hours) Pwl (psi) t (hours) Pwl (psi) t (hours) Pwl (psi) t (hours) Pwl (psi)
0.0109 2,976 0.218 2,611 3.28 1,712 32.8 1,543
0.0164 2,964 0.273 2,536 3.82 1,696 38.2 1,533
0.0218 2,953 0.328 2,469 4.37 1,684 43.7 1,525
0.0273 2,942 0.382 2,408 4.91 1,674 49.1 1,517
0.0328 2,930 0.437 2,352 5.46 1,665 54.6 1,511
0.0382 2,919 0.491 2,302 6.55 1,651 65.5 1,500
0.0437 2,908 0.546 2,256 8.74 1,630 87.4 1,482
0.0491 2,897 1.09 1,952 10.9 1,614 109.2 1,468
0.0546 2,886 1.64 1,828 16.4 1,587 163.8 1,440
0.109 2,785 2.18 1,768 21.8 1,568 218.4 1,416
0.164 2,693 2.73 1,734 27.3 1,554 273.0 1,393
327.6 1,370

www.petroman.ir
c ,:;t; TABLE 4.6 -BUILDUP TEST DATA

~I (flours) Pws (psi) ~, (hours) Pws (psi) ~I (ll0urs) Pws (pSi) .}I (flours) 'pWI (P~~
0.0 1,370 0.546 2,114 546 2,703 437 2.828
0.109 1,586 109 2.418 655 2,717 49.1 2,833
0.164 1.677 164 2.542 8.74 2,737 54.6 2,837
0.218 1.760 218 2.602 109 2.752 655 2.844
0273 1.834 2.73 2.635 16.4 2.777 87.4 2,853
0.328 1.901 328 2.657 21.8 2,793 109.2 2,858
0.382 1,963 3.82 2.673 27.3 2.805 163.8 2.863
0.437 2,018 4.37 2.685 32.8 2.814 218.4 2,864
0.491 2.068 4.91 2.695 38.2 2,822

Froln the time match point, of the MTR for the drawdown test.
0.<XX>264kAt V2 4.2 Analyze the drawdown test data as completely
Lf=( MP ) as possible using Ramey's type curves. Can data in
ct>JlC
I (t OL/) MP the L TR be analyzed with these curves? Why?
4.3 Analyze the drawdown test data using
[ (0000264)(4 5)(0062) '12 McKinley's type curves. Estimate k, k wb' and E. Can
=' .i ] the data in the L TR be analyzed with these curves?
(0.3)(0.23)(3 x 10- )(0.01) Why?
-59 7 f 4.4 Analyze the buildup test using the Horner
-.t. plotting technique. Estimate (I) k, s, E, 'wbs' and'i
Exercises at the beginning and end of the MTR, (2) jJ from the
E .MBH and modified Muskat techniques, and (3)
xam~les 4.1 and 4.2 were based on a portIon of the reservoir pore volume (using jJ before and after the
followIng ~ala for a drawdown test followed by a drawdown lest).
J1rcs~llrc btliidup tcsl. 4.5 Analyze thc buildtlp lest as completely as
q = 5(x) STOll) (con~tanl), pos~iblc using Ramcy's typc curvcs. Is there a shut-in
ct>= 0.2, time, Almax' beyond which the type-curve technique
Jl = 0.8 cp, should not be used? Why?
", = lOx 10-6 psi-I, 4.6 Analyze the buildup test using McKinley's
.Pi = 3,<xx> psi, type curves. Estimate k M'b' k, and E. Is there a shut-
, '" = 0.3 fl, in time beyond which the type-curve technique
h = 56 ft, should not be uscd? Why?
80 = 1.2 RU/Sl.U, 4.7 In the buildup lest analyzed in Example 4.3,
A 11"11 = 0.022 sq ft, does a linear flow region appear? If so, analyze the
p = 50 Ibm/cu ft, data using the conventional equations for linear flow
single-phase liquid, in a reservoir. Does a. radial flo~ region appear? If
liquid/gas interface in wellbore, and so, analyze the data usIng conventlon~1 methods:
r well centered in a cylindrical drainage area with 4.8 A d~awdown test was ru~ In. a vertIcally
fractured oIl well; the results are gIven In Table 4.7.
, t' = I ,<XX>ft. Using the Gringarten et 01. type curve, estimate
The drawdown (esl da(a are presented in Table 4.5; f.racture length and ~ormation pe!meability. I~ntify
buildup lest dala (I = 327.6 hours) are given in linear flow and r?dlal. now reglo?s and vertf~ the
Tablc 4.6. p type-cur~e analysIs with conyentlo.nal analysl~ of
these regions. As part of the conventIonal analysIs of
4.1 Using conventional analysis techniqucs (P"i the radial now region, estimate 'i at the beginning
v~.. log t p'IOl in ETR and MTR, PMi vs. I plot in and end of the MTR and estimate fracture length
I.IR). c~tlmatc k, S, E, tIl"II.~' Vl"'t (assumc from skill-factorcalculalion. Thclesl dala were as
cylindrical reservoir), and 'i allhe beginning and end follows.

TABLE 4.7 -FRACTURED WELL DRAWDOWN


TEST DATA

t (hours) Pwl (psi) , (hours) Pwl (psi) t (hours) Pwl (psi)


0 4,000 1.5 3,932 20 3,823
0.15 3.982 2.0 3.922 30 3.803
0.2 3,978 3.0 3,907 40 3,789
0.3 3,975 4.0 3,896 50 3,778
0.4 3,969 5.0 3,886 60 3,768
0.5 3,965 6.0 3,879 80 3,755
0.6 3,960 8.0 3,866 100 3,744
0.8 3.957 10 3,856
1.0 3,950 15 3,837

" -'" www.petroman.ir


ANALYSIS OF WELL TESTS USING TYPE CURVES -r---" -75

q = 200 STB/D (constant). 6. ~t..'J.;lnlc.':-,R.~I. "Esllmalinf fir-" Effl':lcn..'~ from Af-


B = I 288 RB/STB lerno,,-Disloned Pressure Buildur Dala," J. Pt'l. T«h.
h = 12f' (June 1974)696-697.
t, 7. Grln~arten, A.C., Ramey, H.J. Jr., and Raghavan, R.:
41 = 0.1. "Unsleady.Slate Pre~sure Distributions Cruled by a We]!
II- = O.S Cpo With a Sinile Infinile-Conducti\ily Vertical FraCture," Soc.
C = 20xI0-6psi-l.and Pel.Eng.J.(AugI974)347-360;Trans,AI~IE,2S7.
I llb I d . d .. I .. bl II 8. Agar"al, R.G.: "A Ne" Method To Accounl for Producing
~e ore un oa Ing Istortlon neg 19l e at a Time Effects When Dra~do~-n Type Curves are Used to
tImes. Ana]yzc Pressure Buildup and Other Test Data," paper SPE
9289 presented at the SPE 55th Annual TechnicalConference
References and Exhibition,-held in Dallas, Sept. 21-24, ]980.
9. Ramey, H.J. Jr.: "PracticaJ Use of Modem Well Test
I. Earlougher, R.C Jr.: Advances in H.ell Tesl Analysis, Analysis," paper SPE 5878 presented at the SPE-AIME 51st
Monograph Series,SPE. DaJlas(1977)5. Annual TechnicaJ Conference and Exhibition, New Orleans.
2. Ramey, H.J. Jr.: "Shon- Time Well Tesl Data Interpretation Oct. 3-6, 1976.
in the Presenceof Skin Effecl and Well boreStorage," J. Pel. 10. AJ-Hussainy, R., Ramey, H.J. Jr., and Cra~.ford, P.B.: "The
Tech.(Jan. 1970)97-104; Trans., AIME, 249. Flq~ of Real Gases Through Porous Media," J. Pel. Tech.
3. Agarwa], R.G., AJ-Hussainy, R., and Ramey, H.J. Jr.: "An (May 1966)624-636; Trans.. AIME, 2.31.
Investigation of Well bore Storage and Skin Effect in Unsteady II. Wattenbarger, R.A. and Ramey, H.J. Jr.: "Gas Well Testing
Liquid flo~': I. AnaJ)1icaJ Treatment," Soc. Pel. Eng. J. With Turbulence, Damage, and WeUbore Storage," Trans.,
(Sept. 1970)279-290; Trans.. AIME, 249. .AIME,(I960) 243,877.887.
4. Wattenbarger, R.A. and Ramey, H.J. Jr.: "An Investigation 12. Holditch, S.A. and Morse. R.A.: "The Effecu of Non-Darcy
of Well bore Storage and Skin Effect in UnsteadyLiquid flow: flo~' on the Behavior of HydraulicaJly Fractured GasWells,"
II. Finite-Difference Treatment," Soc. Pel. Eng. J. (Sept. J. Pel. Tech. (Oct. 1976)1169-1178.
1970)291.297; Trans.. AIME, 249. 13. Gringanen. A.C.. Ramey, H.J. Jr., and Raghavan, R.:
5. McKinley, R.M.: "WeUbore Transmissibility from Af- "Pressure AnaJysis for Fractured "'ells." paper SPE 4051
ternow-Dominated Pressure Buildup Data," J. Pel. T«h. presented at the SPE.AIME 47th AnnuaJ fall Meeting, San
(July 1971)863-872; Trans., AIME, 251. Antonio, Oct. 8-11,1972.
www.petroman.ir
I l
Chapter 5
Gas Well Testing

S.l Introduction
This c.hapte~di~cussesdeliv:rability tests of g.aswells. I,(.() = 1,(.(-) -50 300 & !!.&:!.
The dISCussionIncludes basic theory of transient and
pseudosteady-state flow of gases, expressed in terms
P"'f P , T
sc
kh n r
rLl ( ~'" )
of the pseudopressure I,(.(p) and of approximations
to the pseudopressure approach that are valid at high -0.75 +S+D/qg /. ] (5.3)
and low pressures. This is followed by an
examination of flow-after-flow, isochronal, and Eqs. 5.1 and 5.3 provide Ihe basis for analysis of
modified isochronal deliverability tests. The chapter gas well tests. As noted in Sec. 2.11, for P > 3,CXX>
psi,
concludes with an introduction to the application of these equations assume a simpler form (in terms of
pseudopressurein gas ~ell test analysis. pressure, p); for p < 2,000 fsi, they assume another
S.2 BasIc.'. Theory of Gas Flow In Reservoirs procedures
simple formfor(inanalyzing
terms of gas
p ).~'ell testswe
Thus, with equations
can develop
Investigations 1.2 have shown thaI gas flow in in- intermsOf~(pj,p,andp2.1nmostofthisl.'haPter,finile-ac[ing
reservoirs can be expressed by an our equations will be written in terms of p2 -nol
equation similar to that for flow of slightly com- because p2 is more generally applicable or more
pressible liquids if pseudopressure I,(.(p) is used accurate (the e~uatio;}s in I,(.best fit this role), butnstead
of pressure: because the p equations illustrate the general

p q T[ method and permit easier comparison with older


~(P"1) =1,(.(Pj) +50,300-!f..:!:K.:. 1.151 methods of gas well test analysis that still are used
Tsc kh widely.
Before developing the equations, let us generalize
I 688 ~#l..c.r 2
.Iog(~ I ~ )-(S+D/qg/) ] , Eq.
uniform 5.1 to drainage-area
model a drawdownpressure test starting that
<PI from
may any
~

kt much lower than initial pressure (p,) after years of

(5.1) production:
~Ierethe pseudopressure is defined by the integral

,P P
~(P"'f) = ~(jJ) + 50,300~

sc
!ff
lr1.151
I,(.(p)=21 -dp. (5.2)
J JJ.Z
PJ
e term Diqgl reflects a non-Darcy flow pressure 'Iog ~8 (j)#l.DCtD~
kt (
)-(S+D/qg/) . ]
5-i.e., it takes into account the fact that, at high !
near the producing well (characteristic of (5.4) j
g~ gas production rat~s), ~arcy's law does not where p=jJ for ail r at tp=0. For p<2,1XX> psia, '
'dJct correctly the relationshIp between flow rate pZg ~constant =.upZpg for most gases; in this case,I
pressure drop. As a first approximation, this 2 2 2
litional pressure drop can be added to the Darcy's ~(p) = -(~ -~ ).
pressure drop, just as pressure drop across the #l.pZP8 2 2.:red
zone is, and D can be considered constant. Subslituting into Eq. 5.4,
~ absolute value of q" Iq I, is used so that the
n D!q,! is positive for eitter production or in- q #l.-Z- T
ion. ~'---,~"'D.t~ - )
Pwf2 =jJ2 + 1,637~X"'~~2I..:. r log( '1,688~#l.-c
'or stabilized flow3 (r, ~ r ~\ www.petroman.ir
GAS WELL TESTING ~IJ 77

_ ( s+Dlq.rI )] (5.5)
1.151
For stabilized now, ~4

p",j2=p2_I,422q.r~pZP.rT
kh [ ln(!L)
rw I
-0.75+S+Dlq,l} (5.6) qg ~I

Eq. 5.6 is a complete deliverability equation. Given a


value of flowing BHP, Pwj' corresponding to a given I
pipeline or backpressure, we can estimate the rate q, t, t2 t3 l4 !
at which themust
parameters well be
willdetermined
deliver gas. However,
before certain
the equation t ~ i'

can be used in this way:


I. The well flowed at rate q until rj ~r e
(stabilized flow). In this case, note ttat Eq. 5.6 has p
the form

p~-pwj2=aqg+bqg2, (5.7) I

where
[ ( r )-
~ -Z -T
a=I,422"'P..pg-ln -!-- -0.75+s, J (5.8) f?vf
kh rw Wf ,4
and tl l2 t3 4
~-z- T
b=I,422"'P~pg-
kh D. (5.9) t .
The constants a and b can be determined from
flow tests for at least two rates in which q and the Fig. 5.1 -Rates and pressures in flow-after-flowtest. -
corresponding value of P wj are measure~; p also
must be known.
2. The well flowed for times such that rj ~r e
(transient flow). In this case, we will need to estimate
kh, S, and D from transient tests (drawdown or
buildup) modeled by Eq. 5.5 (or some adaptation of
it using superposition); these parameters then can be ~~:~!_~~.! I
combined with known (or assumed) values of p and + I ,.'i,!
rein Eq. 5.6 to provide deliverability estimates. : iSlOPE ;; STAB" E
The gas flow rate qg' used in Eqs. 5.1 through 5.7, i '-'I"", : -!- ~L'".E~A~llITY
should include all substances that are flowing in the p2 -P 2 I : :uRvE
vapor phase in the reservoir, with their volumes wf i :;
~.xpressedat standard conditions. These substances ;/ : ..-,_AesCL~E OPEN..
Include the gas produced as such at the surface, and ,.~ FLOW JTE~TIAL rAQF)
condensate and liquid \\'ater produced at the surface a -
that existed in the vapor phase in the reservoir. -g
~alculatio~ of the vap,or equivalent of condensate is Fig. 5.2-Empirical del!verability plot for flow.after-flow
discussed In Appendix A of Ref. 4. Craft and test,
Hawkins5 summarize the calculation of the vapor
equivalent of produced fresh (non formation) water.
Most of the remainder of this chapter provides
detailed information on testing procedures that lead
to estimates of the parameters required to provide
deliverability estimates- This discussion is based on
recommendations in the ERCB gas well testing
5.3 Flow-Arter-Flow Tests
manual.4

In this testing method, a well flows at a selected


constant rate until pressure stabilizes-i.e.,
pseudosteady state is reached. The stabilized rate and
pressure are recorded; rate is then changed and the

www.petroman.ir
~
78 WELL TESTING

TABLE 5.2 -STABILIZED FLOW TEST ANAL YSIS

TABLE5.1-STABILIZEDFLOW P., qp p2_p.,2 (p2_p2.,)/qp


TEST DATA ~ (MMscf/D) (pSia2) (psia2/MMScf/D)
4082 0 --
~ P., (PSIa) qg (MMscf/DI 4031 4288 4.138 9649
1 403.1 4.288 394.0 9265 11.391 1,229
2 394.0 9.265 378.5 15.552 23.365 1.502
3 378.5 15552 362.6 20.177 35.148 1.742
4 362.6 20.177 14.7 AOF 166.411 -

well flows until the pressure stabilizes again at the


new rate. The process is repeated for a total of three Example 5.1- Stabilized Flow Test Anal
y:sis
or four rates.
Rates and pressures in a typical test follow the Problem. The data in Tabl.e 5.1 w~re reported for a
pattern indicated in Fig. 5.1. Two fundamentally flow-after-flow (or four-point) test In Ref. 6. At each
different techniques can be used to analyze these test rate, pseudosteady ~tate was- reached. Initial (i.e.,
data. before the test) shut-In BHP, p, was determined to be
408.2 psia. Estimate the AOF of the tested well using
Empirical Method (I) the empirical plot and (2) the theoretical flow
An empirical observation -with a rather tenuous equation, In addition, plot deliverabilities estimated
theoretical basis -is that a plot of ~2 = p2 -Pwl using the theoretical equation on the empirical curve
vs. q (Fig. 5.2) on log-log paper is approximately a plot.
straig~t line for. many wells in w~ich the Solution. We prepare a table of data (Table 5.2) to be
pseudosteady
after-flow teststate IS reached
sequence. at each rate
The equation In aline
of the flow-
in plotted for .,both empirical and theoretical analyses.
this plot is I. Empirical Method. From a plot of (jJ2 -Pwf)
vs. qg on 10g-lo~ paper, and extrapolation of this
q g = C( yn1- p w12 ) n (5 10) plot to p2_Pwj =166,411 (where Pw1 =0 Psig or
14.7psia),AOF~60MMscf/D.
This plot is an empirical correlation of field data. The slope of the curve, l/n, is
As in any other empirical correlation, there js sub-
distance risk
stantial of error
beyond the in extrapolating
region in which the
dataplot a large
were ob- I/n= log
IV~ (p2
\P -P
-Pwj 2)12 -log
-IV& (jJ2
\P -P-Pwf 2)I(
tained. Unfortunately, such an extrapolation is log qg.2 -log qg.1
frequently required. To estimate the absolute open
flow potential (AOF) -the theoretical rate at which I ( 105
the ~'ell would produce if the flowing pressure P"'j og 103 )
were atmospheric -it may be necessarv to ex- = = 1.449.
trapolate the curve far beyond the range ortest data. log ( ~1.77 )
An AOF determined from such a lengthy ex-
trapolation may be incorrect.
The constants C and n in Eq. 5.10 are not con- Thus, n = 0.690. Then,
stants at all. They depend on fluid properties that are q
, pr~ssure (and, th~s, tim.eJdependen.t. According,ly, ,if C= -2"Og 2 n ..
! thIs type of dellverablllty curve IS used, perIodic (p -Pwf )
retesting of the ~ell will sho~' changes in C and
perhaps in n. 42.5
We must emphasize that deliverability estimates = ~ 0690 = 0.01 508.
! based on this plot assume that pressures were (10-) ,
stabilized (r/~rt') during the testing period used to Th th ., ld I. b.r ..
construct the plot. If this is not the case, stabilized us, e empmca e Ivera I Ity equatIon IS
deliverability estimates from the curve can be highly
misleading. qg = 0 .-V'
01 ~08{,;2 -2 Pwf )°.690.

Theoretical ~ethod These data are plotted in Fig. 5.3.


Eq. 5.7 suggests that we plot (p2 _p",j2)/qg vs. qg; 2. Theoretical Method. The theoretical deliver-
the result (for pseudosteady-state flow) should- be a ability equation is
straight line with slope b and intercept a. Becausethis
line has a sounder theoretical basis than the log (p2 -Phf2)/qg =a+bqg'
Ap2 -log qg plot, it should be possible to extrapolate
it t.o det~r!Dine ~OF with less erro~ and to correct Fig. 5.4 is a plot of(jJ2 -Pwl)/qg vs. qg for the test '
dellverabll!ty estImates for changes In JAp' 'Pi' etc., data. T~'o points on the best straight line through the i
more readIly. data are (2.7; 900) and (23.9; 1,900). Thus, ;
..
www.petroman.ir
GAS WELL TESTING 79I

I,
I '
0 !

L1::
U I
U') j
!
~
00::::
tt
i ..::: ;
" """ I j
II -"..
PwI.14 7 DO'.
THEORETICAL
ACF ~
:::== ~ I I
It
'

N VI
2 I~ I 0- I
8. I -I
-!
I 0" I I,
N I ~ .i
"+- .i-I !
~ I r N i

NI I~ I ~ I~
IC- ! ! ! I: I
I !

IJ
I
; (\J
10-
'-'"
.
I

! "EMPIRICAL
/
ACF I
" () 100 0 4 1 16 20 24

qg ,MMSCF/D qg ,MMSCF/D

Fig, 5.3-Stabilized gas well deliverability test, Fig. 5.4-Stabilized deliverabilitytest, theoretical flow
equation. constantdetermination,

900 = a + 2.7 b, lower-permeability reservoirs. where it frequently is


impractical to achieve 'j =, l' during the test. An
1,900 = a + 23.9 b. isochronal test is conducted by flowing a well at a
fiXed rate, then shutting it in until the pressure builds
Solving for a and b, we find that a = 773 and up to an unchanging (or almost unchanging) value,
b = 47.17. Thus, the theoretical deliverability jJ. The well then is flowed at a second rate for the
equation is same length of time, followed by another shut-in,
etc. If possible, the final flow period should be long
47.17 q g 2 + 773 q g = (p2 -P wf2). enough to achie..e stabilized flo~'. If this is im-
possible or impractical, it is still possible to predict
We can solve this quadratic equation for the AOF: the stabilized deliverability characteristics (with
increased potentiaJ for error, of course).
47.17qg2+773qg-166,411=O. In obtaining data in this testing program, it is"
essential to record flowing BHP, P \of' as a function
The solution is of time at each flow rate.
Fig. 5.5 illustrates rate and pressures in an
qg = AOF = 51.8 MMscf/D. isochronal testing sequence.This figure illustrates the
following important points about the isochronal
We also can determine points on the d~liverability testing sequence.
curve as calculated from the theoretical equation: 1. Flow periods, excepting the final one, are of
p2 -P\of2 = 47.17 qg 2 + 773 qg' See Table 5.3. These equallengt~ [i.e.,.fl = (f3 -f~) = (f 5.- f~)~ (/7 -16.»)'
results are plotted In Fig. 5.3. The plot is almost 2. Shut-In penods have the objectIve of letting
linear, but there is sufficient curvature to cause a P ~ jJ rather than the objective of equal length. Thus,
15.8070error in calculated AOF. in general, (f2 -fl) ~ (f4 -f3) ~ (/6 -f5)'
3. A final flow period in ~'hich the well stabilizes -".-
5.4 Isochronal Tests (i.e., 'j reaches, t at time I,) is desirable but not
The objective of isochronal testing 7 is to obtain data essential.
to establish a stabilized deliverability curve for a gas The most generaJ theory of isochronal tests is
well without flowing the well for sufficiently long to based on equations using pseudopressure. However,
achieve stabilized conditions (r j 2::,l') at each (or, in we will once again present the theory in terms of the
some cases, any) rate. This procedure is needed for low-pressure approx.imations (0 these equations ~

www.petroman.ir
~
80 WEll TESTING

I
" q

q
q
qg Q.

l, l2 t, l. t t6 t7
TABLE 5.3 -THEORETICAL DEUVERABIUTIES t

qg p2 -Pw/2 ~ f
(MMsC/D) (psia2) W

4.288 4,182
9265 11,210 to. t2 l3 t. ls l6 t7
15552 23,430
20 177 34,800
30 65,640 t ~
40 106,400
49.8 = AOF 166,600 Fig. 5.5 -Rates and pressures In isochronal test.

equations) because (I) they are somewhat simpler points (q , p2 -PWf2) obtained at that time at
and less abstract than equations in pseudopressure several 'different rates, and a truly stabilized
and (2) they allow direct comparison with more deliverability curve can be drawn when r j ~ r ~.
conventional analysis methods 7 based on plots of These assertions can be made more quantitative if
(jJ2 -Pwf2) vs. q on log-log paper. we note that for flowing time t, at each rate, there
Eqs. 5.5 and !.6 provide the basic method for corresponds a drainage radius, rd=crj, that is in- I
interpreting isochronal
For transient flow (rj tests.
< r ~), dependent of rate. Admittedly anticipating a log
venient result, we let r d = 1.585 rj (but the p vs. con-r ,'

2 -2 p.-Z -plot constructed in the solution to Problem 1.2 shows


( P wf = P + 1,637 ~ Tq g that such ~ r d approxi~ates qu.ite.clos.ely the .point
beyond whIch no apprecIable fluId ISbeIng drained). ;
At time tt,
l
. lOg ( 1,688<bp.pCtprW2
kt
)_(~..:.!!lg-zJ
'. 1.151~'
)J 2- -2 ~
Pwf -P +1,422 kh Tqg
~
I

"
(5.5) [ I I 688'"
In (.,
-c
"p.p~/p.w )
-r 2
I
I A

For stabilized flow (rj ~r ~), .T kll

Pwf-., =jJ2 -1,422 p. -.-Tq '..g


'-P"PE [In( -!-)r -(S+Dlqgl) J.
kh rw
1 Because I

-O.75+(s+Dlqgl). (5.6), "9,

J rd2=(1.585)-r;2 ..I

In addition to the flow equations, an important <2 !


theoretical consideralion in understanding iso- = (1.58~) kt I !
chronal tests is the radius-of-investigation concept. 948 (j>p.pCtp j
We observed previously that the radius of in- i
vestigation achieved at a given time in a flow test is -kt I
-,
independent of flow rate and, thus, should be the 377 (j>ppctp
same at a given time for each flow rate in an
isochronal test. Further, the radius of investigation at we may write Ihe transient flow equation as
a given time can be considered to be proportional to a 2 -2 "PpZpg Tq, r 'rdl
d~ainage radius at that time, because it is near (but Pwf = P -1,4,,2 kh lln(- )
slightly less than) the point beyond which there has rw
been no appreciable drawdown in reservoir pressure .
a?d thus no fluid drainage. Accordingly, at a given -O.75+(s+Dlq OJ.:
tIme, the same portion of the reservoir is being ,
drained at each rate and, as a good approximation, Compare with the stabilized deliverability equation:
stabilized flow conditions exist to a point just beyond 7j
r=rj. Thus., a deliverability curve can be drawn at Pwf2=p2_I,422PpZPb.q~
each fixed tIme (hence, the name isochronal) through kh rw )
[ ln(.!.!- I

www.petroman.ir
".,1"- 1:'.11
P
GAS WELL TESTING 81

[ -2 -2 ] TABLE 5.4-ISOCHRONAl TEST DATA

P -(14.7) Pw/
DuratIon or Pws Qg

---I Test (hours) (pSi a) (MMscf/D)


I .-
I-STABILIZED Initial shut-In 48 1.952 -

N '+-
SL I
I
POINT First
First
flow
shut.in
12
15
1.761
1.952
2.6
-

~ = I I Second flow 12 1.694 3.3

l3~ Second Shut.in 17 1.952 -


I I Third flow 12 1.510 5.0

N t2: Third shut-In 18 1.952 -


10- I Fourth flow 12 1.320 6.3

t ~ Extended flow (stabilized) 72 1,151 6.0


I I Final shut-In 100 1.952 -
I
~ :
0 I
-: ~g = AOF TABLE 5.5 -ISOCHRONAL TEST ANAL YSIS

qg ~2 -Pw/2 (~2 -Pw/2 )/qg


(MMscf/D) (psia2) (psia2/MMscf/D)

10 9 q 9
2.6
3.3
709.000
941.000
273.000
285.000

5.0 1,530.000 306.000


Fig. 506 -Empirical deliverability plot for isochronal test. 6.3 2.070,000 328.200

-0.75 + ( s+D I I)] stabilized flow ~d transient flow written in the form

q, .below. For stabtltzed flow. .

The equations are identical in form because we have -2 -2 -+b 2 (5 7) ;

definedatime-dependentdrainageradius,rd'as P Pwf -oq, qg' "

( kt ) Y2 where .
,
rd= ..

3774»#J.pctp #J..Z- T
0 o=1422"'P~pg.ln--O.75+s [ ( r, ) ] 5.8 :,
Thus, we conclude that, at each fixed tIme t I' an .kh r w ,( ) .'

analysis of different rates used in an isochronal test d

should be possible just as it is for a stabilized test -an *

except, of course, the data will not yield a truly #J.pZpg TD ..


tb ' l o ddl ' b o l o Th o. . bl nl ' f b=I,422 (5.9)
s a I Ize e Ivera I Ity curve. IS IS poSSI e 0 y I kh

stabilized data are available or if they can be F .


estimated. or transIent fl ow,

,"

p2_Pwf2=Otqg+bqg2, (5.11)
Analysis of Test Data:

One Rate Continued to Stabilization where b has the same meaning as for stabilized flow

E.xperience 7 shows that reasonably satisfactory and where or' a function of time, is given by ..

results can be obtained with the empirical method

using
I. The
the following
best straight
procedure.
line is drawn through the 0/-
-I
.kh
422!.£!:iK!.
[~ 2
I( n 1,688
kt
4>#J.pcrprw2
) +
s. ]
points <.02 -Pwf2, qg) obtained at a fixed value of

time with the different rates used in the isochronal (5.12)

testing programo The data are plotted on log-log

paper, just as when analyzing a stabilized Thus.

deliverability curve.

2. Lines should be drawn for several values of p2_PWf2=0/qg+bqg2,rj<re' (5.11)

time t, and the slope tin should be established for

each isochronal deliverability curve. and

3. A line with the slope Iin determined from the


...0 -2- 22 ._~
nonstablllzed, fIXed-tIme curves then IS drawn P -Pwf =oqg+bqg ,rj:2:r,. (5.7)

through the single stabilized point, (qg' p2 -Pw!)'

This establishes the stabilized deliverability curve. An analysis method of isochronal tests consistent

Once the stabilized deliverability curve is determined, with the theoretical equations follows.

AOF is established in the usual way, as indicated in I. For a j"rxed value of t, determine b from a plot

Fig. 5.6. of <.02 -Pwf2)/qg vs. q-K'

The theoretical method for analyzing isochronal 2. Using the stabilIzed data point {qgs' <.02-

test data is based on the theoretical equations for Pw!~]' determine 0 from

www.petroman.ir .~ .
www.petroman.ir
I. GAS WELL TESTING 83

S'
t / q
9
l ql
__~~__r~4
q, q iW
EXTEND
PERIOD

......
~S I= S P" P~L Pws !?,s3 ~S4

0 Pwf2 ~~~
Pw PWf3 ~~S(STA8LE)

qg ~
TIME .
Fig. 5.9 -Skin factor determination, Fig. 5.10 -Rates and pressures in modified isochronal
test

where dra",'do\\n le~ts run at different rate~ or buildup lests

a= 1,422
Il'Z'
"'p~pg'
T[In(-!-)-O.75+s,
r ]

(5.8)
following

then plot s'


dra",'do",'n

vs. QI!;
tests

extrapolation
al different

10 q'!
rates.

=0
We

pro\'ides
~an

kh rw an eslimate of true skin fa~tor. s (~ee Fig. 5.9). The


and drainage radius r e mu~t be estimated from expected
-1l-Z -TD ""ell spa~ing (or kno",'led~e of re~ervoir geometry in a
b= 1,422-'""P~pg' (5.9) small or Irregular reservoir).
kh The ~onstants a and b determined in thi~ way then
We also noted that the transient equation has the ~an be substiluled inlo the stabiliz;d deli~erability
form equalion. Eq. 5.7. If a plol of log (p- -P'I'f-) vs.log
QI! also is desired. data points 10 be plolted ~an be
p2 -pwf2=a{Qg+bQg2, (5.11) determined from theequalion.

where 5.5 Modified Isochronal Tests

a {,= I 422!:J!..~
kh [ ~2 In( kt
1,68841p.-c{-r 2
)+ ]
S
The objective of modified iso~hronal
oblainlhesamedataasinanisochronaltestwithout
tests is to

ppw ' h . I hh ' ' d .

u~lng t e ~ometlme~ engt y ~ ut-in perlo s requlr ed

, (5.8) for pre~~ureto slabilize complelel~' before ea~h no",'


lest is run, -.
The objective of determining Ihe stabilized flow In the modified isochronal test (Fig, 5.10), shut-in
equation I:an be a~hieved if the constants a and b can periods of the same duration as the now periods are
be delermined. We note that Ihe ~onstant b ~an be used, and the final shut-in BHP (P..'s) before the
determined from the iso~hronal test data as beginning of a new now period is used as an ap-
illuslrated [b~' plolling (p2 -p..,{2 ) /QI! vs. QR for proximation to p in the test analysis procedure. For
fi,xed value of ( and determining the slope. b. of the exam~le. for the first now period, use (i>2-
resulting beSI straighl line]. Constant a, ho\\ever, is Pwf-' )=(PwS,12 -tWf,,2); for the. second flo:-"
more troublesome: The onl~' salis factory means of perIod, use (P~,2 -Pwf.22). OtherwIse, the analysIs
determining a is through knowledge of each term in procedure is the same as for the "true" isochronal
Ihe defining equation for Ihis quantilY. Thus, we test.
need estimate~ of kh. s. and r 1" (Other quantities in Note that the modified isochronal procedure uses
Eq. 5,8 are usually available.) approximations. Isochronal tests are modeled exactly
Since an isochronat test-consisis of a series of by rigorous theory (if reservoir and fluid properties
drawao\\n and buildup lests, kh and s usually can be cooperate); modified isochronal tests are not.
determined from them. Delermination of kh is However, modified isochronal tests are used widely
straightfor""ard in prin~iple; determination of s is because they conserve time and money and because
less straighlforward. Recall that a single test provides they have proved to be excellent approximations to
only an estimate of s' =S+DQRi. Accordingly, 10 true isochronal tests.
determine s, we must analyze at least two tests: either

www.petroman.ir
84 WELL TESTING

TABLE 5.6 -MODIFIED ISOCHRONAL TEST DATA

Duration or P..
P., qg
Test (hours) (psia) (MMscf/D)
Pretest shut.in 20 1.948 -
First flow 12 1.784 4.50
First shut.in 12 1.927 -TABLE 5.8 -THEORETICAL STABILIZED
Second flow 12 1.680 560 DELIVERABILITIES
Second shut-in 12 1.911 -
Third flow 12 1.546 6.85 qg p2 -P ,2
Third shut-In 12 1.887 -(MMscf/D) (Psla~)
Fourth flow 12 1.355 8.25
E~tended flow (stabilized) 81 1.233 8.00 ~.~ 1 ~.~
Final shut-In 120 1.948 -6:85 1:794:000
8.0 2.274.000
TABLE 5.7 -MODIFIED ISOCHRONAL TEST ANAL YSIS 10.8 3.660.000

qg p.. P., Pw.2_p.,2


~~~} J.P~ (psial (Psla2) (Pws2-Pw,2)/qg 0
4.50 1.948 uBi 612.048 136.011 i:L::
560 1.927 1.680 890.929 159.094 U
68S 1.911 1546 1,261.805 184205 (/)
8.25 1.887 1.355 1.724.744 209.060 ~
8.00 ~
(stabilized)
.HOle thaI I' the1.948.
true current1.233
reservOIr 2.274.415
pressure. IS used lor the stabIlIZed N

test analysIs <J:


V5
Q.. .I

C'

I -~
I N

N
"--_!!'_:'!.~_7.._~!..~
:
~I
I NIJ)

<t ' : ~
-: : QS. 0 2 .6 .
U) i : I -
N
Q..
-..a'
Iv-
T)£OfIET
,
I
:
.q g . MMSCF/D

-' I
I
~ STASlL'ZED: ! Fig. 5.12 -Modified isochronal test analysis. theoretical
I i OEUVERAel-rrY " : flow eq uation, constant determination.
N ." : .',I
~ ", : .,
I

:
TRANSIENT:
DELIVERASILlTY
:
:..
; this value IS A OF 8 MM
= 1 0 .sc. flD
i : , For the theoretical method, we establish the
: i ACF. 108MMSCF/D i con~tant b from. the. slope of a plot of <Pws2-

10'1 ~VM~r-,~g ~ Pwf )/qgvs.qg;mthlscase,


243,(XX} -48,(XX)
qg' MMSCF/D b= 10 = 19.500, ..

using data from Fig. 5.12. Then,


Fig.5.11-Modifiedisochronaltestanalysis. ( -2 2 ) b 2
a=.. IP -Pwf
--J~ Js-vq.s -~.

qgs
-2,274,415 -(19,500)(8.0)2
Example 5.3 -Modified Isochronal -8.0
Test Analysis
Problem. Estimate the AOF from the data in Table = 128,300.

5.6 obtai!1~d in a modifi~ isochronal test,4 using Thus. the equation of the stabili d d r b1"
both emplTlcal and theoretical methods. curve is ze e Ivera I Ity

Solution. We first prepare the data for plotting


(Table 5.7). Fig. 5.11 shows the data riot for the p2 -PWf2 = 128,300 qg + 19,500 qg2.
empirical method. This is a plot of <Pws -Pwf2) vs.
qg on log-log paper. The transient points are used to Solving this equation for the AOF (q when p2 -
establish the slope of the curve, and a line with the PWf2 = 3. 790.(xx}), we find that it is lqual to 11.0
same slope is drawn through the single stabilized MMscf/D.
point. The AOF is the value of qg when Pws 2 -p,":~2 It is also of interest to calculate points on the
= p2 -Pw! = 1,9482 -14.72 = 3,790,(XX) psia; stabilized deliverability curve and to plot them on the

www.petroman.ir
GAS WELL TESTING '...'1111- 85

TABLE 5.9 -GAS PROPERTIES TABLE 5.10 -PSEUDOPRESSURE


FOR EXAMPLE 5.. FOR EXAMPLE 5..

P 1'9 pllAZ ~(P}


(psia) (Cp) z (psia/cp) P (pSia) (pSia2/cp)
--;-so 0:01238 ~ 12.290 150 1.~ X106
300 0.01254 0.9717 24.620 300 7.381 X 106
450 0.01274 09582 36.860 450 1.660 X 107
600 0.01303 0.9453 48.710 600 2.944 X 107
750 0.01329 0.9332 60..70 750 4.582 X 107
900 0.01360 0.9218 71.790 900 6566 X 107
1,050 0.01387 0.9112 83.080 1.050 8.888 X 107
1,200 0.01428 0.9016 93.205 1,200 1.154x1OS
1.350 0.01451 0.8931 104.200 1,350 1..51x1OS
1,500 0.01485 0.8857 114,000 1,500 1.779 X 108
1,650 0.01520 0.8795 123.400 1.650 2.135x108
1.800 0.01554 0.8745 132.500 1,800 2.518x 1OS
1,950 0.01589 0.8708 140,900 1,950 2.929 X 108
2.100 0.01630 0.8684 148.400 2,100 3.363 X 108
2.250 0.01676 0.8671 154,800 2.250 3.817 X 108
2.400 0.01721 0.8671 160.800 2.400 ..291 X 1OS
2.550 0.01767 0.8683 166.200 2.550 4.781 X 108
2.700 0.01813 0.8705 171.100 2.700 5.287 x 108
2,850 0.01862 0.8738 175.200 2.850 5.807 X 108
3.000 0.01911 0.8780 178,800 3.000 6.338 X 1OS
3,150 0.01961 0.8830 181,900 3,150 6.879 X 108

empirical
5.8. Thesedata plot.plotted
data are The values are given in Table
in Fig. 5.11. For p = 150psia '

5.6 Use of Pseudopressure 1J(150)= 2JP E. dp


in Gas Well Test Analysis P~ ilZ
Accuracy of gas well test analysis can be improved in
some cases if the pseudopressure y,(p) is used instead [E. + E.
of approximations writt~n in t~rrns of p~essure or =2 i~~£.~~~~J (150-0)
pressure squared. In thIS sectIon, we discuss the 2
calculation of pseudopressure and provide an in-
troduction to direct use of this quantity in gas-well
dra~down test analysis. Detailed discussion, in- = 2[0 + 12,290] (150)
cludmg systematic development of working 2
equations and application to drawdown, buildup,
and deliverability tests, is provided in Ref. 4. - 1 844
-.x 106 pSI.2
a / cpo
Calculation of Pseudopressure
Gas pseudopressure, y, (p) , is defined by the integral For p = 300 psia,

.p p 1J(300)= 1.844 x 106 ..


y,(p)=2~ -dp, {5.2)
pj IlZ
where PJ IS some arbitrary low base pressure. To +2
( 12,290 +2 24,620 )(300-150)
evaluate .",{p) at some value of p, we can evaluate the
integral in Eq. 5.2 numerically, using values for p. 6 .2
and z for the specific gas under consideration, = 7.381 x 10 psla /cp.
evaluated at reservoir temperature. An example will .., .
illustrate this calculation. ProceedIng m a similar way, v.:ec~n construct Table
5.10. These results are plotted m Fig. 5.13.

Example 5.4 -Calculation of Gas Drawdo~n Test Anai)'sis Using Pseudopressure


Pseudopressure Transient flow at a constant rate from an infinite-
P bl C 1 I h d .I. { ) f acting gas reservoir is modeled by Eq. 5.1.
ro em. a cu ate t e gas pseu opressure 'Y p or
a reservoir containing 0.7 gravity gas at 200"F as a_. + 50 300~ ~
: function of pressure in the range ISO to 3, ISO psia. y,(Pwj) -y,(p/) , T kh
t Gas properties as functions of pressure are given in sc

, Table 5.9. 2
Solution. We will select Pp =0 and use the fact that, r
'll.15110g [ 1,688~Il;CI;rW
as Pp -0, p/ilZ-O. We will use the trapezoidal rule kt
for our numerical integration.
I
."~
--'" www.petroman.ir
86 WELL TESTING
.
TABLE 5.11- DRAW DOWN TEST DATA TABLE 5.12 -DRAWDOWN TEST DATA
FOR CURVE MATCHING
~(P., )
'(hours) P./(psla) (Psia2/cP) ~(P,I-(P.,)
0 3.000 6.338x 108 , (hours) (psla2/cP)
0024 2.964 6.210 x 108 0.024 0.128 x 108
0.096 2.920 6.055 x 108 0.096 0.283 x 108
0244 2,890 5.947 x 1oB 0.244 0.391 x loa
0.686 2.866 5864 x 108 0.686 0.474 x 1oB
2.015 2.848 5.801 x 108 2.015 0.537 x 108
6.00 2.833 5.747 x 108 6.00 0.591 X 108
17.96 2.817 5.693x1oB 17.96 0.645x1oB
53.82 2.802 5.640 x 108 53.82 0.698 x 1oB
161. 2.786 5.585 x 108 161 0.753 x 108
281 2.777 5.553x1oB 281 0.785 x 108 I
401 2.771 5532x108 401 0 806 x 108
521 2.766 5517x108 521 0821x108
641 2.763 5505x108 641 0.833 x 108
761 2.760 5.494 x 108 761 0.844 x 1oB
881 2.757 5.485 x 108 881 0.853 x 108

-(S+Dlqg I)]} (5.1) Exomptet 55.-no A IYSlS


..i" G rlI
OJ as,.,. eII
This equation describes the MTR in a gas well test, Drowdown Test Using Pseudopressures
l just as Eq. 3.1 describes flow of a slightly com- Problem. A constant-rate drawdown test was run on
! pressible liquid. In general, of course, drawdown a gas well with the properties given below. results are
tests in gas wells also have ETR's (usually dominated shown in Table 5.11. '

by wellbore storage distortion) and L TR' s (in which


boundary effects become important). Analysis of a p = 3 (XX)psia ..,
gas well test using pseudopressure as the dependent ~ -0'19 ' I :
variable is illustrated in Example 5.5. Swi : 0:21 i,
Vw = 286 cu ft,
h = 10 ft,
: 1 T =
rw =
200.F,
0.365 ft, I
I IJ.i = 0.01911 cp, I

i qg : I ,(xx) Mscf/D,
,
I
"tg -0.7,
Cti=0.235xIO-3psi-t,
I
I

Cf'
Q
X
:

/ I
!
i:
drainage area = 640 acres (square), and
well centered in drainage area.
This gas is the same as that analyzed in Example 5.4;
:
..

U
Q.. 6- j
..
N ilh:
~". .
~
U") ,.1;1!
:'" ; ~' I
Q.. 41-
II ,
i-!
0 '
~
I !I ~"r
Q.'

-0: I u
I ~ I """"
3- 2 ~, I
=3. "~,.""
-.""' ' ~

-! 'l_- --~ ::::--~


0 ~ 00 '0 0 Q) ~

0 !<XX) 2000 3(XX)

'
FLOWING TIME, hr
PRESSURE, PSI A ! I
Fig. 5.14 -Drawdown test analysis using pseudo- !!
I
Fig. 5.13 -Pseudopressure ys. pressure. pressures. !
I
I
I www.petroman.ir
GASWELLTESTING I 87

TABLE5.13-STABILIZED
DELIVERABILITY
TESTDATA
Rate p.,
Test (MMsCf/D) (psia)
initial buildup -3.127
1 3.710 3.087
2 5980 3.059
3 8191 3.035
4 14.290 2.942

TABLE5.15-GAS WELL
TABLE5.14-ISOCHRONAL DELIVERABILITY BUilDUP TESTDATA
TESTDATA ~t (days)--p.s (PsiaL
q (Mscf/D) P (psia) P., (psia) t (hours) 0001883
0 2542
2.430
983 352.4 344.7 0.5 0.003392 2.600
977 342.4 1.0 0.005738 2.650
970 339.5 2.0 0.009959 2.692
965 3376 3.0 0.01903 2.726
2.631 352.3 329.5 0.5 0.04287 2.756
2,588 322.9 1.0 0.1144 2.785
2.533 315.4 2.0 03289 2.814
2.500 310.5 3.0 0.9724 2.843
3,654 351.0 318.7 0.5 2.903 2,872
3,565 309.5 1.0 7.903 2.896
3,453 298.6 2.0 12.90 2.907
3.390 291.9 3.0 17.90 2,913
4.782 349.5 305.5 0.5 22.90 2.917
4.625 293.6 1.0 27.90 2.920
4.438 279.6 2.0 32.90 2.921
4,318 270.5 3.0 37.90 2,922

accordingly, we can determine 1t-(pw/) foreachpw/' .-(I ,637)(1 ,(XX»(660)


These values are included in Table 5.11. From these = (11.18 x 106)(Im = 9.66 md.
data, determine formation permeability and ap-
parent skin factor. From Eq. 5.1, we see that the apparent skin factor,
Solution. The first step in the analysis procedure is to $' =$+ D!qg I. is
plot 1t-(pw/) vs. log t. This plot is shown in Fig. 5.14. , 5
The curve shape suggests wellbore storage distortion $ = 1.1 I -m [
1t-(Pi)-1t-(PI ---=-I
hr)
up to t ~ I hour, and boundary effects starting at
about 200 hours. A log-log plot of ~~ = [1t-(Pi) -k
~(Pw/)J is useful to confirm this suspicion. Thus, -IOg(;;:..~ -2)+3.23
we tabulate (Table 5.12) and plot ~1t-vs. t. ~Jl.ictir w ].
Qualitative curve matching of log ~~ vs. log I with Thus, ..I
Ramey's type curv.e (no~ shown) indicates an end ~o ,
wellbore storage distortion at about I hour for skin $ = 1.151 -11 [
6.338 X 10818-5.833
x 106- X 108 '
factors in the range 0 to 5 regardless of CsD. con- .
firming the indication on the 1t-(p w/) vs. log I plot.
The absolute value of the slope of the MTR line is -10
g[ (0.19)(0.01911)(2.35
~;:;"ft ft.-. _._9~~ x 10 -4)(0.365)2 ]
m=[(5.944-5.497)X 108J/4

= 11.18x 106 psia2/cp-cYcie. ]


+3.23 = -0.21

The well isofapparently slightly stimulated.


Eq. 5. I shows that the proper interpretation of this Radius investigation at the beginning and end of
slope is the MTR is found from Eq. 1.25:

T
P sc !!-I.:- k V2
m=50,300- (1.151). r.= (948
~ t )
T sc kh t ~Jl.iCti
Thus. for Psc = 14.7 psia and T sc = 520oR,

k=I,637~
q T
[
= ("948)(0.19)(0.0191
1)(2.35 x ~
(9.66)(1) ] Yz

-
www.petroman.ir
BB
--
;,'
_ 0.--

~., WELLTESTING
-I"

= 109 ft at start of MTR (t = I hour), In the 214-hour test, the rate was 1,156 Mscf/D,
the shut-in pressure was 441.6 psia, and the flowing
and BHP was 401.4 psia. Using the data in Table 5.14, (a)
200 Y2 determine the AOF with both empirical andi
r 1= (109)( -) theoretical methods, and (b) establish plots (on the
I same graph paper) of the empirical and theoretical
stabilized deliverability curves.
= 1,550 ft at end of MTR. 5.3. Confirm o/I(p) results stated in Example 5.4
for pressures in the range 450 :5P ~ 3,150 psia.
The distance xe from the well to the edge of the 5.4. The well discussed in Example 5.5 was
640-acre square in which it is centered is 2,640 ft; produced at 2,000 Mscf/D for 90 days and then shut
thus, the time at which the observed deviation from in for a pressure buildup test. Data obtained in the
the MTR occurs agrees qualitatively with the time at buildup test are given in Table 5.15. Determine
which boundary effects should begin to appear. formation permeability and apparent skin factor
using an analysis procedure based on equations
Ex .written in terms of pseudo pressure, o/I(p).
erclses
5.1. The data in Table 5.13 (from Ref. 6) were References
obtained on a well believed to be stabilized at each I. AI-Hussainy, R., Ramey,H.J. Jr., andCrawford,P.B.: "The
rate. Using equations in p2 (strictly speaking, not Flowof RealGasesThroughPorousMedia," J. hI. Tech.
applicable in this pressure range), estimate the AOF (May1966) 624-636;Trans.,A/ME,237. .
using (a) the empirical method and (b) the theoretical 2. W~ttenbarger, R.A. and Ramey,H.J. Jr.: "Gas Wc;I!Testing
h d With Turbulence,Damage,and WellboreStorage, J. P~t.
met 0 '. .Tech. (Aug. 1968)877-887; Trans.,AIME,143.
Also, do the following: (c) plot the theoretical 3. Dake,L.P.: Fundamentalsof ReservoirEngineering, Elsevier
deliverability curve on the same graph paper as the ScientificPublishing
Co., Amsterdam (1978).
empirical curve; (d) since p2 equations are not ac- 4. Theoryand Practicr ofth~ T~ting ofGas W~IIs,Ihi.rdedition,
curat at th O I I d I and outll .ne a Pub. ECRB-7.5-34,Energy Resourcesand ConservationBoard,
e .IS pressure eve, eve '?P .Calgary, Alta. (197.5).
theoretical method based on equations In p; and (e) .5. Craft, B.C. and Hawkins,M.F. Jr.: Applied htro/~um
apply equations in p to these data; in particular, Reservoir Enginnring, Prentice-Hall Book Co., Inc.,
calculate the AOF. Englewood Oiffs, NJ(19.59).
5.2. Cullender7 presented data from an 6. Back Pressur~T~~tfor Natural Gas W~IIs,Revisededition,
..Isochronal test and from an earlier, longer test th at RaIlroadCommIssion of Texas(19.51).
7. CullenderM.H.: "The IsochronalPerfonnanceMethodof
led to approximate stabilization in 214 hours test Delermini~gthe FlowCharacteristics of GasWells," Trans.,
time. AIME(19.5.5) 204,137-142.
www.petroman.ir
.

90 WELL TESTING

a3SERVATION
~LL and
BOTTCf.4HCX..E 0.000264 kl
~SSURE \ TIME LAG 10 = .,.
".. 4Il/.c,r,:,
"-"-"___fACTIVE WELL
--
--Fig. 6.3 can be used in the following way to analyze
RATE AT q interference tests.
ACTIVEWEU. I. Plot pressure drawdown in an observation well,
~=Pi-Pr' vs. elapsed time I on the same size log-
log paper as the full-scale, type-curve version of Fig.
TIM E. 6.3 using an undistor.ted curve (the reader can
prepare such a curve easily).
..2. Slide the plotted test data over the type curve
Fig. 6.1 -Pressure ,esponse In Interferencetest. .1 h . f d (H ' I d .
unt. a matc IS oun. onzonta an vertlca I
sliding both are required.)
3. Record pressure and time match points.
.,
(Po) MP' ~~P and [(lolrfJ)MP,/MP]'
4. Calculate permeability k in the test region from
the pressure match point:

k=141.2:!!!!!- (Po)~p.
h (~)~p
5. Calculate oct from the time match point:

- ( 0.000264k )1 ..,. IMP


/PC,-2 I.
2 IJ.T (lolrfJ)MP
rj r

E.\"ample6.1-1nterference Test
2r. + r in WaterSand
I Problem. An interference test was run in a shallow-
water sand. The active well, Well 13, produced 466
Fig. 6.2 -Region investigated in interference test. STB/D water. Pressure response in shut-in Well 14,
which was 99 ft from Well 13, was measured as a
function of time elapsed since the drawdown in Well
13 began. Estimated rock and fluid properties in-
clude 1/ = 1.0 cp, B... = 1.0 RB/STB. h = 9 ft, r... = 3
in., and 0=0.3. Total compressibility is unkno\\'n.
Pressure readings in Well 14 ~.ere as given in Table
6.1. Estimate formation permeability and total
compressibility.
Solution. We assume that the aquifer is ..
r =:::::::::::==i'ot c,o 107 '0' 0' homogeneous, isotropic, and infinite-acting; we use
rOlo"
.the Ei-function type curves to estimate k and c,. Dala
I to be plotted are presented in Table 6.2. The data fit
: I the Ei-funclion type curve ~'ell. A patr of march
, points are (..l/=128 minutes, lolrfJ=10) and
0 (~= 5.1 psi,po = 1.0). (See Fig. 6.4.) Thus,
a:
k= 141.2~ (PO)~1P
h (~) ~IP

10" (141.2)(466)(1.0)(1.0) (1.0)


KI" I KI M)Z KI' co = -
(9.0) (5.1)
"'
, t 0 /r 02
,. = 1,433 md,
Fig. 6.3 -Exponential integral solution. .
and
0.000264 k (/Mp/60)
c, = .1._2 -,. ,_2, I
IPr I/. (lolro)MP i

!,~~1'~ti;"~ ~~--,., c.:c," I


www.petroman.ir
THEAWELLTESTS 91

(0.~264)( 1 433)( 128/60) TABLE6.1 -PRESSURE/TIMEDATA


= 11\.",nn';/1 1\~/1i\' FROMINTERFERENCE TEST
(0.3)(99) (1.0)(10)
.)t P..
= 2.74 x 10 -S psi -I .(minutes) (pSla)
I D 148.92
5 148.92
25 144.91
6.3 Pulse Testing 40 14372
~ulse tests6 have the same obj.ectives as conventio~al 1~ ~:~:~~
Interference tests -to determIne whether well pairs 200 139.72
are in pressure communication and to estimate k and 300 13870
tpc, in the area of the tested wells. The tests are 400 137.99
conducted by sending a coded signal or pulse 580 13712
sequencefrom an active well (producer or injector) to
a shut-in observation well. The pulse sequence is
created by producing from (or injecting into) the TABLE6.2-INTERFERENCETESTDATA
active well, then shutting it in, and repeating that FORLOG.LOGPLOT
s~quenc~ i~ a regular pattern. An example is in- .)t ~=P, -P.,
dlcated In Fig. 6.5. (minutes! (psla)
The reason for the sequence of pressure pulses is 0 --0
that we readily can determine the effect of an active 5 0
well on an observation well amid the established 25 4.01
trend in reservoir pressure and random perturbations ~ ;.~
(noise)to that trend. Highly sensitive pressure gauges 100 74;
usually are required to detect these small coded 200 9.20
pulses, which may have magnitudes of less than 0.1 300 10.22
psi. 400 1093
Pulse testing has several advantages over con- 580 11.80
ventional interference tests:
I. It disrupts normal operations much less than
, interference
which testing
may range does.
from It hours
a few lasts to
a minimum
a few days.time, I -

! 2. There are fewer interpretation problems caused


by random noise and by trends in reservoir pressure
asthey affect pressure response at observation wells.
3. Pulse test analysis usually can be based on
r simple solutions to the flow equations -specifically,
superposition of Ei-function solutions, which assume 0
infinite-acting, homogeneous reservoirs. In many'
cases, longer interference tests require that boun-
u;
a. 10 4 ..,,-o":~;;--or-
daries be taken into account. ..
Analysis techniques for pulse tests usually are '+- ~:~~ POINT
based on simulating the pressure response in an ~ A~ .'51 psi
observation well with the familiar Ei-function I to 0
solution to the diffusivity equation, using super- 6--~. I ..

position to model the rate changes in the pulsing II At...128min


sequence. From the simulations of pulse tests, charts
relating key characteristics of the tests to reservoir G- 10
properties have been developed.7 Before we discuss <J
Ihese charts (Figs. 6.7 through 6.14) and their ap-
plication. it will be useful to introduce nomenclature
used in pulse test analysis, using the system of
Earlougher I (and his schematic pulse-test rate and
pressure-response history).
Fig. 6.6 illustrates the time lag I L which is the time
elapsed between the end of a pulse and the pressure
peak caused by the pulse. The radius-of -investigation 0I
concept prepares us to expect a time lag. A finite .K) 100 ~
period of time is required for a pulse caused by
producing an subsequent
well. and the active welltransient
to movecreated
to a responding
by a shut- ~ t ' m in .

.in period also requires a finite time period to affect Fig. 6.4-Interference test data from waterreservoir.
pressure response.
The amplitude L\p of a pulse can be represented

I
'- www.petroman.ir
OTHERWELLTESTS 91

(0.(xx)264)( I 433)(128/60) TABLE6.1 -PRESSUREITIMEDATA


= 11\"/nn';/1 1\\/ln\ FROMINTERFERENCE TEST
(0.3)(99) (1.0)(10)
.It P..
= 2.74 x 10 -S psi -I .(minutes) (pSla)
0 148.92
5 14892
25 144.91
6.3 Pulse Testing 40 14372
Pulse tests6 have the same objectives as conventional 1~ ~:~.~~
interference tests -to determine whether well pairs 200 139:72
are in pressure communication and to estimate k and 300 13870
tbc( in the area of the tested wells. The tests are 400 13799
conducted by sending a coded signal or pulse 580 137.12
sequence from an active well (producer or injector) to
a shut-in observation well. The pulse sequence is
created by producing from (or injecting into) the TABLE6.2-INTERFERENCETESTDATA
active well, then shutting it in, and repeating that FORLOG.LOGPLOT
s~quenc~ in. a regular pattern. An example is in- .It ~=P, -P.,
dlcated In Fig. 6.5. (minutes) (psia)
The reason for the sequence of pressure pulses is 0 --0
that we readily can determine the effect of an active 5 0
well on an observation well amid the established 25 4.01
trend in reservoir pressure and random perturbations ;g ~.~
(noise) to that trend. Highly sensitive pressure gauges 100 745
usually are required to detect these small coded 200 920
pulses, which may have magnitudes of less than 0.1 300 10.22
psi. 400 1093
.580 1180
Pulse testing has several advantages over con-
ventional interference tests:
I. It disrupts normal operations much less than
interference testing does. It lasts a minimum time, I -
which may range from a few hours to a few days. .
2. There are fewer interpretation problems caused
by random noise and by trends in reservoir pressure
as they affect pressure response at observation wells.
3. Pulse test analysis usually can be based on I,
simple solutions
superposition to the flow solutions,
of Ei-function equations -specifically,
which assume 0 '
cases, longer interference
infinite-acting, homogeneoustestsreservoirs.
require that
In boun-
many. 0-
in 10 A --,,"",c-::!~-: .r
I daries be taken into account. ~ 'II
Analysis techniques for pulse tests usually are '+- ~:~~ ~INT
based on simulating the pressure response in an ~ A~ .51 psi
observation well with the familiar Ei-function ~ lo .10 ..
solution to the diffusivity equation, using super- 0- -~
position to model the rate changes in the pulsing II At ./28min
sequence. From the simulations of pulse tests, charts 0- !
relating key characteristics of the tests to reservoir <J 10
properties have been developed.7 Before we discuss
these charts (Figs. 6.7 through 6.14) and their ap-
plication, it will be useful to introduce nomenclature
used in pulse test analysis, using the system of
Earlougher 1 (and his schematic pulse-test rate and
pressure-response
history).
Fig. 6.6 illustrates the time lag t L which is the time
elapsed between
peak caused thepulse.
by the end of a pulse
The and the pressure
radius-of-investigati?n 0.1 .
concept prepares us to expect a time lag. A finIte 0 100 KXX>
period of time is required for a pulse caused by
producing an subsequent
well, and the active welltransient
to movecreated
to a responding
by a shut- ~ t ' m in .

.in period also requires a finite time period to affect Fig. 6.4-Interference test data from waterreservoir.
pressure response.
The amplitude Ap of a pulse can be represented ~i,

~\.
www.petroman.ir
..LI'

92 ""~~:"~ilil
WELL ,. TESTING ; I

I ,

RATE IN q I 3 4 ! 5l~6
ACTIVE q q q q RATE IN

WELL ACTIVE
WELL
0 ~~t.-J
TIME TIME-
Fig. 6.5 -Typical rate schedule in pulse test Fig. 6.6 -Pressure response in pulse test.

conveniently as the vertical distance between two was 26 ft; and porosity, ~, was 0.08. In the test
adjacent peaks (or valleys) and a line parallel to this following rate stabilization, the active well was shut
through the valley (or peak), as illustrated in Fig. 6.6. in for 2 hours, then produced for 2 hours, shut in for
The length of the pulse period and total cycle 2 hours, etc. Production rate, q, was 425 STB/D and
length (including both shut-in and flow periods) are formation volume factor, B, was 1.26 RB/STB. The
represented by Alp and Arc' respectively. amplitude Ap of the fourth pulse (Fig. 6.15) was
Analysis of simulated pulse tests shows that Pulse 0.629 psi, and the time lag was 0.4 hour. From these
I (the first odd pulse) and Pulse 2 (the first even data, estimate k and It>c,.
pulse) have characteristics that differ from all
... I responses, aII
sub sequent pulses; beyond these mltla S I r d 6
6 013U Ion. T 014
.
an aJ
U.yze thF.e f our
6 th fi Ise, weduse FIgs.
13pu . .
odd aJ hhave ..
I pulses similar
1 hcharacteristics and all even ,., t I ".. )2. S
A."D (an dlngh kg.. Irhst to etermme '
pu ses so ave slml ar c aractenstlcs. L "'" c ' an t us , we note t at
W~ no:"", define dimensionless ~ariables tha.t are F' = Ar I At = 2/(2 + 2) = 0 5

used m Figs. 6.7 through 6.14, whIch were desIgned pc. ,

~or quamitative analysis of pulse tests: dimensi~nless I Ar = 04/4 = 0 I


ume lag, (tL)D=0.000264 ktL/~.IlC,r;; dlmen- IL c. ..
sionless distance between active and observation
wells, r D = r I r IV; and dimensionless pressure- Then, from Fig. 6.13,
response amplitude, ApD =khAp/141.2 qBp. (with 2
sign convention that Aplq is always positive). ApD (t L l.:lIc) = 0.00221,
Figs. 6.7 through 6.14 provide the correlations to
be used in pulse test analysis. Figs. 6.7 and 6.8 are to and
be used for the first odd (i.e., first) pulse; Figs. 6.9 qB Ap (t I AI ) 2 ,
and 6.10 for the first even (i.e., second) pulse; Figs. k=141.2 -f"/~ I~.j- /
6.11 and 6.12 for all other odd-numbered pulses hAp(ILIAtc)
(third, fifth, etc.); and Figs. 6.13 and 6.14 for all
other even-numbe~ed pulses (fourth, sixth, etc.). The (J41.2)(425)(1.26)(0.8)(0.00221)
figures use the rauos (I) F' = pulse length (~t p) to = 2: ..
cycle length (At c) and (2) time lag (t L) to cycle (26)(0.629)(0.1)
length (~t c). The figures appropriate for a given -817 d
pulse number are used to obtain values of -m.
tlpD(tLI.ltc)2 and [(tL )DlrbJ, which are then F F. 614
used to provide estimates of k and <Pc,. rom Ig. .,

k=]41.2~D(tL~¥, (tL)Dlrb=O.09I.
hAp(tLIArc)
Thus,
0.000264 kt L 0.000264kl L
11>c,= --., -2-. ~cl= -, -2
p.r-[ (I L ) DI r DJ p.r-{ (fL )DlrDJSxample

6.2 illustrates how these figures are applied.

= (0.000264)(817)(0.4)
In Q\/Q~~\2,/\ "",'

:xample6.2-Pulse Test Analysis (0.8)(933) (0.091)Jroblem.

A pulse test was run in a reservoir in which


distance between wells, r, was 933 ft. Formation
viscosity, p.,was 0.8 cp; formation thickness, h, Then,
= 1.36x 10-6

;
I
Iluid he

www.petroman.ir ! 1.11
I~ 1
--1
OTHER WELL TESTS

0003

N
r-a
" ~ 0.00
oJ
.=.,
~ 0.0025
~
1&1
a
::> 0.0020
~
J
~
~ 0.9.
ct 0.001 .
1&1
II!
Z
a
~ 0.0010
1&1
It
1&1
II! 0.0005
.J
::>
~
0 171t S17.t
10-1 I

(TIME LAGI/(CYCLE LENGTH). tL/6tc

Fig. 6.7 -Relation between time lag and response amplitude for first odd pulse.'

0.2

-
N 0.17
0
, 0
~
-- oJ 0.15

13
ct
.J 0.125
1&1
~
~
II! 0.100
II!
I&J
.J
Z
Q 0.075
II!
Z
I&J
~
is o.o~o

0.02~
7 ..I
to.'
(TIME LAG)/(CYCLE LENGTH). tL/6tc

Fig. 6.8 -Relation between time lag and cycle length for first odd pulse. 1

www.petroman.ir
WELL'

0.004~

N
Y 0.0040
~ oJ
~ 0 0.003
Q.
~
!oJ
0 0.00
::>
~

oJ
Q.
~ 0.002
4

!oJ
VI
Z
~ 0.0020
VI
!oJ
~
!oJ
VI 0.001~
oJ

::>

Q.

0.0010
'8' 45.'8'
10-1 I

(TIME LAG)/(CYCLE LENGTH). tL/OtC

Fig. 6.9 -Relation between time lag and response amplitudes for first even pulse. 1

0.200

N 0.17
0
'-~ oJ O.I~ ..
-~

~ .
4 .
oJ 0.12'

!oJ
~
~
VI 0.10
VI
!oJ
oJ
Z
~
z 0.07' :.
!oJ
~
0 0.0'0 ;

0.025 .:

'8' '8'
10-1 1

(TIME LAG)/(CYCLE LENGTH). tL/OtC

Fig. 6.10 -Relation between lime lag and cycle length for first even pulse.'

www.petroman.ir
v.
OTHER WELL TESTS 95

0.003

N
~
U
~ 0.003
"- .J
~
Q
Q. 0002
<3

I&J
0
-; 0.0020
J
a-
~
4 0.00"
\oj
I/)

~
a-
I/) 0.0010
I&J
a:
\oj
I/)
-l 0.0005
!
0
4517.. Z
10-1 1
(TIME LAGI/(CYCLE LENGTH I. tL/c.tC

Fig. 6.11- Relation between time lag and response amplitude for all odd pulses
after the first.'

0.20

N 0.17

'"..-
Q

-
-:, 0.1'

0
4
-l 0.12
I&J ..
~
~
I/) 0.10
I/)
I&J
-l
Z
Q 0.07'
I/)
Z
\oj
~
25 0.0'0

0.02' 5 I 7..
I

(TIME LAGI/(CYCLE LENGTH). tL/c.tC

Fig. 6.12 -Relation between time lag and cycle length for all odd pulses after
the first.'

www.petroman.ir
~_IIIIIIIII'I r"".' WEllTESTING

0.00.'

N
r-I
U
..0.00

~ .J
...=,
Q 0003
Q.
q
I&J
0
:> 0.003
~

~
~
4 0.002
I&J
(/)
Z
~ 00020
(/)
I&J
~
I&J
':'J O.COIS

!
00010 1 I . 10-1 I
(TIME LAG)/(CYCLE LENGTH). tL/OtC

Fig. 6.13 -Re!ation between time lag and response amplitude for all even
pulses alter the flrst.'

0.200

N 0.17S
..Q
"-
Q

..
-., O.ISO

Ii4 ..
.J 0.12S
I&J
~ -.
~
(/) 0.100
(/)
I&J
.J
Z
~ 0.07'
Z
I&J
~
0 O.OSO

0.02Ss I' ~"-"5'.'"


10-1 I

Fig. 6.14 -Relation (TIME time


between LAG)/(CYCLE
lag and cycleLENGTH).
length for tL/OtC
'all even pulses after

the first_'

www.petroman.ir
OTHER
WELL
TESTS 97
i

~ ~ ,..)J;;i ,F ,;--!i.,\
IJ
U) H
W
g:: A DE
K

t TIME-
Fig. 6.15-Schematic of pressure response in pulse test. Fig. 6.16-Schematic 01arilistem test pressurechart.

1.36 X 10 -6 imposed on the pressure gauge. The testing device is


c{ = = 17 x 10 -6 psi -I. then removed from the hole (Point J to Point K).
0.08 The initial flow period is usually brief (5 to 10
minutes); its purpose is to draw down the pressure
.slightly near the well bore (perhaps letting any mud-
6.4 Dnllstem Tests filtrate-invaded zone bleed back to or below static
A drillstem test (DST)8.9 provides a means of reservoir pressure). The initial shut-in period, often
estimating formation and fluid properties before 30 to 60 minutes, is designed to let the pressure build
completion of a well. Basically, a DST is a temporary back to true static formation pressure. This initial
completion of a well. The DST tool is an shut-in pressure on a DST may be the best
arrangement of packers and valves placed on the end measurement made of static reservoir pressure.
of the drillpipe. This arrangement can be used to The second flow period is designed to capture a
isolate a zone of interest and to let it produce into the large sample of formation fluid and to draw down
drillpipe or drillstem. A fluid sample is obtained in the pressure in the formation to the maximum
the test; thus, the test can tell us the types of fluids distance and extent possible within the time that is
the well will produce if it is completed in the tested possible to allow for the DST -frequently 30 minutes
formation. to several hours. The second shut-in period is
With the surface-actuated valves on a DST device, designed to obtain good pressure buildup data so that
it is possible to have a sequence of flow periods formation properties can be estimated. In addition,
follo~'ed by shut-in periods. A pressure recorder on comparison of the final (or extrapolated) pressure
the DST device can record pressures during the flow from the second shut-in period to the initial shut-in
and shut-in periods. The pressures recorded during pressure can indicate that pressure depletion has
the shut-in periods can be particularly valuable for occurred during the DST and that the well thus has
estimating formation characteristics such as per- been tested in a small, noncommercial reservoir. The
meability Ithickness product and skin factor. These desired length of the second shut-in period varies
data also can be used to determine possible pressure from equal to the second flow period (for high-
depletion during the test. permeability formations) to twice the length of the ..
To illustrate how a typical DST is performed, we second flow period (for low-permeability for-
will examine a schematic chart (Fig. 6.16) of pressure mations).
vs. time from a test with two flow periods and two Theory much like that used for an ordinary
shut-in periods. pressure buildup test following production at con-
At Point A, the tool is lowered into the hole. stant rate is used for analyzing the shut-in periods on
Between Points A and B, the ever-increasing mud- a DST. This is true even though the flow rate
column pressure is recorded; at Point B, the tool is preceding a shut-in period in a DST usually decreases
on bottom. When the packers are set, the mud continuously. Usually, the average production rate
column is compressed and a still higher pressure is can be used as a good approximation in buildup test
recorded at Point C. The tool is opened for an initial analyses; this average rate of production is deter-
flow period, and the pressure drops to Point D as mined by dividing the fluid recovery by the length of
sho"'n. As fluid accumulates in the drillstem above the flow period.
the pressure gauge, the pressure rises. Finally, at To analyze the buildup test, we plot Pws vs. log
Point E, the well is shut in for an initial pressure (tp+A/)/.lt, wherelp isnowtheactua!.n°wi~gtime
buildup test. After a suitable shut-in period, the well at the average rate q. The permeabilIty/thIckness
is reopened for a second final flow period, from product is found from the relationship kh = 162.6
Point G to Point H. This final flow period is qBJJ./m. Usually, a fluid sample will not yet have
followed by a final shut-in period (from Point H to been analyzed in the laboratory; accordingly, ,
Point I). The packers are then released, and the correlations (Appendix D) relating JJ.and B to j
hydrostatic pressure of the mud column is again produced fluid properties must be used. j
J
, I ~
I
www.petroman.ir
I
98 WELLTESTING

TABLE6.3-DRILLSTEMTESTDATA pressure is recorded. Follo~'ing sample collection,


shut-in pressures are recorded as they build up with
~t p~s '
(minutes) IPSI) tIme.
t --0-- 3S-O- Flow into the sample chamber is probably ap-
c 5 965 proximately spherical (i.e.. into a point rather than
10 1.215 spread uniformly across an entire productive in-
15 1.405 terval). For this reason, the shut-in test cannot be
~~ 1,590 analyzed as in a DST, although theory based on
30 ~.~~ steady-state spherical flo~' may explain ~;reline test
35 1:740 buildup pressure satisfactorily in some cases. t I The
40 1.753 device is useful for obtaining samples of formation
45 1.765 fluid and estimating initial formation pressure;
extensive use of the device for this latter application
Static reservoir pressure is found by extrapolating has beenreported in the literature. II
the buildup tests to infinite shut-in time. In a dual .
shut-in test. we have the opportunity to extrapolate Exercises
both buildup tests to infinite shut-in time and to 6.1 Determine the duration of an interference test
compare the estimated static reservoir pressure. If the required to achieve a pressure drawdown of 25 psig at
static reservoir pressure from the final shut-in period the observation well for the reservoir described in
is significantly lower than that from the initial shut-in Example 6.1 if the active ~'ell produces 500 STB/D
period, it is possible that the reservoir was partially throughout the test. What will be the radius of in-
pressure depleted even during the relatively short vestigation at this time? If the skin factor in the
DST, implying that the formation tested is probably active well is 2.0, what will be the drawdo~'n in the
noncommercial. active well?
Skin factor is calculated from the conventional 6.2 For the pulse test described in Example 6.2.
skin-factor equation: given the results of the test analysis of the fourth
pulse, determine the time lag and pressure response
(p -p ) for (a) the first odd pulse; (b) the first even pulse; (c)
s= 1.ISI[ \PI hr -Pwfl the third pulse; (d) the fifth pulse; and (e) the sixth
m pulse.
6.3 JohnstQn-Schlumberger9
reports data below
,k, from a DST:
-log (~ ) + 3.23j. (2.4) initial flow period = S minutes,
#J.Ctw Inltla Ihs ut-in
... ' peno
. d30
= minutes,
'

There is a further complication in DST analysis. At final flo.w per~od = 60 m~nutes,and


the time of the test, reservoir rock and fluid final shut-In perIod = 4S minutes.
properties that appear in Eq. 2.4 may not yet be Data obtained in the final shut-in period were as
known accurately. This is particularly true of given in Table 6.3. In the initial shut-in period, the
porosity, ct>, and total compressibility, Ct. Ac- pressure reached and remained stable at 1,910 psi.
cordingly, one may be forced to use the best available Total fluid recovered in both initial and fmal flow
estimates and to recognize that skin-factor and periods filled 300 ft of 2 Vl-in.-lD drill collars (0.0061
radius-of-investigation calculations, which also bbl/ft) and 300 ft of 4'il-in. drillpipe (0.0142 bbl/ft).
depe.nd on these p.roperties, may be s.ubject to The produced fluid '''as 35°API oil with a measured ..
consIderable uncertainty. Use of Eq. 2.3 Instead of gas rate of 47 Mscf/D al Ihe surface (assumed
Eq. 2.4 (Eq. 2.3 includes the sometimes important solution 2as). Formalion temperature was measured
~ermlog (tp+;~t)/tp]toc~lculatesm.aybejustified at l20°F~ PorosilY is estimaled to be 10;'-0; total
In. some cases If other data In the equatIon are kno~'n compressibility is 8.~ x 10 -6 psi -I; wellbore radius
with unusual accuracy. is 4.5 in; and formation thickness is 10 ft.
6 5 W. ..Estimale formation permeability, skin factor, flow
.Irellne Formation Tests efficienc\", and radius of invesligation achieved in the
In many areas. hole conditions prohibit use of DST's lest. .
as temporary wellbore completions. In these areas, 6.4 Smolen and Litsey II propose that "borehole-
and in others where the costs of Ihe required numbers corrected," stead\"-state, spherical flow into the
of DST's for complete evaluation are prohibitive, wireline formation"tester can be modeled by
~'ireline formation tests 10.11frequently are used in ,
formation evaluation work. k = 3,300 qlJ./~. 1
A wireline formation tester is, in effect. a sample
chamber of up to several gallons capacity combined where
wit~ pressure gauges. The.test chal.nbers are forced k = permeabililY, md, I
against the borehole wall In a sealing pad, and the q = flow rate, cm3Is (reservoir conditions).
formation is perforated by firing a shaped charge. Jl. = fluid viscosity (usually mud filtrate), cp, I
The signal t.o fi~e the charge is Ir~nsmitted o.nlogging and .I
cable. fluid IS collected durIng sampling, and ~ = drawdown from formation pressure. psI.~

www.petroman.ir
OTHER WELL TESTS 99

A test showed that formation pressure was 3,850 psi. 4. Vela, S. and \1cKinley, R.\1.: "How Areal Heterogeneities
Pressure was dra~.n down in the sample chamber to Affect Pulse.Test Results," Soc. P~t Eng.J. (June 1970)181-
an apRroximatelv constant 1 850 psi by withdrawing 191;Trans..AI~E. 249.
3 ., ., 5. Jargon, J.R.: Effect of ~ellbore Slorage and Wellbore
10 cm of filtrate (JJ.= 0.5 cp) from the formatIon In Damageat Ihe Active Well on ImerferenceTest Analysis," J.
16 seconds. P~t. T~ch.(Aug. 1976)851.858.
Estimate formation permeability from the test 6. Johnson,C.R., Greenkorn, R.A., and .~oods, E.G.: "Pulse-
data. Testing:. A New Method for Descnblng Reservoir Flow
...Properties BetweenWells," J. Pet. Tech. (Dec. 1966) 1599-
Derive an equatIon for steady-state sphencal flow I~; Trans.,AI ME, 237.
and show that it has the form k = constant x qBIJ./(r 7. KamaJ,\1. and Brigham, W.E.: "Pulse-Testing Responsefor
(Pi -Pw/)]. State the assumptions required for the UnequalPulse and Shut-In Periods," Soc. Pet. Eng. J. (Oct.
equation to model a wireline formation tester flow 1975)399-410; Trans.,~1\1E. 259.
test 8. Edwards, A.G. and Wlnn, R.H.: "A Summary of \1odern
.Tools and Techniques Used in Drillstem Testing," Pub. T-
References 4069. Halliburton Co., Duncan,OK (Sept. 1973).
9. .'Revie-. of Basic Formation Evaluation," Form J.328,
1. Earlougher, R.C. Jr.: Adl'anc~s in ".~/I T~st Analysis. Johnston-Schlumberger,Houston (1976).
Monograph Series,SPE, DaJlas(1977)5. 10. Schultz, A.L., Bell, W.T., and Urbanosky, H.J.: "Ad-
2. \1atthews, C.S. and Russell, D.G.: Pr~ssure BuIldup and vancemems in Uncased-Hole, Wireline Formation-Tester
Flow T~stsin W.ells,\1onograph Series,SPE, Dallas (1967)t. Techniques,"J. P~t. T~ch.(Nov. 1975)1331-1336.
3. Theoryand Pract;~ o/the TestIngo/Gas Wells,third edition, 11. Smolm, J.J. and Litsey, L.R.: "Formal ion Evaluation Using
Pub. ERCB-75-34. Energ~. Resourcesand Conser\'ation ~.ireline Formation Tester PressureData," J. Pet. Tech.
Board, Calgary, Alia. (1975). (Jan. 1979)25-32.

www.petroman.ir
m

~."."
!
..,
i ..,.

, I
Appendix A
Development of Differential
Equations for Flow in Porous Media I

t
Introduction I
In this appendix, we develop some of the basic =pu-,.~YI1Z+PUyAK.lz+pu;:.lx.1y- [pux
differential equations that describe the no~' of fluids
in porous media. Results presented include equations + ~(pU.f) ]~YI1Z -[PUy + ~(pu.y) ]AKAz
for three-dimensional flow of slightly compressible
liquids, and for radial flo\\! of slightly compressible -[PU: + ~(pU;:) ]AK~Y
liquids, gases, and simultaneous flow of oil, water, I
and gas. In developing these equations, we start with = -.1 (pu x) ~Y~ -~(pu \I) A.\',lz -~ (pu.) ;
continuity equations (mass balances); then we in- ." I

trodu~e flow l~ws (such as Darcy's. law) ~nd ap- 'AK~y. j


propnate equatIons of state for the fluId considered. To determine the rate at which fluid accumulates i
within the element, we first note that the mass within
I, Continuity Equation for the element (of porosity 0) at a given time is !
! Three-Dimensional Flo,,' p(!J~.ly,lz (Ibm/cu ft x cu ft = Ibm). Thus, the rate at
To develop continuity equations, we use a mass which this mass changes over a time interval ~/ is
balance on athesmall
balance has element
following of porous material. The
form. (pO 1+,),1
~r -pti> I) Ax.1_~',lz, ..

(rate of mass flow into element) -(rate o~ mass where time, r, is in hours. Accordingly, the mass .i
f flow out of element~ = ~rate of accumulation of balance becomes !
mass wIthin element). !
Our element is shown in Fig. A-I. It has dimen- -~(pu.\.).1.v.:lz-~(pUy)~.l.:-~(pu;:)AK~y
sions .:lx,f ~.v, and .1.-:,
. in theh x, y, and
d .' ~ coordinate p~.I.;1+J.r - p tl>1
system; or convenIence, t e coor mate system IS = ~'~.~',lz.
oriented such that gravitational forces are in the ( -)~ ~/
direction. We denote the components of the If we divide each term by A.\".l.~',lzand take the limit I
\'olumetric rate of flow per unit cross-sectional area as .1r,:l.\',.l)', and 11Z-0, the result is i
(cu~ic feet .per ~our-s9uare feel or feet per hour are ~ ~ ~ = -~ (
typlcalenglneenngumts)byux,u.,andu;:. a .+a + ,pq,).
The rate at which mass enters ihe element in the .\' lX' ~ a~ ar :
direction is pu ~v~ (Ibm/cu ftxft/hrxsq ft= (A.I) !
."C -I
lbm/hr);. the rate. at ,!"hi~h the mass leaves the Continuitv Equation for Radial Flow
element m the x dIrectIon IS [pux+~(pux)].ly.1:.. ...
Similar expressions describe rates of mass entering From a mass balance sl.mll.arto that ~sed to develop
and leaving in the y and ~ directions. The result of E.q. ~.l, the. comlnulty equation for one-
adding these expressions is dImensional, radIal flow can be sho\\"nto be

(rate of mass flow into element) ~~(rpur)=-~(tI>p), (A.2) !


-(rate of mass flow out of element) r ar at ijj
www.petroman.ir
DEVELOPMENT OF DIFFERENTIAL EOUATIONS FOR FLOW .~~1:?j;;~,;i{~J;"
IN POROUS MEDIA
~.- '" 101

t / introduce additional assumptions. First, we restrict


tu +-.d(f tUx our analysis to slightly compressible liquids -those
with constant compressibility. c. where c is defined
by the equation
t +-6(tu>- -I dV_1 dp
-~Uy-

6z
t Ux+-A('Ux)
)-
Uy
1
Y c=v"dj;-pdP'
For constant compressibility
A.6 gives
c, integration of Eq.
(A.6)

/' tlUt x "1 p=poeC(P-P,,). (A.7)

.where Po is the value of p at some reference pressure I


~ Fig. A.1 -Element
balance.
of porous medium used for mass p.0 Eq. A.7 describes most single-phase liquids !I
adequately.
If \\'e now introduce Eq. A- 7 into Eq. A.4 and !'
where u, is the volumetric flow rate per unit cross- assu~e. that (I) k.f = k.".= k;: = constant, (2)
section area in the radial direction. Eq. A.2 is less gravitatIonal forces are negligIble, (3) ct>= constant,
general than Eq. A.I; in particular. radial flow onl.v and (4).Il = constant, Eq. A,~ becomes
is assumed [i.e., there is no flow in the Z or 8
"direction" in a cylindrical (r,z.8) coordinate ~
0 l
eC(P-p,,) ~
oP I+ 0 l oP
a ec(p-p,,) a I
system], Y Y

Flow La,,'s +-e0 I C (p - P,,-


) op /
Liquid flo\\ usually is described by Darcy's law. This OZ OZ
law. when applied in the coordinate system orien-
tation "e have ~hosen to .describe three=dimensional -ct>/l ~
flow, becomes, In field Units. -0.000264 k ot [e c(p-P..) J.

k op
U.f = -0.001127 -:! -, Sim plifvin
-/l ox ., g
-02p 02p
-;:-;+-,.+;::r 02p .
u" = -0.00 1127 ~ ~, o.\"- oy- OZ ~
./l oY
I( op 2 ( op 2 ( op 2/ 1
+c - ) + - ) + -
k. op ox oY OZ)

u.=-0.001127~(-+0.OO694p
~ /l OZ
), (A.3)
=
-I.
'I'/lC ~
~ '
I '.' d. d 0.000264 k ot ' "
Th k h b'
e i are t e permea I Itles In Irection i an p
denotes pressure (psi). In Eq. A.3, we ha\e assumed If we further assume that ,I
that the .\' and y directions are horizontal, so that 0 ~ 0 2 0 2
gravity acts on!y i~ the.: direction.. .C I( .:E I + ( -E) + ( -E)
After equation,
substituting theseisequations Into the con- o.\" oy OZ\\'ith
J
tinuity the result is negligible compared other terms in the ..

~ ( ~ ~ ) 0 ( k "p oP ) equation (w~ich requires either small C or small


0\' a\" + -;- --;- pressure gradients, or both), then
..Il' dY /l IiY ..,
o-p o-p 02p OjlC op
--:; + ;-::2 + ;::r = (A.S)
a k.p op OX- o.v OZ 0.<XX>264
k 01
+ a:: 1-;- ( az + O.00694p)J For radial flow, the corresponding equation is
loop ct>/lC op
I 0 ~ a;: (r a;:) = O.<XX>264 kat. ""."'" (A.9)
= 0.000264 at (ct>p). (A.4) Eqs. A.S and A.9 are diffusiyity equations.
Analytical solutions to Eq. A.9 are known for several
For radial flow, the result is simple boundary conditions; these solutions are used
I I 0 rpk ap I 0 for most \vell test analysis. We must remember that
I -a(--!-a)=-(Pct». (A.S) Eqs, A.Sand A,9arenotgeneral; instead, they are
r r /l r O.<XX>264
ot based on several important assumptions, including
Single-Phase Flow of Slightly (I) the single-ph~e .l}quid flo~ing has small and
..constant compressibility; (2) k IS constant and the
Compressible Fluids same in all directions (isotropic); (3) ct>is constant;
To solve Eqs. A.4 and A.S analytically, we must and (4) pressure gradients are small.
I ., !
~ IAj
-Ilj~'~ www.petroman.ir
~

102 WELLTESTING

Single-Phase Gas Flow to Eqs. A.8 and A.9 prove to be accurate ap-
For gas fl ow characterlze
. db v D arcv ' s I aw andf or a proximatesolutionsofEqs.A.12andA.13.
gas described by the equation'of stat~, Simulraneous Flow of Oil.
.\1 p \\'ater, and Gas
P=--,
RT Z In this section. we outline a detailed derivation of an
Eq. A.4 becomes, for constant ctIand k and negligible equation describing radial, simultaneous flo~ of oil,
gravitational forces, gas, and water. More complete discussions and more
~ ~ ~ ~ ~ ~ general equations are given by Matthews and
Q
-(
p dp
--+
Q
---+
P dp Q
( P dp
---Russell ~.and Martin. 3
a.\" 'lJoZg a.\") ay (. IJoZgay) az IJoZgaz) We assume that a porous medium contains oil,
water. and gas, and that each phase has saturation-
= ctI ~(. l?-). (A.IO) ~ependent effecti\e perr:neability (koo k",o and kg);
O.<XX>264k at Zg tIme-dependent saturatIon (Soo S and SoS);
..pressure-dependent formation volume factor (80,
In this section only, we use Zg to denote the real gas B ,,'0and B ); and pressure-dependent viscosity (p.oo
law compressibility factor so that we can ~ontinue to p , and p.g 1. When gravitational forces and capillary
use the symbol Z for one of the space variables. For pressures are negligible. the differential equation
gases, p.and Zg are functions of pressure and cannot describing this type of flow is
be assumed constant ~xcept. i~ special cases. To I a ap <PC I ap
reduce Eq. A.IO to a form sImIlar to Eq. A.8, we --(r-)= -, (A.14)
define a pseudopressure, I ~ (p), as follows: r ar ar O.<XX>264 Al at

.p p
~'here
~(P)=2) -;-dp, (A.II) -~ ~ ~
P-Zg
~'herePJ isalowbasepressure.
Pi, Now, AI- p.o + p.g + p , (A.IS)

d(l?-) cI=Soco+S...c..+SgC~+cf' (A.16)


~ ( l?- -Zg ap
al Z.~) -dp at and

cpap co
-I dBo+ ~ dRs (A.I/)
--
= ~ -Bo dp Bo dp
.Zg at' Note in Eq. A.17 that effective oil compressibility
since depends not only on the usual change in liquid
I d( l?- ) volume ~ith pressure but also on the change with
C = -~ = ~ Zg pressure of the dissolved GOR. RS'
.~ P dp p dp A similar expression can be written for water

Al so note th at compressibilitv:
.
a I a.I ~ -I dB... ~ dRSI"
-;-~- -:;- ¥,up
-a = --,2p ap C"'
B... dp
+
B ...dp
(A.18)
(It up t IJoZg at
and Exercises
a.o -2p ap A.I Derive Eq. A.2; state the assumptions.
a; -~ ~ .required in.this derivation. .
S'.I ' .A.2 Derive Eq. A.S; state the assumptIons
Iml ar expressIons apply for a~/ay and a~/az. required in this derivation.
Thus, Eq. A.IO becomes A.3 Derive Eq. A.9; state the assumptions
a ( a~ ) a ( a1f, , a a~.) required in this derivation.
~ a;+a;a;)+az(a:;:: A..4o.erive. Eq: ~.13; state the assumpt!ons
requIred In thIs derivatIon. Compare the assumptIons
-ti>,uc I{ a~ required to derive Eq. A.9 with those required to
-0.<XX>264k at. (A.12) deriveEq.A.13.

For radial flo\\, the equivalent of Eq. A.12 is References


I a (. a~ ) ctllJ.Cg a.,; I. AI-Hus':liny.R..R:lmey.H.J.Jr..andCra~(ord.P.B.:"The
; a; r ~ = 0.000264 kat. (A.13) Flo~.of Re:l~Gasc:ThroughPoro~sMroi:l." J. PI'I. Tn.h.
(May1966)6~4-636. Truns..AI\IE. 237.
Eqs. A.12 and A.13 are similar in form to Eqs. A.8 2. Malthe\,s.. C.S. andRussell.D.-G.:Prl'ssurl'Buildl/!!andRull.
and A. 9, but there is one important di fference. The Tl'sls.lIIlll'lls. Mono~~:lph
Series..SPE.Dalla~(196,)I. .
coe ffi IClent
." 3. Marlin. J.C.:
and"Slmplltiro Equallons of FloworIn Multipha.~
Gas Dri\e Ii
ctI,uc/0.000264 k IS constant In Eqs. A.8 Reser\oirs (he Theorelil:al Foundalion
and A.9; in Eqs. A.12 and A.13, it is a function of Pr~surc: Buildup ~al\'si~." Trails.. AI ME (1959) 216. m- i

www.petroman.ir
Appendix B
I Dimensionless Variables I

Introduction
It is convenient and customary to present graphical or
or tabulated solut~ons t~ flow equ~tions, such.as Eq. ap qB/l
A.9,intermsofdlmenslonlessvanables.lnthlsway, -I =-. (8.4)
... bl I I t.
It IS paSSI e to present compact y so u Ions or
f a ar
'w
0.00708 khr w
wid.e range of parameters cp, /l, C, and k, and This boundary condition arises from Darcy's law in
variables r, p, and t. the form similar to that used in Eq. A.3:
In this appendix, we show how many of the B
dimensionless variables that appear in the. well- u = -0.001127 ~ ~ = ~ .
testing literature arise logically and directly In the' /l ar 2rhr
differential eq~~tions (and in. their i".itial and Our objective in this analysis is to restate the
bou~dary condItIons) that descrIbe flow In parous differential equation, and initial and boundary
medIa. conditions in dimensionless form so we can deter-
Radial Flow of a Slightly -mine the ~ime~sionless .vari~bles and parameters that
..charactenze thIs flow situatIon and that can be used
Compressible Fluid to characterize solutions. These dimensionless
In this section, we identi~y the dimensi.onless parameters and variables are not unique (i.e., more -
variables and parameters requIred to charactenze the than one choice can be made for each). Further, we !
solutions to the equations describing radial flow of a want to emphasize that these dimensionless variables'
slightly compressible liquid in a reservoir. We assume are defined rather than derived quantities. These " "
that Eq. A.9 adequately models this flow. ideas will become clearer as we proceed. ~.
Specifically, we analyze the situation in which (I) We define a dimensionless radius, rD=r/rw (any ,
pressure throughout the reservoir is uniform before other convenient reference length, such as r e' could
production; (2) fluid is produced at a constant rate have been used). From the form of the differential
from a single well of radius r w centered in the equation, we also note that a convenient definition of
reservoir; and (3) there is no flow across the outer dimensionless time is tD = 0.000264 kt I o/lcr!. ..
boundary (with radius r e) of the reservoir. Stated The initial and boundary conditions suggest that a
mathematically, the differential equation, and initial convenient definition of dimensionless pressure is
and boundary conditions are
I a
--r-
( )
a~
=
CPIi.C a~
-,
PD = 0.OO708kh(Pi-P)
qB/l
.
r ar ar 0.000264k at ..
\JII.ith this definition, the boundary condItion (Eq.
(8.1) B.4) becomes

att=O,p=pjforallr, -qBIJ apD'


0.OO708khr...~i, -qBIJ
= 0.00708khr". ,
(1.1
atr=re,q=Ofort>O, (8.2) or simply
a -
or a;: I =1.
ap 'D=} ...
-I =0, ,.. (B.3) Expressed In terms of dimensionless variables, the
ar , differential equation and its initial and boundary
~
conditions become
-0.001127(2rrwh) k ap I I a aPD )=ar;'
apD
atr=rw,q= B;a;,.fort>O ~~(rD~ (8.5) j

.. :
www.petroman.ir
I
104 ",,- -:- WELL
TESTING

po=Oforallro atlo=O. (8.6) andl:ompressibilityhasthe fundamental units


ap Compressibility:.[lt~/m).
0,
~I =Oforlo>O. (8.7) Then. ..
'n"..'o"IJ.' 0.<XX>264kl [l-)[t)
-IapO
10=
~~cr; m
[I) I --- lllt2 1(l " )
[1).
aro' =lforlo>O. (8.8) It m
'n. I
The implication is that any solution, Po, of Eq. 8.5 Thus, 10 has units of unity, or. more plainly. is
is a unique function of r 0 and 10 for fixed r £¥; no dimensionless. Similarly,
other dimensionless
dimensionless variablesequation
differential appear inoreither
in the
the Po = 0.00708 khB (p I -p)
dimensionless initial or boundary conditions q ~
describing this particular problem. Thus, if we wish m
to present solutions to Eq. 8.5, we could do so [l ~](l)1 ~ I
compactly either by tabulating or by plotting Po at -It -(I].
ro=1 (r=r...) as a function of the variable 10 with I l3 ! 1 m
the parameter r[H. In fact. such tables and graphs , t (I) lt !
have been prepared and presented; this problem is
precisely the ~'.ell known van Everdingen a~d Hurst Thus. Po also is dimensionless.
constant terminal-rate problem. for which they Again. ~'e stress the rea~on for introduction of
present solutions as functions of 10 and r£¥. some~hat unnatural dimensionless quantities. They
We have implied that the groups of parameters and allo~' solutions for wide ranges of k. h, c. I, rt' r ..., q,
varia.ble~ ~~at arise from the diff.e~ential equ:ation ~. and B to be presented I:ompactly (tables or graphs)
and tts InItIal and boundary condItIons are dlmen- as functions of a minimum number of variables and
~ionless. .We ~'ill now provide the proof for the case parameters. Such tabulations and graphs are in
just considered. widespread use in well testanal~"Si~.
Dimen~ionless variables are
ro =rlr ..., Radial flow With Constant BHP
0 <XX>264
k To illustrate further ho~' the choice of appropri.ate
tD = .t dimensionless variables depends on the specIfic
" ,
op.Cr..,: differential equation, and initial and boundary
and conditions, we now determine appropriate dimen-
0 00708 kh ( ,- ) sionless variables for radial now of a slightly
Po = .P, P. compressible liquid through a ~'ellbore of radius r...
qBIJ. from a reservoir of radius r t. There is no now across
Obviously, r 0 is dimensionless. To sho~' that 10 and the outer boundary (at r= r t); initial.pressure,.p" is
Po are dimensionless. we introduce the symbol (), uniform before ~roduction; and nowlng 8HP In ~he
which denotc~ "has units of." let m denote mass. l. wellbore. P../. IS he.ld constant .once production
length. and t. time. The quantities that appear in 10 begins. The mathematIcal problem ISthen
and Po have the following basic units: I a ap 0IJ.C ap
(r-)= -, (8.9)
k -(l-), r ar ar O.<XX>264 k at ..
1 -(tJ.
0 -(IJ(i.e..dimensionless). p=p,at/=Oforallr. (8.10)
IJ. -(miLt).
c -(lt~/mJ. p=p...ratr=r...forallt>O. (8.11)
r.., -(LJ,
h -(lJ, ..~I =Oforallt>O. (8.12)
P -(m/Lt-J, ar '
q -(l3/t), and '..
B -(I). As before. our approach ~'ill be to eliminate all
.parameters and variables \\'ith dimensions from the
A co~~e.nt on t~e Units of I?ress~r_eiP. and differential equation, and initial and boundary
compressIbIlity. c ~whlc~ ha~ the units pSI) .may be conditions. The appropriate definition of r 0 is again
helpful: Pressure IS defIned as force .per unIt ~rea. r = rlr .; for t ,it is
From Ne~'Ion's second law, force IS mass times 0 ..0
acceleration; thus, force has the fundamental units O.<XX>264 kt
f Force:. [ml/t 2J. 10 =. olAL'r!
I Thus, pressure has the fundamental units The~e definitions will eliminate all parameters from
Pressure:. (ml/t2l2J or (m/lt2), ~q. ~.9..The appropriate definition of Po for this
~-- situation I~
'- Append'.
C I

www.petroman.ir
1
--

DIMENSIONLESS VARIABLES '.1:~' 105


-:,;;;eo

PD -Pi -P ( 0.00)264, k )),1 qdl.

Pi-Pwf
With these definitions, the mathematical statement 4>JJ.c,,; 0

of the problem becomes. Therefore,

.!:.- a'D
'D
(
.!-.- 'Da'D ~ )-~
-aiD' (8.13)
tbCh,: (Pi -Pw
Qp=I.119~"fw\i-pWflQPD'
)
(8.20)

PD=OatID=Oforall, , (8.14) The implication of our analysis is that cumulative


D production, Q (stock-tank barrels), can be deter-
PD=lat'D=lforalll >0, (8.IS) mined fr~m ~mensionless cum~lative production,
D QpD' "'hlch IS based on solutIons to Eqs. 8.13
a through 8.16. Since QpD is based on these solutions,
-1!.Q ! -it is a function of I D and, [N only (for, D = I); thus,
a'D I -0 for all ID >0. (8.16) it appears possible that Q D could be tabulated (or
ro.,..".=,o.. plotted) as a function of~D for selected values of
Thus, dimensionless pressure PD is a function of 'IN. In fact, this has been done by van Everdingen
dimensionless time, ID' and dimensionless radius'D and Hurst; this problem is their well known constant-
for a given value of the dimensionless parameter, D pressure case.
since no other variables or parameters appear in the The QpD values also can be interpreted as
differential equation, or initial or boundary con- dimensionless cumulative water influx from a radial
ditions. However, the appropriate definition of aquifer into a reservoir if , w is interpreted as the
dimensionless pressure is different in this constant- reservoir radius and, e as the aquifer radius.
pressure case from the constant-rate case-with the
appropriate definition dictated by the boundary
conditions.
E
xercise
.
8ecause (his cons(ant-pressure case is of con- 8.1 Consider (he radial flow of a gas expressed in
siderable prac(ical importance, \\'e proceed further terms of (he pseudopressure, ;(t, by Eq. A.13. Define
with our analysis. It is of in(eres( (0 de(ermine in- dimensionless pseudopressure as
s(an(aneousvolume
produced production
for thisrarecase.
andIns(an(aneous
cumula(ive -;(tD=S0300kh T sc T(;(ti-;(t),
production rare q is -,qgPsc
0.001127 (2~, wh) k ap dimensionless time as
q = -B -a I ' (8.17) -0.00)264 kl
JJ. , r. ID- 2'
oJJ.iC
Ii' h.
and cumula(ive production Qp is and

!
I .1 'D='/'"., I
QP=- 24 ~ qdl. (8.18) T =
reservoir (emperature, .R, ~:..
.~ ...T sc =
s(andard-condi(ion (empera(ure, .R, ~;
I( IS con~'eruent to define a dimensIonless productIon PIc =
standard-condition pressure, psia, """':
rate, qD. qg -, s tlO\\. rate Ms cf/ D ,
2a= I
qBII. lI.i =
gas viscosity evaluated at original ..
q D = 0.00708 kh (p, -P -). reservoir pressure, cp, and
Th I WJ Cgl = gas compressibility evalua(ed at original
en, reservoir pressure, psi -1

qD = ~ I (8.19) The cylindrical reservoir is initially at uniform


a'D ro- I pressure. Pi; there is no flow across the ou(er
, boundary of radius 'e: and flow rate into the
\\e also defIne a dImensIonless cumulatIve produced well bore of radius, h' is constant (expressed at
volume, QpD. standard I.'onditions).
,ID B Write the differen(ial equation, and initial and
QpD = \ q D d'D = I .11. 1 bou~dary c~ndi.tions. in dimensional form; (hen
Q 0.OO708Ah (Pi-Ph/) re\\Tltethemmdlmenslonlessform.

..,

---~I www.petroman.ir
(" C
" In'
'AP"~\,.,t- .~
S'~;'. " .-tMl1 as

.."

Appendix C
Van Everdingen and Hurst Solutions to
Diffusivity Equations

Introduction
In Appendix B, we sho"'ed that solutions to dif- evaluated at 'D = 1, Eq. 1.6 shows the functional
ferential equations describing flow in a petroleum form of /(1 D" eD) -an infinite series of exponentials
reservoir for given initial and boundary conditions and Besselfunctions. This series has been evaluated I
can be expressed compactly using dimensionless for several values of, eD over a wide range of values
variables and parameters. In this appendix, we of ID. Chatas2 tabulated these solutions; a
examine four of these solutions that are important in modification of Chatas' tabulation is presented in
reservoir engineering applications. Tables C.l and C.2.
Some important characteristics of this tabulation
C:°nstant Rate at Inner Boundary, include the following.
No Flow Across Outer Boundary I. Table C.l presents values of PD in the range
This solution of the diffusi\ity equation models 1D < I,(XX) for an infinite-acting reservoir. For ':
radial flow of a slightly compressible liquid in a ID <0.01, PD can be approximated by the relation
homogeneous reservoir of uniform thickness;
reservoir at uniform pressure Pi before production; PD =2.fiD/r. (C.2)
no
production
flow across
at constant
the outerrate
boundary
q from (at
the r single
= r e); well
and 2. Table C.l IS valid for fimte reservoIrs WIth

(centered in the reservoir) "'ith wellbore radius 'MI. ID <0.25 r;D.


The solution-pressure as a function of time and 3. For 100 <ID<0.25 r;D' Table C.I can be
radius for fixed values of re' '.." and rock and fluid extended by use of the equation
pr°J:'erties.- is expressed most conveniently in terms P =0.5(ln 1 + 0.80907). (C.3)
of dImensionless variables and parameters: D D

(This equation is identical to Eq. 1.10 for 5=0. It


PD=/(fD,rD"eD)' '..'.."...'.".' .(C.I) begins to become slightly less accurate at ID=0.0625
h r;D' but there is no simple approximation between
were this value and 1D =0.25 ~D. Fortunately, this slight ..
0.OOi08 kh(Pi -p) loss in accuracy is not of sufficient magnitude to
PD = q8J1.' cause problems in most practical applications.)
4. Table C.2 presents PD as a function of 1D for
-0.<XXJ264kf
I D -4>IJ.C r~ ' 1.5<'eD<10.
(a) For values of 1D smaller than the value listed
-'" for a given reO' the reservoir is infinite acting, and
r D -r/r ..., Table C.l should be used to determinePD.
(b) For values of 1D larger than the largest value
and listed for a given reD (or, more correctly, for 25< 1D
-and 0.25 '~D < 1D)' PD can be calculated from2
'eD-rt'/r.."
2(1D + 0.25)
Eq. C.l states that PD is a function of the variables PD = 2 1
1D and r D for a flXed value of the parameter, eD. The ' eD -
most important solution is that for pressure at the
wellbore radius (,=, MIor'D = I): (3':D-4r:D In' eD-~D -I)
-4(~n -1 )2' (C.4)

,
PDI, =f(/D"~D)' 4(eD-I)
D.I (c) A special case of Eq. C.4 arises when r;D.I; i
When expressed in terms of dimensionless pressure then, I

~ www.petroman.ir ~I
,

VAN EVERDINGEN AND HURST SOLUTIONS TO DIFFUSIVITY EQUATIONS 107

TABLEC.1 -Po VI. to -INFINITE RADIALSYSTEM, Estimate the pressure on the inner boundary of the
CONSTANTRATEATINNERBOUNDARY sand pack at times of 0.00 I, 0.0 I, and 0.1 hour.
-.!.S!- -P..P- ..!.sL -P..P- ~ -p..p- Solution. We first calculate 10 and r ~D:
0 0 0.15 0.3750 60.0 2.4758
0.0005 0.0250 0.2 0.4241 70.0 2.5501 r~D=IO/I=IO,
0.001 0.0352 0.3 0.5024 80.0 2.6147 0.<XX>264kl
0.002 0.0495 0.4 0.5645 90.0 2.6718 1D = ~ --2
0.003 0.0603 0.5 0.6167 100.0 2.7233 ~~ I' MI
0.004 0.0094 0.6 0.6622 150.0 2.9212
0.005 0.0774 0.7 0.7024 200.0 3.0036 (0 <XX>264)(1<XX»
0.006 0.0845 0.8 0.7387 250.0 3.1726 =" 1=4x 103 I.
0.007 0.0911 0.9 0.7716 300.0 3.2630 (0.3)(2)(0.11 x 10-3)(1)
0.008 0.0971 1.0 0.8019 350.0 3.3394 .
0.009 0.1028 1.2 0.8672 400.0 3.4057 Then, the folloWIng data result.
0.01 0.1081 1.4 0.9160 450.0 3.4641 I
,"
0.015 0.1312 2.0 1.0195 500.0 3.5164 1 i
0.02 0.1503 3.0 1.1665 550.03.5643 (hour) ID Po SourceofpD 1
0.025 0.1669 4.0 1.2750 600.0 3.6076 --
0.03 0.1818 5.0 1.3625 650.0 3.6476 0.001 4 1.275 TableC.I (reservoir
0.04 0.2077 6.0 1.4362 700.0 3.6842 infinite acting)
0.05
0.06 0.2301
0.2500 7.0 1.4997 800.0
8.0 1.5557 750.0 37505
3.7184 0.01 40 2.401 Tabl e C .(r
2 ~D = 10)
0.07 0.2680 9.0 1.6057 850.0 3.7805 0.1 400 9.6751 Eq. C.4
0.08 0.2845 10.0 1.6509 900.0 3.8088
0.09
0.1
0.2999 15.0 1.8294 950.0 3.8355
0.3144 20.0 1.9601 1,000.0 3.8584 Note th at f or 1 D =. 400'D1 >0 .~D
25 ~ - (025 )( 10)2
-.:
.
30.0 2.1470 = 25; thus, Eq. C.41s usedto calculate PD.
40.0 2.2824 2(1D + 0.25)
50.0 2.3884 PD = 2
NotesFor'O<0.01.PO.2"o" ~D-I
For100<10<0.25
'~D-PO.0511n
'0+ 0809071
(3r:D -4r:D In r ~D -U;D -I)

21 -4(r;D -1)2 k,
J.

PD=~+lnr~-¥4. (C.5) =9.6751. -


r~D ,.~
5. The PD solutions in Tables C.! and C.2 also A rearrangement of the definition of PD results in I:
apply to a reservoir of radius, MIsurrounded by an qB 1
aquifer of radius
water influx from rthe
e when there
aquifer is athe
into constant rateFor
reservoir. of P =P 1-141.2-.!:-PD
kh
this case, values of, eD in the range 1.5 to 10.0, as in
Table C.2, are of practical importance. For most well (141.2)(1.0)(1.0)(2)
problems, r ~D is larger than 10.0, and the aQproxi- = 15-(1<XX»(0.5) PD
mations given by Eq. C.3 for lOO<ID <0.25 ~D and
Eq. C.5 for 0.25 ~D <I D are the really useful results = 15-0.565 PD.
of the van Everdingen and Hurst analysis.
6. When analyzing a variable-rate problem with Thus, we have these values:
the P D solution, we use superposition just as we did .
with the Ei-function solution in Sec. 1.5. 1 (hour) P (psia)
0.001 14.28
E.\"ampleC.l- Use of PD Solutions ~.~I I~.~
for No-Flow Boundary ..
Problem. In a large laboratory flow experiment, Constant Rate at Inner Boundary,
fluid was produced into a I-ft-radius perforated Constant Pressure at Outer Boundary
cylinder from. a sand-packed model with a rad~us of This solution of the diffusivity equation models
10 ft. No fluId flo~ed across the external radIus of radial flow of a slightly compressible liquid in a
the. m.odel. PropertIes o;f the sandpack and produced homogeneous reservoir of uniform thickness;
fluId mclud~ the followIng. reservoir at uniform pressure Pi before production;
k = I darcy, unchanging pressure, also Pi' at the outer boundary
h = 0.5 ft, (at r=, ~); and production at constant rate q from the
p. = 15 psia, single well (centered in the reservoir) with wellbore
~ = 2 cp, radius ~MI'The solutions, PD (evaluated a.t' D = I), as
q = 1.0 STB/D, a function of 1D for fiXed values of r ~ In the range
B= I.ORB/STB, I.S<'~D<3,<xx>, are given in Table C.3. The
cP = 0.3, and dimensionless variables PD' 'D' 'D' and r ~ have the
c1 = 0.11 x 10 -3 psi -I. same definitions as in the previous section.

I
I
www.petroman.ir
~
C'-!~:~
...
TABLE C.2 -Po Ys. to -FINITE RADIAL SYSTEM WITH CLOSED EXTERIOR BOUNDARY,
CONSTANT RATE AT INNER BOUNDARY
'.0 = 1.5 '.0 = 2.0 '.0 = 2.5 '.0 = 3.0 '.0 = 3.5 '.0 = 4.0
-
!.P---EJL-!.P---ER-!.P---ER-!.P---ER-!.P---ER-!.P---ER-
0.06 0.251 0.22 0.443 0.40 0.565 0.52 0.627 1.0 0.802 1.5 0.927
0.08 0.288 0.24 0.459 0.42 0.576 0.54 0.636 1.1 0.830 1.6 0.948
0.10 0.322 026 0.476 0.44 0.587 0.56 0.645 12 0857 1.7 0.968
0.12 0.355 0.28 0.492 0.46 0.598 0.60 0.662 13 0.882 1.8 0.988
0.14 0.387 0.30 0507 0.48 0.608 0.65 0683 1.4 0.906 1.9 1.007
0.160.420 0.320.522050 0.618 0.70 0.703 1.5 0929 2.01.025
0.18 0.452 034 0536 0.52 0.628 0.75 0.721 1.6 0.951 2.2 1.059
0.20 0.484 0.36 0.551 0.54 0.638 0.80 0.740 1.7 0.973 2.4 1.092
0.220.516 0.38 0.565 0.56 0.6470.850.758 1.8 0.994 2.61.123
0.240.548 0.400.5790.58 0.6570.90 0.776 1.9 1.014 2.81.154
0.26 0.580 0.42 0.593 0.60 0.666 0.95 0.791 2.0 1.034 3.0 1.184
0.28 0.612 0.44 0.607 0.65 0.688 10 0.806 225 1.083 35 1.255
0.300.644 0.460.621 0.700.710 1.2 0.8652.50 1.130 4.01.324
0.350.724 0.48 0.6340.750.731 1.4 0.9202.751.176 4.51.392
040 0.804 0.50 0.648 0.80 0.752 1.6 0.973 3.0 1.221 5.0 1.460
0.45 0.884 0.60 0.715 0.85 0.772 2.0 1.076 4.0 1.401 5.5 1.527
0.50 0.964 0.70 0.782 0.90 0.792 3.0 1.328 5.0 1.579 6.0 1.594
0.55 1.044 0.80 0.849 0.95 0.812 4.0 1.578 6.0 1.757 6.5 1.660
0.60 1.124 0.90 0.915 1.00 0.8325.0 1.828 7.01.727
0.65 1.204 1.0 0.982 2.0 1.215 8.0 1.861
0.70 1.284 2.0 1.649 3.0 1.506 9.0 1.994
0.75 1.364 3.0 2.316 4.0 1.977 10.0 2.127
0.80 1.444 5.0 3.649 5.0 2.398
'.0=4.5 '.0=5.0 '.0=6.0 '.0=7.0 '.0=8.0 '.0=9.0 '.0=10.0 .
-2.0 !.P---E-L!.P---ER-!JL-ER-!.P---EJL!.P---EJL!.P---ER-~~
1.023 3.0 1.167 4.0 1.275 6.0 1.436 8.0 1.556 10.0 1.651 12.0 1.732
2.1 1.040 3.1 1.180 4.5 1.322 6.5 1.470 8.5 1.582 10.5 1.673 12.5 1.750
2.2 1.056 3.2 1.192 5.0 1.364 7.0 1.501 9.0 1.607 11.0 1.693 13.0 1.768
2.3 1.702 3.3 1.204 5.5 1.404 7.5 1.531 9.5 1.631 11.5 1.713 13.5 1.784
2.4 1.087 3.4 1.215 6.0 1.441 8.0 1.559 10.0 1.653 12.0 1.732 14.0 1.801
2.5 1.102 3.5 1.227 6.5 1.477 8.5 1.586 10.5 1.675 12.5 1.750 14.5 1.817
2.6 1.116 3.6 1.238 7.0 1.511 9.0 1.61311.0 1.69713.0 1.768 15.01.832
2.7 1.130 3.7 1249 7.5 1.544 9.5 1.638 11.5 1.717 13.5 1.786 15.5 1.847
2.8 1.144 3.8 1.259 8.0 1.57610.0 1.66312.0 1.73714.0 1.803 16.01.862
2.9 1.158 3.9 1.270 8.5 1.607 11.0 1.711 12.5 1.757 14.5 1.819 17.0 1.890
3.0 1.171 4.0 1.281 9.0 1.638 12.0 1.757 13.0 1.776 15.0 1.835 18.0 1.917
3.2 1.197 4.2 1.301 9.5 1668 13.0 1.810 13.5 1.795 15.5 1.851 19.0 1.943
3.4 1.222 4.4 1321 100 1.698 14.0 1.845 14.0 1.813 16.0 1.867 20.0 1.968
3.6 1.246 4.6 1.340 11.0 1.757 15.0 1.888 14.5 1.831 17.0 1.897 22.0 2.017
3.8 1.269 4.8 1.360 12.0 1.815 16.0 1.931 15.0 1.849 18.0 1.926 24.0 2.063
4.0 1.292 5.0 1.378 130 1.873 17.0 1.974 17.0 1.919 190 1.955 26.0 2.108 ..
4.5 1.349 5.5 1 424 14.0 1.931 18.0 2.016 19.0 1.986 20.0 1.983 28.0 2.151
5.0 1.403 6.0 1.469 15.0 1.988 19.0 2.058 21.0 2.051 22.0 2.037 30.0 2.194
5.5 1.457 6.5 1.513 16.0 2.045 20.0 2.100 23.0 2.116 240 2.096 32.0 2.236
6.0 1.510 7.0 1 556 17.0 2.103 22.0 2.184 25.0 2.180 260 2.142 34.0 2.278
7.0 1.615 7.5 1598 180 2.160 24.0 2267 30.0 2.340 28.0 2.193 36.0 2.319
8.0 1.719 8.0 1.641 19.0 2.217 26.0 2.351 35.0 2.499 30.0 2.244 38.0 2.360
9.0 1.823 9.0 1.725 20.0 2.274 28.0 2.434 40.0 2.658 34.0 2.345 40.0 2.401
10.0 1.927 10.0 1.808 25.0 2.560 30.0 2.517 45.0 2.817 380 2.446 50.0 2604
11.0 2.031 11.0 1.892 30.0 2.846 40.0 2.496 60.0 2.806
12.0 2.135 12.0 1.975 45.0 2.621 70.0 3.008
13.0 2.239 13.0 2.059 50.0 2.746 80.0 3.210
14.0 2.343 14.0 2.142 60.0 2.996 90.0 3.412
15.0 2.447 15.0 2.225 70.0 3.246 100.0 3.614
NotesFor10smatlerIhanva'lJ~s
listed,nthIstablefora gIven
reO reservO"
IS,nl.nlleactIng
~lndPO,nTableC1
For2S<10 and'0 largerthan.aluesonlable
.IriO
PO-' -11 - 3f~O-.f~oln'eO-2f~O-1
(';'+2/01 ~fio -1)2 .
..
ForWellsonbOunded
r.servoofs
WIth
2 .1.
feD
Po,2/0+lnf.O-" I
f~O I
I i
www.petroman.ir
-

VAN EVERDINGEN AND HURST SOLUTIONS TO DIFFUSIVITY EQUATIONS 109

TABLE C.3 -Po YI. to -FINITE RADIAL SYSTEM WITH FIXED CONSTANT PRESSURE
AT EXTERIOR BOUNDARY, CONSTANT RATE AT INNER BOUNDARY

'_0 = 1.5 '_0 = 2.0 '-0 = 2.5 '_0 = 3.0 '_0 = 3.5 '_0 = 4.0 '_0 = 6.0
to Po to Po to Po to Po to Po to Po to Po

O:OSOo:2"3O"f;iii"~~o:5O"2-O:-SO00-O:-SO0:620~D:8()24:0~
0.055 0.240 0.22 0.441 0.35 0.535 0.55 0.640 0.60 0.665 1.2 0.857 4.5 1.320
0.060 0.249 0.24 0.457 0.40 0.564 0.60 0.662 0.70 0.705 1.4 0.905 5.0 1.361
0.070 0.266 0.26 0.472 0.45 0.591 0.70 0.702 0.80 0.741 1.6 0.947 5.5 1.398
0.080 0.282 0.28 0485 0.50 0.616 0.80 0.738 0.90 0.774 1.8 0986 6.0 1.432
0.090 0.292 0.30 0.498 0.55 0.638 0.90 0.770 1.0 0.804 2.0 1.020 6.5 1.462
0.10 0.307 0.35 0.527 0.60 0.659 1.0 0.799 1.2 0.858 2.2 1.052 7.0 1.490
0.12 0.328 0.40 0.552 0.70 0.696 1.2 0.850 1.4 0.904 2.4 1.080 7.5 1.516
0.14 0.344 0.45 0.573 0.80 0.728 1.4 0.892 1.6 0.945 2.6 1.106 8.0 1.539
0.16 0.356 O.SO 0591 090 0.755 1.6 0.927 1.8 0.981 2.8 1.130 8.5 1.561
0.18 0.3670.55 0606 1.0 0.778 1.8 0.955 2.0 1.013 3.0 1.152 9.0 1.580
0.20 0.3750.60 0.6191.2 0.815 2.0 0.980 2.2 1.041 3.4 1.190 10.0 1.615
0.22 0.381 0.65 0.630 1.4 0.842 2.2 1.000 2.4 1.065 3.8 1232 12.0 1.667
0.24 0.386 0.70 0.639 1.6 0.861 2.4 1016 2.6 1.087 4.5 1.266 14.0 1.704
0.26 0.390 0.75 0.647 1.8 0.876 2.6 1.030 2.8 1.106 5.0 1.290 16.0 1.730
0.28 0.393 0.80 0654 2.0 0.887 2.8 1.042 3.0 1.123 5.5 1309 18.0 1.749
0.30 0.396 0.85 0.660 2.2 0.895 3.0 1.051 3.5 1.153 6.0 1.325 20.0 1.762
0.35 0.400 0.90 0665 2.4 0.900 3.5 1.069 4.0 1.183 7.0 1.347 22.0 1.771
0.40 0.402 0.95 0.669 2.6 0.905 4.0 1.080 5.0 1.225 8.0 1.361 24.0 1.777
0.45 0.404 1.0 0.673 2.8 0.908 4.5 1.087 6.0 1.232 9.0 1.370 26.0 1.781
O.SO 0.405 1.2 0.682 3.0 0.910 5.0 1.091 7.0 1.242 10.0 1.376 28.0 1.784
0.60 0.405 1.4 0.688 3.5 0.913 5.5 1.094 8.0 1.247 12.0 1.382 ~.O 1.787
0.70 0.405 1.6 0.690 4.0 0.915 6.0 1096 9.0 1.2SO 140 1.385 35.0 1.789
0.80 0.405 2.0
1.8 0.692
0.692 4.5
5.0 0.916
0.916 6.5
7.0 1.097
1.097 10.0
12.0 1.251
1.252 16.0
18.0 1386
1.386 40.0
SO.O 1.791
1.792 -
2.5 0.693 5.5 0.916 8.0 1.098 14.0 1.253
3.0 0.693 6.0 0.916 10.0 1.099 16.0 1.253
'_0=8.0 '_0=10.0 '_0=15.0 '_0=20.0 '_0=25.0 '_0=30.0 '_0=40.0 '
to Po to Po to Po to Po to Po to Po to Po
7-:0- 1:""499
10:0 1-:651"""20:0
~ ~ i148 -so-:o2:"389-ro:-o ~ 120 2:B13
7.5 1.527 12.0 1.730 22.0 2.003 35.0 2.219 55.0 2.434 80.0 2.615 140 2.888
8.0 1.554 14.0 1.798 24.0 2.043 40.0 2.282 60.0 2.476 90.0 2.672 160 2.953 :
8.5 1580 16.0 1.856 26.0 2.080 45.0 2.338 65.0 2.514 100.0 2.723 180 3.011 .-
9.0 1.604 18.0 1.907 28.0 2.114 SO.O 2.388 70.0 2.550 120.0 2.812 200 3.063
9.5 1.627 20.0 1.952 30.0 2.146 60.0 2475 75.0 2.583 140.0 2.886 220 3.109
10.0 1.648 25.0 2.043 35.0 2.218 70.0 2.547 80.0 2.614 160.0 2.950 240 3.152
12.0 1.724 30.0 2.111 40.0 2.279 80.0 2.609 85.0 2.643 165.0 2.965 260 3.191
14.0 1.786 35.0 2.160 45.0 2.332 90.0 2.658 900 2.671 170.0 2.979 280 3.226
16.0 1.837 40.0 2.197 50.0 2.379 100.0 2.707 95.0 2.697 1750 2.992 300 3.259 ..
18.0 1.879 45.0 2.224 60.0 2.455 105.0 2.728 100.0 2.721 1800 3.006 350 3.331
20.0 1.914 50.0 2.245 70.0 2.513 110.0 2747 120.0 2.807 2000 3.054 400 3.391
22.0 1.943 55.0 2.260 80.0 2.558 115.0 2.764 140.0 2.878 250.0 3.150 450 3.440
24.0 1.967 60.0 2.271 90.0 2.592 120.0 2.781 1600 2.936 300.0 3.219 500 3.482
26.0 1.986 65.0 2.279 100.0 2.619 125.0 2.796 180.0 2.984 3500 3.269 550 3.516
28.0 2.002 70.0 2.285 120.0 2.655 130.0 2.810 200.0 3.024 400.0 3.306 600 3.545
30.0 2.016 75.0 2.290 140.0 2.677 135.0 2.823 220.0 3.057 450.0 3.332 650 3.568
35.0 2.040 80.0 2.293 160.0 2.689 140.0 2.835 240.0 3.085 500.0 3.351 700 3.588
40.0 2.055 90.0 2.297 180.0 2.697 145.0 2.846 260.0 3.107 600.0 3.375 800 3.619
45.0 2064 100.0 2.300 200.0 2.701 1SO.0 2.857 280.0 3.126 700.0 3.387 900 3.640
SO.O 2.070 110.0 2.301 220.0 2.704 160.0 2.876 3000 3.142 8000 3.394 1,000 3.655
60.0 2.076 120.0 2.302 240.0 2.706 180.0 2.906 350.0 3171 900.0 3.397 1.200 3.672
70.0 2.078 130.0 2.302 2600 2.707 200.0 2.929 400.0 3.189 1.000 3.399 1,400 3.681
80.0 2.079 140.0 2.302 280.0 2.707 240.0 2.958 450.0 3.200 1.200 3.401 1.600 3.685
160.0 2.303 300.0 2.708 280.0 2.975 500.0 3.207 1.400 3.401 1,800 3.687
300.0 2.980 600.0 3.214 2.000 3.688
400.0 2.992 700.0 3.217 2.500 3.689
500.0 2.995 800.0 3.218
900.0 3.219

Noles For to sm.ller th.h values IISled in thIs table '°' a 9,ven '_0 'eservo.' IS InfInIte aCllno Find Po In Table C 1

i For to '.'ger Ihan values IISled in Ihis lable. Po i In '_0

~ IIJ

. www.petroman.ir
~-
TABLE C.3 -CONTINUED

'.0=50.0 '.0=60.0 '.0=70.0 '.0=80.0 '.0=90.0 '.0=100.0 '.0=200.0


'0 Po to Po to Po to Po to Po to PD to Po
200 ~ ~ i2S7 ~ 3.512 600 3.603 800 3.747 1.000 3859 1.500 4.061
220 3.111 400 3.401 600 3603 700 3.680 900 3.806 1.200 3 949 2.000 4.205
240 3.154 500 3.512 700 3680 800 3.747 1.000 3.858 1.400 4026 2.500 4.317
260 3.193 600 3602 800 3.746 900 3.805 1,200 3.949 1.600 4.092 3.OCKJ4.498
280 3.229 700 3.676 900 3803 1.000 3.857 1,300 3.988 1.800 4.150 3.500 4.485
300 3.263 800 3.739 1.000 3.854 1.200 3.946 1.400 4.025 2.000 4200 4.OCKJ4.552
350 3.339 900 3.792 1.200 3937 1.400 4.019 1.500 4058 2.500 4303 5.OCKJ4.663
400 3.405 1.000 3.832 1.400 4.003 1.500 4.051 1.800 4.144 3,000 4379 6.OCKJ4.754
450 3.461 1.200 3.908 1.600 4.054 1.600 4.080 2.000 4.192 3,500 4.434 7,OCKJ4.829
500 3.512 1.400 3.959 1.800 4.095 1.800 4130 2.500 4285 4.OCKJ 4478 8.OCKJ 4.834
550 3.556 1,600 3.996 2.000 4127 2.000 4.171 3.000 4.349 4.500 4.510 9.OCKJ 4.949
600 3.595 1.800 4.023 2.500 4.181 2.500 4.248 3.500 4.394 5,000 4.534 10.OCKJ4.996
650 3.630 2.000 4.043 3,OCKJ4.211 3.000 4.297 4.000 4.426 5.500 4.552 12.OCKJ5.072
700 3.661 2,500 4.071 3.500 4.228 3.500 4.328 4.500 4.448 6,000 4.565 14.000 5.129
750 3.688 3,000 4.084 4.OCKJ4.237 4.000 4347 5,000 4.464 6.500 4579 16.OCKJ5.171
800 3.713 3,500 4.090 4.500 4.242 4,500 4.360 6.000 4.482 7.000 4583 18.OCKJ5.203
850 3.735 4.000 4.092 5.000 4.245 5.000 4368 7.000 4.491 7,500 4.588 2O,OCKJ5.227
900 3.754 4.500 4093 5.500 4.247 6.000 4.376 8.000 4.496 8.000 4.593 25.OCKJ5.264
950 3.771 5.000 4094 6.OCKJ 4.247 7,000 4.380 9.000 4.498 9.000 4.598 3O.OCKJ5.282
1.000 3.787 5.500 4094 6.500 4.248 8,000 4.381 10.000 4.499 10.000 4601 35.OCKJ5.290
1.200 3.833 7,000 4.248 9.000 4.382 11.000 4.499 12,500 4604 40.OCKJ5.294
1.400 3.862 7,500 4.248 10.000 4.382 12.000 4.500 15.000 4.605
1.600 3.881 8.OCKJ 4.248 11.000 4.382 14.000 4.500
1,800 3.892
2,000 3.900 -
2,200 3.904
2.400 3.907
2,600 3.909
2.800 3910

'.0 = 300.0 '.0 = 400.0 '.0 = 500.0 '.0 = 600.0 '.0 = 700.0 '.0 = BOO.O
-.!.J?- -E.!L -.!.J?- -.PJL ~ -.PJL ~ -.PJL -.!.J?- -E.!L to-.PJL
6.000 4.754 15.000 5.212 20.000 5.365 40.000 5.703 50.000 5.814 70.OCKJ5.983
8.000 4.898 20.000 5.356 25.000 5468 45.000 5.762 60,000 5.905 SO.OCKJ6.049
10.000 5.010 30.000 5.556 30.000 5.559 50.000 5.814 70,000 5.982 9O.OCKJ6.108
12.000 5.101 40.000 5.689 35.000 5.636 60.000 5.904 80.000 6.048 100.OCKJ6.160
14.000 5177 50.0005.781 40.000 5.702 70.0005979 90.0006.105 120.OCKJ6.249 ..
16.000 5242 60.0005.845 45.0005.759 80.000 6.041 100.0006156 140.OCKJ6.322
18.000 5.299 7000C 5.889 50000 5.810 90.000 6.094 120.000 6.239 160.OCKJ6.382
20.000 5.348 80.000 5.920 60.000 5.894 100.000 6.139 140.000 6305 1SO.OCKJ6.432
24,000 5.429 90.000 5.942 70.000 5.960 120.000 6.210 160.000 6.357 2OO,OCKJ6.474
28.000 5.491 100.000 5.957 80.000 6.013 140.000 6.262 180.000 6.398 250.OCKJ6.551
30.000 5.517 110.000 5.967 90.000 6.055 160.000 6.299 200.000 6.430 300.000 6.599
40.000 5606 120.000 5.975 100.000 6088 180.000 6.326 250.000 6.484 350.OCKJ6.630
50.000 5652 125.000 5.977 120.000 6.135 200.000 6.345 300,000 6.514 400.000 6.650
60,000 5.676 130,000 5.980 140.000 6.164 250.000 6.374 350,000 6.530 450,OCKJ6.663
70.000 5.690 140.000 5.983 160.000 6.183 3OO.OCKJ 6.387 400.000 6.540 5OO.OCKJ6.671
80.000 5.696 160.000 5.988 180.000 6.195 350.000 6.392 450.000 6.545 550,OCKJ6.676
90.000 5.700 180.000 5.990 200.000 6.202 400.000 6.395 500.000 6.548 600.000 6.679
100.000 5.702 200.000 5.991 250.000 6.211 500.000 6.397 600.000 6.550 700.000 6.682
120.000 5.703 240.000 5.991 300000 6.213 600.000 6397 700.000 6.551 800.000 6.684
140,000 5.704 260,000 5.991 350.000 6.214 800.000 6.551 1.000.000 6.684
150,000 5.704 400.000 6.214

'"
;
it
www.petroman.ir
VANEVERDINGEN
ANDHURST
SOLUTIONS
TODIFFUSIVITY
EQUATIONS 111

TABLEC.3-CONTINUED
I '.0 =900.0 '.0 = 1.000.0 '.0 = 1.200.0 '.0 =1.400.0 '.0 = 1.600.0 '.0 = 1.800.0
, to Po to Po to Po to Po to Po to Po
i a~ 6-:-o-:i91~ i161 ~ 6:""5():; ~ 6:""5():; i"5(iO5) i619 ~ 6-:7"10
9.0(104) 6.108 1.2(105) 6.252 3.0(105) 6.704 2.5(105) 6.619 3.0(105) 6710 4.0(105) 6854
1.0(105) 6.161 1.4(105) 6.329 4.0(105) 6.833 3.~105) 6.709 3.5(105) 6.787 5.0(105) 6.965
1.2(105) 6251 1.6(105) 6.395 5.0(105) 6918 3.5(105) 6.785 4.0(105) 6853 6.0(105) 7.054
14(105) 6.327 1.8(105) 6.452 60(105) 6975 4.~105) 6849 5.0(105) 6.962 7.0(105) 7.120
1.6(105) 6.392 2.0(105) 6.503 7.0(105) 7013 5.~105) 6.950 6.0(105) 7.046 8.0(105) 7.183
1.8(105) 6447 2.5(105) 6.605 8.0(105) 7038 6.~105) 7026 7.0(105) 7.114 9.0(105) 7238
2.0(105) 6.494 3.0(105) 6.681 9.0(105) 7056 7.~105) 7.082 8.0(105) 7.167 1.0(106) 7.280
2.5(105) 6.587 3.5(105) 6738 1.0(106) 7067 8.~105) 7123 9.0(105) 7.210 1.5(106) 7.407
3.0(105) 6.652 4.0(105) 6.781 1.2(106) 7.080 9.~105) 7.154 1.0(106) 7244 2.0(106) 7.459
4.0(105) 6.729 4.5(105) 6.813 1.4(106) 7.085 1.~106) 7.177 1.5(106) 7.334 3.0(106) 7489
45(105) 6.751 5.0(105) 6.837 16(106) 7088 15(106) 7229 2.0(106) 7364 4.0(106) 7495
50(105) 6.766 5.5(105) 6.854 1.8(106) 7089 2.~106) 7.241 2.5(106) 7.373 5.0(106) 7495
55(105) 6.777 6.0(105) 6.868 1.9(106) 7089 2.5(106) 7.243 3.0(106) 7376 5.1(106) 7.495
60(105) 6.785 7.0(105) 6.885 2.0(106) 7.090 3.~106) 7.244 35(106) 7.377 5.2(106) 7.495
70(105) 6.794 8.0(105) 6895 2.1(106) 7.090 3.1(106) 7244 4.0(106) 7378 5.3(106) 7.495
8.0(105) 6.798 9.0(105) 6.901 2.2(106) 7.090 3.2(106) 7.244 4.2(106) 7.378 5.4(106) 7.495
9.0(105) 6.800 1.0(106) 6.904 2.3(106) 7090 3.3(106) 7244 4.4(106) 7378 5.6(106) 7.495
10(106) 6.801 1.2(106) 6.907 2.4(106) 7090
'.0 = 2.000.0 '.0 = 2.200.0 '.0 =2.400.0 '.0 = 2.600.0 '.0 =2.800.0 '.0 = 3.000.0
to Po to Po to Po to Po to Po to Po
4~ 6:8"54 ~ 6:""966 6~ ~ ~ 7:1"34" 8~ "i2O1 1~ 7:3-;-2
5.0(105) 6.966 5.5(105) 7.013 7.0(105) 7. 134 8.~105) 7.201 9.0(105) 7260 1.2(106) 7.403
6.0(105) 7.056 6.0(105) 7.057 8.0(105) 7.200 9.~105) 7.259 1.0(106) 7.312 1.4(106) 7.480
7.0(105) 7.132 6.5(105) 7.097 9.0(105) 7.259 1.~106) 7.312 1.2(106) 7.403 1.6(106) 7.545
8.0(105) 7.196 7.0(105) 7.1331.0(106) 7.3101.2(106) 7.4011.6(106) 7.5421.8(106) 7.602
9.0(105) 7.251 7.5(105) 7.167 1.2(106) 7.398 1.4(106) 7.475 2.0(106) 7.644 2.0(106) 7.651
1.0(106) 7.298 8.0(105) 7.199 1.6(106) 7.526 1.6(106) 7.536 2.4(106) 7.719 2.4(106) 7.732
1.2(106) 7.374 8.5(105) 7.229 2.0(106) 7.611 1.8(106) 7.588 2.8(106) 7.775 2.8(106) 7.794
1.4(106) 7.431 9.0(105) 7.256 2.4(106) 7668 2.0(106) 7631 3.0(106) 7.797 3.0(106) 7.820
1.6(106) 7.474 1.0(106) 7.307 2.8(106) 7.706 2.4(106) 7.699 3.5(106) 7.840 3.5(106) 7.871
1.8(106) 7.506 1.2(106) 7.390 3.0(106) 7.720 2.8(106) 7.746 4.0(106) 7.870 4.0(106) 7.908
2.0(106) 7.530 1.6(106) 7.507 3.5(106) 7.745 3.~106) 7.765 5.0(106) 7.905 4.5(106) 7.935
2.5(106) 7.566 2.0(106) 7.579 4.0(106) 7.760 3.5(106) 7.799 6.0(106) 7.922 5.0(106) 7.955
3.0(106) 7.584 2.5(106) 7.631 5.0(106) 7.775 4.~106) 7.821 7.0(106) 7.930 6.0(106) 7.979
3.5(106) 7.593 3.0(106) 7.661 6.0(106) 7.780 5.~106) 7.845 8.0(106) 7.934 7.0(106) 7.992
4.0(106) 7.597 3.5(106) 7.677 7.0(106) 7.782 6.~106) 7.856 9.0(106) 7.936 8.0(106) 7.999
5.0(106) 7.600 4.0(106) 7.686 8.0(106) 7.783 7.~106) 7.860 1.0(107) 7.937 9.0(106) 8.002
6.0(106) 7.601 5.0(106) 7.693 9.0(106) 7.783 8.~106) 7.862 1.2(107) 7.937 1.0(107) 8.004
6.4(106) 7.601 6.0(106) 7.695 9.5(106) 7.783 9.~106) 7863 1.3(107) 7.937 1.2(107) 8.006
7.0(106) 7.696 1.0(107) 7863 1.5(107) 8.006
8.0(106) 7.696

Some important properties of these tabulated c, = 20 x 10 -6 psi -1 ,


solutions include the following: r w = 0.45 ft, ..
I. For values of tD smaller than the smallest value /" = 0.8 cp,
listed for a given reD' the reservoir is infinite acting, h = 12 ft, and
and Table C.I should be used to determine PD' B = 1.25 RB/STB.
2. For values of 1 D larger than the largest value ..
listed for a given r (or for 1 >~ ) Calculate pressure In the wellbore for production

PD~lnreD'
eD D eD'
(C.6)
afterO.I, 1.0, and lOO.Odavs.
. .- ~
SolUtIon. Smce ID- 0 .VVV264 k/1 ct>/"c,rw2 (I .In
hours), then
Example C.2 -Use of PD Solutions ID = (0.00)264)(80)(24)_1 days -
for Constant-Pressure Boundary (0.21 )(0.8)(20 x 10 -6)(0.45)2

Problem. An oil well is producing 300 STB/D from a


water-drive reservoir that maintains pressure at the =7.45 x 105t.
original oil/water contact at a constant 3,00) psia.
Distance from the well to the oil/water contact is 900 Also
ft. Well and reservoir properties include the '

following.
reD-
-r e -900 - 2,vvv.
/VVI

ct>= 0.21, r", 0.45


k = 80 md, Thus, we obtain the following.~

www.petroman.ir
r
1 12 WELL TESTING

TABLE C.4 -OpO Ys. to -INFINITE RADIAL SYSTEM. CONSTANT PRESSURE AT INNER BOUNDARY

DimenSIon. D,menSion. DImenSion. D,menSion. DimenSIon.


Dimension. less DImenSIon. less DimenSion. less DimenSIOn. less DimenSIon. less
less CumulatIve less CumulatIve less Cumulative less Cumulative less Cumulative
Time ProductIon Time Production TIme PrOductIon Time PrOduction Time Production
-'0 __~P-e- _!O_- -~p'p- --'-P-- -9-Po- '0- _°2.0- -.!JJ-- CpO
0.00 0.000 98 42.433 740 226904 1.975 528337 8.900 1.986796
001 0112 99 42781 750 229514 2.000 534145 9.000 2.~.828
005 0278 100 43129 760 232.120 2.025 539945 9100 2.026438
0 10 0404 105 44858 770 234.721 2.050 545 737 9.200 2.046227
015 0520 110 46574 775 236020 2.075 551522 9.300 2.065996
020 0606 115 48277 780 237318 2.100 557299 9.400 2.0857«
025 0689 120 49968 790 239912 2. 125 563~ 9.5OC 2.105473
0.30 0758 125 51648 800 242501 2.150 568830 9.600 2.125.184
040 0898 130 53317 810 245086 2.175 574585 9.700 2.144878
050 1020 135 54976 820 247668 2.200 580 332 9.800 2.164555
060 1 140 140 56625 825 248957 2.225 586072 9.900 2.184.216
070 1251 145 58265 830 250245 2.250 591 086 10.000 2.203861
080 1359 150 59895 840 252819 2.275 597532 12.500 2.688967
0.90 1469 155 61517 850 255388 2.300 603252 15.000 3.164.780
1 1569 160 63 131 860 257953 2.325 608965 17.500 3.633368
2 2447 165 64737 870 260515 2.350 614672 20.000 4.095800
3 3.202 170 66336 875 261795 2.375 620372 25.000 5.005.726
4 3.893 175 67928 880 263073 2.400 626~ 30.000 5.899508
5 4539 180 69512 890 265629 2.425 631755 35.000 6.780247
6 5153 185 71090 900 268 181 2.450 637437 40.000 7.650096
7 5743 190 72661 910 270729 2.475 643113 50.000 9.363.099
8 6314 195 74226 920 273.274 2.500 648781 60.000 11.047.299
9 6.869 200 75785 925 274545 2.550 660093 70.000 12.708358
10 7.411 205 77338 930 275.815 2.600 671379 75.000 13.531.457
11 7940 210 78886 940 278353 2.650 682640 80.000 14.350.121
12 8457 215 80428 950 280888 2.700 693877 90.000 15.975389
13 8964 220 81965 960 283.420 2.750 705<m 100.000 17.586.284
14 9461 225 83.497 970 285948 2.800 716~ 125.000 21.560.732
15 9949 230 85023 975 287211 2.850 727«9 1.5(105) 2.538(104)
16 10434 235 86.545 980 288473 2.900 738598 2.~105) 3.308(104)
17 10913 240 88062 990 290995 2.950 749725 2.5(105) 4006(104)
18 11.386 245 89575 1.000 293.514 3.000 760.833 3.~105) 4.817(104)
19 11855 250 91084 1.010 296.030 3.050 771.922 4.~105) 6.2671104)
20 12319 255 92589 1.020 298543 3.100 782992 5.~105) 7699(104)
21 12778 260 94090 1.025 299799 3.150 794042 6.~105) 91131104)
22 13233 265 95568 1.030 301053 3.200 805075 7.~105) 1.0511105)
23 13684 270 97081 1.040 303.560 3.250 816<m 8.~105) 1.1891105)
24 14131 275 98.571 1.050 306.065 3.300 827088 9.~105) 1326<105)
25 14.573 280 100057 1.060 308.567 3.350 838067 1.~106) 1462t105)
26 15.013 285 101540 1.070 311~ 3.400 849028 1.5(106) 2.126(105)
27 15450 290 103019 1.075 312.314 3.450 859974 2.~106) 27811105)
28 15883 295 104495 1.080 313.562 3.500 870903 2.5(106) 3427(105)
29 16313 300 105968 1.090 316055 3.550 881816 3.~106) 4.064!105)
30 16.742 305 107437 1.100 318.5015 3.600 892. 712 4.~106) 5.3131105)
31 17 167 310 108904 1.110 321.032 3.650 903594 5.~106) 6.~105)
32 17590 315 110.367 1.120 323517 3.700 914459 6.~106) 7.7611105)
33 18.011 320 111.827 1.125 324760 3.750 926309 7.~106) 8.965(105)
34 18429 325 113284 1.130 326000 3800 936. 1« 8.~1061 1016(106)
35 18845 330 114738 1.140 328.480 3.850 946966 9.~106) 1.134{106)
36 19259 335 116189 1.150 330.958 3.900 957773 1.~107) 1252<106)
37 19671 340 117638 1160 333433 3.950 968.566 1.5(101 1828(106)
38 20080 345 119083 1.170 335.906 4.000 9793« 2.~101 2398(106)
39 20488 350 120526 1.175 337142 4.050 990.108 2.5(107) 2961(106)
40 20894 355 121966 1.180 338376 4.100 1.000858 3~107) 3517(1061...
41 21298 360 123403 1.190 340.843 4.150 1.011595 4.~107) 461~106)
42 21701 365 124838 1.200 343.308 4.200 1.022318 5.~1071 5.689(106)
43 22101 370 126270 1.210 345.770 4.250 1.033028 6.~101 6758{106)
44 22500 375 127699 1.220 348.230 4.300 1.043724 7.~101 7816(106)
45 22.897 380 129126 1.225 349460 4.350 1.054409 8.~107) 8866(106)
46 23291 385 130550 1.230 350688 4.400 1.065.082 9.~101 9.911(106)
47 23684 390 131972 1.240 353.144 4.450 1075.743 1.~108) 1095(107)
48 24076 395 133391 1.250 355597 4.500 1.086390 1.5(108) 1.6O4{107)
49 24466 400 134808 1.260 358048 4.550 1.097024 2.~108) 2. 108{107)
50 24855 405 136223 1.270 360496 4.600 1.107.646 2.5(108) 2.6071107)
51 25.244 410 137635 1.275 361720 4.650 1.118.257 3.~108) 3. 1~107)
52 25.633 415 139.045 1.280 362942 4.700 1. 128854 4~108) 4071(107)
53 26020 420 140453 1.290 365386 4.750 1. 139.439 5.~108) 503 (107)
54 26.406 425 141859 1.300 367828 4.800 1. 150012 6.~108) 5.98 1107)
55 26791 430 143.262 1.310 370267 4.850 1.160.574 7~108) 69281107)
56 27.174 435 144.664 1.320 372.704 4.900 1.171 125 8.~108) 7865(107)
57 27555 440 146.064 1.325 373.922 4.950 1. 181666 9.~108) 87971107)
58 27935 445 147461 1.330 375139 5.000 1. 192. 1~ 1.~109) 9725(107)
59 28314 450 148856 1.340 377.572 5.100 1.213222 1.5(109, 1429(108)
60 28691 455 150.249 1.350 380003 5.200 1.234203 2.~109) 1~108)
61 29068 460 151460 1.360 382.432 5.300 1.255.141 2.5(109) 2.3281108)
62 29.443 465 153029 1.370 384859 5.400 1.276.037 3.~1091 2.771(108)

Noles For/O' 0010pO=2'/01'

For /0 . 200 0pO' -.29881 + 202566' 0


Into

www.petroman.ir
VAN EVERDINGEN AND HURST SOLUTIONS TO DIFFUSIVITY EQUATIONS -.113

TABLE C.. -CONTINUED


D,menSIon. D,menSIon. D,mens,on. D,mensIon. DimenSion
DImenSIon. less D,menSion. less D,menSIon. less D,menSIon. less D,menSIon. less
less CumulatIve less CumulatIve less Cumulative less CumulatIve less CumulatIve
TIme ProductIon TIme Production T,me ProductIon Time Production TIme Product,on
-.!.~ -_<?eo!?- '0 CpO _'0 CpO '0 _f?pp ---~Q- -C~o
63 29818 470 1~.416 1.375 386.070 5.500 1.296893 4~109) 3645(106)
64 JO192 475 155801 1.380 387283 5600 1.317709 5.~109) '51~108)
65 JO565 480 157184 1.390 389705 5.700 1338 486 6~109, 53681108,
66 JO937 485 158565 1.400 392125 5.800 1359225 7~109) 62~108)
67 31308 490 159945 1.410 394 543 5900 1.379927 8~10Y) 7 ~10S)
68 31679 495 161322 1.420 396959 600) 1400593 9~10~ 79O9110S,
69 32048 500 162698 1.425 398167 6.100 1421224 1~10' ) 8747(108)
70 32'17 510 165444 1.430 399373 6200 1441820 1.5(10'°) 128&109)
71 32785 520 168 183 1.440 401786 6.300 1462383 2.~10'0) 1697(109)
72 33 151 525 169549 1.'50 404 197 6400 1.482912 2.5(10 'O) 21031109)
73 33517 530 170914 1.460 406606 6500 1503408 3~10'0) 2505(109)
74 33883 540 173639 1470 409013 6600 1523872 4~10'0) 3299(109) i
75 34247 550 176357 1475 410214 6700 1.544305 5~10'0) '0871109) l
76 34611 560 179069 1.480 411418 6800 1.564706 6~10'0) 4868(109) i
77 34974 570 181774 1.490 413820 6.900 1585077 7~10'0) 5643110~)
78 35336 580 184473 1.500 416220 700) 1.605418 8~10'0) 6.414(109)
79 35697 590 187166 1.525 422.214 7.100 1.625729 9.~10'0) 7183110~)
80 36058 600 189852 1.550 428196 7200 1.646011 1.~10") 7948110~)
81 364'8 610 192533 1.575 434168 7300 1666265 15/10'" 117/10'°)
82 36777 620 195208 1.600 440128 7400 1.686490 2.~10") 155110'0)
83 37.136 625 196544 1.625 446077 7500 1.706688 2.5/10'1) 192(10'°)
84 37494 630 197878 1.650 452016 7.600 1.726859 3.~10") 2.29110'°)
85 37851 640 200542 1.675 457945 7.700 1.747002 4.~10'" 3.02(10'°)
86 38207 650 203201 1.700 463863 7800 1.767120 5.~10") 375110'0)
87 38563 660 205854 1.725 469771 7900 1.787212 6.~10"1 447'10'°)
88 38919 670 208502 1.750 475669 8.00) 1807278 7~10") 519110'°)
89 39272 675 209.825 1.775 481558 8.100 1.827319 8.~10") 5.89110:O)
90 39626 680 211145 1.800 487437 8.200 1.847336 9.~10") 658110°)
91 39.979 690 213784 1.825 493.JO7 8.300 1.867329 1.~10'2) 728110'°,
92 40331 700 216.417 1.850 499167 8400 1.887298 15/10'2) 108110'"
93 40684 710 219046 1.875 505019 8500 1 .907243 2~10 12) 142110")
94 41034 720 221670 1.900 510861 8.600 1.927166
95 41385 725 222980 1.925 516695 8700 1.947065
96 41735 730 224289 1950 522.520 8800 1966942
97 42084

( Pwf no flow across the outer boundary (at r=r~); and I


(days) (D PD Source (psia) constant BHP Pwf at the single producing well
~ 7.45 X 104 ~ 0.5 (In t -i735 (centered .in the reservoir) with wel.lbore ra~ius r "'.
+ 0.8~) The solution -pressure as a function of time a~d I.
1.0 7.45 x 10S 7.161 Table C.3 2,684 radius ~or f~ed values of r~, r w' and .rock 3:"d fluid ., I
100.0 7.45 x 107 7.601 In r 2,665 pro~rtles.-ls expre~sed most conveniently In terms II
~D of dimensionless variables and parameters: I
Note that for r~D=2,OOO, the reservoir is infinite PD=!«(D,rD,reD)' ",..~
acting at t D = 7.45 X 104. This means that Table C.I
rather than Table C.3 is to be used to determine P D; where
however. f?r this (D;' Eq. C.3, which "extends:' Pi-P , ..
TableC.I,lsappropnate.For(D=7.45xIOS,PDIS PD='- -.
found in Table C.3; for (D=7.45XI07, PD is Pi-P"'f
calculated from Eq. C.6. Note also that
B 0.<XX>264 kt
-q II. 'D= -,__2'
P"'f""Pi -0.00708 khPD ~#J.Clr",
rD=r/r""

(300)(1.25)(0.8)
= 3.CXX>-(0.00708)(80)(i2)PD r~D=r ~/r ",.

For this problem, instantaneous rate q and


=3.CXX>-44.14PD' cumulative production Qp are of more practical
importance than P D' and these quantities can be
Constant Pressure at Inner Boundary derived from the fundamental solutions, PD' In
, Appendix B, we showed that a dimensionless
No Flo'" Across Outer Boundary production rate, q .and dimensionless cumu!ative
This solution of the diffusivity equation models production, QpD' c~n be defined as
radial flow of a slightly compressible liquid in a qB
homogeneous reservoir of uniform thickness; qD = II. .
reservoir at uniform pressure Pi before production; 0.00708 kh (Pi -Pwf) I
I I'

~ www.petroman.ir ,'~)"I
..114 ,'" WELL TESTING 1

,
I

and For reD = ~ and t D ~ 100. q D can be approximated


as3
QpD = I ID q D dt D -( -4,29881 + 2.02566 t D)
0 QpD- lntD .

B Since both qD and QpD are based on solutions


= 111"..- L_2 /- _\Qp' (PD) that, for rD=1 (r=rw)' depend only on tD
, 1.119 (j>cI hr w (Pi -P wf) and r~D' q D and Q~D also should depend only on t D
and reD' Table C.5 confirms this expectation. For
Dlmenslon!ess cumulative production. QQ' IS given reD and tD' QpD is determined uniquely.
pres~nted I.n Tables C:.4 and C.5. Table. C.41s ~or Although the QI!D solutions can be used to model
Infimte-actlng reserVOirs, and Table C.5 IS for fimte individual well problems. they more often are used to I
reservoirs with 1.5 ~ r eD~ I x 1~6. For r ~D ~ 20, model water influx from an aquifer of radius r ~ into :
values for both qD and Q D are given. In Table C.5, a petroleum reservoir of radius r w for a fixed I
f~r values of t D smalle! t~a~ the. small~st value for a reservoir pressure. Pwf' Superposition is used to I
given r~D' the reservoir IS Infimte-actlng and Table model a variable pressure history, as illustrated in :
C.4 should
those in thebetable
used.forFora values of t D oflarger
given value r~D' than
the Example C.4. I !.

reservoir has reached steady state, and Example C.3 -Use of QpD Solutions .
QpD = (~D -1)/2. Problem. An oil well is produced with a constant
'.
TABLE C.5 -Cpo Ys.to -FINITE RADIAL
CONSTANT SYSTEM
PRESSURE WITH BOUNDARY
AT INNER CLOSEDEXTERIORBOUNDARY. I

'.0=1.5 '.0=2.0 '.0=2.5 ~~ '.0=3.5 '.0=4.0


!.fl.-~-.!LE.eJL-.!LE.eJL-.!L~-.!L!}..pJ; !L.9.PJl..
0.05 0.276 0.05 0.278 0.10 0.408 0.30 0.755 1.00 1.571 2.00 2.442
0.06 O.~ 0.075 0.345 0.15 0.509 0.40 0.895 1.20 1.761 2.20 2.598
0.07 0.330 0.10 0.404 0.20 0.599 0.50 1.023 1.40 1.940 2.40 2.748 .
0.080.354 0.125 0.458 0.250.681 0.60 1.143 ~1.60 2.111 2.60 2.893 I
0.09 0.375 0.150 0.507 0.30 0.758 0.70 1.256 1.80 2.273 2.80 3.034 j
0.10 0.395 0.175 0.553 0.35 0.829 0.80 1.363 2.00 2.427 3.00 3.170
0.11 0.414 0.200 0.597 0.40 0.897 0.90 1.465 2.20 2.574 3.25 3.334
0.12 0.431 0.225 0.638 0.45 0.962 1.00 1.563 2.40 2.715 3.50 3.493
0.130.446 0.250 0.678 0.50 1024 1.251.791 2.60 2.849 3.753.645
0.14 0.461 0.275
0.150.4740.300 0.715
0.751 0.55
0.60 1.083
1.140 1.50 1.997
1.752.184 2.80
3.00 2.976
3.098 4.00 3.792
4.253.932 .
0.16 0.486 0.325 0.785 0.65 1.195 2.00 2.353 3.25 3.242 4.50 4.068 I .
0.17 0.497 0.350 0.817 0.70 1.248 2.25 2.507 3.50 3.379 4.75 4.198
0.18 0.507 0.375 0.848 0.75 1.299 2.50 2.646 3.75 3.507 5.00 4.323 I :,
0.19 0.517 0.400 0.877 0.80 1.348 2.75 2.772 4.00 3.628 5.50 4.560 I 1

0.20 0.525 0.425 0.905 0.85 1.395 3.00 2.886 425 3.742 600 4.779
0.21 0.533 0.450 0.932 0.90 1.440 3.25 2.990 4.50 3.850 6.50 4.982 I :
0.22 0.541 0.475 0958 0.95 1.484 3.50 3.08J 4.75 3.951 7.00 5.169 .~
0.23 0.548 0.500 0.983 1.0 1526 3.75 3.170 5.00 4.047 750 5.343 ; !
0.24 0554 0550 1.028 1.1 1605 400 3.2J7 550 4.222 800 5.504 ..' I
0.25 0559 0.600 1.070 1.2 1.679 4.25 3.317 6.00 4.378 8.50 5.653 'I
0.26 0.565 0.650 1.108 1.3 1.747 4.50 3.381 650 4.516 9.00 5.790 ; I
0.28 0.574 0.700 1.143 1.4 1.811 4.75 3.439 7.00 4.639 950 5.917 i
0.300.5820.750 1.174 1.5 1.870 5.00 3.491 7.50 4.74910 6.035 .
0.32 0.588 0.800 1.203 1.6 1.924 5.50 3.581 800 4.846 11 6.246
0.34 0594 0.900 1.253 1.7 1.975 6.00 3.656 850 4.932 12 6.425 I
0.36 0599 1.000 1295 1.8 2.022 6.50 3.717 9.00 5.009 13 6.580
0.38 0.603 1.1
0.400.606 1.2
0.45 0.613 1.3
1.330
1.358
1.382
2.0
2.2
2.4
2.106
2.178
2.241
700
7.50
8.00
3.767 9.50 5.078 14
3.809 10.00 5.138 15
3.843 11 5.241 16
6.712
6.825
6.922
II i
j
0.50 0.617 1.4 1.402 2.6 2.294 9.00 3.89J 12 5.321 17 7.004 i !
0.60 0.621 1.6
0.7006231.7
1.432
1.444
2.8
3.0
2.340
2.380
10.00
11.00
3.928 13
3.95114
5.385 18
5.43520
7.076
7189
I '
i,.I
0.80 0.624 1.8 1.453 3.4 2.444 12.00 3.967 15 5.476 22 7.272
2.0 1.468 3.8 2.491 14.00 3.985 16 5.506 24 7.332
2.5 1.487 4.2 2.525 16.00 3.993 17 5.531 26 7.377
3.0 1.495 4.6 2.551 18.00 3.997 18 5.551 30 7.434
4.0 1.499 5.0 2.570 20.00 3.999 20 5.579 34 7.464
5.0 1.500 6.0 2.599 22.00 3.999 25 5.611 38 7.481
7.0 2.613 24.00 4.000 30 5.621 42 7.490
8.0 2.619 35 5.624 46 7.494
9.0 2.622 40 5.625 50 7.497
10.0 2.624

www.petroman.ir I
EVERDINGEN
AND
HURST
SOLU~10NS
TODIFFUSIVITY
EOUATIONS .-1_- 115

.BHP
initially of at2,<XXJ
2,500 psi a for
The 1,0 hour from a reservoir I,<XXJ = 2,<XXJ.
psia, reservoir is finite; there is 'tD = OS
no flow across the outer boundary. Other rock, fluid, ,
and well properties include the following. There is no entry in Tablr; C.5 at this (D. Thus, the
B = 1.2 RB/STB, reservoir is infinite acting, and from Table C.4,
II. = I cp, QpD =44.3. Then,

'wk =
= 0.294md,
O.Sft, Qp = 1.1 19 (j)cth'w2 (p/-Pw/) QpD/B

h = IS ft,
~ = 0.15, =(I,119)(0.IS)(20XI0-6)(IS)(0.S)2
c t = 20 x 10 -6 psi -I , and
't = 1,<XXJft. .(2,SOO-2,<XXJ)(44.3)/1.2

Calculate the cumulative production, Qp' in barrels. = 0.233 STB.

Solution.
from eitherWe will calculate
Table t D and
C.4 or Table C.S. reD and read Q pD Example C.4 -Analysis of Variable

t D = 9-:~~~ Pressure History With QpD Solution


~IJ.Crr w Problem. A well is completr;d in a reservoir with an
initial pressure of 6,000 psi. The well can be con-
= (0.<XXJ264)(0.294)( I) = 103 sidered centered in the cylindrical reservoir; there is
(0.IS)(I)(20x 10-6)(0.S)~' no flow across the outer boundary, Reservoir, fluid,
and well properties include the following.

TABLE C.5 -CONTINUED

'.0=4.5 '.0=5.0 '.0=6.0 '.0=7.0 '.0=8.0 '.0=9.0 '.0=10.0


~E-ESL~.-9-PJL-.!Q.-~l~lI2-~~~!JL~
2.5 2.835 3.0 3.195 6.0 5.148 9.00 6.861 9 6.861 10 7.C17 15 9.965
3.0 3.196 3.5 3.542 6.5 5.440 9.50 7.127 10 7398 15 9.945 20 12.32
3.5 3.537 4.0 3.875 7.0 5.724 10 7.389 11 7.920 20 12-26 22 13.22
4.0 3.859 4.5 4.193 7.5 6.002 11 7.902 12 8.431 22 13.13 24 14.09
4.5 4.165 5.0 4.499 8.0 6.273 12 8.397 13 8.930 24 13.98 26 14.95
5.0 4.454 5.5 4.792 8.5 6.537 13 8.876 14 9.418 26 1C.79 28 15.78
5.5 4.727 6.0 5.074 9.0 6.795 14 9.341 15 9.895 28 15.59 30 16.59
6.0 4.986 6.5 5.345 9.5 7.047 15 9.791 16 10.361 30 16.35 32 17.38
6.5 5.231 7.0 5.605 10.0 7.293 16 10.23 17 10.82 32 1710 34 18.16
70 5.464 7.5 5.854 10.5 7.533 17 10.65 18 11.26 34 1782 36 18.91
75 5.684 8.0 6.094 11 7.767 18 11.06 19 11.70 36 1852 38 19.65
8.05.892 8.5 6.325 12 8.220 19 11.46 20 12.13 3819.19 40 20.37
8.5 6.089 9.0 6.547 13 8.651 20 11.85 22 12.95 40 19.85 42 21.07
9.0 6.276 9.5 6.760 14 9063 22 12.58 24 13.74 42 20.48 44 21.76
9.5 6.453 10 6.965 15 9.456 24 13.27 26 14.50 44 21.09 46 22.42
10 6.621 11 7.350 16 9.829 26 13.92 28 '15.23 46 2169 48 23.07
11 6930 12 7.706 17 10.19 28 14.53 30 15.92 48 22.26 50 23.71
12 7.208 13 8.035 18 10.53 30 15.11 34 17.22 50 22.82 52 24.33
13 7.457 14 8.339 19 10.85 35 16.39 38 18.41 52 2336 54 2494 ..
14 7.680 15 8.620 20 11.16 40 17.49 40 18.97 54 2359 56 25.53
15 7.880 16 8.879 22 11.74 45 18.43 4520.26 562A39 58 26.11
16 8.060 18 9.338 24 12.26 50 19.24 50 21.A2 58 2488 60 26.67
18 8.365 20 9.731 25 12.50 60 20.51 55 22.46 60 2536 65 28.02
20 8.611 22 10.07 31 13.74 70 21.45 60 23.40 65 26..18 70 29.29
22 8.809 24 10.35 35 14.40 80 22.13 70 24.98 70 2752 75 30.49
24 8.968 26 10.59 39 1493 90 22.63 80 26.26 75 2848 80 31.61
26 9.097 28 10.80 51 1605 100 23.00 90 27.28 80 29.36 85 32.67
28 9.200 30 10.98 60 16.56 120 23.47 100 28.11 85 30.18 90 33.66
30 9.283 34 11.26 70 16.91 140 23.71 120 29.31 90 30.93 95 34.60
34 9.404 38 11.46 80 17.41 160 23.85 140 30.08 95 31.63 100 35.48
38 9.481 42 11.61 90 17.27 180 23.92 160 30.58 100 32.27 120 38.51
42 9.532 46 11.71 100 17.36 200 23.96 180 30.91 120 34.39 140 40.89
46 9.565 50 11.79 110 17.41 500 24.00 200 31.12 140 3592 160 42.75
50 9.586 60 11.91 120 17.45 24031.34 1603704 180 44.21
60 9.612 70 11.96 130 17.46 280 31.43 180 37.85 200 45.36
70 9.621 80 11.98 140 17.48 320 31.47 200 38.« 240 46.95
80 9.623 90 11.99 150 17.49 360 31.49 240 39.17 280 47.94
90 9.624 100 12.00 160 17.49 400 31.50 280 39.56 320 48.54
100 9625 120 12.00 180 17.50 500 31.50 320 39.77 360 48.91
200 17.50 360 39.88 400 49.14
220 17.50 400 39.94 440 49.28
440 3997 480 49.36
480 39.98

'- www.petroman.ir i~\/t


r:.
116 WELL TESTING

k = 31.6 md. from three wells. each beginning to produce when


<> = 0.21. P wj is changed and each producing with pressure
rw = 0.33 ft. drawdown equal to the difference in pressures before
r~ = 3.300 ft. and after the change:
80 = 1.25 RB/STB.
h = 20 ft.
1J.0 = 0.8cp.and Weill producesfor(18-0)=18months
c( = 20xl0-6psi-l. with (Pi-Pwjl)=6.<XX>-5.500=500psi.
Well 2 produces for (18-6) = 12months
The well produ.ced for 6 months with. flowing BHP with (Pwj I -Pwj2) = 5.500 -4.500= 1.<XX>
psi.
Pwj of 5.500 pSI. for 6 more mon~hs WIth Pwf = 4.5~
PSI. and for 6 more months WIth Pwj=5.(XX) pSI. Wel~3producesfor(18-12)=6months .
Calculate cumulative production after 18 months of with (Pwj2 -Pwj3)=4.500-5,<XX> = -500psl.
production from this well. .For . well. reO = 3.300/0.33 = 10.CXX>.
this
Solution.
problem. Superposition is required to solve
We can calculate the cumulative this rD =' 0 CXX>264
., k t
production by adding the cumulative production <>IJ.C
{r ~

TABLEC.5-(CONTINUED)
qo and CpoY5.to -FINITE RADIALSYSTEMWITH CLOSEDEXTERIORBOUNDARY-
CONSTANT PRESSURE AT INNER BOUNDARY

'.0=20 '.0=50 '.0=100


~-

-.!P- -9.P- -EoPJL to ~ -EoPJL to -9.P- Opo


100 0.3394 42.91 600 0.2652 189.0 2.000 0.2304 532
130 0.3174 52.76 800 0.2915 241 3,000 0.2179 757
160 0.2975 61.98 1,000 0.2393 290 4,000 0.2070 969
200 0.2728 73.38 1.300 0.2220 359 5,000 01967 1.171
240 0.2502 83.83 1.600 0.2060 473 6,000 0.1869 1.363
300 0.2197 97.91 2.000 0.1865 502 8,000 0.1686 1.718
400 01770 117.7 2.400 0.1682 573 1x10. 0.1536 2.000
500 0.1426 133.6 3,000 0.1543 667 1.3x10. 0.1304 2.461<103
600 0.1148 146.4 4.000 0.1133 795 1.6x10. 0.1118 2.82~103
700 0.0925 156.7 5,000 0.0833 895 2 x 10. 0.0910 3.23 x 103
800 0.0745 165.1 6.000 0.0682 974 2.4 x 10. 0.0741 3.56 x 103
1.000 0.0483 177.1 8.000 0.0418 1,082 3 x 10. 0.0645 3.94 x 103
1.300 0.0483 187.8 1x10. 0.0254 1.148 4x10. 0.0326 4.37x103
1.600 0.0132 193.4 1.3 x 10. 0.0120 1.201 5 x 10. 0.0195 4.62 x 103
2.000 00056 1969 1.6x10. 0.0056 1.227 6x10. 0.0117 4.77x103
3.000 0.(XX)6 199.2 2 x 10. 00021 1.241 8 x 10. 0.0042 4.92 x 103
2.4 x 10. 0.0006 1.246 1 x 105 0.0015 4.97 x 103
3 X 10. 0.0002 1.249 1.1 x 105 0.0009 4.98 x 103

'~0=200 '.0=500
~- '.0=1.000
~-

to -9.P- Opo to ~ Opo to ~ Opo


1x10. 0.1943 2.19x103 1x105 0.1566 1.75x10. 3x10. 0.1773 5.89x103
1.3x10" 0.1860 277x103 1.3x105 0.1498 2.21x104 4x10. 01729 764x103 .
1.6x10. 01820 3.33x103 1.6x105 0.1435 2.65 x 104 5x104 0.1697 9.35x103
2x10. 0.1742 4.04 x 103 2x105 0.1354 3.21x104 1x105 0.1604 1.76x10.
2.4x10~ 0.1668 4.72x103 2.4x105 0.1277 3.73x10. 2x105 0.1518 3.32 x 10.
3x10. 0.1562 5.69 x 103 3x105 0.1170 4.47x10. 3x105 0.1464 480x10.
4x104 0.1401 7.17x103 4x105 0.1012 5.56 x 104 4x105 0.1416 6.24 x 10.
5 x 10. 0.1236 8.50 x 103 5 X 105 0.0875 6.50 x 104 5 X 105 0.1371 7.64 x 10.
6x10' 0.11269.68x103 6x105 0.07567.31x104 6x105 0.13278.98x10.
8x10. 0.0905 1.17x10. 8x105 00565 8.62 x 10. 7x105 0.1285 1.03x105
1x105 0.07281.33x10. 1x106 0.04229.60x10. 8x105 0.12441.16x105
1.3x105 0.0524 1.52x10. 1.3x106 0.0273 1.06x105 9x105 0.1204 1.23x105
1.6x105 0.0378 1.65x10. 16x106 00176 1.13x105 1x106 0.1166 1.40x105
2x105 0.0244 1.78x10' 2x106 0.0098 1.18x105 1.4x106 0.1024 1.83x105
2.4 x 105 0.0138 1.86 x 10' 2.4 X 106 0.0055 1.21 x 105 2 X 106 00844 2.39x 105
3x105 000821.92x10. 3x10° 0.0023 1.23x105 24x106 0.0741 2.71x105
4x105 0.0028 1.97x10" 4x1OS 0.0005 1.25x105 3x106 0.0610 3.11x105 -"
5x105 0.0009 1.99 x 10' 5x106 0.00011.25x105 4x106 0.04423.63x105
5 x 106 0.0320 4.01 x 105
7 X 106 0.0167 4.48 x 105
8.4 X 106 0.0106 4.67 x 105
1 X 107 0.0063 4.80 x 105
I; 1. For,O smaller than.alues I,SledIn thISlablefora g'ven',0' re_' IS,nl,n'le acllng F,ndOpOin TableC. 1.4 X 107 0.0017 4.95 x 105
2. For,O larger than'aluesllSled In th,slable OpO; V.o,2 -1/2 2 X 107 0.0004 4.99 x 105
3 For,O larger I~n .alues "Sled ,n IhlSlableqo' 00 3 X 107 0.0000 5.00 x 105

, ~. www.petroman.ir
VAN EVERDtNGENAND HURST SOLUTIONSTO DIFFUSIVITYEOUATIONS 117

(0.00)264)(31.6)(730 hr Imonth)(t months) Q D = 1.06 x 107,


= (0.21)(0.8)(2xI0-5)(0.33)~ ~p=(8.19Xl0-6)(-500)(I.06XI07)

= 1.664 X 107 (t months). = -0.434x 105.


Then
ForWelll,tD=(I.664xI07)(18)=3.0xl08. Qp=Qpl +Qp2+Qp3
From Table C.5, = 1.04 x 105 + 1.55 x 105 -0.434 x 105
QpD=2.54XI0', =2.16xl05STB.
Q
p =I.119ct>c{hr; (Pi-Pwf)Q pDIB E .
.xercises
=(I.119)(0.21)(2xl0-5)(20)(0.33)~ C.l An oil well is producing from a reservoir
.(p -P .) Q 11.25 (r e = 900 ft) with no. fluid. ~rossing th: outer boun-
I ..1 pD dary of the reservoir. Initial reservoir pressure is
= 8.19 x 10 -6 (Pi -Pwf) Qpo 3,00) psia. Pressure in the well bore is maintained at
=(8.19x 10-6)(500)(2.54x 10') 2,00) ~sia. Well and reservoir properties include the
following:
= 1.04x 105STB. It> = 0.21,
k = 80md,
ForWeIl2,cO=(I.664x I07)(I2)=2.0x 108. CI = 20xlO-6psi-l,
QpD=1.89xIO', rw=0.45ft,
IJ. = 0.8 cp,
Qp =(8.19x 10-6)(l,oo)(1.89x 107) h = 12 ft, and
= 1.55 x 105 STB. B = 1.2 RB/STB.
, Calculate instantaneous rate and cumulative produc-
For Well 3, CD=(I.664x 10 )(6)= 1.Ox 108. tion after 1.0 day of production.

TABLE C.5 -(CONTINUED)


-'.0 :2.<XX> '.0 :4.<XX> '.0: 1.0(10.)
to --.9£-- Opo to -..:9.P- OpD to -9-9- Opo
1 X 105 0.1604 9 x 105 0.1366 3 x 106 0.1263 4.06x 105
2 X 105 0.1520 1 x 106 0.1356 4 x 106 0.1240 5.31 x 105
3x105 0.1475 1.3x106 0.1333 5x106 0.1222 6.76x1OS
4x105 0.1445 6.27 x 10. 1.6x106 0.1315 2.26 x 105 6x106 0.1210 7.76 x 105
5x105 0.1422 7.70x10. 2x106 0.1296 2.78x105 8x106 0.1188 1.02x106
6x105 0.1404 9.11x10. 2.4x106 0.1280 3.30 x 105 1x107 0.11741.25x106
7x105 0.1389 1.05 x 105 3x106 0.1262 4.06 x 105 1.2x107 0.1162 1.49x106
8x105 0.1375 1.19x105 4x106 0.1237 5.31x105 1.4x107 0.1152 1.72x106
9x105 0.1363 1.33x105 5x106 0.1215 6.54 x 105 1.6x107 0.1143 1.95x106
1x106 0.1352 1.46x105 6x106 0.1194 7.74x105 1.8x107 0.1135 2.17x106
1.3x106 0.1320 1.86x105 8x106 0.11551.01x106 2x107 0.11282.40x106
1.6x106 0.1291 2.25 x 105 1x107 0.1118 1.24x106 2.4x107 0.1115 2.85 x 106
2x1OS 0.1254 2.76x1OS 1.2x107 0.1081 1.46x106 3x107 0.1098 3.51x106
2.4x106 0.1216 3.26 x 105 1.4x107 0.1046 1.67x106 4x107 0.10714.60x10& ..
3x106 0.1166 3.97 x 105 1.6x107 0.1012 1.87x106 5x107 0.1050 5.66 x 106
4x106 0.1084 5.10x105 18x107 0.0979 2.07 x 106 7x10;" 0.0998 7.70x106
5 x 106 0.1008 6.14 x 105 2 X 107 0.0948 2.27 x 106 8 X 107 0.0975 8.69 x 10&
7 x 106 0.0872 8.02 x 105 2.3 X 107 0.0902 2.54 x 106 9 X 107 0.0952 9.65 x 106
1x107 0.07011.04x106 3x107 0.08033.14x106 1x10s 0.0930 1.06x107
1.3x107 0.0563 1.23x106 4x107 0.0681 3.88 x 106 1.2x10s 0.0887 1.24x107
1.7 x 107 0.0421 1.42 x 106 5 X 107 0.0577 4.51 x 106 1.4 X 10s 0.0846 1.41 x 107
2x107 0.0339 1.53x106 7x107 0.0415 5.49 x 106 1.7x10s 0.0788 1.66x107
2.4x107 0.0253 1.65x106 8x107 0.0352 5.87 x 106 2x10s 0.0734 1.89x107
.1%/t'? 1lt'/N 1;7-"%/1/'".9%/pl tltJ8 6,lPXJp6 24x1p8 o.~ 211xl07
4"KI't1'(
G1'tt'5
/4YK~ (KrvrII'
tf'tfS//6# k// /~ dr'
5X107 0.0038 1.95X10: 1.2x10s 0.0181 6.90 x 106 4x10s 0.0458 3.06 x 107
7X10s 0.0009 1.99x10 1.4x10s 0.0130 7.21x106 5x10s 00362347x107
1x10 0.0001 2.00 x 106 1.7x10s 0.0079 7.52 x 106 6x10s 0'0286 3'79x107
1.3x108 O~OO~ 2.00 x 106 2x10s 0.00487.71x106 7x10s 0:02264:04x107
2.3x10s 0.0029 7.82 x 106 8x108 0.0178 4.24x107--
2.6 x 10s 0.0018 7.89 x 106 1 X 109 0.0111 4.53 x 107
3x10s 0.0009 7.94 x 106 1.4x109 0.0043 4.82 x 107
-4 5 X 10s 0.0002 8.00
0.0000 7.99 x 106 32 X 109 0.0011 5.00
0.0001 4.96x 107
107 .'I.S

1 FFOf'Osm~lIerth~nv~lu
2 I 8 Sli Sled Inlh'SI~ble'or~glvenrfO.'eservolrls,n"nlle~ctlngFindO
.Of'O ~rge' than values lIsted In thIs labIa. °pO- (1_0,2_112 pO inT~bleC.
3 For,O larg-r thanvalueslisted in thislable. "0. 00

www.petroman.ir
I
I -
1 18 WELL TESTING

C.2 A single oil well is producing in the center of a d> = 0.2


circular. full water-drive reservoir. The pressure at k = 83 r'nd
the.oil/~ater contact is c.onstant at 3.340 p:sia and the C( = 8 x 10~6 psi-I.
radial distance to the oil/water contact IS 1.500 ft. r = 3(xx) ft (reservoir radius)
The well produced for the first 15 days at a rate of r: = 30.(xx) ft (aquifer radius)'.
500 STB/D. the next 14 days at 300 STB/D. and ~he !l = 0.62 cpo and
last day at 200 STB/D. Calculate the cumulative h = 40 ft.
production and well bore pressure at the end of 30
days. C.4 If. in Exercise C.3. the reservoir boundary
41 = 0.20. pressure suddenly dropped to 2.704 psia at the end of
k = 75 rod. 100 days. calculate the total water influx at 400 days
CI = 17.5xI0-6psi-l. total elapsed time.
r w = 0,5 ft.
!l = 0.75 cp, References
h =
B = 15 ft.RB/STB.
1.2 and I. van E'erdingen. A.F.and Hurst. W.: "The Application of the
Laplace TransformatIon to Flow Problems in Reservoirs,.
C.3 An oil reservoir is initially at a pressure. Pi' of Trans.,AIME (1949)186,305-324. .

2.734 psia. If the boundary pressure is suddenly 2. Chatas. A.T.: "A Practical Treatment of Nonsteady-State
lo".ered 10 2.724 psia and held there. calculate the Flow Problemsin ReservoirSystems," Pel. Eng (Aug. 1953)
I .. fl .. f 100 B-44through B-56.
cumu atl\e water In ux Into the rese!volr a ter .' 3. Edwardson, M.J. el al.: '.Calculation of Formation Tern-
200. 400, and 800 days. ReservOIr and aquifer perature DisturbancesCaused by Mud Circulation," J. Pel.
properties include the following. Tech.(April 1962)416-426; Trans.,AIME. 225.

TABLE C.5 -(CONTINUED)

'eO=2.5(10.) '_0=1.0(105) 'eO=2.5(105) '_0=1.0(10&)


'0 --.9..P.-- ODO '0 -.9JL ODO '0 -.9JL ODO '0 -.9JL ODO
3 X 101 0.1103 1.4 x 108 0.1017 2 x 109 0.0897 2 x 10'0 0.0813
4 x 101 0.1086 2 x 108 0.100) 3 x 109 0.0881 3 x 10'0 0.0800
6 x 101 0.1064 6.77 x 106 2.4 X 108 0.0990 4 x 109 0.0870 4 x 10'0 0.0791
7 x 10:' 0.1054 7.83 x 106 3 X 108 0.0980 3.10 x 107 5 X 109 0.0861 4.51 x 108 6 X 10'0 0.0778
8 x 107 0.1047 8.86 x 106 3.5 X 108 0.0971 3.59 x 107 6 X 109 00854 5.37 x 108 8 X 10'0 0.0770-
9 x 107 0.1041 9.93 x 107 4 X108 0.0966 4.07 x 107 7 X 109 0.0849 6.22 x 108 1 X 10" 0.0763 7.95 x 10'0
1x10B 0.10351.10x107 5x108 0.0956 5.03x107 8x109 0.0844 7.07x108 1.3x1011 0.0756 1.02x10'O
1.4x10G 0.1016 1.51X107 6x108 0.0948 5.98x107 1X10'0 0.0836 8.75x108 1.6x10" 0.0750 1.25x10'O
2x108 0.09932.11x107 7x108 0.0941 6.93x107 1.4x10'0 0.0824 1.21x109 2x10" 0.07431.55x10'O
2.6x10G 00973 2.70x107 8x108 0.0935 7.87x107 2x10'O 0.0809 1.70x109 2.4x10'1 0.0737 1.84x10'O
3 x 108 0.0960 3.09 x 107 8.4 X 108 0.0933 8.24 x 107 3 x 10'0 0.0787 2.49 x 109 3 X 10" 0.0730 2.28 x 10'0
3.3 X 10G 00950 3.37 x 107 9 x 1oB 0.0930 880 x 107 4 X 10'0 0.0766 3.27 x 109 4 X 1011 0.0719 3.21 x 10'0
3.6x10. 0.0940 3.66 x 107 1x109 0.0925 9.73x107 4.4x10'0 0.0757 3.58x109 5x10" 0.0709 3.72x10'O
4 x 100 0.0927 4.03 x 107 1.4 X 109 0.0911 1.34 X 108 4.7 X 1010 0.0751 3.80 x 109 6 X 10" 0.0697 4.42 x 10'0
4.4 x 100 0.0915 440 x 107 2 X 109 0.0896 1.80 x 108 5 x 10'0 0.0745 4.03 x 109 7 X 10" 0.0686 5.11 x 10'0
5x10G 0.0896 4.94x107 3x109 0.0877 2.77x108 5.4X10'0 0.0737 432x109 8x10'1 0.06766.32x10'0
5.4x10c 0.08045.30x107 4x109 0.0861 3.64x108 6x101O 0.0725 4.76x109 1x10,2 0.0656 7.13x10'O
6 x 100 0.0866 5.82 x 107 5 X109 0.0845 4.49 x 108 7 x 10'0 0.0705 5.48 x 109 1.2 X 1012 0.0636 8.42 x 10'0
64x10. 00855 6.17x107 6x109 00829 5.33x108 7.4x10'0 0.0698 5.76x109 1.4x10,2 0.0617 9.67x10'0
7x10. 0.08376.67x107 7x109 0.08146.15x108 8x10'0 0.0686 6.17x109 1.5x10'2 0.0607 1.03x10,2
74 x 100 0.0826 701 x 107 8 X 109 0.0799 6.95 x 108 8.4 x 10'0 00679 6.42 x 109 ..
8x10. 0.08097.50x107 9x109 0.07847.75x108 9x101O 0.0668 6.85x109 >1.5x10,2no1determined.
84 x 10" 00798 782 x 107 1 X 10'0 0.0770 8.52 x 108 1 X 10" 0.0658 7.51 x 109
9x100 0.0782 8.29x107 1.3x10'O 0.0728 1.08x109 1.1x10" 0.0632 8.15x109
1x103 0.0756 9.06x107 1.6x101O 0.0689 1.29x109 1.3x101' 0.0593 9.38x109
1.3x103 00683 1.12x108 2x101O 0.0639 1.56x109 1.6x10" 0.0551 1.11x101O
1.6x10' 0.0616 1.32x108 2.4x10'0 0.0594 1.80x109 2x10" 0.0494 1.32x10'0
2 x 10' 0.0538 1.55 x 108 3 X10'0 0.0531 2.14 x 109 2.4 X 10" 0.0443 1.51 x 10'0
2.4x109 0.0469 1.75x10B 4x10'O 0.04412.62x109 3x10" 0.03761.75x10'O
3 x 109 0.0382 2.00 x 108 5 X 10'0 0.0366 3.03 x 109 4 X 10" 0.0286 2.08 x 10'0
4 X 109 0.0272 2.33 x 108 6 X 10'0 0.0304 3.36 x 109 5 X 10" 0.0217 2.33 x 10'0
5 X 109 0.0193 2.56 x 108 7 X 10'0 0.0253 3.64 x 109 7 X 10" 0.0126 2.67 x 10'0
6 X 109 0.0138 2.72 x 108 8 X 10'0 0.0210 3.87 x 109 1 X 10'2 0.0055 2.92 x 10'0
8x109 0.0070 2.92 x 108 9x10'0 0.0174 4.06x109 1.3x10'2 0.0024 3.04x10'O
1 x 10'0 00035 302 x 108 1 X 10" 0.0145 4.22 x 109 1.6 X10'2 00011 3.09 x 10'0
1.4x10'c
2x10'O 0.0009 3.10x108
0.0001 3.12x108 1.3x10"
1.6x10" 0.0083 4.55x109
0.00484.74x109 2X10'2 0.00043.11X10'O i
3 x 10'0 0.0000 3.12 x 108 2.4x10"
2 X 10" 0.0023
0.0011 4.88 x 109
4.94x109 .
3 x 10" 0.0004 4.98 x 109 0

3.4 x 10" 0.0002 4.99 x 109


4 X 10" 0.0001 5.00 x 109
Noles 1 For,Osmalle'IlIln .alueslisled.nIlloslablelorlo.ven'_0. reservoIr
oS.nfinlleaClono
F.ndOpO.nTlbleC.
2 F"'.Ola,oerlllan,"lueSIISledlnlll'slable.OpO' (/eOI2_, 12
3 Fc':Ola,oe,Illan,"IueslisledInIII.s'able.QO'00 i
I
I I~ I ~

.--~ www.petroman.ir I,J,,6


rr(ji:"~'rt,, t~tirt.at. ~"11Y. ana ,I'
"rv,"..' ,.".1f,;~- " ~ \c1$e i)f 5Ia
",. {~\'~\'edc.
.f '-'-1;:, '-f"" ~...'
~ut!Ofl... GO 1\..
~

..;,' !i.~':) ::11( ~ .".,..-~ ~~-

.-,';,,!Wf.il'.
.1 -EsrimQ/i

Appendix D
Rock and Fluid Property Correlations

Introduction
Pressure transient test analysis requires knowledge of Tec' and pressure. p~, of an undersaturated crude
reservoir fluid properties such as viscosities, com- oIl with gravity of 30 API (specific gravity = 0.876 at
pressibilities, and formation volume factors. In
addition, f formation
I . d f
compressibility
property requent y requIre
is I a. rock
F
or test ana yslS. or
60°F).

-285
...
Sol U Ion. From
.".
F.Ig. 0 - 1, T.or --an1160o R d
most of these properties, laboratory analysis Ppc -psla.
provides the most accurate answer; however, in many
cases, laboratory results are not available. and the Bubble-Point Pressure of Crude Oil
test analyst must use empirical correlations of ex- ..
perimental data. ~ The test anaJ~stmay need to estimate the. sa.turatlon
This appendix provides a summary of correlations o.r bubble-.polnt pressure of a cr.ude oil In some
that have proved useful for test analysis. These clrcum~ta~ces -e.g., to determine whether ,a
correlations are selected from those presented by reservOir IS satu.rat~d or und~rsa~u~ated at a certaJ.n
Earlougher; 1 his collection of correlations is pr~ssure. ~tandlng s corre~atlon IS. useful for thIs
probably the best and the most complete in print at estimate; Fig. O-~ sh,owsthis c~rrelatlon.
the time of this writing. We assumethat the reader of To. use Standing s co~r~latlon, one m,ust k".ow
this text has completed a studv of the fundamentals solution G<?R, gas gravity, stock-tank Ol~ gravity,
of reservo.r fl ' d t .. s h d.. d and reservoir temperature. Example 0.2 Illustrates
Ih UI proper les. ucd stu I.les bprovl f h. I t'
d .1 .' . .l. ef use 0 t IScorre a Ion.
etal on t e meaning, origin. an app Ica I Ity 0
these correlations. Readers not familiar with this
r basis are referred to texts bv Amvx et al.2 and
McCain.3
figures fromIn Earlougher's
this appendix.collection
'we si~plY
andreproduce
illustrate ~ ..
the use of each with an example. i °.
Uu 1300 !:.IITT
Pseudocritical Temperature and E I-Go
1200 : .!: T i

Pressure of Liquid Hydrocarbons ~ i 1100 ~Sl'ClDoc*,rlcAL


0~ Tl'.~f"*ArCl*l'
I. Pseudocritical temperature (T pc) and pressure (ppc) g~ 1000 ,, ' ,
, of undersaturated crude oils can be estimated from III
Ifj III
a: '°0 .,I I .
presented
an approximate
in Fig. correlation
0-1. Thesedeveloped
pseudocritical
by Trube4
properties
and Go ~
Go 100 1 ': ,

are required to estimate compressibility of an un- I- 700 , ~~f"ClDoc*'rICAL


dersaturated crude oil. as illustrated by Example 0.5. :! 100 ,~*l'ssu*l"
To use Trube's correlation, one must know the i: 500' , .I ! .,
~ specific gravity oof the undersaturated reservoir liquid ~ i 400 ' :
:; corrected to 60 F. Example 0.1 illustrates use of the i a.. : :
correlation. U III 300 , I
8~ 200 I
:J
III'(/)U) 0.12 0.11 0.70 Q" 0.71 Qat 0.11
Example D.l- Estimation of : ~ SP£CIFIC GRAVITY
OF~DERSATURATED RESERVOIR
P d .. I T dP GoLIOUIOATRESERVOIRPRESSURE
CORRECTEDTO60°F
:s-eu ocrltlca I emperature an ressure
for Undersaturated Crude Oil Fig. D.1 -Approximate correlationof liquid pseudocritical
Problem: Estimate the pseudocriticaJ temperature. pressureand temperaturewith specific gravity.'
L., 1.
www.petroman.ir
120 WELL TESTING

Example D.2 -Estimation of To determine solution GOR, one must know


Bubble-Point of Crude Oil bubble-point pressure, reservoir temperature, stock-
P bl. , tank oil gravity, and gas gravity. Example D.3
ro e~. Estimate the, bubble-I?olnt pressure of a illustrates use of Standing's5 correlation for
crude 011from a reservoIr producIng at a GOR of 350 t" at solut"on GOR
scf/STB (believed to be solution gas only) with gas es 1m mg I .
gravity 0.75 and oil gravity 30. API from a reservoir
with temperature 200.F.
Example D.3 -Estimation of Solution GOR
Solution. On Fig. D-2, we start on the left by ex- 0 0 .
tending a horizontal line from the assumed GOR of Problem. Estlm,ate ~he solution ~R of a crude 011
350
f
sc
h"
f/STB
0
rom t IS pOInt, we
d
to t
h
e
10
Ine
f
or
. al I.me th at ter-
raw a vertic
a gas gravIty
"
0
f 0 75
.;
from
0
a reservoir
pSla, temperature
0 075
wIth
200 .F
bubble-poInt
01
,01 gravIty
.
pressure
. 30 API ,an d gas 1,930

minates at the line for a stock-tank oil gravity of 30. gravIty 0 .


API; we then draw a horizontal line from that point Solution. In Fig. D-2, we start at the lower right by
that terminates at the intersection with the 200.F extending a vertical line from the bubble-point
reservoir temperature line. Finally, we draw a ver- pressure to the 200.F reservoir-temperature line. We
tical line from this point that intersects the bubble- next draw a horizontal line to the left that terminates
point pressure scale at 1,930 psia. Thus, the at the intersection with the line for tank-oil gravity of
estimated bubble-point or saturation pressure of the 30. API. We then draw a vertical line from this point
crude oil is 1,930 psia. to intersect the 0.75 gas-gravity line. Finally, we draw
a horizontal line from that point to the GOR scale on
S I . GOR the left and read the GOR to be 350 scf/STB.
0 otlon
Frequently. the saturation pressure of a reservoir oil
is known, but solution GOR at saturation pressure. .
(required for some test analyses) is unknown. Fig. D- 011 Formation Volome Factor
2 also can be used for this estimate. Fig. D-3 can be used to estimate the formation

£'--1.£
~£Q"'.£D
,. -' ~ .' ~-..
." -~ Ao , , ,.,.. .'
.110 c... .,.. 'r ., a", oN
.-" *' .--" ., ~'AP,
...,ccou-c
s ,." .' .._f, , ,..
~ -"N'r -., "'. .I.5D
-_.,
u. ..~. ---,oak?
-"-', .f,.0"
...»"API
Ny '- ~.
-*' ""~'r KoN .,.. .w...
-~. , '.
-,... '-'0 P$JA

Copyriv"t /952
C"e.ron Researc" Company
Reprinted br PermissIon

Fig. 0-2 -Bubble-point pressureor dissolvedGORof oilo"

www.petroman.ir
ROCK AND FLUID PROPERTY CORRELATIONS 121

EXA~Pr.(

~£OtJ'~ED
~,..,.."
,. -", "..,.~
,.I".. -.."'.
.' 100." i'
'.'" ." JSOCI"8..~, "'.~" .,. 0-7J,
~ .1.,...", ." JO'AP'
PROCEDURE
S,.".'" .f "" I." ,.-. .' "" ,AG...
-"w """."'."" ..", ...JSO cr..
' ""."" 0.7S ".-- .."
-' ...,-'-'.""
P_..~ """."" .". JO .API
""'. ...,-"k.'
""~'" M. 100'" r".
,-.,.. -"." .."'.. .."",..~ /0..
, ..,'., -, ..',." of"..". ..,

Copyr;9'" 19S2
C"evron Re.earc" Company
Reprinted ~y Perml..ion
I"ORMATION
OO,UME.f
1U8.,
E POINT
"Ou'D-

Fig. D.3-Oil fOfmationvolumefactor.'

10-1 volume factor 80 of saturated crude oil, For this


.estimate, one must kno~. solution GOR, gas gravity,
.'\. "\." stock-tank oil gravity, and reservoir temperature.
Example 0.4 illustrates use of this correlation, which
u~ " ...I '$E'IIDO~E'DUCE'D
-' also was developed by Standing.
>-" " rl'.'E'~ATII~E'.
T,r
'"\ 'l I I
g ~ '~' :. , :; E.\"amp/eD.4-Estimation ofOi/
~ ~"'"-' ;', ~ ~,' : Formarion.Vo/umeFacto: i
~ 10-1 Problem. EstImate the formatIon volume factor of a
~ .salurated crude oil from a reservoir wilh solution ..
0 "~ ' '\.~, I GOR of 350 scf/STB, 2as 2ravitv of 0.75, tank oil
u "... --.,
..gravity of 30' API, and reservoir temperature of ~
0 .., OO'
III !, '" '\.1 1./0 -. F
u
:> ~ ""."" 00
0 , 1 I ...j. '" I".j ~'" Solution. In Fig. 0-3, ~'e start at the upper left and
~ 2-r-./-'! I r '\.."" ' "-J 0'0 e.xtenda horizontal line from a solution GOR of 350
g " i ~1 ~ ~ I ;" ~~, scf/STB to intersect the line for gas gravity of 0,75.
~ 10-S ,..l","" ;80 \\.e then extend a vertical line from this point to
~ .0..70 intersect the 30' API tank-oil-gravity line. From this
.I point of intersection, we dra~ a horizontal line to
--~.:.+~ --~ -0.10 inlersect the line for a reservoir temperature of
.' .o~~ ~OO'F. Finallv, we dra~' a vertical line from that
T ~ -:-- '. ..II '. .04.. point to the formalion-volume-factor scale at the
I 10 102 lower right and read 80 = 1.22 RB/STB.
PSEUOOREOUCEOPRESSURE,Ppr

Fig. D.4-Correlation of pseudoreduced compressibility Com pressibility of UndersaturatedOil


for an undersaturated 011.'
The correlation in Fig. 0-4, developed by Trube,4
can be used to estimate the compressibility, co' of an

L ,www.petroman.ir-.-"
122 WEll TESTING

10
,
,

'='
,
1

,
4. .
In

~
...
~ I
~
U
In 10"
-,
.-' ...6
C~
~ .. .
---2: l&-

10-
I

~~~ .(J

.J
m
5

~ I~.(/) ..,
..m
4
.m
(aRs_' R, ~ ~
I \dP)T: (O.83p + 21.751
3
10') 1.0 1.2 1.4 1.6 1.8 2.0
10 I ., 102 ., 10J o' OIL FORMATION VOLUME FACTOR,
GAS IN SOLUTION, Rs. SCF/STB Bo. RES BBL/STB .j!
Fig. D.S-Change of gas in solution in oil with pressurevs. Fig. D.6-Change. of oil formation volume factor wi,thgas
gas in solution.' In solution vs. 011formationvolumefactor.

undersaturated crude oil. By definition, Co = Cpr/Ppc , so


The basic information required to use this figure is C = 0.001/285 = 3.51 x 10 -6 psi -1.
specific gravity of the undersaturated crude oil, 0
these
reservoir
values,
temperature,
we can and
estimate
reservoir
pseudocritical
pressure. Given
tem- Compressibility of Saturated Crude 011

perature (T pc) and pressure (P~), pseudoreduced The apparent compressibility of saturated crude oil is
temperature (T pr = T/ T pc:) and pressure <fp'r.= significantly higher than that of undersaturated ?il.
plPec) ~nd, thus, ps~udoreduce~. ~ompresslt)lllty The reason is that a pressure drop results!n evol~t~on
(c prJ, which leads to 011compressibility (co) from of gas from the oil; the total volume of oil remaJrnng
the definition actually decreases with pressure decline (although the
C =C I .d:nsity of the remainin~ liquid oil actually decrease.s """,
0 pr Ppc slightly). The net result IS that the total volume of oil ~;,
Example D.5 illustrates this sequence of calculations. and evolved gas becomes greater as pressure drops,
leading to an apparent compressibility of the system
that is appreciably higher than that of liquid oil ..
Example D.5 -Estimarion of alone. In equation form, .'.
Undersaturated Oil Compressibility C = -..!.- ~ +!!L ~ ", (D.l)
Problem. Estimate the compressibility, co' of an 0 Bo dp Bo dp
undersaturated crude oil with 30° API gravity (0.876 The first term accounts for the volume change in the
specific gravity) at a re~ervoir temperature of 200°F liquid caused by (I) vaporization of some. of the
and pressure of 5,000 psla. liquid and (2) increase in density of the remaJnder of
Solution. In Example D.I, we found that the the liquid. The. de~ivative .dBo /dp is a positive
pseudocritical temperature of a 30. API oil was number, so vaporization domm~tes. The second term
T pt. = 1180°R and that the pseudocritical pressure accounts for the volume o~cupled by gas evolved.as
was p = 275 psia. Thus, pressure decreases ~or dlssolve~ as .p~essure I~-
Tpr=TITpc=(200+460)/I,I60=0.569,
pc creases).
term is positive.
The derivative
Further,
dR sldp
its numerical
lS positIve,value
so this
is

and greater than that of the term -(I/Bo) (dBoldp).


Ramey6 proposed correlations that lead to an
Ppr =Plppc = 5,000/285 ='7.5. estimate of Co for a saturated oil; these correlations 'I
From Fig. D-4, then, are given in Figs. D-5 and D-6. To use these i
correlations, we must note that Eq. D.I can be
cpr = 0.001. written as

.IL
www.petroman.ir
r
ROCK AND FLUID PROPERTY CORRELA TIONS 123

c = -~ ~ + ~ ~ '0.;
0 Bo dp Bo dp :

= ~ ~(Bg-~). (D.2}
Bo dp dRs ,

Fig. 0-5 provides an estimate of dRsldp; Fig. 0-6


estimates dBoldRs'
To estimate co' one needs values of reservoir
pressure, p, solution GOR, R s (which, in turn, can be ~
estimated from Fig. 0-2), tank-oil specific gravity i I
("Y0) and gas gravity ('Yg >, oil formation volume = '0
factor, Bo (which can be estimated from Fig. 0-3), ~
and gas formation volume factor. Bg. (Bg can be !
calculated from reservoir temperature, pressure, and ~
gas gravity, which leads to the real-gas-law deviation ~
factor z: Bg = 0.00504 Tzlp RB/scf.) Example 0.6 ~
illustrates use of these correlations. ~ '0

ExampleD.6 -Estimation of Saturated


.-
"
~

Oil Compressibility
Problem. Pressure in an oil reservoir has dropped
below the initial bubble point to 2,500 psia. Reservoir ,
fluid characteristics include the following:
"Yo = 0.825 (400 API),
"Yg = 0.7,
T = 200°F = 66OoR,and
z = 0.851.
Estimate
current saturation
the apparent
pressure
compressibility
of 2,500 psia.
of this oil at its 000.
_h. -aT 00".-a_~ --

Solut~on. To evaluate co' we must determine each Fig. 0.7 -Dead oil viscosity at reservoir temperatureand
term In Eq. 0.2. atmosphericpressure.'

C0 = B~ ~d p ( B g -~ dR ).
a s
From knowledge of p, 'Yg' 'Yo' and T, we can
estimate Rs from Fig. 0-2, as in Example 0.3; the dB t:;:-
result is R s = 640 scf/STB. From knowledge of Rs' ~..J..:!.!L X 104 = 5.6.
'Yg' 'Yo' and T, we can estimate Bo from Fig. 0-3, as dRs 'Yg
in Example 0.4; the result is Bo = 1.36 RB/STB.
We can calculate Bg since T, p, and z are known: Thus,
dB I
Bg =0.00504 Tzlp --2- =5.6x 10-4,\-2.:L
dRs io
= (0.00504)(660)(0.851)/2,500

=0.001132RB/scf. =(5.6xI0-4>..j-
rOT
0.825
From the inset in Fig. 0-5, = 0.516 x 10 -3 RB/scf.
dRs
dP = (0.83pRs
+ 21.75) We no\\' can calculate co:

I dRs! dBo'
\ C0 ---I
- IB e' --I
- 640
--a
B dp -dR
s
J ~ .~ ,

-[(0.83)(2,500) + 21.75]
I
= 0.3052 scf/STB-psi. = (~ )<0.3052)(0.001132-0.0005 16)
(This result also could be read from the curves
plotted in Fig. 0-5.) =0.138 x 10-3 psi-I
From Fig. 0-6,

www.petroman.ir
124 WELL TESTING

100
.
.
.E.AM~LE:
,
PWoeLEM: FINO THE GAS.SATURATEO VISCOSITY OF
A C-uOE OIL HAVING A SOLUTI~ GAS/OIL _ATIO OF 600

.Cu FT / BeL AND DEAD OIL VISCOSITy OF ,~O C~, All

.AT THE SAME TEM~ERATURE.

~ROCEOURE: lOCATE I.~O C~ ON THE DEAD OIL VISCOSITY


I
SCALE (ABSCISSA! AND GO UP VE_TICAllY TO THE 500

GAS/OIL ~ATIO liNE. THEN GO lEFT HO~IZONTAllY TO

;:; .READ THE ANS.E~, 0.~8 CP, ON THE GAS-SATURATED

~ OIL VISCOSITY SCALE 10~0INATEI


III
III
OJ
~
a. a.
V 10

j~
0'4 ..
0~ '
..4
~ III .
..Z 0
.
~ 4
1
III" I
4 ~
0 ;)

..4
..
0 ~ .
,.. a.
..
.."
~ 2
0"
..-
v~
>
> .
0 10

...
~

III
..,
~
..4

J:. .

01
...,.. ...,.. .14"'"
OS 14 10 ,~

VISCOSITY OF DEAD OIL, CP

(AT ~ES:~VOIR TEMPERATURE AND ATMOSPHERIC PRESSVRE)

Fig. D-S -Viscosity of gas.saturated crude oil at reservoir temperature and pressure. Dead oil
viscosity from laboratory data or from Fig. 0-7.1

Oil Viscosity is 350 scf/STB, oil gravity is 30. API, and reservoir

Viscosity of saturated crude oil can be estimated temperature is 200.F.


Chew Beal's
from and Connally's
correlation dataS-
';' (Fig, (Fig.
D- 7) D-8).
combined
Viscosity
with ~olutlon. From FIg. J?-7, the dead o~1 VIS~OSlty, #l.od'

estimates for undersaturated oil require [he same IS 2.15 cp'. From FIg. D-8, the VIS~OSlt~ of gas-
figures plus an additional correlation of Beal's, Fig. s~turated .011 (#l.ob) at th~ bubble pol.nt IS. 1.0 cpo
D-9. Fig. D- 7 provides an estimate of dead (gas-free) Finally, using da:a from Fig. D-9, the VISCOSIty?: the
loil viscosity, #l.od; Fig. D-8 provides an estimate of undersaturated 011 (#1.0) at a pressure of 5,000 pSI IS
gas-saturated oil viscosity, #l.ob' at saturation ,5000-1 930
p!essu.re;. and Fig. D-9 provide.s an estimate of #1.0= 1+ (0.067)(' ,) = 1.21 cpo
VISCOSIty Increase above bubble-point pressure. 1,000
These viscosity estimates require knowledge of
reservoir temperature, oil gravity, solution GaR, ...
and, in the case of an undersaturated oil, bubble- Solubility of Gas In Water
point and reservoir pressures. Example D.7 Solubility of natural gas in water can be estimated
illustrates this estimation procedure. from correlations of Dodson and Standing, 9 --~-

presented in Figs. D-IO and D-II. Fig. D-IO gives the


solubility of natural gas in pure (nonsaline) water;
Example D. 7 -Estimation of Oil VIscosity Fig. D-II provides a means of correcting solubility in
Problem. Estimate the viscosity of an undersaturated pure water for brine salinity.
oil at a reservoir pressure of 5,000 psia and at the To estimate solubility of gas in water, one must
oil's saturation pressure of 1,930 psia. Solution GaR know reservoir temperature and pressure and total

www.petroman.ir
~OCK ANDFLUID PROPERTYCORRELATIONS
'" -- !if, :: 125

.J
m
m
'- 20
~
10.
:)
u
! 16
.a:
iii .
Q: 2 Q
:) ~
V! ~
V! 102 a: 12

'&:' .a:
0. ~
~
Z ~
0 1 '" 8
0. ~
III 10 ~
-I Z
IIUj.
m 0
-
:)0. ~ 4
II .:)
0 -I
J: O 2 0
0- 111
a:'
II. 0. I 0
U
III .260
c
I/) .TEMPERA~E. -F
III
~ 2 Fig. 0.10-SOlubility of naturalgas in purewater.1

Ja:
F4~~~~~~~
)0 10-1
... CORRECTIONFOil BRINE SALINITY
ii .III
I V!
0
>u .ZIIIIO
1 Q a:
-l- ~
m 2-.-ccr ~

~
,
10
~Z
~-- Z
a: J J
0.9
~
10-1 2 ..I 1 ..10 1 ..101 1 ~ a: ; 08 100'¥"
\lISCOSITY OF GAS-SATURATED CRUDE 0 10 20 30 40
AT BuBBLE POINT PRESSURE, CP TOTAL SOLIDS IN BRINE. ppm .IO-~

Fig. 0-9 -Rate of increase in oil viscosity above bubble- Fig. 0-11-Correction of natural gas solubility for
point pressure-1 dissolvedsolids,' !

solids
this content of
estimation the water. Example 0.8 illustrates
procedure. 0-11, to
used This estimatethe
estimate requires the same
solubility information
of gas in water ':
(reservoir temperature and pressure and total solids .
E I D 8_£ ' ..r co~tent. of the water), Example 0,9 illustrates the J
xamp e ,'.
Gas Solubility In Water
.stlmatlon OJ estimatIon procedure. ~'"\
I
'ii,;
Ii",
Problem. Estimate the solubility of natural gas in a .i
formation temperature
reservoir "-ater with 20,fXXJppm dissolved
of 200°F, solids,
and reservoir ..1
E.x-ampleD.9-Estlmatlon of Water '

pressure of 5,<XX>
psia. Formation Volume FaClor
Solution. From \Fig. 0-10, the solubilitv (R ,) of Problem. Estimate the formation volume factor of
natural gas in pure water at 200°F and 5,OOOs~iais brine with 20,~ ppm dissolved solids. at a tem-
20.2 scf/STB. From Fig. 0-11, the correction factor perature of 200 F and pressure of 5,fXXJpsla.
(Rs.._IRs..p) for 20,OOO-ppmsalinity at 200°F is 0.92. Solution. From Fig. 0-12, the formation volume
Thus, factor of pure water is 1.021 RB/STB; for gas-
R s..- = (R s..p ) (R sWI R sw ) saturated pure. water, it is.1 .030 RB:STB. Br!ne with
~ -., p 20,fXXJ ppm dIssolved solids contains less dIssolved
-(.0.2)(0.92) gas than pure water; from Fig. 0-11, the ratio of
" = 18.6 scf/STB. this
solubility
value in
to brine
interpolate
to that between
in pure water
volumeis 0.92.
factors
Using
for

pure and gas-saturated water, then,


Water Formation Volume Factor Bw = 1.021 + (1.030 -1.021)(0.92)
The formation volume factor Bw can be estimated by
using Fig. 0-12, an additional correlation developed = 1.029 RB/STB.
by Dodson and Standing,9 along with Figs. 0-10 and

I
.II
r -\1 www.petroman.ir
126 ..WELL
TESTIN
. ~;;"
[;
-:.. 40
C/I

.-
Q.

Q
.3
loe J
u

>-
~ ~'" ...
C/I -~ = 3.2
" 1.05 ~
~ Vi
g:.
.-NRC WATCR " ~~ :'+ ' C/I
~

.~ ---WATER PLUS NATURAL"" Q. 28


~ ..-,~ ~
1.04 0
.I.)

~ g:
"-
I.) '"
4 ~ ~ 24
I.; 1.03 .~ 60 100 140 180 220 260
~ -~ !.J.; .TEMPER.ATURE. 8F
..J
O ~- -,-- .~ FIg. D.13-Compressibility of gas-freewater.'
> ---,- ++J

~
Z
0
-'-
~~
1.02
._-,--,.
--T
-1$0
~ --I-
~ ~

L.o..'
~
>- 1.3
...>-
..J-
0
g: 1.01
-, "
~ m..J
-M
C/I'"
Ii.~:';:
g:.~
'"
~ 1.00
,.
-,.- ,...
~

iiij
;~
i+1-j
lOoO- ~
..::J
Qa:::J
4Q.
g: ~~
Q.a:
u 0
Vi 12
I ;zc
"O,
"~".,
""

~ ~'- I.) 1.1 ,.c!'


::;-:~ -" ~ a: :;';,:'
~ ;.; , ~ E ~ '" !,"',
0" --'-~ --1 ~ Hf:!:J~ ~i ~i.:.
.0 1000 2000 ~ ~ ~ ~ 1.00 5 10 15 20 \c'
PRESSURE. p, PSIA GAS-wATER RATIO, CU FT leeL .
.,-
Fig. 0-12-Formation volume factor for pure water, gas- Fig. D.14-Effect of dissolved gas on water com-
free andgas-saturated.' pressibility.'

t
Compressibility of Water S,(XX) psia. Oil in the reservoir has a saturation
in Undersaturated Reservoirs pressure of 1,930 psia.
The formation water that occurs in an un- Solution. We found in Example D.8 that solubility of
dersaturated oil reservoir will not release gas as gas in water at the stated conditions is 18.6 scf/STB.
pre~sure is d~creased; this wo.uld l.eadto formati.on of From Fig. D-13. the compressibility (cwp) of pure
or Increase In a gas saturation In the reservoir. In (gas-free) water is 2.96 x 10 -6 psi -1 .At a gas/water
such a case. we ~'ill assume that the formation water ratio of 18.6 scf/STB. the correction factor for gas in
is saturated with gas at reservoir pressure. One solution is 1.16(Fig. 0-14). Thus.
implication of this is thai, as gas is released from ..
solution in the water, it is assumed 10 be redissolved c w = C"'p (c ,,/c "p) "
in the undersaturated oil. =(2.96xIO-6)(1.16)
For. this assumed system behavior. Dodson and -4 -6.-1
Standlng's9 correlations (given in Figs. D-13 and D- -3. 3 x 10 pSI.
14) can be used to estimate the compressibility of
water in an undersaturated oil reservoir.
These water-compressibility estimates require Compressibility of Water
knowledge of reservoir temperature and pressure and in a Saturated Reservoir
formation--:vater salinity. E.xample 0.10 illustrates In a saturated reservoir, gas released from solution in
the calculatIon procedure. the formation water either ~'ill begin to form or will
increase a gas saturation as reservoir pressure is
Example D. la-Estimation of Water ~owered; as Ramey6 pointed out, thi~ ~~amaticall.y ~-
CompressibllilY i a Und I I d Increases the apparent ~a~e.rc?mpresslblllty. In thIs
R .n n ersa ura e case. the water compressIbilIty IScalculated from '~
eSerVOlr .Ii
Pro blem. Estimate
.I the compressibility of a forma- c w = ---+ dB". --a-
B. dRSIII (D.3) .,I
tion water containing 20,(XX}ppm dissolved solids in Bw dp Bill dp
a reservoir with temperature 200.F and pressure The term -(I/B".) (dBw/dp) is still determined :1
"

1
L
www.petroman.ir
i
ROCKANDFLUIDPROPERTY
CORRELATIONS "' I 127

using Fig. 0-13 (for gas-free water) and Fig. 0-14 (to o.
correct for the effect of gas in solution). Ramey's
correlation,6 presented in Fig. 0-15, is used to -=
estimate dRsw/dp for fresh water, and Fig. D-II is r Q
used to correct for the effect of salinity on dRsw/dp. ~
f This compressibility estimate requires knowledge !
of formation-water salinity, reservoir temperature ~ QOO4
and pressure, and formation volume factor of the gas
dissolved in the water. Example 0.11 illustrates this -:t
estimation procedure. n
~ ~Q. QOO2
~
Example D.ll- Estimation of Water
Compressibility in a Saturated Reservoir 00 1000 ~ ~ ~ ~
Problem. Estimate the apparent compressibility of PRESSuRE,PSIA
for.ma~ion water containing. 30,(XX) ppm dissolved Fig. D.15-Change of naturalgas In solution in formation
solIds In an undersaturated 011reservoIr at 200.F and water with pressurevs. pressure.'

21

20
r~'I..'ro ~ roooo
I 'r.' ,.- I
."..110. '... ~...
'IO..I".!... !...
Go I. 1'1..000. 10... !...
U
.'" 17
: I
w
~ '
.,
J
.,
~ -
W 1

Go
U 14
i
W
i: 13 I
.,
i
~ IZ 'or~~..orcooorc'lo.'.c'oo III
'oo..'ro .S '."'
0Z 1.1 .ors...ro ."",c r '0 .a,.rs .", .
C .0' co."o.ro rl.tO,.t.'.".,
~W 10 ..'cos". ." r..r..'to "t,s"ar
~J "P.' .,., - f..'

~ 0'
w
Go
2
w --
~ 0&
~
g 07
~
If)
IaI oa
/ w,sco'", I,..) .' I.,. '.rs",.r .t..o. I't".
., s."'O."O. ..rs'...( 0' ..'r. ..o.r lit"
~
~
c
>- 0$
~
Vi
0 04
(,)
If)
>: 03

02

r 01
00
.0 ac ac tOO 120 140 lac lac 200 220 240

TE~PERATuAE, -,.
fig. D-16-Water viscosity atvarioussalinities and temperatures.'

www.petroman.ir
"I
128 WELL TESTING

! 700 Example D. 12-Estimation of


oJ~ .w'Sl'CLLANCO(l. W'ater Viscosity
c ..:so G'ASCS
~ ~ 6~0 .C.t'$A'l"'1; Problem. Estimate the viscosity of water containing
~ Q. It'tVID,f 20.00) ppm (2070)dissolved solids at 200"F and 5,<xx>
u ..
0111 psla.
0«600
~~ Solution. From Fig. 0-16. the viscosity IJ.r" of water
~~ with 211ioNaCI at 200" F and atmospheric pressure is
~ ~50 0.32 cpo The correction factor f for 5,<XX>-psia
pressure is 1.016 (inset at upper right of figure).
Thus.
~oo
« /l... =/lr"f=(0.32)(1.016) =0.33 cp,
0
oJu.
C 4~0
~ ~Q. Pseudocritical Properties of Gas
~.
i ~ Pseudocriticaltemperature, Tpc:' and pressure, Ppc.
g ~ 400 are useful quantities that allow application of
g: generalized correlations of gas properties needed in
~!' 3~0 applications. The most accurate values of these
Go~ quantities are calculated from compositions of
~ -natural gas mixtures.2.3 Approximate values can be
300 determined from a correlation developed by Brown
0.6 0.1 OJ 0.9 1.0 1.1 1.2 et 01.II This correlation, presented in Fig. 0-17, is
GASGRAVITY,Yv' (AIR 81) based on gas gravity, 'Yg; values of critical properties
Fig. D.17-Correlation of pseudocritical properties of depend on whether the gas is from a gas condensate
condensate well fluids and miscellaneous reservoir (the "condensate well tlwds" curve) or
naturalgaseswith fluIdgravity.' from a relatively dry gas reservoir (the
"miscellaneous gases" curve). Example 0.13
illustrates use of this correlation.

2,5~ psia.. Formation volume factor of the 0.7. Example D. J3 -Estimation of


gravltygaslsO.001132RB/scf. P d ,. IG P t.
-:seu ocntlca as roper les
Solution. From Fig. 0-15,. for fresh water, Problem. Estimate Tpc and Ppc for a dry gas of
dRs...ldp = 0,0033. The correction factor for the gravitv(f )07
effect of salinity (Fig. 0-11) is 0.875; thus, for brine, .g .,

dRs",ldp = (0.0033)(0.875) =0.0029 scf/STB-psi. Solution. From Fig. 0-17, Tpc=390oR and
From Fig. 0-12, using procedures outlined in p pc = 665 psia.
Example 0.9, B", = 1.033RB/STB. From Figs. 0-13
andO-14,usingproceduresofExampleO.l0, G L D t ..
as- aw evla Ion Fac t or ( 1.-Factor)
-~ ~ =(3.13x 10-6)(1.11) and Gas Formation Volume Factor
B... dp Application of the real gas law,

=3.47xl0-6psi-l. pV=znRT, (0.4)


Th to relate pressure, volume, and temperature for gases ..
us, requires values of the deviation factor z. One ap-
c = -~ ~ + ~ ~ plication of particular importance is in calculating
'" B... dp B... dp gas formation volume factor, Bg, from

Bg = 0.00504 TZlp (RB/scf). (0.5)


= 3 47 x 10 -6 + (0.001132)(0.0029) Fig. 0-18, developed by Standing and Katz, 12 can
.(1.033) be used to estimate z. One must know pseudoreduced
pressure, Ppr' and pseudoreduced temperature, Tpr'
= 6.65 x 10 -6 psi -I. to determine z. By definition,

Ppr=plppc (0.6)
Water0-16,
Viscosity and.
Fig. first presented in the Ilteraiur-eby~Mii:-~- T -TIT ~,~ (0 7) ~ --~J

thews and Russell,lO allo\\s estimates of water pr -PC" : i


viscosity as a function of reservoir temperature, In these definitions, pressures must be expressed in
salinity, and reservoir pressure. Example 0.12 psia (psig + 14.7) and temperatures in degrees I
illustrates use of this correlation. Rankine (8F + 460). I
To use this correlation, one must know reservoir I

www.petroman.ir
ROCK
AND
FLUID
PROPERTY
CORRELATIONS 129

0 I & S 4 5 7
1.1 1./
~£IIDO-~£D(JC"D
r£MP£~A~£,
r, .
".0
10
6..
6" .0
i4
6.6

0..

0..
, ~ .,J

NO.7 ,..

~
~
U I."
.1: 0.. ,.-
Z
0
~
4
~ I.'
0
1/1 0.5 1.4
4
~
oJ
4
..,
~ o. l.a .
,./
..
OS 1.2

I I 1.1

4-
1.0 {'II 1.0

I. I

1.06

0.' 0.-
..10 II 1& 13 14 I.
PSEUDO-REDUCED
PRESSURE,
Ppr
Fig. 0-18 -Gas-law deviation factor for natural gases as a function of pseudoreduced pressure and temperature.' I
!

I I.
www.petroman.ir
,
.i
130 WELL TESTING I:":
IfIit'

I ..7
Q
,~
,;'
U "
...k >-
...
,.-
Q. .J
u ? m '
>-" gj
~
~
-1&1
~ .
! s Q.
~ ~
~ 0
I&J U
f
~ 0
'
0 1&1
U U
5 :)
0 0
1&1 1&1
U ~
:) 0 I
0 0
1&1 :)
I&J
~ I cn
0 Q.
0
:)
I&J
~ .II
Q. "
10-1
I
I 10"
3
., .7 .. 10 15
PSEUOOREOUCEO PRESSURE. Ppr PSEUOOREOUCEO PRESSURE. Ppr
Fig. D.19A -Correlation of pseudoreduced compressibility Fig. 0-198 -Correlation of pseudoreduced compressibility
for natural gases.1 for natural gases. 1
GASGMVITY. Y,. IAIRa'.OOOI-
o. 10 ,.. 1.0 ...,» a..
00'.
OOt. ' ...1
Q
~II , CO.
§~
C..
a.
00'
u 0.005
!.;
"-
u>
..
. 0..
..." 10
& .S"
:t-
o QO' 00 .1 00 I ~
~ .""co,
"'ax . 0011 ..
OJ
~
.. C 00'0
101
Z
0
..000.
C
..
..
..
-000.
0 -
..> 000"
U
QOO
0 10 so .0 .0 .0 00 80 80 -
MOLECULAR WEIGHT. M
Fig. 0-20 -Viscosity of natural gases at 1 atm.'
.1
---~
www.petroman.ir
~
ROCKANDFLUIDPROPERTYCORRELATIONS 131

temperature and pressure and pseudocritical tem- '0


perature and pressure (from either composition or
gas gravity). Example D.14 illustrates use of this
correlation.
"

ExampleD. 14-Estimation of Gas-Law


Deviation Factor and Gas Formation
Volume Factor "

Problem. Estimate.: for O.7-gravity gas in a reservoir


with temperature 200°F and pressure 2.500 psia. Use
this value of z to determine the gas formation volume.
factor Bg.
Solution. In Example D.13, we found that ~
T pc = 390"R andppc =665 psia. Then. ~"
Tpr=TITpc=(200+460)/390=1.69. g4 I'
and ~ j
Ppr =plppc =2.500/665=3.76. ~ "0
>

From Fig. 0-18, ,=0.851. Then,


Bg =0.00504 T,/p 00
= (0.00504)(660)(0.851 )/2,500
= 0.001132 RB/scf.
00

Gas Compressibility
Figs. D-19A and O-I9B (developed by Trubel3) lead
to estimates of gas compressibility, Cg' In these 10
figures (which cover different ranges of the in-
dependent variables), pseudoreduced com-
pressibility, cr. is plotted as a function of
pseudoreduced pressure, Ppr' with the parameter '0, '0 i
pseudo reduced temperature,
compress!b!l!ty.is Tpr'
defined as cpr Pseudo
=C"Ppc; reduced
thus, gas ~S~UOO"~DUC~D
T~..PE"ATU"~.
T.. iI

compressIbIlity ISfound from the relatIon Fig. D.21A-Effect of temperature and pressure on gas
-viscosity_'
Cg-Cprlppc' (D.8)
Use of Figs. D-19A and D-19B requires kno\\'ledge
of reservoir temperature and pressure and reservoir conditions. From knowledge of reservoir
pseudocritical temperature and pressure of the gas temperature and gas gravity (or its equivalent,
(from either composition or gravity). Example 0.15 molecular \\-eight). we can estimate the viscosity of a
illustrates use of this correlation. hydrocarbon gas, /lgi' at atmospheric pressure...
Insets in Fi2. 0-20 allo\\' corrections to this viscosity
..for nonhyd;ocarbon components of the gas. Figs. D-
ExampleD.15-Estlmatlon of Gas 21A and D-218 (two different \\'ays of plotting the
Compressibility same data) permit calculation of gas viscosity at
Problem. Estimate the compressibility of a 0.7- reservoir temperature and pressure, given viscosity at
gravity gas at a reservoir temperature of 200"F and atmospheric pressure. pseudoreduced temperature,
pressure of 2,500 psia. Tpr' and pseudoreduced pressure. Ppr' Example
S I .
0 utlon.
I
n
E
xamp e
I D 14
.,
f
\\'e oun
d h
t at
0.16 illustrates application of these figures.
T pr = 1,69, Ppr = 3.76, and Ppc = 665 psia for these
conditions. From Fig. D-19A, cpr =0.26. Thus, Example D.16-Estimation of Gas
Cg =cprlppc =0.26/665 Viscosit.v
= 0 00039 .-1 Probiem. Estimate the viscosity of a -o.7..gravity -
.pSI. hydrocarbongas(noHzS,N2,orCOz)at200°Fand
2,500 psia.
Gas Viscosity Solution. From Fig. D-20, the viscosity It 1 of 0.7-
Figs. D-20, D-21A, and 0-21 B (from the work of gravity gas [molecular weight = 0.7 x 28.~ = 20.3
Carr (Or01.14)can be used to estimate gas viscosity at Ibm/(lbm-mole») at 200°F and atmospheric pressure

www.petroman.ir
-~

,
132 WELL TESTING

e.

'.0

0
~
~
"-
~ 4.0
~
0 .i.-
~
~
~
)..
I- --.---
U)
0
(.)
II) 3.0
>

2.0

, i

1.0
I J 4 '8'1' I 14
1.0

PSEUOOREOUCEO
10

PRESSURE, Ppr
"po"

,1
!

Fig. 0.21 B -Effect of pressure and temperature on gas viscOSity.' ...

is 0.01225 cpo In Example D.14, we found that. at Formation compressibility is a complex function "
these conditions. T pr = 1.69 and Ppr = 3.76. Thus, of rock type, porosity. pore pressure, overburden ,!
from Fig. D-2IA or 0-218, J.lg/J.lgl= 1.45. At 200°F pressure, and, in general, the stresses in different
and 2,500 psia, gas viscosity is directions in the formation. No reliable correlation
-( / of this quantity with the controlling variables has
ILg-J.lg J.lgi)J.lgl been presented in the literature; indeed, laboratory
= (0.01225)( 1.45) determinations of cf are difficult, and many reported
= 0.0178 cp values of this quantity are doubtless erroneous
.because conditions in the field were not duplicated in
the laboratory. A much-used correlation, developed
.by Hall. IS is presented in Fi2. 0-22. This correlation
Formallon Compressibility relates Cf to a single variabl~-porosity. As reported -~~ "--
Formation compressibility, Cf' is defined as by Earlougher, I this correlation is known to be
I 0V incorrect by an order of magnitude or more in
C f = --( .:..:..2.) , (D.9) specific situa(ions. Thus, \\'hile (he correlation is easy
V p op T to use, the result may be seriously in error for IftJ~ .
where V p = pore volume of porous medium. given applica(ion.

~.- -, I"j

www.petroman.ir
ROCK
AND
FLU::::~:RELATIO:..~I.I- -:-
Use of [his correlation is illustrated in Example
0.17. The result may be of no greater accuracy than .T
simply assuming Cj:4x 10-6 psi-l,sinceonlyone .~~~l ~
of the many variables affecting C f has been taken into f -; .i ~
., I~ I.-i .:
account. vI """.
.,
~. T
I \.~\l.~ .:I
ExampleD,17-Estimationoj
-'=i I
-i ;'~;
'1:; e ,i ,1

Formation Compressibility lit I i ~...::..;: I =1 ~

Problem, Estimate the formation compressibility Cf .f .! ~-!- o_..;-~ li--


for a reservoir with 20070 porosity. ~, J : !,i.i

Solution, From Fig, D-22,cf=3,6XIO-6 psi-[. ~i 1 ..~.::

~.-Iu . .'-"-
Exercises.
Results of pressure transient test analysis sometimes 00 1 .I I 0 II
'--1
10 II I. .
are combined with rock and fluid properties to 'O-Ollff-'tIU.f
calculate the following quantities: Fig. D-22 -Formation compressibility. 15

Total reservoir flow rate,


qRt =qo.l!o +qwB... + (qg -Rsqo/l,(xx»Bg. Water salinity = 27,500 ppm,
Total mobIlity, Sw = 0.25,
At=kol#l.o+kwl#l.w+kgl#l.g, S = 005
Total compressibility, S~ = 0:70: and
Ct =coSo +cwSw +CgSg +cf' It> = O.IS.
The following exercises require calculation of q Rt'
~t' and Ct for two cases. References
0.1. Calculate qRt' At, and Ct for an under- ...
saturated""bil reservoir with the following properties. [. Earlougher, R:C. Jr.: Adl.'ances In "'~I Test AnalYSIS,

I qo
q w
=
=
100 STB/D,
20 STB/D,
Monograph Senes, SPE, Dallas ([977) 5.
2. Arnyx, J.W., Bass, D.M. Jr., and Whiting, R.L.: Petrolt'Um
Reservoir Engineering: Ph_vsical Proper/ies, McGraw-Hili
Book Co., Inc., Nev.. York City (1960). .
..3. McCain, W.D. Jr.: 77Ie Propernes of Petroleum Fluids,
, qg = qoRs (reservoIr produces dIssolved gas Petroleum Publishing Co., Tulsa (1973).
only), 4. Trube, A.S.: "Compressibility of Undersaturated
Reservoir pressure = 4,00) psia, Hydrocarbon Reservoir Fluids," Trans., AIME (1957) 210,
Reservoir [em perature = 220.F, 341-3~.. .
R = 400 scf/STB 5. Standing, M.B.: Volumetric and Ph~e BehaVIor of Oil Field
s 0 7 ' Hydrocarbon Systems, Reinhold Publishing Corp., New York
i' g = ., City (1952).
i'n = 0.S5, 6. Ramey, H.J. Jr.: "Rapid ~Iethods for Estimating Reservoir
W 1.. 25 rv,,-, (2 5 U7 N Cl) Compressibilities," J. Pet. Tech. (April 1964) 447-454;
ater sa tnlty = ,vvv ppm ..0 a , Trans., AIME. 231.
ko = 20 md, 7. Seal, C.: "The Viscosity of Air, Water, ~alural Gas, Crude ..
k... = 0.93 md, Oil and It~, Associated Gases at Oil-Field Temperatures and
k = 0 (no free-gas sa[uration), Pressur~s, Trans., AI~IE (1946) 16~: 94-~IS. ..
g -0 IS 8. Chew, J. and Connall)., C.A. Jr.: A VIscosity Correlation
4> -., for Gas-Saturaled Crude Oils," Trans., AIME (1959) 216, 23-
So = 0.65, 25.
S..' = 0.35, and 9. Dodson, C.R. and Standing, M.B.: "Pressure-Volume-
S = O. Temperature and Solubility Relations for Natural-Gas-Water
g Mixtures," Drill. and Prod. Prac., API (1944) 173-179.
0.2. Calculate q Rt' At, and Ct for a saturated oil 10. Matthews, .C.S., and Russell, D.G..: Pressure Buildup and
reservoir with the following properties: Flow Te~ts In Jfells, Monograph Senes, SPE, DaJlas (1967) 1,
Appendix G.
q = lOOSTB/D, II. Brown. G.G., Karz, D.L., Oberfell, G.G., and Alden, R.C.:
I q 0 = 5 STB/D .Vatural Gasoline and the Volalile Hydrocarbons. NaturaJ
"., Gasoline Assn. of America, Tulsa (1948).
qg = 250 Mscf/D, 12. Standing. M.B. and Katz, D.L.: "Density of Natural Gases."
Oil gravity = 3S' APi, Trans., AI ME (1942) 146,140-149.
Gas gravity = O.S, 13. Trube, A.S.: "Compressibility of Natural Gases," Trans.,
Reservoirpressure
Reservo.r temperature = 2,OO)psia,
= 200.F 14. AIME(1957)210,355~357.
Carr, N.L., Kobayashi, R., and Burrows, D.B.: ViSCOSityof

k = 1100 md' Hydrocarbon Gases Under Pressure," Trans., AIME (1954)


0 .181, 264-272.
k... = 3.3 md, IS. Hall, H.N.: "Compressibility of Reservoir Rocks," Trans.,
kR = 7.25 md, AIME(1953) 191, ~.311.

www.petroman.ir
:J ;} j;!:$tr\ioii i~
I.;~"{, ..'"
t"" ""~
l;..'co
~"~I,"",,
""t.."" """

Appendix E
A General Theory of Well Testing

The purpose of this appendix is to summarize an or


a.pproach to well test analysis that is not limited 0 00708 kh (p -p )
eIther to wells in infinite-acting reservoirs or to wells , ~~~.. \Yi ~f'wf~ =
centered in cylindrical reservoirs. This theory is q8jJ.
stated more clearly in terms of dimensionless
variables, which we have largely avoided in the text 21
because they are abstract and thus are less easily ( :-4 + In r De -0.75) + s
understood than other approaches. For some ap- rDe
plications. though, use of dimensionless variables is -+ I >0 25 2 (E 2)
essential-and
We have seen general
that fortheory
I,S 948
is such
q,JJ.C
an
(r e2
area.
/ k (infinite- -PD s'D .rDe' .

acting reservoir) and for constant-rate flow for a well Even more generally. we can wnte
centered in a cylindrical reservoir. 0.00708 kh (Pi -Pw~ --
2 8 -PD(ID)+S. (E.3)-
P-I -P wf= -70 .kh
6~ kl ) -2s ].Thus.
[ln ( !.688q,JJ.C(r~ givenq ajJ.value ID' there exists a rule for.

(I II) determining PD for a well centered in a cylindrical


and for I> 948 q,p.c I r e2 I k (pseudosteady state), reservoir-either expression
general Eq. E.I
includes or Eq.
transientE.2. Thus,
flow this
and

q8jJ. 0.<XX>527 kl pseudosteady-state flow as special cases.


Pi -Pwf= 141.2-[ .; -2 -The value of this generalization becomes clearer if
kh q,p.c(r e now we reconsider a subject we introduced in Chap.
, r 3 -n changes in rate in the producing history of a
+In(-!..-)-0.75+s].
r w -The (1.12 modified) well(Fig.E-I).
total pressure drawdown at the well. expressed
We can generalize by writing these equations in terms in terms of dimensionless variables. is found by .
of dimensionless variables: superposition. as we showed in Chap. 3 for the.. ,
0.00708 kh (p -P ) I 0.<XX>264 kl special caseof transient flow. Here. note that rate ql
_-'::--=-=-:':T' ~ = -(In .; -2 acts for time. I; (q2 -ql) for time (I-II)' ...; and
q jJ. 2 q,jJ.c(rw (qn-qn-l) for time (I-In-I)' Thus. we write, in

+ 0.809) + s. general.
or 0.00708kh[Pi-Pw (I D»)
0.00708kh(p,-p
""iYwf' ) I
=-(lnID+0.809)+s ~-'~~~"~i-f'Wf~=(ql-O)[PD(ID-O)
jJ. I:
q8jJ. 2' I

+s)+(q2 -ql)[PD(ID-IDI)+S)+ '"


=PD+S,ID,S0.25rDe2. (E.I)
+ (qn
kh -qn-1 )(PD (ID -ID.n-1 )+sJ,
and~708kh q8jJ.
(Pi-~ = [ (r
-Zelr w) 2 or.morecompact~.
~00708 [Pi ~Pwf(~Q!.! = n

8jJ. [ j=1L' ~j j'.


( 0.000264 kl ) ( ---+s,
re ) 3 ] ( )] +qns, .
.~:;:..- -2- +In PD ID-IDj (E.4)
q,JJ.C(rW r... 4

-~_o- www.petroman.ir :I'!


~

A GENERAL THEORY OF WELL TESTING


135

where
Aqj =q; -qj-l (and qo -0),
Q.n,.t
tLX)-0. I -
Eq. E.4 is general- i.e., it applies to a reservoir in Q. ~
which, for some values of t D -t Dj' P D can be the
pseudosteady-state solution, and, for other values of
t D -t Dj' P D can be th~ transient solutio~.. 3 4
As an example, consider a pressure buildup test In n-1 n
a cylindrical reservoir. Let q I = q; q2 = 0;
tDI =tpD+AlD;andtm-tDI =AlD.Then, t
0.00708 kh (Pi -Pws) Fig. E.1- n rate changes in well's producinghistory.
BII. =qPD (tpD +AlD)

-qPD(.1tD)' I
-2 [In (tD +AlD) +0.809] +PD(tD+AlD)'
If and only if flow is transient for total time
tpD+AlD,then , , (E.6)
0.00708 kh (Pi -Pws) -~ [1 Now, for some small values of 61/D:S61/Ds'
BII. -2q n (tpD+AlD) In(tD+~tD) =In(tD) and PD(tD+~tD) =
PD(tD)'
+ 0.8(1)] -~ q(ln I1tD + 0.809) For olt D:S AIDs' then,
2 "
or 0.00708kh (Pi-Pws) = I
-In ( t+~
-+PD(tD) ) I1

qBIL 2 AI
p;-pws=70.6-ln qBII. ( .pU'
t D+~D
-u ) I
kh AID -2 [In (2.246 tD)]

.~ -162.6k;;-log
-qBII. (~AI ) .2 = ~ In (~AI ) +constant.. (E.7)

In fact, the arguments leading to Eq. E.4 un- The implication of Eq. E.7 is simply that for suf-
derstate its generality. For constant-rate flow in ficiently small AI, a plot of P ws vs. In [( t + ~) / ~J or
cylindrical reservoirs, PD can be calculated for all t D log [( t + ~) / ~J will be linear and will have a slope,
from simple equations -but the method is not m, related to permeability. This linear relationship .
restricted to cylindrical reservoirs. It applies to any exists, of course, only for sufficiently small values of i
drainage configuration for which P D can be 11t. Once we have established the linear relationship,
calculated as a function of t D (using finite-difference tho~gh, we can extrapolate it to larger times. In
simulation or any other convenient means). particular, we ca4ne~traP.olatePws to (t+~) / ~ = I.
We now examine a useful method 1-3 for deter- Matthews et al. did this and chose to call the ex-
mining P D as a function of t D for more general trapolated pressure p'. Eq. E.7 shows that
reservoir shapes; this method uses the Matthews- 0.00708 kh I "
Brons-Hazebroek functions4 developed for use in B (Pi -p') = PD (t D) --In (2.246 tD)' ..i I
determining average reservoir pressure. We start by q JJ. 2
no
0 f ting thlasthfor
genera a pressure buildup test in a reservoir
ape, ..., , , , (E. 8)

A material balance shows that


( .- ) c, V p (Pi -jJ) =5.615 qBt/24
000708
.P, kh Pws
B=PD(tD+~D)- --
q IL -c,Ah<l>(p;-p), (E.9) I
P D (~D ), , (E.5) where A is the area drained by a well (square feet), j
(Eq. E.9 is valid regardless of drainage-area shape.)
For ~ID sufficiently small (e.g., fJD :s0.25 r[)e2 in a .No.\\" define tDA as 0.~264 kt/<I>ILC,A. Sub-
cylindrical reservoir), flow will be transient regard- stltutlng for I from Eq. E.9 gives
less
= 1/2(lnAlD+O,809).lfwenowaddandsubtract
of drainage-area configuration, and PD ~AlD) tDA= 0000264
'= kt (0 .(p;-p)
000264) (kh) (24)

1/2 In (t D +.1l D) on the right side of Eq. E.5 written <l>1LC,A (5.6 I 5)(qBIL)
at these sufficiently small values of ~ D' the result is
0.00708 kh (Pi -Pws) = ! In (~) -0.00708 kh (Pi -p)
qBIL 2 ~ -2rqBIL '
I
I
I
www.petroman.ir
:r!l~
136
," ",;;r:'{;
~~
..
' :.
:~;' WELL TESTING

or been used to determine the values in Table 1.1 in the


-column "Use Infinite System Solution With Less
2 r t DA _0.00708kh(pj-p)
-qBp. (EIO) thanlOJoErrorfortDA<,"thusprovidinguswitha
means of determining the upper limit of applicability

of the relationshipPD(tD)=1/2 (In tD+O.809) in


Then, using Eqs. E.8 and E.ID, reservoirs of general drainage-area configuration.

-I 0.00708 kh (p -p') + 0.00708 kh


B (p.I -p) = forNow we turn ourP D
determining attention to developing
(t D) after a method
pseudosteady:state
qBp. q II. conditions have been established. Brons and Millers
I pointed out that, at pseudosteady state,
-PD(tD)+-ln(2.246tD)+2rtDA'
2 PDMBH (tDA) = I n (C AIDA )
Simplifying,
=lnCA+lntDA' (E.14)
(p' -p)B
70 = 47ft DA + In (2.246 t D) -2 P D (t D) where C A is a sh~pe factor whose value ~epends o,n
.6 q p. the specific draInage area configuration. ThIs

equation implies a linear relationship between


=PDMBH(tDA)' (E.II) PDMBH and In tDA' with intercept In CA varying
from case to case. The \-1BH charts (Fig. 2.12) show
Note that the left side of Eq. E.II is the ordinate of that this linear relationship does exist and that the
the MBH plots (thus, we give it the name PDMBH)' intercept does depend on the specific drainage-area
Note also that tDA =0.000264 ktltPlJ.C(A is the ab- configuration. Further, the time at which PDMBH .
scissa of the MBH plots. becomes a linear function of In t DA establishes the
Eq. E.II allows us to use the MBH charts to beginning of pseudosteady-state flow.
determine P D (t D) for the drainage-area con- The task at hand is to show that Eq. E. J4leads to a
figurations considered in constructing the charts. To method for determining PD(ID) for general
understand why this is so, note that Eq. E.II can be drainage-area shape for pseudosteady-state flow. If
put in the form we substitute the value of P~BH (IDA) from Eq.
I E.14 into Eq. E.12, the result is
PD(ID) = 2 'l't DA + -In (2.246ID) _
2 P D (I D) -rl2 DA + 1/21n.(2 246 t D ) -
I
--PDMBH(IDA)' (E 12) -1/2In(C A tDA)
2

For time, ID' sufficiently small that no boundary =2rIDA +1/2In(~!.Q)


effects have appeared (transient flow), C A IDA
I or
PD (I D) = 2 In (2.246ID)' ID <I DB' 2.246 A
PD (tD) =2rIDA + 1/21n (-;:;:-::-r). (E. IS)
ForID<IDB' then, CArw

P D (I D) -I
-21n (2.246 I D) Thus,beweestablished
can have established a rule drainage
for general by which shape
PD (I D)
in
pseudosteady-state flow, Table 1.1 gives the times ..
=2 'KtDA+2 ~ I n (2.-ID
"~6 ) ("Exact for IDA <" and "Less Than.JOio Error for
tDA<")atwhichEq.E.15canbeapplled..
Finally, there is the problem of how to establish
PD (I D) for general drainage-area co~fi.8uration
I when there is a gap between the upper limIt of ap-
-2PDMBH (IDA)' plicability of the transient solution and the lower
or limit of Eq.
solution. applicability
E.12, whichofapplies
the pseudosteady-state
at all limes, can be
P DMBH (IDA) = 4rt DA used to fill this gap:
PD (t D) =2'Kt DA + 1/21n (2.2461 D)
(p' -p)
=,lD<IDB' (E.J3) -1/2PDMBH ( IDA' )
70.6 qBp. d h . h .
To avoid both tDA an ID on t e rig t Sldeo f the
~q. E.13 s~ows that I." tr~nslent flo~ (mfintte- same working equation, we can rewrite this result as
acting !ese~vo~r), P DMBH ISa Imear functIon of IDA' 2
Thus, In princIple, a plot of PD\IBH vs. IDA could be PD (I D) =2rlDA + 1/21n (2.246 IDAA/r. )
used to determine the time IDA up to which a -1/2p (I)... (£.16) -~,
reservoir of general shape is infinite acting (by ob- DMBH DA .
serving the time at which a de\iation from linearity Thus, values of PDMBH could be read from their
occurs). In principle, then, this technique could have charts at a desired value of IDA' and PD (/p> could ,

"
I'
www.petroman.ir .!..A:
r AGENERALTHEORYOF WELLTESTING 137

be calculated for use in subsequent reservoir analysis. References


~lougherprovides.vaJuesofPD(tD) ~orgeneraJ I. Ramey. H.J. Jr. and Cobb. W.M.: "A General Pressure
draInage-area shapes In Ref. 6 AppendIx C. For Buildup Theorv for a Well in I Oosed DrainageArea," J. Prt.
applications. the reader should find the desired PD Tech. (Dec. 1971)1493-1505;Trans.. AIME, 251.
values in that reference. 2. Cobb. W.M. and Dowdle. W.L.: "A Simple Method for
In summary. this appendix has shown that well test Determining Well Pressurein Oosed RectangularReservoirs."
aJ . h . ,.. d
.an yS!S t~ rnques ar~ not Iml~e t? we ~ center
II ed J. Prt. Tech.(Nov. 1973)1305-1~.
3. Dake, L.P.: Fundamentals af ResenlOirEngineering. Elsevier
In cylIndrIcal reservoIrs or to InfinIte-actIng reser- Scientific Publishing Co., Amsterdam(1978).
voirs. One can derive or find in the literature 4. Matthews, C.S., Brons, F., andHazebroek.P.: "A Method for
dimensionless-pressure values for general drainage- Determination of Average Pressurein a BoundedReservoir,"
area shapes; one can combine these values (using Trans.. AIME (1?5.J)201,187:191: .
.. k . b .5.
s~perposltlOn) to ta e I.nto account any ar Itr~y rate
Brons, F. and MIller, W.C.: A Simple Method for CorrectlDl
Spot PressureReadings," Trans.,AIME (1961)222,803-805.
hIstory before and durIng the test. Thus. the title of 6. Earlougher. R.C. Jr.: Adl.oonm in Well Test Analysis,
L the appendix: A GeneraJ Theory of Well Testing. Monograph Series,SPE, Dallas(1977)s.

www.petroman.ir
Appendix F
Use of Sf Units
in Well-Testing Equations

This Appendix summarizes the changes required to A more complete table of conversion factors. em-
solve the equations stated in the text by using In- phasizing application to well-testing problems. is
ternational 5ystem (51) metric units. To show the given by Earlougher.1
necessary changes and to allo~' the interested reader Table F-2 summarizes oilfield (customary) and
to apply the 51 unit system to typical well-testing preferred 51 units (practical) for single variables and
problems. this Appendix has four major parts: (I) groups of variables of major importance in well test
conversion factors from "oilfield units" to 51 units analysis.
are tabulated in Table F-I for the units used in the
text; (2) a summary of preferred 51 units for major
variables is given in Table F-2; (3) major equations in
the text are restated in Table F-3. with constants
given in both <:,ilfield and 51 unit.s: an~ (4) ans~ers to TABLE F-2 -CUSTOMARY A~D \IETRIC UNITS .,

all examples In the text are given In 51 units. In fOR ~IAJOR VARIABLES IN EQUATIONS
addition. in the Nomenclature. preferred 51 units are
given (in parentheses) for each quantity used in the CuSI(\mary Practical
text. tJnil Metric Unit
Compre~sibiliIY.(i ~i ; kPa I
TABLEF-I -CONVERSION FACTORS DensilY.p Ibm,cuft kg/m'
Gas no~ rare.q. Mscf D m'!d
Gasvi!ico~iIY.,.. ,,-p "Pa.'
To Convert Liquid no~ raft. q.. and q.. BID m'/d
~~ ~ ~ulliJ1lyBY tnver!il:_- Liquidvisco,iIY." cp mPa.s
acre!i m~ 4.04; E + 03 2.471 E -04 Permeability. k md md
bbl m' 1.590 E-OI 6.21XJ Pre!i\ure.p ~i kPa
cp mPa.s 1.0 1.0 Pseudopres~ure;~(p) ~i~;...p MPa~/Pa.s -
cp "Pa.!i 1.0 E+03 1.0 E-03 Radius.r fl m
cu fl m' 2.832 E -02 3.532 E +01 Slope. III ~i/,,~...le kPa/cyde
ft m 3.048 E-OI 3.281 Temperalure. T oR K
md m.m~ 9.869 E-OI 1.013 Thickness.h ft m .
psi kPa 6.895 I..SO E-OI Time. ( hr b ~~
oR K 5.555 E-OI 1.80 Volume. V bbl m'
5Qfl m~ 9.24X>E-02 1.076 E+OI Wellboresforageconslanl. C, bbl/psi m'!kPa

www.petroman.ir
USEOFSIUNITSINWELL-TESTINGEaUA::~~ 139-

TABLE F-3 -~AJOR EQUATIONS WITH CONSTANT VALUES IN CUSTOMARY AND SI ~ETRIC

Numerical Value of Constants


Equalion in Order of Appearance
Number ("I' "2' "3 .-.)
in TexI Equalion Customary 51
.,
a. P I ap ~IJ.C' ap
1.1 -,+--=-- 0.<XX>264 3.557xI0-6
or. r ar "Ik at
qBIL [ 2to 3 3
1.6 Pwf=Pi-"I- -:-y+lnrrO-- 141.2 1.866xI0
kh rrO 4
~ , .,
e-ao'oJ.(a r > '
+ 2 '""' I n rO I
i.J22 2 \
n=1 an fJI (anrrO> -JI(an>])
.,
1.7 P=P+"
I
I -£1
qBIL
kh
{ -"2~1L"lr
kt
) 70.6 9.33x 102

948 7.036 X 104

1.9 ~s="I~(~-I)ln(2.) 141.2 1.866xI03


kh ks r ".

1.11 Pi -P..f= -"I -In 1


qB1L ( ""OIL",r;,
..-ls ) r 706..X 933 102
kh kt 1.688 1.253x 105

I 12 P ",=P,-cl- c2kt_2 +In


qB1L1 .L- ( ---141.2
rr ) 3 1 1.866x 103
.WJ I kh ~"" r r .4
r- It'.. 0.<XX>527 7.1 X 10-6

1.13 ~=_C-J!!!! -0.234 4.168xI0-2


at CI VP

1.16 P-Pwf=c.-ln qBIL / ( ---+s


rr ) 3 I 141.2 1.866x 103
kh r.., 4

qB1L c2 kt
f =c l -.,+In
1 ( rr ) 3 I 3
1.17 P-P
'" kh ~ r: r---+s 4 I 141.2 1.866xI0
1J.C"t' w 0.<XX>527 7.lxIO-6

1.19 Js q = kJh 141.2 1.866x 103


P-P"f I(
"I BIL In ---
rr ) 31
r... 4 ..

1.20 P-P..rj=".- qB1L1 1-In ( 10.06 A


,... _2)-
3
-+5 I 141.2 1.866x 103
kh 2 C.4 rw 4

1.21 J= ".kh 0.00708 5.356xI0-4


II
BIL -In(
10.06 A
-, ) --+
3
s I
2 CAr... 4
1.26 CS="I~!£ 25.65 101.98
p g

1.29 Po=c,kh(Pi-PW> 0.00708 5.356x!0-4


qjBp.
qB p.t '/:
1.46 pj-Pwf=cl -(-) 4.064 6.236
hLf kq,c, :::;],.
kt '/j 4 -~;.,."
-,.. .

1.47 r;=() 948 7.036x10


c I ~IJ.C', ,'~-~ 1:

r -i:.-.
if!1- -".u .- www.petroman.ir
'!'"'l_::~ :i~'F~
140 WEll TESTING

TABLE F-3 -MAJOR EQUATIONS WITH CONSTANT VALUES IN CUSTOMARY AND SI METRIC(Conld.)

NumericalValue of Constants
Equation in Orderof Appearance
Number (CJ'C2'C3 ...)
in Text Equation Customary 51-

1.48 Is=CltPlJi:,r;/k 948 7.036x 104

1.52 lp =cINp/qlasl 24 24

2.1 P"'S=Pi-C/~log
qBIJ. [ (lp+;1/)/;1/ ] 162.6 2.149x103

2.2 m=c.- qBIJ. 162.6 2.149x103


kh
2 I (Plhr-P"f) ( k
.4 s=I.151 In -Iog.~. )+cll +3.23 5.10

2.7 PD= clkh(Pws-Pwj) 0.00708 5.356x 10-4


qBIJ.

2.8 ID= ~ 0.000264 3.557x 10-6


tPlJ.'.,rIt.

2.9 CsD= ~ 0.894 0.159


« , h It'
2.10 .1/t'=~/(1 +A//lp) --

2.11 C = qB
--24 AI 24
s ci ~
2.121D=50CsDeo.14S --

CseO.14S 2.247x 106


2.13 I '"b s =c J (kh/lJ.) 170,000

2.14 r",o=r",e-s --

2.15
2.16 Lj=2r",o
(~)s=0.869m(s) --

2.17 E= ~ = P-P"f- (~)s --


Jideal P-P",j
2.18 'E=P.-P",j-(~)s
.
P -Pwl
--

2.19 s= hI-sd+sp --
hp

2.20 sp= ( !!!. -1)[ In( !!i-Jiji- ) -21 --


hp rw kv
qB IJ. 1/2
2.21 mL =C, -(-) 4.064 6.236
hLf kcPc, -;c-.:

2.22 10g(LI) = ~ 1( PWj-Plhr ) +IOg.-!!.-


2 m
-CI
cPlJi:
I
I 2.63 4.497

www.petroman.ir
-
USE OF SI UNITS IN WELL. TESTING EOUA TIONS 141

TABLE F.3 -MAJOR EQUATIONS WITH CONSTANT VALUES IN CUSTOMARY AND SI ,\fETRIC(Contd,)

Numerical Value of Constants


Equation in Order of Appearance
Number (CI'C2'C3.")
in Text -Equation ~~iomar; -J s;t-
--~ --
2.24
.
10g(jJ-pws) =Iog (cl --.,
QBIJ. ) c2k~ 118.6 227.37
kh 4>IJ.C
fr*e 0.00168 -0.8385

2.25 Pi -Pws = -ci QBIJ. [


-In / c2<PIJ.C
I r~ / -2.s } 70.6 9.33 x 102
kh k(/p +..\1)

-c, (-q)
BIJ. /
kh
( c2<PIJ.C
In --~~
~
) -25 1 1.688 1.253 x 105

-ci ~ Eil ~3<P1J.C~


kh k(lp +~) 1 3.792 2.814x 10S

-c, (-q)
BIJ.
-Ei ( -C3<PIJ.CfL2 )
kh k~1

2.28 ~"ws=-CI~
kh { -Ei(~~<PIJ.CI!:!.
k..1/ )/ 70.6 9.33xI02
3.792 2.814x 105
2.29 L=J~l~§~ 0.(XX)148 1.944 x 10-6
<PIJ.C
I

2.30 VR= ~4Np)(Bo~ --


(P2-P,)Cf<P
2.31 Pw/=Pi+C. ~~/IOg(~)_~+DqR2./ -162.6 2.149
kh kIp 1.151 1.688 11.638

23"
.-Pw/
2 = .2 ~lqglJ.j~i!:
P, + kh / 1og ( ~kl ) _15+Dqf2.
1.151 / I ..637 I 508
p 1.688 11.638

2.33 P..'s=P'-c, qB,IJ.'


'g-,f/l"! /
log ( :£-.:...:::.
I +..1/ /
I kh ~I
) 162.6 2.149

2.3~
.
s =s+Dq,~ (PI hr -P..,,)
= /.151 / --og 1 ( k 2 ) +CI / 3..23 2 10

.
In (/)lJ.iC fir..,

2.35
., ., q IJ.z,T
P"i.'s=Pj-c.,gl"l""log (::£-.:-.=:.
t +~I
kh 011 ) 1.637 1.508

(2 -2 k
2.36 S'=S+Dqg=I.151
1 ~~~-IOg(:,,--1)+CI1
m <PlJ.jclirw 3.23 2.10

2.37 Pw/=Pj +CI qRf


-log / ( C2r1>cl~
---16 ) 5 / 26..149x 2 10 3
AI h AI I 1.151 1.688 1.253x105

2.38 P...s =Pi -ci ~qR ( I +AI )


log::£-.:-.=:. 162.6 2.149x 103
Alh ~ -

2.39 qRI=qoBo+ (qg-- qoRs )Bg+q..,Rw --


I .(XX)

k k k
2.40 AI = -£ + ~ + ::l --
1J.0 IJ.w IJ.g

www.petroman.ir
~

142 WELL TESTING

TABLE F-3 -MAJOR EQUATIONS WITH CONSTANT VALUES IN CUSTOMARY AND 51 METRIC(Contd.)

Numerical Value of Constants


Equation in Order of Appearance
Number (CI,C2,C3"')_~
in Text Equation Customary SI
-.
2.41 At =cl ~ 162.6 2.149x 103
mh

2.42 ko=cl ~~ 162.6 2.149x 103


mh
( qoRs
qg --BgJlg
C'I
)
2.43 kg =c, .162.6 2.149
mh
I ,<XX> I.<XXJ
2.44 kw =Ct qwB",Jlw ]62.6 2.149x 103
mh

2.45 s= 1.151 ! PI hr -Pwf -IOg(~)+CI


I 3.23 5.10
m t/>Ctr-w

3.1
/ ( c20jJ.Cr2.
P"j=Pi+CI"kh qBJl log ---k~)-0.869s I 162.6 2.149x 103

1,688 1.253x 105


3.2 1 wbs= (Ct +c2s)C s 200,<XXJ 2.644 X 106
kh/Jl
I 2 ,<XX> ] .586 x ]05
3.4 k=cl qBJl
-162.6 2.149x 103
mh

3.5 s=].1511(Pi-Plhr)-IOg(~)+CI 1 3.23 5.10


m ct>Jlctr;

3.7 11"=~-~1~~Q:::! 3,800 2.82x]05

3.8 Pi-pwr=CIJJ.j!
q kh l log(~~~d
kl
) +0.869sl J 162.6 2.149x]03

1,688 1.253x 105

3.s=].151.'
9 /( Pi-P"'I"
-"J ) I
-;--Iog(~ ,k ) +cl I 3.23 5.10"
q Ihrm 4>Jlctr,o;,

3.11 .Pi-P".r=c.!!!.; t (qj-qj-I)log(I-lj-t) 162.6 2.149x103


qn kh \. j= I qn

+IIOg(~)-c2+0.869s1)
I ct>jJ.C,
"'. ) 3.23 5.10

JlB n j\
3.12 Pi-P,,'s=cl -E (qj-qj-I)log(/-lj-l) 162.6 2.149x103
kh )=1
3.15 Pi -P":s =cl- q2BIJ. j
-log
ql ( /1+12+AI
.pt ' 'p~ ' -.)+Iog ( 1'1+.11
'po: ' -) / 162.6 2.149x 103
kh q2 1p2 + At At

3.18 Pwf=Pi-CI~llog(~)-C2+0.869sl 162.6 2.149x103


kh ct>1J.',rw

-CI~
kh llog(!P.~ At' ) +~IOg(At')
ql
1 3.23 5.10

www.petroman.ir
-,
USE OF SI UNITS IN WELL.TESTINGEOUATIONS 143

TABLEF.3 -MAJOR EQUATIONS WITH CONSTANT VALUES IN CUSTOMARY AND 51 METRIC(Conld.)

Numerical Value of Constants


Equation in Order of Appearance
Number
. (CI'C2'C3 ...)
-In Tex! -Equation -5;:ustomary -Sl-

3.19 S=I.151/ QI(~~)-log(~)+CI


(ql -Q2) m ~JlC(r; J 3.23 5.10

3.20 Pi =PIII./1+m/IOg( ~ )-CI +0.869s} 3.23 5.10


~JlClr III

b' k
3.21 S=I.151
/ m'
--10g( -"-
~JlC(r; )+CI } 3.23 5.10

4.1 Cs=cl Aillb


-25.65 101.98
p

4.2 Cs=clllbVlllb --

4.3 CsD =cl Cs/lPc(hr; 0.894 0.159

4.4 k=cl ~( PO) 141.2 1.866x 103


h Pi -Pili! MP

4.5 clk
cPcl=-:;2 ( -0.<XX>264
t ) 3..557X 10-6
.1£'11110 MP

4.6 tD= clkt _2 0.<XX>264 -3.557xIO-J


~/liCgir III

4.7 1,!-D=,!!,Tscr1,!-(Pi)-1,!-(P~ 50.300 3.733


clPscQg T

4.8 s' =S+Dlqgl --

Jp P
4.9 1,!-(p)=2 dp --
/l(p)~(p)
p,
4.10 Po = _kh(Pi -Pw/)- 141.2 1.866
ci Qg/liBgi ..

4.11 Bgi =cl ~iT


-5.04 0.351
Pi

4.12 k=cl ~~( PO) 141.2 1.866


h Pi-Pw! MP

4.13 lPc(j-clk
---r
( -0.<XX>264
t 3.557 x 10 ~
/lir... 10
)MP -J
I

kh(,p, ~ -2)
4.14 Po= -P..of} 1.422 1.309
cIQg/li~iT

4.15 k=cl Q /liZiT


..g.-,"" ( :2_- PD 2 ) 1.422 1.309 (
h Pi Pili! MP i

C k 1 i
4.16 lPc(i= -~ (-) O.<XX>264 3.557x 10-3
/lirlll 10 MP

www.petroman.ir
--.I.
-
144 WELL TESTING

TABLEF.3 -MAJOR EQUATIONS WITH CONSTANT VALUES IN CUSTOMARY AND SI METRIC(Conrd.)

Numerical Value of Cons(ants


Equation in Order of Appearance
~umber = (cl' c2' c3 ...) -
~ Tex~ Equation Customary 51
.~~--~.- -
c. kt 6
4.17 tDL = ..0.CXX>264 3.557xIO-
, 41"CL-
r I f
4.18 Lf= I cIk t MP 11/2 0.CXX>264 3.557xIO-6

~Jl.CI {tOL,)MP

5.1 y,{P..t) =y,{Pi) P q T / 1.151 log ( c.,~~JI.I CII r "')


+C) -K.:!L:-
2
50,300 3.733
Tsc kh kt

-(S+D/Qg/)f 1,688 125.3

5.2 y,{p) =2
ip P
-dp --
.~
p,;

5.3 y,(P,'t)=y,(P)-CI&~
Tsc kh f ln(!:.!-)-0.75+S+Dlqgl
r", l 50,300 3.733

5.4 P q T
y,(P..t)=y,(P)+CI-K2L:-I.15110g-"~rp~/P""
/ ( c2 4>JI. ~c
k l
.r
'"
2
) -50,300 3.733
Tsc kh t

-(s+Dlqg I) I 1,688 125.3

5
5 .Pitt 2-
-P -2 +cl qgJl.pZp,T /1
og ( C24>Jl.pCIP ) - ( ~!!J!!-EJ)1 1637
,. 1508
~ kh kt p 1.151 1,688 11.638

5.6 p..j=P.-CI~~
., qp..".T r
/(In~)-0.75+S+Dlqgl I 1,422 1.309

5 .P
7 -~ -P..f 2 = oq g + bq g~ --

5.8
JI. -'" -T
o=clrp"pg'ln-!--O.75+s
I ( r ) / 1,422 1.309
kh r",

5.9 JI.-'"- T D
b=cl rp"pg' 1,422 1.309
kh
5 .qg
10
= C (p~
-.,
-p,,:!' 2 ) n -- ..

5.11 p~-p",}=olq~+bqi --

5.12 °1=Clrp"pg'
JI..~- T I 1-In, ( kt
)
2 +s J 1,422 1.309
kh 2 c2/Pp.pclpr'" 1,688 125.3

.,
6.1 Pi-Pr=-cl-Ei qBp. ( -.70.6
c.,op.cIr~
) 9.33xI0. .,
kh kt
948 7.036 x 104
6.2 PD = --EiI ( -)-ri> --
2 4to

A.I ~ +~ +~=-~(P4» --
ax a>, IJz at

www.petroman.ir
USE OF SI UNITS IN WELL. TESTING EQUATIONS 145

TABLE F-l -MAJOR EQUATIONS WITH CONSTANT VALUES IN CUSTOMARY AND SI METRIC(Conld.)

Numerical Value of Constants


E qua t Ion
.in Order of Appearance
Number (cl,c2,c3" .)
in Text Equation Customary 51

I a a
A.2 --(rpu,)=--(p(j) --
r ar at

A.4 a ( kxp
---+ ap ) a
-~ (k pap-+ ) a
-~ I k.p ( -+
ap 0.00694p )1 O,<XX>264 3.553 x 10-6
ax IJ. ax ay IJ. ay az IJ. az

I a
= --(p(j)
ci at

A.5
I a ( rpk, ap ) I
= --(p(j)
a
0.<XX>264 3.553 x 10-6
r ar IJ. ar ci at

a2 a2 a2 (j) c a
A.8 a? + -a? + a? = ~ £ 0.<XX>264 3.553 x 10-6

A.9
I a
--r-
( ap )= (j)IJ.C ap
--0.<XX>264 3.553x 10-6
r ar ar CI k at

A.13
I
--r-
a ( a1,l- )
=
(j)IJ.C a1,l-
~ -0.<XX>264 3.553x 10-6

rar ar clk at

A.14
I a
--r-
( ap )= li>ct ap
--0.<XX>264 3.553 x 10-6
r ar ar CI At at

ko kp kw -
A.15 At = -+ -R. + ---

1J.0 IJ.g IJ.w

A.16 Ct =Soco +Swcw +SgCg +cf --

A.17 CO = -~ ~ + ~ ~ --
Bo dp Bo dp

I dBw Bp dRsw
A.18 cw=---+ --
Bw dp Bw dp

Tz
D.5 Bg =cl -0.00504 0.351 ..
P
E.I clkh(Pi-Pw/) =!(lntD+C2)+S 0.00708 5.356xIO-4
qBIJ. 2
0.809 -1.062

E.2 clkh(Pi-Pw/) =
( ~+ln'De-o.75)+S 0.00708 5.356xIO-4
qBIJ. rDe

=PD +s, tD >0.25 rDe2

E.3 ci kh(Pi -Pw/) =PD (tD) +S 0.00708 5.356x 10-4


-qBIJ. -

I n
E.4 clkh(Pi-Pwf(tD)]
B = ,1: AqjPD(tD -tDj) I +qns 0.00708 5.356x 10-4
IJ. )=1

E.5 clkh(p;-pws) =PD(tD+~D)-PD(~D) 0.00708 5.356xI0-4


qBIJ.

www.petroman.ir
'I'
146 WELL TESTING

TABLEF-3 -MAJOR EQUATIONS WITH CONSTANT VALUES IN CUSTOMARY AND SI METRIC(Conld.)

Numerical Value of Constants


Equation in Order of Appearance
Number (CI'C2'C3"')
.~
In Text Equation Customary SI
-.
E.6 clkh(p-p)
' I
ws = -In (---[In(ID
/+~ ) I +~D) +0.809] 0.00708 5.356x 10-4
qBIl 2 ~ 2

+PD(/D+~D)

E.7 clkh(p;-pws) =-In


1 (-+PD(ID)--
/+~/ ) I r In(2.246ID) ] 0.00708 5.356XIO- 4

qBIl 2 ~ 2 :/
(
= 2J In ~)
/+~I + constant

cJkh(p.-p.) I
E.8 '=PD(/D)--ln(2.246ID) \ 0.00708 5.356xI0-4
qBIl 2
E.9 CrVp(p;-p)=ClqBI=C,AhlP(pj-p) 0.2339 4.167xI0-2

clkh(p;-p)
E.IO 2"KIDA= 0.00708 3.975 x 10 -2
qBIl
(p' -p) 2
E.II =4"KIDA +In(2.246 'D) -2PD(/D) 70.6 9.33 x 10
clqBIl i;!
"
(~
=PDMBH (IDA) *1
E.12 PD(/D) = 2 "KIDA +0.5 In(%.246 ID) -0.5PDMBH (IDA) --

E.13 PDMBH (IDA) = 4"KIDA = (p' -p) .ID <IDB 70.6 41.25
ci qBIl

E.14 PDMBH (IDA) =In(CAIDA) =lnCA +lnlDA -C3 --

E.15 2.246A
PD(/D=2"KIDA +0.05 In -::::--r ( ) --
CAfw
E.16 PD(/D) = 2 "KIDA+0.5 In(2.246 IDAAlr2w)-0.5PDMBH (IDA) --

Answers to Examples Expressed in 81 Units


..
~
'Examplel.].pi = 17740kPa,PIO = 20464kPa. 1=30hours:pj-pwl
PIOO = 20 684 kPa. 5
( ~.253
) -2s. I
= -933~1 x]O tP.I£C,rW2
Example 1.2.J = 4.612xI0-J m3/kpa'd.kJ = 16 kh tin kl
md,s= 16. -
I = 200 hours: no simple equation can be written.
Example ].3.
Infin". P...,do',.ady',a,.
P\Oud"".ady"a,. I = 400 hours: p -P wi
Solu"on 'Arr,n..ma.", IE,..""
-~~~
Circula,
;,!--~~~
0.10 132 o~ 79.2
~-;--~?ir~ ~;---~~~,i
0.1 I)~
-.= 1.866x103~
kh
1 !In
2
( C10.06A
r 2 ) -~4 +S I.
Squa'...,.n,.,od
0.09 119 OO~ 660 0.1 1)2 A w
~uar.-quadran' 0025 33 0)0 3'/60 06 "'92
Example ].4. I = 75.8 hours.
J "
-~2- ~- -,~~~:k~l. Im3/dl Example ].5. ~ = 1]2.73 kPa.
Circuw 31.62 l.ll3x 10-l .1.11
~ua, n'.'od )0.11 1.213xI0-2 .IIHI
Squar.-quadran,
.I~I) '.116.10-~ 31..1' Example].6.lp = 176 hours.

www.petroman.ir
USEOFSIUNITSINWELL.TESTING
EQUATIONS 147

~
E.~ample2.I.k=48md,Pi=13445kPa,s=I.43.
~
Examf'e6.I.k = 1433md,Ct = 3.974xI0-.
kPa- .
Example 2.2. AI = 6 hours, AI = 50 hours. Example 6.2. k = 817 md, rPct = 1.973x 10-7
Example2.3.k = 7.65md. kPa-l,Ct = 2.465xI0-6kPa-l.

Example 2.4. s = 6.37. r...a = 1.036x JO-4 m, AppendixC


(Ap)s = 2668kPa, E = 0.629./...bs = 7.42hours. ExampleC.I.
I (hour) p (kPa)
Example2.S.s = 12.3,sp = 13.2,sd = -0.18. -0:001 ~
0.01 94.04
Example2.6.jJ = 30413kPa. 0.1 65.71

Example 2.7. jJ = 30 420kPa. ExampleC.2.


I~ Pwl (kPa)
Example 2.8. L = 72.8 m. O.J 18857
Example 2.9. Ar = 5.707x 106 m 2.100.0 1.0 18 506
18375

4 210
Exdam~l~ 2 .
7.k = 9.96 md, s' = 4.84; k = 9.77 ExampleC.3. Qp = 0.0370 m3.
m ,s -..
ExampleC.4.Q = 3.975x 10. m3.
Example2.11. At = 0.0457md/Pa's, ko = 26.2 md, p
k... = 1.49md, kg = 0.782 md, s = 1.50.
AppendixD
ExampleD.I. Tpc = 644K, Ppc = 1965kPa.
Chap. 3
E;;;p.e3.I.k = 7.65md,s = 6.37. ExampleD.2,Pb...b= 133O7kPa.

Example3.2. V p = 4.992x 10sres m3. ExampleD.3. GOR = 62.34 m31m3.


Example3.3. k = 7.44 md, S = 6.02. ExampleD.4. Bo = 1.22 m31m3.

Example3.4.k = 7.65md,s = 6.32,p. = 30385 ExampleD.S. Co =5.09xJO-7kPa-l.


kPa. -S -I
ExampleD.6. CO= 2 x 10 kPa .
E,ample3.S.kh = 31.7md.m.
--ExampleD.7.#l-o = I. 21m Pa.s.

ExampleD.8. Rsw = 3.313m31m3.


Chap. 4
Example 4.1. k = 10.3 md, S = 5.0, Cs = 2.675 ExampleD.9. B... = 1.029m3 1m3.
x 10 -4 m3/kPa. ..
ExampleD.IO.c... = 5.076xI0-7 kPa-l.
Example 4.2. k,,'b = 4.01 md, kl = 8.03 md, E'= 6 I
0.607. ExampleD.II.c... = 0.957x 10- kPa- .

Example4.3. LI = 18.2m, k = 4,5 md. ExampleD.12. #1 = 0.33mPa.s.

Example0.13. Tpc = 216.6K,ppc = 4585 kPa.

~ Example0.14. Bg = 6.355x 10-3 m3/m3.


Example 5.1. qg = AOF = 1.47x 106 m3/d.
Example0.15. Cg = 5.656x10-. kPa-l.
Example5.2. qg = AOF = 2.373 x 10s m3/d.
ExampleD.16.lJ.g = 17.8jlPa.s.
Example5.3.qg = AOF = 3.IJ5x 10s m3/d.
ExampleD.17.cl = 5.22xI0-7 kPa-l.
Example 5.4. Resultsare tabulated in Table 5.10and
are plotted in Fig. 5.13. Reference
Example5.5.k = 9.66md,s' = -0.21. I. Earlougha. R.C. SPE.
MonographSeries. Jr.: DaJlas
Ad~ncrs in5. IJ'~I Tnt AnDII'Sis.
11977) -

www.petroman.ir
AppendixG
Answers to SelectedExercises
Chap.I.
Exercise 1.1. Assumptions: Sufficiently far from t'och well that £i-

x £i(x) In( ,. 781x) functionsolution can be used tor each well.


0.01 -4.038 -4.028
'.. Exercise 1.9. (a) P,.f=2.680 psi; (b) P,hur.in =2.961

0.02 -3.355 -3.335 psi.

0.1 -1.823 -1.725


1.0 -0.219 0.5772
E ;ra
Exercise 1.2.
I 10 - 2 486p .
xerc~ ..p-, Sl.

r (ft)
-Co-- p (psi)
--- Exercise1.11. Ignore prodUCtionbefore long shut-in in
No influence.
0.333 2.812 calculating tp.
1.0 2.837 -
10 2.888 Exercise 1.12. (a) t = 126 hours. t= 192 hours (actual

100 2.940 totalproducingtime)~(b)'

I.(XX) 2.988
3.160 3.<XXJ p (psi)

r.. = 1.9 89 ft. -., Homer


.-

Exercise 1.3. r..>1.989ft. ~ Super- -2~


-Uj9 A pprox-
r (ft) position imation
E .
xerClse I ..(a)t>
4 9 .68 seconds: (b)t~l. 2 I seconds; 100
10 2.760
2.499 2.744
2.484
/C)t~18weeks. I.(XX) 2.97.2 2.974
E .
xerClse I ..r;=1.989
S fi; r=I.490fi:.1p=2.45psl. 2.<XXJ .2.996 2.993 .

Exercise 1.6. t,=25.2davs. Exercise1.3.

E.~ercisel.7. r;unchanged.doubledrawdownat
.Infinire.A...ring ~ ~fKlU"'~ ~ pSS ~.~ J ~ "
eachr. 0 1.1.1 7.9~ 0.5161 163.1

Exercisel.S.(p;-P"f)"II;t/;trA 0 13.1 7.9~ 0.5~61 ~6.~.1


.. ]
B
=-70.6_ QA/.L
kh
[ ln [ -~I. 688 4J/.L(",r.7"
k(t-tA)
-2SA
]
6
~.r;7
11.9 9.:~
-"
05~."!() ~6/.5
u / "1
119 9 '"
Q /.LB
-70.6~Ei [ -,..'~
-948(j>11("r~
] "~ '. 10.6 15~
O ~""S
0.5177
"'
~58.9
kh k(t-tB) ..A I. 1.9K 7~:: O.~I~ ~()Il1

qc/.LB .
-70.6-£,
kh
[ -~-948(j>/.Lclr.~c
k(t-tc)
]
la!
L:J
r:-1
EB
1/.9
3.% 3.1.0
6.6 0.5156
O.~J
161.8
153.0

-
www.petroman.ir
-

ANSWERS TO SELECTED EXERCISES 149

Chap. 2 Chap. 4
Exerciw 2.1. (a) em>r=0.~89: (b) no difference. Exercise 4.1. k=9.68 md: s=4.59: £=0.636;
p"'j=I.150psi:(c)r..> 1.133 ft. 1..h,=7,3 hours: Vp=34.2xI06 cu ft: r..=986 ft
calculated: r t =226 ft (beginning of MTR): rj = 1,010 ft
Exerciw2.3. 1"hJ =7.85 hours. (endofMTR).

Exercise 2.4. MTR begins at -7.85 hours: k=24.5 Exercise 4.2. s=5.0 (for C $0 = 103); 1"~$ -5 hours;
md. k= 10.3md; C, =0.0116 RB/psi.

Exercise 2.5. s=O.064: (i1p), =32 psi; £=0.992: Exercise 4.3. C, =0,01 RB/psi: k,,'h =4.01 md;
r..u=0.~69ft. k=8,03md;E=0.606.

Exerciw 2.7. p=~.308 psi (p'method): p~4.405 psi. Exercise 4.4. (I) k=IO.1 md. s=5.05. £=0.594.
(modified Muskat method). l..h, =6.9 hours; (2) p=2.854 psia (p' method).
p=2.864 psia (modified Muskat method); (3)
Exercise 2.9. k=9.20 md; s+Dq,o:= -0.952. VR=3.01 x 107 resbbl.

Exercise 2.10. k,,=32.5 md: k".=3.48 md: k~=1.18 Exercise 4.5. Cso=103: s=5; k=9.92 md;
md:A,=80.52md/cp;s=-2.15. ' VR=3.0IxI07resbbl.

Exerciw 2.11. L=64 ft. Exercise 4.6. Cs =0.0103: k"~ =4.13 md: k=8.26
md: £=0.£i>7.
Exercise 2.12. (a) in psi:
Exercise4.8. Type-curve analysis: Lf=200 ft: k= 15.2
.11(days) md. Conventional analysis: k= 15.4 md: Lf= 144 ft.
~ -.!!- ~ ~ --!.C!- Squarerootanalysis: Lf=232 ft.
1.0 ~.837 ~.948 ~.973 2.99~
10 ~.862 2.948 ~.973 2.99~
10~ ~.888 2.948 ~.973 2.992 Chap. S
10'
10.&
~.914
~.9J9
~.949
~.9~1
~.973
~.973
~.99~
2.99~
E . 1
xerclse 5 ..(a) AOF=107.0 MMscf/D: (b)
10~ ~.965 ~.965 2.976 2.993 AOF= 100.0MMscf/D.
10" ~.988 ~.988 2.988 2.994
107 3.(XX) 3.<XX>J.<XX>3.<XX> Exercise 5.2. (a) AOF =6.6 MMscf/D (empirical
(b) ~1 (days) !J-J!!l method); (b) AOF=5.6 MMscf/D (theoretical method).

0 ')00
0 Exercise5.4. k= 10.3 md; s'=0.533.
.1 -
1.0 629
10 1.989 Chap. 6
E . 2 3 -., -.Exercise 6.1.1=851 hours: rj=12.510 ft: ~=68.6
x~rclse .1 .p=4.418 pSI (usIng Ip.u): p=4.411 pSI psi.
(using lp)'

.
E xerclse 2 ..11th,
14 5h
-ours. - 48 d
. k -m. ri-
371 ft
- (at
._ Exercise6.3. k=IOI md:s=-2.10:
r =290ft
£=1.46:
..
beginning). r, =813 ft (at end): s= 10.93.~, =950 psi. ' .

£ =0',-+.30;p =4.325 psi (MBH p* method): p =~.325 psi Exercise 6.4. k=0.51 md.
(modifIed Muskat method).

Chap. 3 Appendix A
Exercise3.1.k=9.55md:s=4.45:A=67.9acres. E . Al la a
xerclse (rpu,)=--(pq,).
r r at
Exercise3.2. k=II.1 md:s=4.14.
E .
33
xerclse ..k=12.52md:s=4.
7
I:p
* 3 0 .I
~4. 8 pSI. ExerciseA.2.
a ( rpkr ap ) = I a
-(pcb).
r ar Il ar 0.<XX>264
at
Exercise 3.4. (a) t Plotting
(hours) Function ExerclseA.3.
.I --r-a ( ap ) = IPlI.c -.ap
0.5 -0.301 r ar ar 0.<XX>264kat
1.5 0.428
2.5 1.331
ExerciseA.4.
I a
--r-
( a", ) = q,IlC~
'-.
a",
(b)P"f=2.105 psi. r ar ar 0.<XX>264k at

www.petroman.ir
150 WELL TESTING

Appendix B ExerciseC.3. Cumulative


.Water
Exercise8.1. Dimensionalform: Influx

.:~( )
r a,
r~
ar
= ~JJ.£'~~.
0.(xx)264k at .100
°t (days) (res bbl)
3.278 X 104
Dimensionless fonn' 200 5.377 x 104
.400 8.959 x 104

~~ ( ro~ ) =~. 800 1.J.68x 104


'0 a,o a,o ato" ExerciseC.4. Cumulative water influx=2.342x 105
res bbl.

Appendix C Appendix D
ExerciseC.I. q,,=979 STB/D. Exercise D.I. qRI=145 RB/D. A,=35.0 md/cp.
£', =9.55x 10-b psi-I
l1Np = 1.056 STB.
Exercise D.l. qRI=414 RB/D. A, =655 md/cp.
ExerciseC.l. Pllf=3.150psia:Np=II.900STB. c,=1.74xI0-4psi-l.
www.petroman.ir
Nomenclature
a = 1.422 /l.r:r~T [(In r..
--0.7 ) 5 +$ ] ...,.,of gravIty.
g = acceleration ftlsec. (m/s.)
kh r ". g (' = gravitational units conve~ion factor.
A = drainage area of well. sq ft (m2) 32. 17 (Ibm/ft)/(lbf-s~). dimensionless
Af = fracture area. sq ft (m 2) h = net fonnation thickness. ft (m)
AR = reservoir area. acres (m2) J = productivity index. STB/D-psi
A,,~ = wellbore area. sq ft (m2) (mJ/d.kPa)
.-.TD J actual = actual or observed well productivity
b = 1.422 /I.,,~,,~ index. STB/D-psi (m J/d .kPa)
kh J idcal = productivity index with penneability
b' = intercept of(p,-P"f)/qn plot. psi/STB-D unaltered to sandface. STB/D-psi
(kPa/m J /d) (m J /d' kPa)
B = fonnation volume factor. J R = gas-well productivity index. Mcf/D-psi
res vol/surface vol (m J /d' kPa)
B J( = gas fonnation volume factor. RB/Mscf J I = Bessel function
(m3/mJ) k = ~servoir rock penneability. md
B.~i = gas fonnation volume factor evaluated k f = formation permeability
at Pi. RB/Mscf (m3/mJ) (McKinley method). md
B" = oil fonnation volume factor. RB/STB k~ = penneability to gas. md
(mJ/mJ) kH = horizontal penneability. md
B ". = water fonnation volume factor. RB/STB kJ = reservoir rock penneability (based on
(mJ/mJ) PI test). md
c = compressibility. psi -I (kPa -I) ko = penneability to oil. md
Cf = fonnation compressibility. psi -I (kPa -1 ) k s = penneability of altered zone
c x = gas compressibility. psi -I (kPa -I) (skin effect). md
c xi = gas compressibility evaluated at original k v = vertical permeability. md
reservoir pressure. psi -I (kPa -I) k". = penneability to water. md
c.~". = compressibility of gas in wellbore. psi -I k "iJ = near-well effective permeability
(kPa -I) (McKinley method). md
-Co = oil compressibility. psi -I (kPa -I ) L = distance from well to no-flow
cpr = pseudo reduced compressibility boundary. ft (m)
c, = 5"c n +5 "C ". +5.~c.~ +C f Lf = length of one wing of venicaJ frncture. ft
=total compressibility. psi -I (kPa -I) (m)
C,i = total compressibility evaluated at Pi. m = 162.2 qBpikh=absolute vaJue of slope of
psi -I (kPa -I) middle-time line. psi/cycle (kPa .cycle)
CIf' = total compressibility evaluated at p, psi -I m' = 162.6 Bpikh = slope of drnwdown curve
(kPa -I) with (Pi -PMf)/q as abscissa.
c" = water compressibility. psi -I (kPa -I) psi/STB/D-cycle (kPa/mJ /d .cycle)
C,,~ = compressibility of liquid in wellbore. m" = slope of P;-s or P~f plot for gas well,
psi-I (kPa-l) psia2/cycle (kPa'cycle) -
c"1' = compressibility of pure (gas-free) water. mL = slope of linear flow graph. psi/hr'~
psi-I (kPa-l) (kPa.h'~)"
C = perfonnance coefficient in gas-well mmax = maximum slope on buildup curve of
deliverability equation fractured well. psi/cycle (kPa .cycle)
C A = shape constant or factor m true = true slope on buildup curve uninfluenced
C s = wellbore storage constant. bbl/psi by fracture. psi/cycle (kPa. cycle)
(m J IkPa) M = molecular weight of gas
C sO = 0.894 C s/ct>cIhr! =dimensionless n = inverse slope of empirical gas-well
wellbore storage constant deliverability curve
D = non-Darcy flow constant. D/Mscf (d/mJ) P = pressure. psi (kPa)
£ = flow efficiency. dimensionless p = volumetric avernge or static drainage-area
~ pressure. psi (kPa)
--.£i(-x) = -J (e -u /u)du p* = MTR pressure trend extrapolated to .." -

x infinite shut-in time. psi (kPa)


=the exponential integral Po = 0.00708 kh(Pi-P)/qB/I.=
F' = dl p/dl (' = ratio of pulse length to dimensionless pressure as defined for
cycle length constant-rnte problems

www.petroman.ir
,~'-".. .
152
POMBH = 2.303(p*- p)/m. dimensionless -'11111-
half-length WELLTESTING

P, = original reservoir pressure. psi (kPa) I ~nd = end of MTR In drawdown test. hours
PMT = pressure on extrapolated MTR. psi (kPa) If I = time at which late-time region begins.
P" = arbitrary reference pressure. psia (kPa) hours
P pi' = pseudocritical pressure. psia (kPa) = lag time in pulse test. hours
P pr = p~udoreduced pressure I p = cumulative production/most recent
Pr = pressure at radius r. psi (kPa) production rate = pseudoproducing time.
Po, = standard-condition pressure. psia (kPa) hours
(frequently. 14.7 psia) I pss = time required to achieve pseudosteady
P..f = flowing BHP. psi (kPa) state. hours
P..., = shut-in BHP. psi (kPa) I, = time for well to stabilize. hours
PI hr = pressure at I-hour shut-in (or flow) I..bs = wellbore storage duration. hours
time on middle-time line (or its T = reservoir temperature. oR (OK)
extrapolation). psi (kPa) T pc = pseudocritical temperature. oR (OK)
q = flow rate. STB/D (m3/d) T pr = pseudoreduced temperature
q0 = dimensionless instantaneous flow rate at T sc = standard condition temperature. oR (OK)
constant BHP (usually S200R)
qx = gas flow Idte. Mscf/D (m3/d) u = flow rate per unit area (volumetric
q.~, = total gas flow rate from oil well. Mscf/D velocity). RB/D-sq ft (m3/d.m2)
(m 3/d) V p = reservoir pore volume. cu ft (m 3)
Qp = cumulative production at constant BHP. V R = reservoir volume. bbl (m3)
STB (m3 ) V... = wellbore volume. bbl (m3)
BQ x = distance coordinate used in linear flow
Qpo = ~ analysis. ft (m)
1.119fi>c,hr;'(p;-p..f) Y1 = Bessel function
=dimensionless cumulative production z = gas-law deviation factor. dimensionless
R = universal gas constant z; = gas-law deviation factor evaluated at
Rs = dissolved GOR. scfgas/STB oil (m3/m3) pressurep;. dimensionless
Rs... = dissolved gas/water ratio. Zpg = gas-law deviation factor evaluated at-po
scf gas/STB water (m3/m3) dimensionless-
Rs..p = solubility of gas in pure (gas-free) water. an = rootsofequationJ1(anr~o)Y1(a~)
scf gas/STB water (m3 /m3) -J I (an)Y I (anr ~O)=O
r = distance from center of wellbore. ft (m) 'YR = gas gravity (air= 1.0)
r.l, = transient drainage radius. ft (m) 'Y0 = oil gravity (water= 1.0)
rd = radius of drainage. ft (m) ~ p = oil production during a time interval. STB
r ~ = external drainage radius, ft (m) (m 3)
r~o = r~/r... Ap* = P*-P... psi (kPa)
r; = radius of investigation. ft (m) (Ap)d = pressure change at depanure (McKinley
r s = radius of altered zone (skin effect). ft (m) method). psi (kPa)
r... = wellbore radius, ft (m) (Ap)s = 141.2 qBllfs)/kh=0.869 ms=additional
r "'a = effective wellbore radius. ft (m) pressure drop across altered lone, psi ..
s = skin factor. dimensionless (kPa)
s' = s+Dqx =apparent skin factor from Ap:, = P,,'s -PMT = difference between pressure
gas-well buildup test. dimensionless on buildup curve and extrapolated
s* = log (k/$Jl.c,r;)-3.23+0.869s MTR. psi (kPa)

S = log( ~ ) -3.23+0.869s A~ = t~me elapsed since shut-in. hou~


$IlC,r..- Al = time elapsed since Idte change 10 two-rate
Sg = gas saturation. fraction of pore volume flow test. hours ..
So = oil saturation. fraction of pore volume Al c = cycle length (flow plus shut-In) In pulse
S... = water saturation. fraction of pore volume .test. hours .
I = elapsed time. hours AI" = time at depanure (McKinley method).
10 = 0.(xx)264 kl/tJ>IlCfr; .hours-
= dimensionless time Al end = time MTR ends. hours
lOA = 0.<XXJ264kl/$IlC fA Al p = pulse-period length. hours
=dimensionless time based on dldinage Atr = time at which middle- and late-time
area. A stldight lines intersect. hours
lOLl = 0.<XXJ264b/~lJ.CfLf-
, '1 = 0.<XXJ264 k/~IJ.C=hydrnulic diffusivity.
=dimensionless time based on fldcture sq ft/hr (m~ Ib)

www.petroman.ir
Bibliography
A E
Aiarwal, R.G.: "A New Method To Account for Producing- Time Earlougher, R.C. Jr.: AdL'ancC)" in Wrl! Tt'St AnalysIS,
Effects When Drawdown Type Curves Are Used To Analyze Monograph Series, SPE, Dallas (1977) 5.
Pressure Buildup and Other Test Data,," paper SPE 9289 Edwards, A.G. and Winn, R.H.: ,. A Summary of Modem Tools
prcscnled al the SPE 551h Annual Techntcal Conference and and Techniques Used in Drillstem Testing," Pub. T-4069,
Exhlbillon, Dallas, SePI. 21-24. 1980. Halliburton Co., Duncan, OK (Sept. 197).
Agarwal, R:G., AJ-Hussainy, R., and Ramey, H.J. Jr.: "An Edwardson, M.J. rt al.: "Calculalion of Formation Temperature
Investlgallon of ~ellbore Storage and S~~n Effect In Unsteady Disturbances Caused by Mud Circulation," J. ht. Trch. (April
Liquid Flow -I. AnalytIcal Treatment. Soc. ht. Eng. J. 1962) 416-426' ]; AI ME 225
(Sept. 1970) 279-290; Trans.. AIME, 249. ,raIlS., ,.

AI-Hussainy, R.. Ramey, H.J. Jr., and Crawford. P.8.: "The


Flow of Real Gases Through Porous Media," J. ht. Trch.
(May 1966) 624-636; TraIlS., AIME, 237. G
Amyx, J.W., Bass, D.M. Jr., and Whiting, R.L.: h~rolrum Gladfeller, R.E., Tracy, G.W., and Wilsey, L.E.: "Selecting
Rrservolr EngIneerIng:. Ph.vslcal Proprrtlt'S. McGraw-HIli Book ~ells Which Will Respond 10 Production-Stimulation Treat-
Co., Inc., New York CIIY (1960), ment," Drill. and Prod. Prac, , API, Dallas (1955) 117-129.
Gray, K.E.: "Approximaling Well-to-Faull Distance From
"c. 8 Pressure Buildup Tesls," J. ht. Trch. (July 1965) 761- 767,
Back Pressurr Trst for Natura! Gas !J'ells, Revised edition, Gringarle,n, A.C., Ramey, H.J',~r., and Ra,havan, R.: "Prcssure
Railroad Commission ofTe~as (1951) AnalysIs for Fractured Wells, paper SPE 405 I presenled at Ihe
.SPE-AIME 471h Annual Mecling, San Antonio, Oct. 8-11,
Beal, Carlton: "The ViscosilY of Air, Water, Natural Gas, Crude 1972.
Oil and lIs Trans.,
Pressures," Associaled
AIMEGases
(1946)al 165,94-115.
Oil-Field Temperalures and ..,.
Gnnganen, A.C., Ramey: ~.J. .Jr., and Raahavan, R.. .Un-
..,' sready-Slate Pressure Dislnbutlons Created by a Wdl Wllh I
Brons, F. and Miller,. W.c:;: A Simple Method for Correcting Single Infinite-Conductivity Vcnical Fracture," Soc. ht. Eng.
SPOI Pressure Readings, J. ht. Trch. (Aug. 1961) 803-805; J. (Aug. 1974) 347-360; Trans., AIME, 257.
Trans., AIME, 222.
Brown, George G.. Katz, Donald L., Obcrfell, George G., and
Alden, Richard C.: Nalura! Gasoline and tM Volatile
H.vdrO(.urbons, Natural Gas Assn. of Amcrica, Tulsa (1948). H
Hall, Howard N.: "Comprcssibility of Reservoir Rocu," Trans.,
AIME(1953)198.3~311.
C Hawkins, M.F. Jr.: "A Nole on the Skin Effect," TraIlS., AIME
Carr, Norman L., Kobayashi, Riki, and Burrows, David B.: (1956)207,356-357.
"Viscosity of Hydrocarbon Gases Under Prcssure," TraIlS., Holditch, S.A. and Morse, R.A.: "The Effects of Non-Darcy
AIME (1954) 201, 264-272. Flo" on the Behavior of Hydraulically Fractured Gas Wdls," J.
Carslaw, H.S. and Jaeger, J.C.: Conduction of Hrat in Solids, Prt. Tech. (Oct. 1976) 1169-1178.
second edilion, Oxford atlhe Clarendon Press (1959) 258. Horner. D.R.: "Pressure Buildup in Wells," Proc., Third World
Charas, A.T.: "A Praclical Trealment of Nonstcady-Stlte Flow Pet. Cong., The Hague (1951) Sec. 11,503-523; also Pressurr
Problems in Reservoir Systems," Pet. Eng. (Aug. 1953) 8-44 -Analysis .\1ethods, Reprinl Series, SPE, Dallas (1967) 9,25-43.
8-56.
Chew, Ju-Nam and Connally, Carl A. Jr.: "A ViscosilY
Correlation for Gas-Saluraled Crude Oils," Trans., AIME
(1959) 216. 23-25. J
Cobb, W.M. and Dowdle, W.L.: ..A Simple Method for Jargon, J.R.: "Effect of ~'ellbore Storage and Well bore Damage
Determining Well Pressure in Closed Rectangular Reservoirs," al Ihe A~ive ~'ell on Inlerference Test Analysis," J. ht. T«h.
J. Pet. Tech. (Nov. 1973) 1305-1306. (Aug. 19;6)851-858.
Cobb, W.M. and Smith, J. T.: "An Investigation of Pressure- Johnson, C.R., Greenkorn, R.A., and Woods, E.G.: ."Pulse-
Buildup Tesls in Bounded Reservoirs," paper SPE 5133 Testing: A Ne\\ ~etho~, for Dcscnblng Reservoir Flo'l
presenled al Ihe SPE-AIME ~9th Annual \Iecling, Houston, Properties Bet"een Wells. J. Pel. Tech. (Dec. 1966) 1599-
Oct. 6-9, 1974. ,o\n abridged version appears in J. Prt. Tech, 1604; Trans., A1\IE, 237.
(Aug. 1975) 991-996: Trans., AIME, 259,
Craft, B.C. and Ha"kins, M.F. Jr.: Applied Pelroleum Reserl'oir
Engineering, Prentice-Hall Book Co., Inc., Englewood Cliffs, K
NJ(1959). .
Kamal, \1. and Brigham, W.E,: "Pulse- Testing Response for
Cullender. M.H.: "The Isochronal Performance Melhod of Unequal Pulse and Shut-In Periods," Soc. hI. Eng. J. (Oct.
Determining Ihe Flow Characterislics of Gas Wells," Trans.. 1975)399-410; Trans., AIME, 259.
AIME(1955)200$,137-142. Kalz, D,L. el al.: Handbook of Natural Gas Engineering,
McGra\\-Hili Book Co, Inc,. Ne" York (1959) 411.

D
Dake, L.P.: Fundalnentals of Reservoir Engineering, Elsevier
Scienlitic Publishing Co., Amsterdam (1978). L
Dodson. C,R. and Standing, M.B.: "Prcssure-Volume- Larson, ",C.: "Underslanding the Muskal Melhod of Analyzing
Temperature and SolubililY Relations for Nalural-Gas-Water Pressure Buildup Curves," J. Cdn. ht, T«h. (Fall 1963) 2,
Mixtures," Drill. and Prod, Prac., API (1944) 173-179. 136-141.

www.petroman.ir
BIBLIOGRAPHY 155

M S
Martin, J.C.: "Simplified Equations of Flow in Gas Drive Saidikowski, R.M.: "Numerical Simulations of the Combined
Reservoirs and the Theoretical Foundation of Multiphase Effel:\s of Well bore Damage and Partial Penetration," paper
Pressure Buildup Analysis," Trans., AI ME (1959) 216, 3~-311. SPE 8204 presented at the SPE-AIME 54th Annual Technical
.at th ews, C..,S B rons, F., and Haz .~,..
M -
br ~ k P ." A ,Method for Conference and E.,hibitlon, Las Vegas, Sept. 23-26, 1979.
Determination of Average Pressure in a Bounded Reservoir," Schultz, A.L., Bell, W.T., and Urbanosky, H.J.: "Advancements
Trans., AIME (1954) 201, 182-191. in Uncased-Ho1e, Wireline Formation-Tester Techniques," J,
Mauhews, C.S. and Russell, D.G.: Pressure Buildup and Flow Pet. Tech. (Nov. 1975) 1331-1336.
Teslsin ~'ells, ~10nograph Series, SPE, DaJlas(I967) I. Slider, H.C.: "A Simplified ~1ethod of Pressure Buildup Analysis
McCain, W.D. Jr.: The Properlle5 of Petroleum Fluids, fora Stabilized Well,"Trans.,AIME(1971)27I,1155-1160.
Petroleum Publishing Co., Tulsa (1973). Slider, H.C.: Praclical PFlrol~m R~rvoir Engineering Methods,
McKinley, R.M.: "Estimating Flow Efficiency From Afterflow- Petroleum Publishing Co., Tulsa (1976) 70.
Distorted Pressure Buildup Data," J. PFI. Tech. (June 1974) Smolen, J.J. and Litsey, L.R.: "Formation Evaluation Using
696-697. Wireline Formation Tester Pressure Data," J. PFt. Tech. (Jan.
McKinley, R.M.: "Wellbore Transmissibility From A fter flow- 1979)2.5-32.
Dominaled Pressure Buildup Data," J. Pet. Tech. (July 1971) Standing, M.B.: Volumelric and Phase Behavior of Oil Field
863-872; Trans., Al ME, 251. Hydrocarbon S.vslems, Reinhold Publishing Corp., New York
\1iller, C.C., Dyes, A.B., and Hutchinson, C.A. Jr.: "Estimation (1952).
of Permeability and Reservoir Pressure From Bouom-Hole Standing, Marshall B. and Katz, Donald L.: "Density of Natural
Pressure Build-Up Characteristics," Trans., AIME (1950) 189, Gases," Trans., AI ME (1942) 146,140-149.
91-104. Slegemeier, G.L. and Mauhews, C.S.: "A Study of Anomalous

0 Pressure Buildup Behavior," Trans.,AIME(1958)213,44-SO.


Odeh, A.S.: "Pseudosleady-State Flow Equation and Produc-
livity Index for a Well With Noncircular Drainage Area," J.
Pel. Tech. (Nov. 1978) 1630-1632.
Odeh, A.S. and Jones, L.G.: "Pressure Drawdown Analysis, T
Variable-Rate Case," J. Pel. Tech. (Aug. 1965) 960-964; Theorv and Praclice of lhe Tesling of Gas ~'ells, third edition,
Trans., AI ME, 234. Pub. ECRB-75-34, Energy Resources and Conservation Board,
p Calgary (1975).
Trube, Alben S.: "CompressibililY of Natural Gases," Trans.,
Perrine. R.L.: "Analysis of Pressure Buildup Curves," Drill. and AIME (1957) 210, 355-357.
Prod. Prac., API, DaJlas (1956) 482-509.
.". .,
T 1.-
ru~,
AI '"'--
~n
S
.:
"C
ompresSI IllY
.b I ' '
0f U n d ersatur ated
PInson, A.E. Jr.: Concerning the Value of Productng Time .Used Hvdrocarbon Reservoir Fluids," Trans., AIME (1957) 210, 341-
in Average Pressure Determinations From Pressure- BuIldup 344.
Analysis," J. Pet. Tech. (Nov. 1972) 1369-1370. -

R
Ramey, H.J. Jr.: "Non-Darcy Flow and Well bore Storage Effects
on Pressure Buildup and Drawdown of Gas Wells," J. PFI. V
Tech. (Feb. 1965) 223-233; Trans., AIME, 234. E d. A F d Hurst W F ." Th - Appll 'cation of the

van ver Ingen, ..an ,... .

Ramey, H.J. Jr.: "Practical Use of Modern Well Test Analysis," Laplace Transformation to Flow Problems in Reservoirs,"
paper SPE 5878 presented at the SPE-AIME 51st Annual Trans., AIME (1949) 186. 305-324.
Technical Conference and Exhibition, New Orleans, Oct. 3-6, Vela, S. and McKinley, R.~1.: "How Areal Heterogeneities Affel:\
1976. Pulse-Test Results," Soc. Pel. Eng. J. (June 1970) 181-191;
Ramey, H.J. Jr.: "Rapid Methods for Estimating Reservoir Trans., AIME, 249.
Compressibilities," J. Pel. Tech. (April 1964) 447-454; Trans.,
AIME,23I.
Ramev, H.J. Jr.: "Short-Time ~'ell Test Data Interpretation in
the -Presence of Skin Effect and Well bore Slorage," J. Pel. ..
Tech. (Jan. 1970)97-104; Trans.,AIME,249 W
Ramey, H.J. Jr. and Cobb, W.M.: "A General Pressure Buildup Wattenbarger, R.A. and Ramey, H.J. Jr.: "An Investigation of
Theory for a Well in a Closed Circular Drainage Area," J. Pel. Well bore Storage and Skin Effect in Unsteady Liquid FlolO-II.
Tech. (Dec. 1971) 1493-1505; Trans., AIME, 251. Finite-Difference Tr~atm~nt," Soc. Pel. Eng. J.(Sept. 1970)
"Re\iew of Basic Formation Evaluation," Form J-328, Johnston- 291-297; Trans., AIME, 2~9.
Schlumberg~r,Houston(1976). Wauenbarg~r, R.A. and Ramey, H.J. Jr.: "Gas W~II Testing
Russ~II, D.G.: "Del~rmination of Formation Characteristics With Turbul~nce, Damage, and W~lIbor~ Slorage," J. Pel.
From Two-Rale Flow Tests," J. Pel. Tech. (Dec. 1963) 1347- Tech. (Aug. 1968) 877-881; Trons., AIME, 243.
1355; Trans., AIME, 228. Win~stock, A.G. and Colpius, G.P.: "Ad\ances in Eslimaling
Russell, D.G. and Truiu, N.E.: "Transient Pressur~ Behavior in Gas W~II Deli\erabililY," J. Cdn. Pel. Tech. (July-Sepl. 1965)
V~rlically Fractur~d Res~rvoirs," J. Pel. Tech. (Oct. 1964) I) 1-119. Also, Gas TechnoloE-v, Reprint Series, SPE, Dallas
11;9-1170; Trans., AIME, 231. (1977) 13,122-130.

-.-
www.petroman.ir
Author Index
A H Odeh.A.S.. ~O.60. 62
A~a/'ll/al.R.G.. 20. 49. 75 Ha". H..~.. 132.133
Alden. R.C.. 133 HawkIns.M.F. Jr.. 4. ~O.77. 88. P
AI-Hussainy.R.. 20. 49.75.88. 102 Hazeb~k. P.. 20. 35-39. 41. 46. 48. 49.
Amy~. J. W.. 119. 133 74. 135-137 Pemne.R.L.. 46. 49
Holdilch. S.A.. 75 Pinson.A.E. Jr.. 49
B Homer.DR.. 2.18-21.23-27.29.30.36.
BoL'is.
D.M. Jr.. 133 37.46.48.49.56.58.63.65.72
Hum. W.. 3. 20. 118 R
Beal. C.. 124. 133 Hulchinson.C.A. Jr.. ~5. 49 Raghavan.R.. 75
Bell. W.T.. 99 Ramey. H.J. Jr.. 20. 27. 29. 44. 49.53.
Brigham. W.E.. 99 J 62-64.66-68.74.75.87.88. 102. 122.
Brons. F. 20. 35-39. 41. 46.48.49.74. 126. 127. 133. 137
135-137 Jaeger. J.C.. 20 Rus.\CII. D.G.. 1.20.34-36.49.62.99.
Bmwn. G.G.. 128. 133 Jargon. J.R.. 89. 99 102. 128. 133
Burrow~.D.B.. 133 Jones.L.G.. 60. 62
Johnson.C.R.. 99 S
C
Carr. N.L.. 131. 133 K Safdikowski.
SehullZ. A.L..R.M..
99 33. 49
Ca~law. H.C.. ~O Kamal. M.. 99 Slider. H.C.. ~O.25. 49
Charas.A.T.. /~. 1/8 KalZ. D.L.. 20.128. /33 SmIth.JT.. ~5. 29. 49
Chew. J.. 124. 133 Kobayashi.R.. 133 Smolen.J.J.. 98. 99
Cobb. W.M.. 25. 29.49. 137 Slanding. M.B.. /19-/21. 124-126.
Colpills. G.P.. 53. 62 L 128. 133
Connally. C.A. Jr.. 124. 133 Slegemeier.
G.L.. 62
Crafl. B.C.. 77. 88 Lanan. V.C.. 49
Crawford. PB. 75. 88. 102 Lilsey. L.R.. 98. 99 T
Cullender.M.H.. 88

0 M Tracy. G.W.. 119.


Trube.A.S.. 62 121. 131. 133

Dake. L.P.. 88. 137 Manin. J.C.. 49. /02 Truin. N.E.. 34. 35. 49
Dodson.C.R.. 124-126. 133 Manhews.C.S.. I. /7.20.35-39.41.46. U
Dowdle. W.L.. 137 48.49. 62. 74. 99. 102. 128. 133.
Dyes.A.B.. 25. 49 135-/37 Urbaoosky.H.J.. 99
McCain. W.O. Jr.. /19. /33
E McKinley. R.M.. 63. 68-71. 74. 75. 89. 99 V
Miller. C.C.. 25. 37. 49
Earlougher.R.C. Jr.. I. 14.20.41.49.62. Miller. W.C.. 49. 136. 137 vanEverdin~en.A.F.. 3. 20. /18
75.89.91.99.1/9.132.133.137 Morse.R.A.. 75 Vela. S.. 89. 99
Edwards.A.G. 99 Muskal.M.. 35. 36. 40. 41. 48. 49.74
Edwardson.M.J.. /18 W
N
G Wanenbarger. R.A.. 20. 44. 49. 75. 88
Nisle. R.G.. 4 Whiling. R.L.. 133
Gladfeller.R.E.. 53. 62 Wilsey. L.E.. 62
Gray. K.E.. 43. 49 0 Wineslock.A.G.. 53. 62
G~nkom. R.A.. 99 Winn. R.H.. 99
Gringanen.A.C.. 63. 7/-75 Oberfell.G.G.. 133 Woods.E.G.. 99

..
www.petroman.ir
Subject Index
A bubble-point of crude oil. 119 (or now In porous media.
Absolute open flow. 77-79. 82. 84. 85 dlsunce to Inflow txMIndary. 42 dcvc~nl. 100-102
Acidization. 4. 30 effective wellbo~ mlus. 32 ~iall1ow of nonideal gas. 2
A (terl1ow end of wellbo~ ~torage single-phaseflow of ~5Crvoir oil. 2
buildup lest. with or withoul. 25. 27. 64 dlstonlon. 28. 29. 33 slmul~s flow of oil. gas and water. 2
definition. 24 flow efficiency. 33 to ~I unsteady-Slateflow. 2
dislonlon. 29. 42 fOmlallon comp~sslblilly. 133 Dlffusivlry ~lOn:
duration. 21. 26. 30 (ormation pemleability. 30 dcfim~ of. 2
Albc~ Energy Resoun;e and Con5Crvallon gas comp~ssibiliry. 31 for bIXIrMicdcylioorical ~rvoir. 3
Board. I. 77. 88. 99 gas formal Ion volume factor. 131 (or Infinite cylioorical ~rvoir with
Analysis by/of gas-law deviation (actor. 131 li~= well. 3-6
~l' also Calculalion/estimation o( gas pseudop~ssu~. 85. 86 for .-eudosIeadY-Slatesolution. 6-11
Analysis by/of (examples): gas solubiliry in water. 125 for ~iaI flow in infinite ~rvoir with
buildup test for venically fractu~ well. gas viscosiry. 131. 132 well~ Slorage. 11-13.64
73.74 011(ormatIOn volume factor. 121 solutions 10.3-15.63.91
conSUnl-rale drawdown lest. 51. 52 oil viscosiry. 124. 125 Van Everdingen-Hurst solutions. 106-118
damage near wellbo~. 32. 33 po~ volume. 53 Dimensionless:
drawdown test using McKinley's Iype p~ssu~s bcyund the wellbo~. 5. 6 p~~ solutions. 14
curves. 70. 71 pseudocnllcal gas propenles. 128 II~ lag. 93-97
drawdown test using Ramey's rype pseudocrilical lemperalu~ aoo p~ssu~ for variables. 3. 27. 35. 5 I. 63. 66. 68. 71.
curves. 67. 68 undersaturated crude oil. 119 92. 103-105
drawdown lest with varying rate. 53. 54 radius of investigation. 15 well~ storage constant. 12
flow in generalized ~servoir ~servolr size. 44 Dr:linage area:
geomelry. 8.11 saturated oil comp~ssibility. 123 average~ssu~. 24. 35. 36.40
gas well buildup test. 45 skin factor. 32. 33 cin:ular. 7-11. 29. 39. 51
gas well drawdown test using solutIon GOR. 120 geometry. 29
pseudo~ssu~s. 86-88 undersaturaled oil comp~ssibility. 122 hcuglXlaJ. 8. 9
Horner's approximation. 18. 19 water comp~sslbility In a saturated infimte-xting. 30
ideal p~~ buildup lest. 22. 23 ~servolr. 127. 1:.8 off~r. 9-11. 27
incompletely perforated interval. 33 water comp~sslbiliry In an undersaturated ~ssu~. 21. 64. 76
interfe~nce lest in water sand. 90. 91 ~servolr. 126 shape. 36
isochronal gas welltesl. 82 water formation volume factor. 125 squa~. 8-11. 29. 36. 5I. 71. 72
modified isochronaltesl. 84. 85 water vISCOSity.128 stall<:pressu~. 35. 36.40.46
mulliphase buildup test. 46 Canadian gas well testing manual. 89 Drainage -shape factors:
multirate flow test. ~. 61 Comp~sslbiliry. total system. 2.46 for ~rs. 37-40
n-rate flow test. 59.~ Comp~ibility co~lations: in ~ ~rvoirs. 9. 10
pulse lest. 92-97 crude oil. saturated. 122. 123 in venically fractu~ ~5Crvoirs. 10
stabilized flow test. 78. 79 crude oil. undersaturated. 12L 122 in waler-drive ~rvoirs. 10
two-rate flow test. 59 formation. 132. 133 Drawdown~:
U5Cof Po solutions for constanl-p~ssu~ gas. 128-131 ~l' ~~ drawdown test
txMIndary. 111-113 water. saturaled ~5Crvolr. 126-128 Dtillsteffi tcSIS.I. 97. 98
use of Po solutions for oo-flow water. undersaturated reservoir. 126
txMIndary. 107 Conservation of mass. law of. 2 E
use of Q 0 Constant-rate production. 12. 34. 56. 64
solution"s. 114. 115 Continuity equation: Early-{j~ ~gion. buildup curve. 23-27. 30.
use of super}X)Sition. 18 for mial flow. 100. 101 .35. SO.51. 65. 68.86
variable ~ssu~ history with Q for three-dImensional flow. 100 Ei functK>ll:
solutions. 115-117 po Co~lations: ~l' Exponential integral
well from PI lest. 7. II empirical. of field data. 78 Empiric:21meIhIxI for analyzing ga.~flow test
Assumptions. idealized: in pulse lest analysIs. 92-96 dau. 78. 82. 84. 85
homogeneous ~servoir. 25. 26 relatIng I' and B to produced Exen;\se5:
infinile ~servoir. 24 fluid propenies. 97 analysis of well tests using rype
single-phase liquid. 25 rock and fluid propenies. 119-133 curves. 74. 75
de-elopmcnt of diffe~ntial equa.\ions for
B fluid flow In porous media. lof
D di~nsionless variables. 103
Bessel functions. 3. 6 dnllstem. interfe~nce. pulse. wi~line
Bibliography. 154 Dan;y.s law: lests. 98. 99
Bounded ~servoir: applicability of. 3. 76 flo..tesls. 61. 62
cylindrical. 3 isothennal flow of fluids of small and fluid flow in porous media. 19. 20
p~ssure behavior. 16 constanl comp~ssibility. 2 gas well testing. 88
shape factors for single-well drainage p~ssu~ drop. 76 pressu~ buildup tests. 47-49
a~as. 9.10 Delive~bility: rock and fluid properties co~lalions. 133
Bubble-point p~ssu~ co~lalion. 119. 120 emplncal plot. 81 van Everdingen and Hurst solutions to
BuIldup test: equation. 77-79 diffusiviry equations. 117. 118
Sel' P~ssu~ buildup test isochronal curve. 81. 82. 86 E~poncntial integral:
C modified isochronal curve. 84 argumenlof. 5
slabilized curve. 77. 79-82. 84. 85 con.unt.42
Calculation/estimation of: slOlbllizedequation. SO. 83 definition of. 3
Sel' also Analysis by/of stOlbilized.estimates. 78 scqucoceof. 18
Calcula!ion/estimation of (examples): tests. gas ..ells. 76. 79 solution. 2. 5-8. 14-16. 24. 41. SO. 55.
additional p~ssu~ drop. 32 lransienl curve. 82. 84 89.91
avelOlgep~ssu~ in drainage a~a. 36. 37. Dif(e~ntIOlI equalions: type curves. 90
40.41 describing a flow tesl. 63 valuesof. 4

www.petroman.ir
158 WELL TESTING

F 27.29.35.42. .43.50.51.68. 86 di'l"n.:~ 10 oo-now tNJundary..43


Falloff le,ts: 29. 30-32. 63 Linear now Into fr.lctun:s. 13. 34. 63. ~~:lmplo:. k>g'k>cg~ph. 31
Field rests. 4ualilatlve behavior of. 26. 27 71. 72 e~ampl~. semik>glraph. 28
Flow ~fficlcncy e~tr.apobtlng 10 infinite shut-in lime. 98
o:alculalionof. 32. 33. 69-71 M follo",ng drawdown test at diff~renl
definItion. 32 I2tes. 83
Flow I"w,. 101 Match poinl'. pressure and lIme. 65-74. IX) fur ga, w~lI. 45.77
Flow tests. 50-62. 77-80 Matenal balances. 18.43 for Intinlte.~ing reservoir. 35
Fluw-aft~r-now le'ts. 77-79 MBH pressurefunction. 36-39.41 for well ~r reservoir ~ndary. 42
Formalion ~rmc"bllity: for different w~1I locations in a 2: I for ,,~II near reservoir limil(l). 35
bulk. 29 rectangulartNJundary. 37 ide.algraph. 23
detcnnincd from buildup test. 22. 29. 30 for different well locations in a 4: I in drillst~m testin,. 97
determincd from drawdown test. rectangular tNJundary. 38 in hydl2ulically frxtu~ well. 26. 34
52. 70. 87 for different w~lllocations in a square innuence of aftcrflow on H~r gl2ph. 27
effcclive. SO. 52. 56. 58. 59 tNJundary. 37 mulliphase. 46
e'timalion of. 7. 21. 23. 30. 36. 90 for reclan81esof various shapes. 38 pl()\tmg tcchn~. 22
for infinlle.acting n:.ervoir. 25 for well in center of equilateral fi8ures. 36 precedcd by consunl rate production. 56
from Iwo-r.lle now test,. 59 in a square anti in 2: I rectangle,. 39 pre,,'~dedby In -I) differenl now I2tes. 57
'nl_nI ty~ o:urvc'. 63. 70-73 on a 2: I rectangle and equilateral prec~d.:d by tWo different now rates. 56
In'lnlt~-..cting re.ervoir. 58 triangle. 39 rdl~ hlslory for ~al sySlem. 24
Inlormatlon about. I McKinley's ty~ curves. 68-71 rate hlSt(lry for Kleal system. 22
relaliun to slope. m. 56 Middle-time line. 32-36. 42. 52. 53. 58. 63 5ha~ of. 15
rel:lt)(,n to 'tr.aighi-line slo~. 24. ~6 Middle-lime region. buildup curve. 23-27. IY~ curves. 63-74
F,lrrn..I)(,n volume f"ctor I:orrelation: 29. 30. 34-36. 42. 43. 46. 50-53. 56. v~ni"',,lly fractUredw~lI. 73. 74
~;L,.1~8-131 58.59.63.65.68.86-88 wireline.97.98
oil. 120. 121 MobililY. 100al.2. 46. 47 wllh fOnllalion damage. 25
wal.:r. 125. 126 Model with no afterflow. 25
F.-.acturedwell' buildup lest. 57 with pressurehumping. 58
s Hydraulic fracture,; and drawdown equation. 58. Pre-sure drawdown test:
Hydr.luli"',,lIy frdctured well drawdown tesl. 76 analysis deve~nt. 41
e4uation for MTR ofdr.awdo"n le5t. 53 con,;l:lnt I2te. 51-53. 61. 63. 64. 67.
G now in ItIc reservoir. 64 71.73
ga' now in term of pseudopres.,ure.66 con'~ntional. 58
G..., dclivo:r.lbililV contr.lI:I'. 57 ideal reservoir. 2 declining rate. SO
G;a., 'H'W In n:~~oir,;. 76. 77 infinil~-actin~ re5Crvoir. 44 dimen,ionless.64
GiI' p,;cudopre,-,ure. 45. 128 inter1erenceteslS. 16. 89 £j-functKM ~KM. 24
G..., !;i1tur.llion. immtlbilo:. 25 n-r.aIO:now le'l. 59 eslim;llion of reservoir pore volume. 52
G..., well le,I,. I. 21. 45. 76-88 pres.'iUrebehavior ill any point gas "ell analysis. 85. 86
G;a.,wclliest data: in reservoir. 18 KleaJized coostantrate. SO
i1niIly,", of one r.atecontinued 10 production hi.'ilory of vari;lbl~ r.alewell. 18 in an .>bservarionwell. 89. IX)
'I;lbilization. 81. 82 rate history. well wilh continuou.ly model 0(. 76. 77
;lnal~", when no ,tabilized now changing rate. 17 modifications of equarion for gas. 44. 45
altaincd.82 re,;ervoir. correlillion of. 35 mUllir;ll~. SO
Ga,-I;I" devi;lti(1n faClor. 128. 129. 131 single-pha...enow of oil. 25 ,ha~ of. 15
Grin~;lncn ", at I)(X' curves. 71-74 ';Ieady-,tale rddial for equation. ~ 100al. 17. 31. 32
variable-rdle well. 16 ty~ curves, 63-74
M.x!ified isochmnallest,. 83-85 variable-rase. 53. 54.61.83
H Mooified Muskal melhod. 40. .41 Pressute falk>ff lest:
HclcrlIgcneilic,.1.15.21.25.26.28.51 M uIIIpa...e
h nOW.I,~llcaltonSlor.
--"' fi .. 45- ' 7 S"" Falloff tesl
Humogenl.'(IU' re-ervoir a ,umplion. 25. 26 Mulllr.lle now te,I.. 55-61 Pres,ure humping. 58
H me I t .
buld ' 4 ' 7 9 30
, ' n-rdle 'low I~st. 59. 60 Pres.ure levellD sumJUndlng
(I"\rpoo luple5t._,-.'-'" 'orrnalion.3.s-41
H .6.37.46.56.58.65.7.
. 9 '3 N n-. Dan:y. 44. 76
rl~'5ure I~. 00II-
H '
yur.lu
".
umcr I';lppm~lrn..t)(Jn.
II: ul
,,
U"vlty.
-18. I .-.56
definICiOn
f
o. 2 ~on1Cncl;llun:. 151
Pre"ure response.-.v-
lV\ 9'" 97
..
H .1 I , ., pre"ure lran,~t I~'I:

~ur.lUI IC IIr.l..'Iunng..4.
Hd t. d .,0II ' 3 3~ on 5 non-D;ln:y 110win fr.lclun:. 71 S(,.. Pn:'-'iUred rawdown t~,t and Pre"ure
~ r.lu II:" y rdl'lun: we ..' -.'. 'ov. 0 bid
UI up Ie.;. ..
I p Pnlbl.:m examples:
S.." Calcul;ltioo/eSlimalion of ;lnd
Idcal buildu t ' 1 24 26 - P.:r1or;ltedint~r\;lI. incomplel~. 33 An;ll),i, b)/of
Idc;l1 n:.crvorp e't.
m.1-. I ' Perme;lblIlly: Productlvlty Itxx:X:
I~ging. u-e I(~e..
of. 16. 17
1
."t~~ d " 11"" 3 '"\~'"
nc;lr we ~re. 'V
. 1 1alIOO.
ca..u
. ' 11 ,
Inlmltc cl1nduclivt f t 34 '5 "PP;ln:nt. 70 110" efflClCOCY calculation. 3-
InlinilO: re~rvoir: I y rac ure,. aver.lge. e4u;ltion for. 7. 30 lor general dl3inage-are;l geometry. 8
at:ting. .~5 3K ~ 45 SO 5~ ~6 60 64 dam,,!!e. 3 le't. ;lnaly,is of "ell, from. 7
66.7.4 86 891}()' , ..foml;tlion.I.7.~1-26.29._~O.36.50. P..:ud<ICrilic-.lltem(X'r.ltureandpre"un:
;I"ump,ion. '24 .5~. 56. 58. 59. 63. 70-73 cOrrel;lIK)n:
lh Iln ,..n- 11 ' nc;lr.well.70 crude oIl. 119. I~O
I:vlindnl:;l1
..e
nlulllpl

."e,v'tem. "i '


.~,un:c "" .-'
W II 16 '" I ' '. (X'nne;lI,'ltvlt
h i
Ic~nc,' pn~ul:l.~.
.I~' 8 '
.'. 97
eo
0;1'. I-
' 8

with "- II"'.' d. I n . II Pore volume. Csllm:ltlon of. 53 P~udo pre'-'iUre.66. 76. 79. SO. 85 .
Intcr1o:rence
"" ~re
le,t,.,tordge
I. 16: ~89-91
1;1 0" In.Pon""ty!compre,,ibility pnxJul:t. 89-92 ~udopnxJUt:lion lime. 2. 18.11.30..42
Intem;lti<Jn;l1S,.. f Ut I Pre"ure buildup tC'I. 21~9 P..:udu'tciidy-,;tIle now. 15.25.32.36.37.
1"lChn1n;llte'I,.-' 79-83
em 0 nl ,. I h ' 3
"ctua gr.l!' .-."7."OV.
'" ,n 5,-.J ~3.6-"7 70
"n;lly,i, for v~nil:ally fr.lclun:d ~udo,teiidy-!ilale now c4ualion:
L well, 73. 74 s Pse~J5ICady-stalesolution
IxlUndaryeffect,. 26 ~udu'teady-~ solution. 6-11
liIte-llme line. _~5 dO:lcnninati<1nof P'h,' 31 Pul.c n:,plfl.o;Camplitude. 93-97
liItc-llffiC n:gion. buildup curve. ~3. 2.4. ~6. di,l;loce fnlm ,I<'PCdclUbling. .43 Pul..c le,t,. I. 16. 91-97

www.petroman.ir
SUBJECT INDEX 159

R Simuluneous now of oil. g~. and waler. 2. Ra~y.s. 601.-68


Radial diffusivilY equatIon. 11-13. 64 102
Radial now: Single-phase now: V
continuity
In fracturedequation for.63.100.
reservoir. 72 101 of gas.
of 102 oIl. 2. 2S
reservoir Unlt-slOlX'line. 13. 27-29 .864-6

of a nonideal gas. 2 of slightly com~ible nuid. 21. 10I V


of a slightly compressibk nuid. 103. 104 Skin factor:
with constant
Radius BHP. 104.
of investigation. 10S 18.23.
2. 13-IS. apparent. 33. 87
calculation/estimation of. 23. 27. 28. ~an EvenI~ngen-Hu~ solutions. 3. 1(x)-118
24.29.30. 3S. 42.52.63. SO. 87.89. 30-32.3.5.47. SO-S2. ~. 97 aP'f eqllvalent of produced fresh
91.98 definition of. 5 water. 77
Railroad Commission of Tex». 88 ~pendeocc on recognition of MTR. 24 Van:~;~ wcll. production schedule for.

Ra~y solution.
Ramey.s 27. 29
type curves. 64-68. 87 dl~nslOOless
equation. pressure solutions. 14
16.21.98 V'ISCOSlty
co~ Ia.tlOllS.

Rate history: for damaged or stimulated wells. 7. 13. g~.I~:;


for actual pressure buildup test. 24 22. 30 :~r- 127 128
for buildup test following single for gas wells. 83 Vol ...
now rate. 55 of type curves. 67. 68 u~~ averag~p~ssure. 6

for buildup
now rat~s.lest
55 following twO diff~renl used to charact~rize
Solubility wellbore
of gas in wal~r darnag~.
correlation. 63
124. W
for ideal pressure buildup test. 22 125. 127 ~.;lter saturatK>l1.
immobil.:. 2S
for multirate test. 54 5.>lulion GOR correlation. 119. 120 Well testing:
for single-rate drawdown test. 55 SPE mooograpils on well testing. I. 89 ge~raJ d-.y. 134-137
for two-rate now test. 57 Stabilized now. 76-83 pUrJX'5e.I
References: Stabilized production rate. II SPE Mooograpns. I. 89
analysis of well tests using type curves. 75 Steady-state radial now equation. 4 uo\ingtype curves. 63- 7S
d.:velopmenl of differential ':ljuations for Stimulation. wellbore. I. 13.21.30.31. 33. Wellbo~
nuid now in porous media. 102 63.64.68.69 calculatK>l1of ~re,; beyond. 5. 6
drillslem. interfe~nce. pulse. wi~line Superposition: damage. 1.21.30-34.63.64
tests. 99 principles of. 2. IS-17. 35.41 dimensionless~re solution. 14
no-. lests. 62 use of. 18.40.44.55.77. 91 eff~ive radius. definition. 31. 32
fluid now in porous media. 20 pre~u~ drop near. 32
gas well testing. 88 T schcmalic of ~~ di"tribution near. 5
general theory of well testing. 137 )thematic containing single-phase liquid
Introduction 10 weillesting. I TransmIssibility: or gaJ.. 12
p~~ buildup test. 49 fomtatlOn. 69. 71 schematic with nvving liquid/gas
ro.:k and nuid propeny correlations. 133 near-well. 68-70 Interfxr.12
van Everdingen and Hu~ 'iOlutions to Turbulent p~ssu~ loss. 44 stimulalion. I. 1.3.21. 30. 31. 33. 63. 64.
diffusivity equations. 118 Two-rate flow test. S6-.59 68. 69 --
R.:servoir limits testing. 6. 21. 41-44 Th!oretlcal mettKx! for analyzing g~ flow stOf:lge. II. 89
Reservoir size. .:stimating. 44 test data. 78. 79. 82. 84 'torolge constant. 12. 13. 63-69
Type<urves: slorag~ distonion. 13.27-29.31-33.52
S analysis. 1.63-75.89 54.57. 58.~. 63. 6.5. 67-69. 86. 87'
..fundamentals. 63. 64 temporary "'Ompletion.~.98
ganen n oJ 7 1- 74 1__1 CI\
Sealing fault. 41. 42 Grin
Shape factors....unUdUln2-.}V
single-well draInage 9 10 McKinle y ' s 68 71 "' IreIInc ,ormation
..- ~(S. I. 98. 99

www.petroman.ir
www.petroman.ir

You might also like